Consti II - Pojas and Jumao-As

April 5, 2017 | Author: Reyryan Apor | Category: N/A
Share Embed Donate


Short Description

Download Consti II - Pojas and Jumao-As...

Description

THE FRATERNAL ORDER OF SAINT THOMAS MORE TAU MU Fraternity and SIGMA TAU MU Sorority Ateneo de Davao University College of Law -PRAYER by SAINT THOMAS MORE-

TAU MU TAU MU TAU MU TAU MU TAU MU TAU MU TAU MU TAU MU

Bill of Rights (Definition) It is the set of prescriptions setting forth the fundamental civil and political rights of the individual, and imposing the limitations on the powers of government as a means of securing the enjoyment of those rights. The Bill of Rights is designed to preserve the ideals of liberty, equality and security “against the assaults of opportunism, the expediency of the passing hour, the erosion of small encroachments, and the

TAU MU

CONSTITUTIONAL LAW II

TAU MU TAU MU

The above was written while St. Thomas was imprisoned in the Tower of London.

The Fraternal Ateneo de Davao

TAU MU TAU MU

“Give me the grace, Good Lord To set the world at naught. To set the mind firmly on You and not to hang upon the words of men's mouths. To be content to be solitary. Not to long for worldly pleasures. Little by little utterly to cast of the world and rid my mind of all its business. Not to long to hear of earthly things, but that the hearing of worldly fancies may be displeasing to me. Gladly to be thinking of God, piteously to call for His help. To lean into the comfort of God. Busily to labor to love Him. To know my own vileness and wretchedness. To humble myself under the mighty hand of God. To bewail my sins and, for the purging of them, patiently to sufer adversity. Gladly to bear my purgatory here. To be joyful in tribulations. To walk the narrow way that leads to life. To have the last thing in remembrance. To have ever before my eyes my death that is ever at hand. To make death no stranger to me. To foresee and consider the everlasting fire of Hell. To pray for pardon before the judge comes. To have continually in mind the passion that Christ sufered for me. For His benefits unceasingly to give Him thanks. To buy the time again that I have lost. To abstain from vain conversations. To shun foolish mirth and gladness. To cut of unnecessary recreations. Of worldly substance, friends, liberty, life and all, to set the loss at naught, for the winning of Christ. To think my worst enemies my best friends, for the brethren of Joseph could never have done him so much good with their love and favor as they did him with their malice and hatred. These minds are more to be desired of every man than all the treasures of all the princes and kings, Christian and heathen, were it gathered and laid together all in one heap. Amen”

KITY

TAU MU TAU MU TAU MU TAU MU TAU MU TAU MU

CONSTITUTIONAL LAW II Order of Saint Thomas More Atty. Philip John Pojas/Atty. Rovyne G. Jumao-as, RN University College of Law

scorn and derision of those who have no patience with general principles.” [Nachura, Outline Reviewer in Political Law 2006, p. 81] Significance of the Bill of Rights Government is powerful. When unlimited, it becomes tyrannical. The Bill of rights is a guarantee that there are certain areas of a person’s life, liberty, and property which governmental power may not touch. All powers of the government are limited by the Bill of Rights. [Bernas, The 1987 Philippine Constitution: A Comprehensive Reviewer, p. 22] Classification of Rights 1. Political Rights – granted by law to members of community in relation to their direct or indirect participation in the establishment or administration of the government [San Beda Reviewer 2006, p. 7]. They refer to the right to participate, directly or indirectly, in the establishment or administration of government, e.g. the right of sufrage, the right to hold public office, etc. [Nachura, Outline Reviewer in Political Law 2006, p. 81]; 2. Civil Rights – rights which municipal law will enforce at the instance of private individuals for the purpose of securing them the enjoyment of their means of happiness [San Beda Reviewer 2006, p. 7]. Those rights that belong to every citizen of the state or country, or, in a wider sense, to all its inhabitants, and are not connected with the organization or administration of government. They include the rights to property, marriage, equal protection of the laws, freedom of contract, etc. [Nachura, Outline Reviewer in Political Law 2006, p. 81] ; 3. Social and Economic Rights; 4. Human Rights; The diference between the guarantees of the Bill of Rights and the guarantees that are found in Article XIII on social justice: the Bill of Rights focuses on civil and political rights, whereas Article XIII focuses on social and economic rights. Moreover, the guarantees in the Bill of Rights are generally self-implementing, i.e. they can be appealed to even in the absence of implementing legislation. On the other hand, the social and economic rights guaranteed in Article XIII generally require implementing legislation. [Bernas, The 1987 Philippine Constitution: A Comprehensive Reviewer, p. 22] *Human Rights vs. Property Rights: Human Rights will prevail over Property Rights (Phil. Blooming Mills case) *In a democracy, the preservation and enhancement of the dignity and worth of a person is the central core as well as the cardinal article of ?faith? in our civilization. The inviolable character of man as an individual must be protected to the largest possible extent in his thoughts and in his beliefs as the citadel of his person.

ACADCOM 2010; Contributors: Gene Geocaniga, Jarissa Guiani, Darlene Magabilen TAU MU Page 1 of 179

CONSTITUTIONAL LAW II Order of Saint Thomas More Atty. Philip John Pojas/Atty. Rovyne G. Jumao-as, RN University College of Law

KITY

Ateneo de Davao

are

superior

to

PBMEO vs. PBMCI 51 SCRA 189 (1973)

TAU MU TAU MU TAU MU TAU MU TAU MU TAU MU

Facts: Private respondent was dismissed by her employer for dishonesty. The evidence against her consisted of a check worth P640.00 which was discovered by her co-employee who opened an envelope addressed to respondent. The NLRC

TAU MU

WATEROUS DRUG vs. NLRC 280 SCRA 735 (1997)

TAU MU

Facts: Andre Marti and his wife wanted to have some packages delivered to Switzerland by a forwarding company. In accordance with the SOP of the company, the company inspected the package. It turned out that the package contained marihuana which was neatly stashed to avoid detection. A case was filed against Marti by the State for violation of the Dangerous Drugs Act. Marti contested that there was an illegal search and invoked his right against unreasonable search, therefore, the evidences should be held inadmissible in court. TAU MU Issue: W/N BOR can be invoked against a private entity Held: The court ruled that he cannot invoke this right because the Bill of Rights can only be invoked against the State. It governs the relationship of the State and its citizens. It does not apply to issues between two individuals (Marti and the forwarding company). The search was conducted by a private individual and not a peace officer. The police were just looking as the proprietor did the search of the package.

TAU MU

PEOPLE vs. MARTI 193 SCRA 57 (1991)

TAU MU TAU MU

Bill of Rights can only be invoked against the State

TAU MU TAU MU

Facts: Union wanted to participate in a demonstration. The management allowed them on the condition that it should not be during their shift so that operation will not be hampered. But union wanted all of the members to participate at the same time and they did. As a consequence, the management dismissed the officers of the union. TAU MU Issue: Whether the rights of the workers (right to assembly, expression and petition for redress of grievances) should be given more importance over the rights of the management-property rights Held: The court ruled in favor of the workers. Human rights enjoy primacy over property rights. Not all rights are equal. There is a hierarchy of rights in the Bill.

TAU MU TAU MU TAU MU TAU MU TAU MU TAU MU

BASIC PRINCIPLES Basic Human Rights Property Rights

The Fraternal

ruled that the dismissal was illegal as the only evidence supporting the charge is inadmissible for having been obtained in violation of her right against unreasonable searches and seizures. Held: The evidence is inadmissible. The search was made by a private individual. [As ruled in People v. Marti, the Constitution, in laying down the principles of government and fundamental liberties of the people, does not govern relationship between individuals. The constitutional prescription against unlawful searches and seizures therefore applies as a restraint directed against the government and its agencies tasked with the enforcement of the law. Thus, it could be invoked only against the State to whom the restraint against arbitrary and unreasonable exercise of power is imposed.] The recourse of citizens against such assaults by private individuals is through criminal and civil proceedings, not the exclusionary rule. PEOPLE vs. DOMANTAY 307 SCRA 1 (1999) Facts: Accused was charged with rape with homicide. While detained in a municipal jail, he was interviewed by a radio reporter during which he confessed to the crime. Two or three meters away from the reporters were policemen, but no lawyer assisted the accused. Is the confession of accused to the reporter admissible? Held: Yes. The confession is not covered by Sec. 12 (1) of the Bill of Rights. The Bill of Rights does not concern itself with the relation between a private individual and another individual. It governs the relationship between the individual and the State. The prohibitions therein are primarily addressed to the State and its agents. In this case, the presence of the police officers 23 meters away did not exert undue pressure or influence on accused or coerced him into giving his confession. Accused could have refused to be interviewed, but instead he agreed. PEOPLE vs. MENDOZA 301 SCRA 66 (1999) Facts: Accused is charged with parricide. After the death of his wife, her father came to the house and took her personal belongings as well as a Mission Order and a Memorandum Receipt for a .38 caliber Colt Revolver issued in favor of accused. Accused claims that the documents were procured in violation of his constitutional right against unreasonable searches. Held: the right invoked by accused is a restraint directed only against the government and its agencies. The constitutional protection against unreasonable searches and seizures refers to the immunity of one’s person from interference by government and it cannot be extended to acts committed by private individuals so as to bring it within the ambit of alleged unlawful intrusion. In the instant case, the memorandum receipt and mission order were discovered by accused’s father-in-law. No search warrant is necessary.

ACADCOM 2010; Contributors: Gene Geocaniga, Jarissa Guiani, Darlene Magabilen TAU MU Page 2 of 179

SERRANO vs. NLRC 323 445 (2000)

TAU MU TAU MU TAU MU TAU MU TAU MU TAU MU TAU MU

Facts: Accused committed a crime on May 3, 1982. he was investigated by the police during which he waived his right to a lawyer in writing but without the assistance of counsel. He is now before the Supreme Court challenging the admissibility of his confession on the ground that under the 1987 Constitution the waiver of the right to counsel can only be made with the assistance of counsel. Held: Petitioner’s contention that Art. III, Sec. 12 of the 1987 Constitution should be given retroactive efect for being favorable to him as accused, cannot be sustained. While Art. 22 of the Revised Penal Code provides that “penal laws shall have a retroactive efect insofar as they favor the person guilty of a felony who is not a habitual criminal,” what is being constructed here is a constitutional provision specifically contained in the Bill of Rights which is obviously not a penal statute. A bill of rights is a declaration of the individual rights and privileges which the Constitution is designed to protect against violations by government, or by individuals or groups of individuals. TAU MU

TAU MU

FILOTEO, JR. vs. SANDIGANBAYAN 263 SCRA 222 (1996)

TAU MU TAU MU

Bill of Rights has no retroactive application

The Fraternal Ateneo de Davao

TAU MU TAU MU

Facts: Serrano was head of the Security Checker’s Section of Isetann Department Store. In 1999, as a cost-cutting measure Isetann phased out the entire security section and engaged the services of an independent security agency. Thus, it wrote a notice of termination to Serrano efective on the same day, contrary to the provisions of Art. 283 of the Labor Code which requires a one-month notice. Was the constitutional right of Serrano violated? TAU MU Held: No. The employer’s failure to comply with the notice requirement does not constitute a denial of due process, but a mere failure to observe a procedure for termination. The reason is that the due process clause is a limitation on government power, not on private power such as the termination of employment under the Labor Code. Secondly, the notice and hearing are required under the process clause before the power of organized society are brought to bear upon the individual. Under Art. 283, the failure to comply with the notice requirement makes the termination of his employment merely inefectual, but not illegal. Consequently, he is not entitled to reinstatement. However, he must be paid backwages from the time his employment was terminated until it is determined that his termination is for just cause because the failure to hear him before he is dismissed renders the termination without any legal efect.

KITY

TAU MU TAU MU TAU MU TAU MU TAU MU TAU MU

CONSTITUTIONAL LAW II Order of Saint Thomas More Atty. Philip John Pojas/Atty. Rovyne G. Jumao-as, RN University College of Law

INHERENT POWERS OF THE STATE Fundamental/Inherent Powers (Definition) The fundamental powers of the State are the police power, the power of eminent domain, and the power of taxation. These powers are inherent and do not need to be expressly conferred by constitutional provision on the State. They are supposed to co-exist with the State. The moment the State comes into being, it is deemed invested with these three powers as its innate attributes. [Cruz, Constitutional Law 2007, p.37]. These powers are considered inherent because they belong to the very essence of government and without them no government can exist. A constitution can only define and delimit them and allocate their exercise among various government agencies. A constitution does not grant them. [Bernas, The 1987 Philippine Constitution: A Comprehensive Reviewer, p. 22] Briefly, the police power is the power of the State to regulate liberty and property for the promotion of the general welfare. The power of eminent domain enables the State to forcibly acquire private property, upon payment of just compensation, for some intended public use. By the power of taxation, the State is able to demand from the members of society their proportionate share or contribution in the maintenance of the government [Cruz, Constitutional Law 2007, p.37]. TAU MU Similarities The three inherent powers of the State are similar in the following respects: 1. They are inherent in the State and may be exercised by it without need of express constitutional grant; 2. They are not only necessary but indispensable. The State cannot continue or be efective unless it is able to exercise them; 3. They are methods by which the State interferes with private rights; 4. They all presuppose an equivalent compensation for the private rights interfered with. 5. They are exercised primarily by the legislature. [Cruz, Constitutional Law 2007, p.37].

TAU MU

Differences The three inherent powers of the State difer from each other in the following ways: 1. The police power regulates both liberty and property. The power of eminent domain and the power of taxation afect only property rights; 2. The police power and the power of taxation may be exercised only by the government. The power of eminent domain may be exercised by some private entities; 3. The property taken in the exercise of the police power is destroyed because it is noxious or intended for a noxious purpose. The property taken under the power of

ACADCOM 2010; Contributors: Gene Geocaniga, Jarissa Guiani, Darlene Magabilen TAU MU Page 3 of 179

4.

eminent domain and the power of taxation is intended for a public use or purpose and is therefore wholesome; TAU MU The compensation of the person subjected to the police power is the intangible altruistic feeling that he has contributed to the general welfare. The compensation involved in the other powers is more concrete, to wit, a full and fair equivalent of the property expropriated or protection and public improvements for the taxes paid. [Cruz, Constitutional Law 2007, p.38]

TAU MU TAU MU TAU MU TAU MU TAU MU TAU MU TAU MU

Scope/Characteristics Police power has been characterized as “the most essential, insistent and the least limitable of powers, extending as it does to all the great public needs.” Negatively, it has been defined as “that inherent plenary power in the State which enables it to prohibit all that is hurtful to the comfort, safety, and welfare of society.” [Bernas, The 1987 Philippine Constitution: A Comprehensive Reviewer, p. 23]

TAU MU

As thus defined, the police power easily outpaces the other two inherent powers as instruments of the State in interfering with private rights. The power of eminent domain affects not all of the people directly but only those whose property is needed for conversion to public use. The power of taxation, while imposed on most of the people directly, demands only part of their money as their contribution to the upkeep of the government. Both these powers involve only property rights. By contrast, the police power regulates not only the property but, more importantly, the liberty of private persons, and virtually all the people. It is in this sense that police power may be regarded as infinitely more important than eminent domain and taxation. [Cruz, Constitutional Law 2007, p. 40]

TAU MU

Police Power (Definition) It is the power of promoting public welfare by restraining and regulating the use of liberty and property [Nachura, Outline Reviewer in Political Law 2006, p. 43].

TAU MU TAU MU

POLICE POWER

The Fraternal Ateneo de Davao

TAU MU TAU MU

Limitations Although inherent and indispensible, the fundamental powers of the State are not without restrictions. As ours is a government of limited powers, even these prerogatives may not be exercised arbitrarily, to the prejudice of the Bill of Rights. The presumption in libertarian societies is in favor of private rights and against attempts on the part of the State to interfere with them. “Constitutional provisions for the security of persons and property should be liberally construed.” Hence, the exercise of these fundamental powers is subject at all times to the limitations and requirements of the Constitution and may in proper cases be annulled by the courts of justice. [Cruz, Constitutional Law 2007, p.38]

KITY

TAU MU TAU MU TAU MU TAU MU TAU MU TAU MU

CONSTITUTIONAL LAW II Order of Saint Thomas More Atty. Philip John Pojas/Atty. Rovyne G. Jumao-as, RN University College of Law

Police power is the most pervasive, the least limitable, and the most demanding of the three powers [Nachura, Outline Reviewer in Political Law 2006, p. 43]. It may be exercised as long as the activity or the property sought to be regulated has some relevance to the public welfare. The reach is virtually limitless. The person’s acts and acquisitions are hemmed in by the police power. The justification is found in the ancient Latin maxims, salus populi est suprema lex (the welfare of the people is the supreme law) and sic utere tuo ut alienum non laedas (So use your own as not to injure another's property), which calls for the subordination of individual benefit to the interests of the greater number [Cruz, Constitutional Law 2007, p.40]. TAU MU Owing to the need to protect society from the inordinate assertion of individual liberty, it has been held that the police power may not be bargained away through the medium of a contract or even a treaty. The impairment clause must yield to the police power whenever the contract deals with a subject afecting the public welfare [Cruz, Constitutional Law 2007, p.41]. A law enacted in the exercise of police power to regulate certain government activities or transactions could be given retroactive efect and may reasonably impair vested rights or contracts. Police power legislation is applicable not only to future contracts, but equally to those already in existence. Non-impairment of contracts or vested rights clauses will have to yield to the superior and legitimate exercise by the State of the police power [Nachura, Outline Reviewer in Political Law 2006, p. 43]. The taxing power may be used as an implement of police power [Nachura, Outline Reviewer in Political Law 2006, p. 43]. Eminent domain may be used as an implement to attain the police objective [Nachura, Outline Reviewer in Political Law 2006, p. 43]. TAU MU Exercise of Police Power The national government, through the legislative department, exercises police power. [Bernas, The 1987 Philippine Constitution: A Comprehensive Reviewer, p. 23]. However, Congress may validly delegate this power to the President, to administrative bodies and to lawmaking bodies of local government units. Local government units exercise the power under the general welfare clause (Sec. 16 of RA 7160 and under Secs. 391, 447, 458 and 468 of RA 7160) [Nachura, Outline Reviewer in Political Law 2006, p. 44]. The exercise of police power lies in the discretion of the legislative department. Given a police problem, it is entirely up to the legislature to decide, whether or not, in the first place, it should act against the problem. If it does, well and good; but if it does not, it may not be compelled to do so by judicial process. No mandamus is available to coerce the exercise of the police power [Cruz, Constitutional Law 2007, p.46].

ACADCOM 2010; Contributors: Gene Geocaniga, Jarissa Guiani, Darlene Magabilen TAU MU Page 4 of 179

Tests for Valid Exercise of Police Power 1. Lawful Subject The interests of the public in general, as distinguished from those of a particular class, require the exercise of the power. This means that the activity or property sought to be regulated afects the general welfare; if it does, then the enjoyment of the rights flowing therefrom may have to yield to the interests of the greater number [Nachura, Outline Reviewer in Political Law 2006, p. 45]. TAU MU

TAU MU TAU MU TAU MU TAU MU TAU MU TAU MU TAU MU

Municipal Ordinances (Requisites for Validity): a. Must not contravene the Constitution or any statute; b. Must not be unfair or oppressive; c. Must not be partial or discriminatory; d. Must not prohibit, but may regulate, trade; e. Must not be unreasonable; f. Must be general in application and consistent with public policy [Nachura, Outline Reviewer in Political Law 2006, p. 46].

TAU MU

Additional Limitations (When exercised by delegate): a. Express grant by law ( Secs. 16, 391, 447, 458 and 468, RA 7160 for LGUs); b. Within territorial limits (for LGUs, except when exercised to protect water supply); c. Must not be contrary to law (activity prohibited by law cannot in the guise of regulation be allowed; an activity allowed by law may be regulated, but not prohibited) [Nachura, Outline Reviewer in Political Law 2006, p. 46].

TAU MU

Even if the purpose be within the scope of the police power, the law will still be annulled if the subject is sought to be regulated in violation of the second requirement. In Constitutional Law, the end does not justify the means. The lawful objective, in other words, must be pursuant to a lawful method; that is, both the end and the means must be legitimate. Lacking such concurrence, the police measure shall be struck down as an arbitrary intrusion into private rights [Cruz, Constitutional Law 2007, p.49].

TAU MU TAU MU

2. Lawful Means The means employed are reasonably necessary for the accomplishment of the purpose, and not unduly oppressive on individuals [Nachura, Outline Reviewer in Political Law 2006, p. 46].

The Fraternal Ateneo de Davao

TAU MU TAU MU

The first requisite simply means that the subject of the measure is within the scope of the police power, that is, that the activity or property sought to be regulated afects the public welfare. If it does, the enjoyment of private rights may be subordinated to the interests of the greater number, on the time-honored principle that the welfare of the people is the supreme law [Cruz, Constitutional Law 2007, p.49].

KITY

TAU MU TAU MU TAU MU TAU MU TAU MU TAU MU

CONSTITUTIONAL LAW II Order of Saint Thomas More Atty. Philip John Pojas/Atty. Rovyne G. Jumao-as, RN University College of Law

CASES ON POLICE POWER ERMITA-MALATE HOTEL vs. CITY OF MANILA July 31, 1967 Facts: The city of Manila promulgated an ordinance which regulated the operation of hotels & motels. The ordinance requires the following: 1. Registration in the lobby should be open to public view 2. That the mayor can conduct inspection of the establishment anytime 3. Prohibits the renting of the room more than once within 24 hours 4. Minors should be accompanied by parents or adults Held: the ordinance is valid. The reason of the law is to safeguard public morals due to the alarming increase in prostitution traceable to hotel/motel businesses. The 24 hour limit is reasonable considering that clandestine meetings are usually shorter than legitimate ones. TAU MU TAXICAB OPERATORS vs. BOT 119 SCRA 597 (1982) Facts: The law concerns the phasing out of dilapidated taxicabs. The lawful subject here is public welfare. The lawful method is to phase out old taxis to ensure public safety. Taxicab owners and operators challenged this by saying that it afects their property rights. Held: The court held that the law is not unduly oppressive, therefore valid. Six years cut-of time is sufficient for the owners to have substantial return of their investment. ICHONG vs. HERNANDEZ 101 PHIL 1055 Held: There is substantial distinction between aliens and Filipinos since alien’s loyalty to the country is transitory in nature which is motivated only by some kind of personal interest. The purpose of the law is to prevent aliens from engaging in business in the Philippines. It is observed that there is alien dominance in retail trade. That is the reason why the law intends to give it back to the Filipinos. It also applies to all members of the class, except to Americans, since they are covered by the Parity Amendment found in the 1935 Constitution. This provision authorizes Americans to engage in all trade in the country. The classification also has relevance to the lawful subject and method. TAU MU LUTZ vs. ARANETA 98 PHIL 148 Facts: Walter Lutz, as Judicial Administrator of the Intestate Estate of Antonio Jayme Ledesma, sought to recover the sum of P14,6666.40 paid by the estate as taxes from the Commissioner under Section e of Commonwealth Act 567 (the

ACADCOM 2010; Contributors: Gene Geocaniga, Jarissa Guiani, Darlene Magabilen TAU MU Page 5 of 179

TAU MU TAU MU TAU MU TAU MU TAU MU TAU MU

It is the ultimate right of sovereign power to appropriate, not only the public, but [even] the private property of all citizens within the territorial sovereignty, to public purposes

TAU MU

Eminent Domain (Definition) Also known as the power of expropriation [Nachura, Outline Reviewer in Political Law 2006, p. 46].

TAU MU

EMINENT DOMAIN (Sec. 9)

TAU MU

Facts: Pres. Marcos promulgated an EO which prohibited the inter-provincial movement of carabaos and carabeefs. Anyone caught violating the law will have his carabao and carabeef confiscated. The purpose of the law is to prevent indiscriminate slaughter of carabaos so that they could be preserved for agricultural use by the farmers. The lawful subject is general welfare since the Philippines is an agricultural country. The farmers need them for farming instead of using machineries. (The country was in an energy crisis) Held: The court held that there was a lawful subject which is general welfare. However, the method chosen by the government has no logical connection with the purpose of the law. Prohibiting the inter-provincial transfer of carabaos would not prevent their indiscriminate slaughter. The carabaos can still be killed anywhere without even transferring them. TAU MU

TAU MU TAU MU

YNOT VS IAC 148 SCRA 659

The Fraternal Ateneo de Davao

TAU MU TAU MU

Sugar Adjustment Act), alleging that such tax is unconstitutional as it levied for the aid and support of the sugar industry exclusively, which is in his opinion not a public purpose. Issue: Whether the tax is valid in supporting an industry. Held: The tax is levied with a regulatory purpose, i.e. to provide means for the rehabilitation and stabilization of the threatened sugar industry. The act is primarily an exercise of police power, and is not a pure exercise of taxing power. As sugar production is one of the great industries of the Philippines; and that its promotion, protection and advancement redounds greatly to the general welfare, the legislature found that the general welfare demanded that the industry should be stabilized, and provided that the distribution of benefits therefrom be readjusted among its component to enable it to resist the added strain of the increase in tax that it had to sustain. Further, it cannot be said that the devotion of tax money to experimental stations to seek increase of efficiency in sugar production, utilization of byproducts, etc., as well as to the improvement of living and working conditions in sugar mills and plantations, without any part of such money being channeled diectly to private persons, constitute expenditure of tax money for private purposes. The tax is valid.

KITY

TAU MU TAU MU TAU MU TAU MU TAU MU TAU MU

CONSTITUTIONAL LAW II Order of Saint Thomas More Atty. Philip John Pojas/Atty. Rovyne G. Jumao-as, RN University College of Law

[Bernas, The 1987 Philippine Constitution: A Comprehensive Reviewer, p. 101]. Eminent domain is described as “the highest and most exact idea of property remaining in the government” that may be acquired for some public purpose through a method “in the nature of a compulsory sale to the State.” Being inherent, the power of eminent domain does not need to be specifically conferred on the government by the Constitution. As it happens, however, it is expressly provided in Article III, Section 9, that “private property shall not be taken for public use without just compensation.” This provision is not a grant but indeed a limitation of the power as its negative and restrictive language clearly suggests [Cruz, Constitutional Law 2007, p.62]. Strict Construction against Expropriator The limiting function imposed by the Bill is in keeping with the philosophy of the Bill of Rights against the arbitrary exercise of governmental powers to the detriment of individual rights. Given this function, the provision of Article III, Section 9 should therefore be strictly interpreted against the expropriator and liberally in favor of the property owner. “The exercise of the right of eminent domain, whether directly by the State, or by its authorized agents, is necessarily in derogation of private rights, and the rule in that case is that the authority must be strictly construed.” [Cruz, Constitutional Law 2007, p.63] TAU MU Who May Exercise the Power Congress, and by delegation, the President, administrative bodies, LGUs, and even private enterprises performing public services [Nachura, Outline Reviewer in Political Law 2006, p. 48]. Under existing laws, the following may exercise the power of expropriation: a. The Congress; b. The President of the Philippines; c. The various local legislative bodies; d. Certain public corporations, like the Land Authority and the National Housing Authority; e. Quasi-public corporations like Philippine National Railways, PLDT and the MERALCO. Eminent Domain vs. Destruction from Necessity EMINENT DOMAIN DESTRUCTION FROM NECESSITY The right of eminent The right of necessity domain is a public arises under the laws of right; it arises from the society or society itself. laws of society and is It is the right of selfvested in the state or defense, of selfits grantee, acting preservation, whether under the right and applied to persons or to power of the state, or property. It is a private benefit of the state, or right vested in every those acting under it. individual, and with which the right of the state or state necessity

ACADCOM 2010; Contributors: Gene Geocaniga, Jarissa Guiani, Darlene Magabilen TAU MU Page 6 of 179

of

Property must be for public use

Scope of the Power of Eminent Domain In the hands of Congress the scope of the power is, like the scope of legislative power itself, plenary. It is as broad as the scope of police power itself. It can thus reach every form of property which the State might need for public use. It can reach even private property already dedicated to public use or even property devoted to religious worship [Bernas, The 1987 Philippine Constitution: A Comprehensive Reviewer, p. 22].

Ateneo de Davao

properties [Cruz, Constitutional Law 2007, p.67]. All private property capable of ownership may be expropriated, except money and choses in action [Nachura, Outline Reviewer in Political Law 2006, p. 50]. Expropriation of money would be a futile act because of the requirement for the payment of just compensation, usually also in money. A chose in action is “a personal right not reduced into possession but recoverable by a suit at law, a right to receive, demand or recover a debt, demand or damages on a cause of action ex contractu or for a tort or omission of duty.” It is essentially conjectural both as to its validity and its value [Cruz, Constitutional Law 2007, p.68]. Even services may be subject to eminent domain [Nachura, Outline Reviewer in Political Law 2006, p. 50].

TAU MU

TAU MU TAU MU TAU MU TAU MU TAU MU

b. Private Property Anything that can come under the dominion of man is subject to expropriation. This will include real and personal, tangible and intangible

TAU MU

The issue of the necessity of the expropriation is a matter properly addressed to the RTC in the course of the expropriation proceedings. If the property owner objects to the necessity of the takeover, he should say so in his Answer to the Complaint. The RTC has the power to inquire into the legality of the exercise of the right of eminent domain and to determine whether there is a genuine necessity for it. [Nachura, Outline Reviewer in Political Law 2006, p. 49].

TAU MU

MU

TAU MU

Requisites for the Exercise of the Power of Eminent Domain a. Necessity Questions of necessity or wisdom are essentially political when decided by the national legislature and are usually not subject to judicial review [Cruz, Constitutional Law 2007, p.65]. But when exercised by a delegate, the determination of whether there is genuine necessity for the exercise is a justiciable question [Nachura, Outline Reviewer in Political Law 2006, p. 49]. TAU

TAU MU

Private property may be expropriated for public use and upon payment of just compensation; condemnation of property is justified only if it is for the public good and there is genuine necessity therefor of a public character. Consequently, the courts have the power to inquire into the legality of the right of eminent domain and to determine whether or not there is a genuine necessity therefor [Cruz, Constitutional Law 2007, p.66].

TAU MU TAU MU

Foundation of Right to Exercise the Power of Eminent Domain The foundation of the right to exercise eminent domain is genuine necessity and that necessity must be of public character. Government may not capriciously or arbitrarily choose which private property should be expropriated [Nachura, Outline Reviewer in Political Law 2006, p. 49].

The Fraternal

TAU MU TAU MU

Requires payment just compensation

has nothing to do. Does not require payment of just compensation Cannot require the conversion of the property taken to public use

KITY

TAU MU TAU MU TAU MU TAU MU TAU MU TAU MU

CONSTITUTIONAL LAW II Order of Saint Thomas More Atty. Philip John Pojas/Atty. Rovyne G. Jumao-as, RN University College of Law

Property already devoted to public use is still subject to expropriation, provided this is done directly by the national legislature or under a specific grant of authority to the delegate. A mere general authority may not suffice. In such a case, the courts will have authority to inquire into the necessity of the expropriation and, finding none, refuse to allow it [Cruz, Constitutional Law 2007, p.68]. c. Taking Taking, as the term is commonly understood, imports a physical dispossession of the owner, as when he is ousted from his land or relieved of his watch or his car and is thus deprived of all beneficial use and enjoyment of his property. In law, however, the term has a broader connotation. Taking may include trespass without actual eviction of the owner, material impairment of the value of the property or prevention of the ordinary uses for which the property was intended [Cruz, Constitutional Law 2007, p.70]. Taking, in the constitutional sense, may include trespass without actual eviction of the owner, material impairment of the value of the property for which the property was intended. In People vs. Fajardo (104 Phil 44), a municipal ordinance prohibiting a building which would impair the view of the plaza from the highway was considered taking. The property owner was entitled to payment of just compensation [Nachura, Outline Reviewer in Political Law 2006, p. 50]. Requisites of Taking (Republic vs. Castellvi): 1. The expropriator must enter a private property; 2. The entry must be for more than a momentary period; 3. The entry must be under warrant or color of legal authority; 4. The property must be devoted to public use or otherwise informally appropriated or injuriously afected; 5. The utilization of the property for public use must be in such a way as to oust the owner and deprive him of the beneficial enjoyment of the property.

ACADCOM 2010; Contributors: Gene Geocaniga, Jarissa Guiani, Darlene Magabilen TAU MU Page 7 of 179

When municipal property is taken by the State There is compensable taking depending on the nature of the property. If it is patrimonial property of the municipality, that is, property acquired by the municipality with its private funds in its corporate or private capacity, compensation is required. However, if it is any other property such as public buildings or legua communal held by the municipality for the State in trust for the inhabitants, the State is free to dispose of it at will [Bernas, The 1987 Philippine Constitution: A Comprehensive Reviewer, p. 104]. TAU MU

TAU MU TAU MU TAU MU TAU MU TAU MU TAU MU TAU MU TAU MU

Private property condemned for public use sold to private user – effect (Heirs of Moreno vs. Mactan-Cebu, 2005): The predominant precept is that upon abandonment of real property condemned for public purpose, the party who originally condemned the property recovers control of the land if the condemning party continues to use the property for public purpose; however, if the condemning authority ceases to use the property for a public purpose, property reverts to the owner in fee simple. The government’s taking of private property, and then transferring it to private persons under the guise of public use or purpose is the despotism found in the immense power of eminent domain. Moreover, the direct and unconstitutional state’s power to oblige a landowner to renounce his productive and invaluable possession to another citizen, who will use it predominantly for his own private gain, is ofensive to our laws [Bernas, The 1987 Philippine Constitution: A Comprehensive Reviewer, p. 104]. TAU MU

TAU MU

This original meaning of the phrase has now been broadened to cover uses which, while not directly available to the public, redound to their indirect advantage or benefit [Cruz, Constitutional Law 2007, p.76]. TAU MU

TAU MU TAU MU

d. Public Use As a requirement of eminent domain, “public use” is the general concept of meeting public need or public exigency. It is not confined to actual use by the public in its traditional sense. The idea that “public use” is strictly limited to clear cases of “use by the public” has been abandoned. The term “public use” has not been held to be synonymous with public interest, public benefit, public welfare, and public convenience [Nachura, Outline Reviewer in Political Law 2006, p. 51].

The Fraternal Ateneo de Davao

TAU MU TAU MU

Effect of Police Power and Eminent Domain on Private Property* EMINENT DOMAIN POLICE POWER Property is “taken” Property is “regulated” Transfer of ownership No transfer of ownership The term “taking” must The term “regulation” is be compensated. not compensable *[Bernas, The 1987 Philippine Constitution: A Comprehensive Reviewer, p. 104]

KITY

TAU MU TAU MU TAU MU TAU MU TAU MU TAU MU

CONSTITUTIONAL LAW II Order of Saint Thomas More Atty. Philip John Pojas/Atty. Rovyne G. Jumao-as, RN University College of Law

e. Just Compensation Just compensation is described as a full and fair equivalent of the property taken from the private owner by the expropriator. This is intended to indemnify the owner fully for the loss he has sustained as a result of the expropriation. The measure of this compensation is not just the taker’s gain but the owner’s loss. The word “just” is used to intensify the meaning of the word “compensation,” to convey the idea that the equivalent to be rendered for the property taken shall be real, substantial, full, ample [Cruz, Constitutional Law 2007, p.76]. Expressed diferently, the compensation given to the owner is just if he receives for his property a sum equivalent to its “market value” [Bernas, The 1987 Philippine Constitution: A Comprehensive Reviewer, p. 104]. Market Value (Definition) “Market value” has been described in a variety of ways. It is the “price fixed by the buyer and seller in the open market in the usual and ordinary course of legal trade and competition; the price and value of the article established or shown by sale, public or private, in the ordinary way of business; the fair value of property as between one who desires to purchase and one who desires to sell; the current price; the general or ordinary price for which property may be sold in that locality” [Bernas, The 1987 Philippine Constitution: A Comprehensive Reviewer, p. 104]. Owner (Concept) According to Knecht vs. CA, the term “owner” as applied in eminent domain cases refers to all those who have lawful interest in the property to be condemned, including a mortgagee, a lessee and a vendee in possession under an executory contract [Cruz, Constitutional Law 2007, p.78]. When a parcel of land is taken by eminent domain, the owner of the fee is not necessarily the only person who is entitled to compensation. Every person having interest at law or in equity in the land taken is entitled to share in the award. If a person claiming an interest in the land sought to be condemned is not made a party, he is given the right to intervene and lay claim to the compensation [Bernas, The 1987 Philippine Constitution: A Comprehensive Reviewer, p. 108]. Title to the property does not pass until after payment, except in agrarian reform [Nachura, Outline Reviewer in Political Law 2006, p. 57]. TAU MU

Consequential Damages/Consequential Benefits Consequential damages consist of injuries directly caused on the residue of the private property taken by the reason of expropriation. Where, for example, the expropriator takes only a part of a parcel of land, leaving the remainder with an odd shape or area as to be virtually unusable, the owner can claim consequential damages. On the other hand, if the remainder is as a result of the

ACADCOM 2010; Contributors: Gene Geocaniga, Jarissa Guiani, Darlene Magabilen TAU MU Page 8 of 179

expropriation placed in a better location, such as fronting a street where it used to be an interior lot, the owner will enjoy consequential benefits which should be deducted from the consequential damages. Consequential benefits, like consequential damages, must be direct and particular and not merely shared with the rest of the properties in the area, as where there is a general appreciation of lad values because of the public use to which the condemned properties are devoted [Cruz, Constitutional Law 2007, p.79].

TAU MU TAU MU TAU MU TAU MU TAU MU TAU MU TAU MU

The general rule is that the value must be that as of the time of the filing of the complaint for expropriation (Section 4, Rule 67, Rules of Court).

TAU MU

Property Assessment The property taken should be assessed as of the time of the taking, which usually coincides with the commencement of the expropriation proceedings. Where entry precedes the filing of the complaint for expropriation, the assessment should be made as of the time of the entry [Cruz, Constitutional Law 2007, p.83].

TAU MU

Without prompt payment, compensation cannot be considered “just,” for the property owner is made to sufer the consequence of being immediately deprived of his land while being made to wait for a decade or more before actually receiving the amount necessary to cope with his loss. To allow the taking of the landowner’s properties and in the meantime leave them empty-handed by withholding payment of just compensation while the government speculates on whether or not it will pursue expropriation, or worse, for government to subsequently decide to abandon the property and return it to the landowner when it has already been rendered useless by force majeure, it is undoubtedly an oppressive exercise of eminent domain that must never be sanctioned [Nachura, Outline Reviewer in Political Law 2006, p. 54].

TAU MU TAU MU

Reasonable Period Just compensation includes not only the correct determination of the amount to be paid to owner of the land but also the payment of the land within a reasonable period of time from its taking [Bernas, The 1987 Philippine Constitution: A Comprehensive Reviewer, p. 105]. TAU MU

The Fraternal Ateneo de Davao

TAU MU TAU MU

In such a case, the owner is not restricted to payment of the market value of the portion actually taken. In addition to the market value of the portion taken, he is also entitled to payment of consequential damages, if any to the remaining part of the property. At the same time, from the total compensation must be deducted the value of consequential benefits, if any, provided consequential benefits shall not exceed consequential damages [Nachura, Outline Reviewer in Political Law 2006, p. 51]. If the consequential benefits exceed the consequential damages, these items should be disregarded altogether as the basic value of the property should be paid in every case [Cruz, Constitutional Law 2007, p.79].

KITY

TAU MU TAU MU TAU MU TAU MU TAU MU TAU MU

CONSTITUTIONAL LAW II Order of Saint Thomas More Atty. Philip John Pojas/Atty. Rovyne G. Jumao-as, RN University College of Law

Moreover, the filing of the case generally coincides with the taking. When, however, the filing of the case comes later than the time of taking and meanwhile the value of the property has increased because of the use to which the expropriator has put it, the value is that of the time of the earlier taking. Otherwise, the owner would gain undeserved profit. But if the value increased independently of what the expropriator did, then the value is that of the later filing of the case [Bernas, The 1987 Philippine Constitution: A Comprehensive Reviewer, p. 109]. When eminent domain is exercised by a local government unit, the “amount to be paid for the expropriated property shall be determined by the proper court, based on the fair market value at the time of the taking of the property (Sec. 19, RA7160) precisely because the Rules of Court cannot prevail over RA7160, a substantive law [Nachura, Outline Reviewer in Political Law 2006, p. 56]. TAU MU Determination of Just Compensation The principal criterion in determining just compensation is the character of the land at the time of the taking. The tax declaration is only one of the factors to be used in determining the market value of the property for purposes of arriving at the amount to be paid by way of just compensation [Nachura, Outline Reviewer in Political Law 2006, p. 55] Form of Compensation As explicitly provided by Sec. 16(e), RA 6657, the deposit of compensation must be in “cash” or in “Land Bank bonds,” not in any other form, and certainly not in a “trust account.” Entitlement of Owner to Interest When there is delay in the payment of just compensation, the owner is entitled to payment of interest, if claimed; otherwise, interest is deemed waived. In NAPOCOR vs. Angas, the Supreme Court held that the interest due the property owner is at the rate of 6% per annum, prescribed in Article 2209 of the Civil Code, and not 12% per annum under Central Bank Circular No. 416, because the latter applies to loans or forbearance of money, goods or credits, or judgments involving such loans or forbearance of money goods or credits. The kind of interest involved here is by way of damages, hence Art. 2209 of the Civil Code applies. In some expropriation cases, the Court imposed an interest of 12% per annum on the just compensation due the landowner. It must be stressed, however, that in these cases, the imposition of interest was in the nature of damages for delay in payment which, in efect, makes the on the part of the government one of forbearance. It follows that the interest in the form of damages cannot be applied where there was prompt and valid payment of just compensation. Conversely, where there was delay in tendering a valid payment of just compensation, imposition of interest is in order

ACADCOM 2010; Contributors: Gene Geocaniga, Jarissa Guiani, Darlene Magabilen TAU MU Page 9 of 179

[Nachura, Outline Reviewer in Political Law 2006, p. 56].

TAU MU TAU MU TAU MU TAU MU TAU MU TAU MU

Temporary takeover of private business or utility Temporary takeover by the government extends only to the operation of the business and not to the ownership thereof. As such, the government is not required to compensate the private entityowner of the said business as there is not transfer of ownership, whether permanent or temporary. The private entity-owner afected cannot, likewise, claim just compensation for the use of said business and its properties, as the temporary takeover by the government is in the exercise of

TAU MU

Right to repurchase or re-acquire the property The property owner’s right to repurchase the property depends upon the character of the title acquired by the expropriator, e.g. if land is expropriated for a particular purpose with the condition that when that purpose is ended or abandoned, the property shall revert to the former owner, then the former owner can reacquire the property [Nachura, Outline Reviewer in Political Law 2006, p. 58].

TAU MU

MU

TAU MU

Plaintiff’s right to dismiss the complaint in eminent domain In expropriation cases, there is no such thing as the plaintif’s “matter-of-right” to dismiss the complaint, precisely because the landowner may have already sufered damages at the start of the taking. The plaintif’s right to dismiss the complaint has always been subject to Court approval and to certain conditions [Nachura, Outline Reviewer in Political Law 2006, p. 58]. TAU

TAU MU TAU MU

Writ of Possession The issuance of the writ of possession becomes ministerial upon: a. Filing of a complaint for expropriation sufficient in form and substance; b. Upon deposit made by the government of the amount equivalent to fifteen percent (15%) of the fair market value of the property sought to be expropriated per current tax declaration [Nachura, Outline Reviewer in Political Law 2006, p. 58].

The Fraternal Ateneo de Davao

TAU MU TAU MU

Right of Owner in case of Non-payment of Just Compensation As a rule, non-payment of just compensation in an expropriation proceeding does not entitle the private landowner to recover possession of the expropriated lots, but only to demand payment of the fair market value of the property. However, in Republic vs. Vicente Lim, the Supreme Court said that the facts of the case do not justify the application of the rule. It was held that where the government failed to pay just compensation within five (5) years from the finality of the judgment in the expropriation proceedings, the owners concerned shall have the right to recover possession of their property [Nachura, Outline Reviewer in Political Law 2006, p.57]. TAU MU

KITY

TAU MU TAU MU TAU MU TAU MU TAU MU TAU MU

CONSTITUTIONAL LAW II Order of Saint Thomas More Atty. Philip John Pojas/Atty. Rovyne G. Jumao-as, RN University College of Law

the police power and not the power of eminent domain [Nachura, Outline Reviewer in Political Law 2006, p. 59]. RA7279 (Urban Development and Housing Act of 1992) Under RA7279, lands for socialized housing are to be acquired in the following order: 1. Government lands 2. Alienable lands of the public domain 3. Unregistered, abandoned, or idle lands 4. Lands within the declared Areas for Priority Development, Zonal Improvement Program sites, Slum Improvement and Resettlement sites which have not yet been acquired 5. BLISS site which have not yet been acquired 6. Privately owned lands The mode of expropriation is subject to two conditions, namely: a. It shall be resorted to only when the other modes of acquisition have been exhausted b. Parcels owned by small property owners are exempt from such acquisition [Nachura, Outline Reviewer in Political Law 2006, p. 60] Judicial Review [Bernas, The 1987 Philippine Constitution: A Comprehensive Reviewer, p.112] The exercise of the power of eminent domain is subject to judicial review. The following aspects of the exercise of the power have been subject to judicial scrutiny: 1. The adequacy of the compensation 2. The necessity of the taking 3. The “public use” character of the purpose of the taking Instance where exercise of the power of eminent domain is not subject to judicial review When land is expropriated for subdivision and resale for social justice purposes directly by the legislature and not through an inferior agency of the state, the necessity and public purpose of the taking are not subject to judicial review. Article XIII, Section 4, constitutes a textual commitment of discretion on the subject to the legislature. However, if the power is exercised by an agency possessing power delegated by the legislature or when the issue is the adequacy of the compensation, judicial review is still proper.

Res Judicata The principle of res judicata does not bar the right of the State or its agent to expropriate private property. “The very nature of the eminent domain, as an inherent power of the State, dictates that the right to exercise the power be absolute and unfettered by a prior judgment or res judicata. The scope of eminent domain is plenary and, like police power, can ‘reach every form of property which the State might need for public use.’”

ACADCOM 2010; Contributors: Gene Geocaniga, Jarissa Guiani, Darlene Magabilen TAU MU Page 10 of 179

While the principle of res judicata does not denigrate the right of the State to exercise eminent domain, it does not apply to specific issues decided in a previous case. For example, a final judgment dismissing an expropriation suit on the ground that there was not prior ofer precluded another suit raising the same issue; it cannot, however, bar the State or its agent from thereafter complying with this requirement, as prescribed by law, and subsequently exercising its power of eminent domain over the same property. (Municipality of Paranaque vs. VM Realty Corporation) TAU MU

TAU MU TAU MU TAU MU TAU MU TAU MU TAU MU TAU MU

Essential requisites for LGU to validly exercise the power of eminent domain (Section 19, RA7160): a. An ordinance is enacted by the local legislative council authorizing the local chief executive, in behalf of the LGU, to exercise the power of eminent domain or

TAU MU

Ordinance (Requisite for Valid Expropriation) In Municipality of Paranaque vs. VM Realty Corporation, the Supreme Court declared that there was lack of compliance with Sec. 19, RA7160, where the Municipal Mayor filed a complaint for eminent domain over two parcels of land on the strength of a resolution passed by the Sangguniang Bayan, because what is required by law is an ordinance.

TAU MU

Exercise of Eminent Domain by Local Government Unit By express legislative authority granted by Congress in Section 19, RA7160, local government units may expropriate private property for public use, or purpose, or welfare, for the benefit of the poor and the landless. Thus, in Moday vs. CA, the Supreme Court held that the Sangguniang Panlalawigan of Agusan del Sur was without authority to disapprove Bunawan Municipal Reso 43-89 because clearly, the Municipality of Bunawan has the authority to exercise the power of eminent domain and its Sangguniang Bayan the capacity to promulgate the assailed resolution [Nachura, Outline Reviewer in Political Law 2006, p. 52].

TAU MU TAU MU

The State may not enter into a contract which in efect binds it not to exercise the power of eminent domain. Like police power and the power of taxation, the power of eminent domain is inalienable. “There can be no right to restrain by contract the power of eminent domain, it must also of necessity to follow that any contract by which it was sought to accomplish that result would be inefficacious for want of power.”

The Fraternal Ateneo de Davao

TAU MU TAU MU

Expropriation as substitute for the enforcement of a valid contract Expropriation lies only when it is made necessary by the opposition of the owner to the sale or by the lack of any agreement as to price. Where there is a valid and subsisting contract, between the owners of the property and the expropriating authority, there is no reason for the expropriation.

KITY

TAU MU TAU MU TAU MU TAU MU TAU MU TAU MU

CONSTITUTIONAL LAW II Order of Saint Thomas More Atty. Philip John Pojas/Atty. Rovyne G. Jumao-as, RN University College of Law

pursue expropriation proceedings over a particular private property; b. The power of eminent domain is exercised for public use, purpose or welfare, or for the benefit of the poor and the landless; c. There is payment of just compensation as required under Sec. 9, Article III of the Constitution, and other pertinent laws; d. A valid and definite ofer has been previously made to the owner of the property sought to be expropriated, but said ofer was not accepted. Limitations on power of eminent domain of LGU: (Estate of JBL Reyes vs. City of Manila) Since local governments possess only delegated power of eminent domain, it is subject to limitations specified by law on the delegated power. Thus there are mandatory limits with respect to: 1. The order of priority in acquiring land for socialized housing; and 2. The resort to expropriation proceedings as a means to acquiring it. Private lands rank last in the order of priority for purposes of socialized housing. In the same vein, expropriation proceedings may not be resorted to only after the other modes of acquisition are exhausted. Compliance with these conditions are mandatory because these are the only safeguards of often time helpless owners of private property against what may be tyrannical violation of due process when their property is forcibly taken from them allegedly for public use. TAU MU

CASES ON EMINENT DOMAIN PPI vs. COMELEC 244 SCRA 272 (1995) Facts: The Commission on Elections passed a resolution authorizing itself to procure free space in newspapers which it shall allocate among candidates or for dissemination of information during the 1995 national election. Thereafter, Commissioner Maambong wrote to newspapers letters directing them to provide free space of no less that one-half page form March 6 to May 6, 1995. PPI contends that the resolution and the letters constitute unlawful taking of private property. Held: The letters form the constitutional body have coercive efect as to compel the addressee to donate space. As such they constitute unlawful taking of private property for public use. First, there appears to be no necessity for the taking as it was not shown that the newspapers are not willing to sell space at the usual rate. Second, the taking is without compensation. Third, COMELEC is not empowered under the Constitution or under any legislation to take over private property. TELECOMMUNICATIONS vs. COMELEC 289 SCRA 337 (1998)

ACADCOM 2010; Contributors: Gene Geocaniga, Jarissa Guiani, Darlene Magabilen TAU MU Page 11 of 179

TAU MU TAU MU TAU MU TAU MU TAU MU TAU MU TAU MU TAU MU TAU MU

Facts: Petition for review of a decision of the CA. The SB of the Municipality of Bunawan passed a resolution authorizing the municipal mayor to initiate the petition for expropriation of a parcel of land, belonging to the petitioners. The Sangguniang Panlalawigan disapproved said resolution and returned it with the comment that expropriation is unnecessary considering that there are still other lots available. Issue: W/N a municipality may expropriate private property by virtue of a municipal resolution which was disapproved by the SP Held: The Court finds no merit in the petition and affirms the decision of the CA. Eminent domain is a fundamental State power that is inseparable from sovereignty. It is the government’s right to appropriate in the nature of a compulsory sale to the State, private property for public use or purpose. Inherently possessed by the national legislature, the power of eminent domain may be validly delegated to local governments, other public entities and public utilities. For the taking of private property by the government to be valid, the taking must be for public use and there must be just compensation. The SP’s disapproval does not render said resolution null and void. Their power to declare a municipal resolution invalid is on the sole ground that it is beyond the power of the SB or the mayor to issue. Said

TAU MU TAU MU

MODAY vs. CA 268 SCRA 586 (February 20, 1997)

The Fraternal Ateneo de Davao

TAU MU TAU MU

Facts: GMA Network, Inc., is challenging Sec. 52 of BP Blg. 881, which provides: The Commission shall procure radio and television time to be known as “Comelec Time” which shall be allocated equally and impartially among candidates… For this purpose, the franchise of all radio broadcasting and television stations are hereby amended so as to provide radio or television time, free of charge, during the election period. Petitioner claims that this is a form of taking or property without compensation. TAU MU Held: The argument has no merit. All broadcasting, whether by radio or television stations, is licensed by the government. Airwave frequencies have to be allocated as there are more individuals who want to broadcast than there are frequencies assign. A franchise is thus a privilege subject, among other things, to amendment by Congress in accordance with the Constitution if the common good so requires. In truth, radio and television broadcasting companies, which are given franchises, do not own the airwaves and frequencies though which they transmit broadcast signal and images. “Airtime” is not a finished product which become the property of the company, like oil produced form refining. Thus, no private property is taken. As the government spends public funds in granting broadcasting companies the privilege and in supervising them, it would be strange if they cannot be required to render public service by giving free airtime.

KITY

TAU MU TAU MU TAU MU TAU MU TAU MU TAU MU

CONSTITUTIONAL LAW II Order of Saint Thomas More Atty. Philip John Pojas/Atty. Rovyne G. Jumao-as, RN University College of Law

resolution is valid and binding and could be used as lawful authority to petition for the condemnation of petitioner’s property. The limitations on the power of eminent domain are that the use must be public, compensation must be made and due process of law must be observed. The necessity of exercising eminent domain must be genuine and of a public character. MUNICIPALITY vs. VM REALTY CORP. 292 SCRA 678 (July 20, 1998) Facts: Municipality of Parañaque filed a complaint for expropriation against respondent VM Realty over two parcels of land. The complaint was filed “for the purpose of alleviating the living conditions of the underprivileged by providing homes for the homeless through a socialized housing project.” Parenthetically, it was also for this stated purpose that petitioner previously made an ofer to enter into a negotiated sale of the property with private respondent, which the latter did not accept. The trial dismissed the case. CA affirmed the said decision. Issue: W/N the resolution of the Parañaque Municipal Council is a substantial compliance of the statutory requirement of Sec. 19, RA 7180 in the exercise of the power of eminent domain by the plaintif-appellant. TAU MU Held: The power of eminent domain is lodged in the legislative district branch of the government, which may delegate the exercise thereof to LGUs, other public entities and public utilities. An LGU may therefore exercise the power to expropriate private property only when authorized by Congress and subject to the latter’s control and restraints, imposed “through the law conferring the power or in other legislations.” In this case, Sec. 19 of RA 7160, which delegates to LGUs the power of eminent domain , also lays down the paramenters for its exercise. Thus, the following essential requisites must concur before an LGU may exercise the power of eminent domain: 1. An ordinance is enacted by the local legislative council authorizing the local chief executive, in behalf of the LGU, to exercise the power of eminent domain or pursue expropriation proceedings over a particular private property; 2. The power of eminent domain is exercised for public use, purpose or welfare, or for the benefit of the poor and the landless; 3. There is payment of just compensation, as required under Sec. 9, Art. III of the Constitution and other pertinent laws; 4. A valid and definite ofer has been previously made to the owner of the property sought to be expropriated, but said ofer was not accepted. The local chief executive sought to exercise the power of eminent domain pursuant to a resolution of the municipal council. Thus, there was no compliance with the first requisite that the mayor be authorized through an ordinance. A municipal ordinance is diferent from a resolution. An ordinance is a law, but a resolution is merely a

ACADCOM 2010; Contributors: Gene Geocaniga, Jarissa Guiani, Darlene Magabilen TAU MU Page 12 of 179

TAU MU TAU MU TAU MU TAU MU TAU MU TAU MU TAU MU

Facts: In 1947, The PAF leased the property of Castellvi renewable year to year. The lease was renewed yearly until 1956, Castellvi refused to renew the contract with PAF any longer. In 1959, the government instituted an action to expropriate the property previously leased. Also, in the same year, the property was placed under the possession of the government by virtue of a court order. TAU MU Issue: What are being disputed in this case are two issues: 1. Castellvi and the government cannot agree on the price Government – P 0.20 per sq.meter Castellvi – P15.00 per sq.meter 2. They do not also agree on the date which will be the basis for the determination of the value of the land. Governement – value in 1947 Castellvi – value in 1959 Held: The court held that the value should be determined at the time of the taking. The taking here would mean 1959, where all the elements of taking were present. It cannot be 1947 since it did not satisfy the 2nd and 5th elements.

TAU MU

REPUBLIC vs. CASTELLVI 58 SCRA 336 (1974)

TAU MU TAU MU

TAKING

The Fraternal Ateneo de Davao

TAU MU TAU MU

declaration of the sentiment or opinion of a lawmaking body on a specific matter. The power of eminent domain necessarily involves a derogation of a fundamental or private right of the people. An LGU, like the municipality of Parañaque, cannot authorize an expropriation of private property through a mere resolution of its lawmaking body. The LGC expressly and clearly requires an ordinance of a local law for that purpose. A resolution that merely expressed the sentiment or opinion of the Municipal Council will not suffice. The Court holds that the principle of res judicata, which finds application in generally all cases and proceedings cannot bar the right of the State or its agent to expropriate private property. The very nature of eminent domain, as an inherent power of the State, dictates that the right to exercise the power be absolute and unfettered even by a prior judgment or res judicata. The scope of eminent domain is plenary and, like police power, can “reach every from of property which the State might need for public use. While the principle of res judicata does not denigrate the right of the State to exercise eminent domain, it does not apply to specific issues decided in a previous case. The State or its authorized agent may still subsequently exercise its right to expropriate the same property, once all legal requirements are complied with. To rule otherwise will not only improperly diminish the power of eminent domain, but also clearly defeat social justice.

KITY

TAU MU TAU MU TAU MU TAU MU TAU MU TAU MU

CONSTITUTIONAL LAW II Order of Saint Thomas More Atty. Philip John Pojas/Atty. Rovyne G. Jumao-as, RN University College of Law

NPC vs. GUTTIEREZ 193 SCRA 1 (1991) Facts: NPC erected transmission lines passing through the property of Gutierrez. As a consequence, NPC asked for a 3-meter right of way. In this portion, NPC prohibited Gutierrez from planting anything above 3 meters. According to NPC, they will only pay the rental of the right for way. Gutierrez argued that there is actually impairment of ordinary usage of his property. Therefore, he should be paid for the full value of the property traversed by the line. Issue 1: Whether there was taking under the concept of eminent domain here Issue 2: Should there be a transfer of title Held 1: The court held that there is taking as to the sense that there is impairment of ordinary use, so that the owner is entitled to full compensation for the portion afected. As the property is injuriously afected by an easement of right of way, there is expropriation. Held 2: NPC argued that since they will pay for the whole property afected, then ownership of that property should be transferred to them. The title remains with Gutierrez but NPC should still pay the full amount for the property afected because there was impairment of usage. REPUBLIC vs. PLDT 26 SCRA 620 (1969)

TAU MU

Facts: Republic, through the Bureau of Telecommunications, rented the facilities of PLDT for the use of the government and its offices. Thereafter, the Bureau extended its services to the public. PLDT filed a complaint alleging that said bureau was violating the conditions under which their Private Branch Exchange (PBX) is inter-connected with the PLDT's facilities, referring to the rented trunk lines, for the Bureau had used the trunk lines not only for the use of government offices but even to serve private persons or the general public, in competition with the business of the PLDT, and thereafter disconnected the services of the Bureau. Republic then filed a suit praying in its complaint for judgment commanding the PLDT to execute a contract with plaintif, through the Bureau, for the use of the facilities of defendant's telephone system throughout the Philippines under such terms and conditions as the court might consider reasonable, and for a writ of preliminary injunction against the defendant company to restrain the severance of the existing telephone connections and/or restore those severed. TAU MU Issue: W/N the Republic may compel PLDT to execute a contract to use of its facilities Held: While the Republic may not compel the PLDT to celebrate a contract with it, the Republic may, in the exercise of the sovereign power of eminent domain, require the telephone company to permit interconnection of the government telephone system and that of the PLDT, as the needs of the government service may require, subject to the payment of just compensation to be determined by the court. Nominally, of course,

ACADCOM 2010; Contributors: Gene Geocaniga, Jarissa Guiani, Darlene Magabilen TAU MU Page 13 of 179

TAU MU TAU MU TAU MU TAU MU TAU MU TAU MU TAU MU TAU MU TAU MU

Facts: In 1978, the National Power Corporation (NPC) took possession of private respondent’s land under the mistake in belief that it forms part of the public land reserved for hydroelectric power purposes, and it paid financial assistance to he the City of Marawi. In 1979 private respondent raised his claim of ownership, but it was only in 1990, after more than ten years of beneficial use, when NPC acceded to the fact that the property belonged to respondent. In 1992, NPC instituted expropriation proceedings. At what time should the value of the land be computed, at the “taking” or the date of the filing of the complaint for eminent domain? Held: The general rule is that in determining “just compensation” the value of the property at the time of the filing of the complaint is controlling. Normally, the time of taking coincides with the filing of the complaint. However, where the taking precedes the institution of the case, the time of taking is critical in determining just compensation. In this case, however, while there was entrance by NPC into the property in 1978, taking was not really made at that time. Taking has the following elements: (1) the expropriator must enter a private property; (2) the entrance must be for more than a momentary period; (3) the entry into the property should be under warrant or color of legal authority; (4) the property must be devoted to public use or otherwise injuriously afected; and (5) it must oust owner and deprive him of all beneficial enjoyment of the property. In this case, NPC’s entrance in 1978 was without intent to expropriate or was not made under warrant or color of legal authority, for it believed that the property was public land. Thus, just compensation should be the value at the time of the filing of the complaint in 1992.

TAU MU TAU MU

NPC vs. CA 254 SCRA 577 (1996)

The Fraternal Ateneo de Davao

TAU MU TAU MU

the power of eminent domain results in the taking or appropriation of title to, and possession of, the expropriated property; but no cogent reason appears why the said power may not be availed of to impose only a burden upon the owner of condemned property, without loss of title and possession. It is unquestionable that real property may, through expropriation, be subjected to an easement of right of way. The use of the PLDT's lines and services to allow inter-service connection between both telephone systems is not much diferent. In either case private property is subjected to a burden for public use and benefit. If, under the Constitution, the State may, in the interest of national welfare, transfer utilities to public ownership upon payment of just compensation, there is no reason why the State may not require a public utility to render services in the general interest, provided just compensation is paid therefor. Ultimately, the beneficiary of the interconnecting service would be the users of both telephone systems, so that the condemnation would be for public use.

KITY

TAU MU TAU MU TAU MU TAU MU TAU MU TAU MU

CONSTITUTIONAL LAW II Order of Saint Thomas More Atty. Philip John Pojas/Atty. Rovyne G. Jumao-as, RN University College of Law

MARINE RADIO vs. REYES 191 SCRA 205 (November 6, 1990) Facts: Petitioners are self-described “Filipino entrepreneurs deeply involved in the business of marine radio communications in the country.” They are also operators of shore-to-ship and shipto-shore public marine coastal radio stations, and are holders of certificates of public convenience duly issued by the NTC. Among other things, they handle correspondence between vessel passengers or crew and the public. DOTC unveiled a maritime coastal communications system project, designed to ensure safety of lives at sea. It was set out to provide, among other things, ship-to-shore and shore-to-ship public corresponding, free of charge. Sec. Reyes replied that MARCAPI’s main business concern is public correspondence, which is only fourth in the order of priority of services to be ofered by the present maritime project. Primarily, it will ofer distress and safety communications service which is obligatory in the maritime mobile service. The petitioners brought the instant suit, alleging in essence, that Secretary Rainerio Reyes had been guilty of grave abuse of discretion. TAU MU Issue: W/N the act complained of is equivalent to taking without just compensation. Held: There is no merit in the petition. There can hardly be any valid argument against providing for public corresponding, free of charge. It is compatible with State aims to serve the people under the Constitution, and certainly, amid these hard times, the State can do no less. The Court is not of the thinking that the act complained of is equivalent to a taking without just compensation. It does not seem that the DOTC, by providing for free public correspondence, is guilty of an uncompensated taking. Rather, the Government merely built a bridge that made the boat obsolete, although not entirely useless. Certainly, the owner of the boat cannot charge the builder of the bridge for lost income. And certainly, the Government has all the right to build the bridge. DIDIPIO EARTH SAVERS’ vs. GUZON 485 SCRA 586 (2006) Facts: Sec. 76 of RA 7942 otherwise known as the Philippine Mining Act of 1995, provides: Entry into private lands and concession areas – Subject to prior notification, holders of mining rights shall not be prevented from entry into private lands and concession areas by surface owners, occupants, or concessionaires when conducting mining operations therein. Issue:Is the provision a form of taking which is subject to just compensation? TAU MU Held: Yes. The entry referred to in Sec. 76 is not just a simple right-of-way which is ordinarily allowed under the provisions of the Civil Code. Here, the holders of mining rights enter private lands for purposes of conducting mining activities such as exploration, extraction and processing of minerals. Mining right holders build mine infrastructure, dig mine shafts and connecting

ACADCOM 2010; Contributors: Gene Geocaniga, Jarissa Guiani, Darlene Magabilen TAU MU Page 14 of 179

tunnels, prepare tailing ponds, storage areas and vehicle depots, install their machinery, equipment and server systems. On top of this, under Sec 75, easement rights are accorded to them where they may build warehouses, port facilities, electric transmission, railroads and other infrastructures necessary for mining operations. All these will definitely oust the owners or occupants of the afected areas the beneficial ownership of their lands. Without a doubt taking occurs once mining operations commence.

TAU MU TAU MU TAU MU TAU MU TAU MU TAU MU TAU MU TAU MU TAU MU TAU MU TAU MU

Facts: In 1978, NAPOCOR, without the landowners’ knowledge and prior consent, took possession of the sub-terrain area of their lands and constructed therein underground tunnels. When discovered by the owners in 1992, they asked for compensation. NAPOCOR denied arguing that: 1. The owners’ rights extend to the subsoil only insofar as necessary for their practical interests, and 2. The tunnels merely constitute and easement upon their property, which does not involve any loss or title or possession. Hence, they were not denied the beneficial use of their subject properties as to entitle them to just compensation by way of damages. Issue: Was there taking as to entitle the owners to compensation? Held: Yes, there was taking. The ownership of land extends to the surface as well as to the subsoil under it. First, the argument that the landowners’ right extends to the subsoil insofar as necessary for their practical interests serves only to further weaken the NPC’s case. The theory would limit the right to the subsoil upon the economic utility which such area ofers to the surface owners. Presumably, the landowners’ right extends to such height or depth where it is possible for them to obtain some benefit or enjoyment, and it is extinguished beyond such limit as there would be no more interest protected by law. The owners could have dug upon their property motorized deep wells but were prevented from doing so by NPC precisely because of the construction and existence of the tunnels underneath the surface of their property. They, therefore, still had a legal interest in the subterrain portion insofar as they could have excavated the same for the construction of the deep well. Second, the manner in which the easement was created by NPC, however, violates the due process rights of the owners as it was, without notice and indemnity to them and did not go through proper expropriation proceedings. NPC could have, at any time, validly exercised the power of eminent domain to acquire the easement over the property as this power encompasses not only the taking or appropriation

The Fraternal Ateneo de Davao

TAU MU TAU MU

NPC vs. IBRAHIM 526 SCRA 149 (2007)

KITY

TAU MU TAU MU TAU MU TAU MU TAU MU TAU MU

CONSTITUTIONAL LAW II Order of Saint Thomas More Atty. Philip John Pojas/Atty. Rovyne G. Jumao-as, RN University College of Law

of title to and possession of the expropriated property but likewise covers even the imposition of a mere burden upon the owner of the condemned property. The underground tunnels impose limitations on the use of the property for an indefinite period and deprives them of its ordinary use. Based upon the foregoing, the owners are clearly entitled to the payment of just compensation. TAU MU

REPUBLIC vs. ANDAYA 524 SCRA 671 (2007) Facts: Ismael Andaya is the owner of two parcels of land in Butuan City, which were subject to a 60-meter wide perpetual easement for public highways, irrigation ditches, aqueducts, and other similar works of the government or public enterprise. Out of the 10,380 sq. m. property, 701 sq. m. was subject of the easement. During the expropriation proceedings, Andaya demanded that the consequential damages he must be paid should be passed on the remaining area of 9,679 sq. m. which is the entire property minus the easement. Andaya alleged that the easement would render his entire property unusable and unhabitable. Is he correct? Held: Yes. Taking, in the exercise of the power of eminent domain, occurs not only when the government actually deprives or dispossesses the property owner of his property or of its ordinary use, but also when there is a practical destruction or material impairment of the value of his property. Using this standard, there was undoubtedly a taking of the remaining area of Andaya’s property. True, no burden was imposed thereon and Andaya still retained title and possession of the property. But the nature and the efect of the floodwalls would deprive Andaya of the normal use of the remaining areas. It would prevent ingress and egress to the property and turn it into a catch basin for the floodwaters coming from the Agusan River. Hence, Andaya is entitled to payment of just compensation, which must neither more nor less than the monetary equivalent of the land. TAU MU CARLOS SUPERDRUG vs. DSWD 526 SCRA 130 (2007) Facts: Carlos Superdrug questioned the constitutionality of the Expanded Senior Citizens Act of 2003, which grants to senior citizens 20% discount on their purchase of medicines. According to CS, the law is confiscatory because it compels drugstore owners to grant the discount and will result in a loss of profit and capital since it tailed to provide a scheme whereby drugstores will be justly compensated for the discount. Issue: Was there a valid taking? Held: Yes. The Senior Citizens Act was enacted primarily to maximize the contribution of senior citizens to nation-building, and to grant benefits and privileges to them for their improvement and

ACADCOM 2010; Contributors: Gene Geocaniga, Jarissa Guiani, Darlene Magabilen TAU MU Page 15 of 179

well-being as the State considers them an integral part of our society. The law is a legitimate exercise of police power which, similar to the power of eminent domain, has general welfare for its object. Police power is not capable of an exact definition but has been purposely veiled in general terms to underscore its comprehensiveness to meet all exigencies and provide enough room for an efficient and flexible response to conditions and circumstances, thus assuring the greatest benefits. PUBLIC USE

TAU MU TAU MU TAU MU TAU MU TAU MU TAU MU TAU MU TAU MU

Facts: The President of the Philippines issued a Letter of Instruction instituting an nationwide slum improvement and resettlement program and to adopt slum improvement as a national housing policy. The Metro Manila Zonal Improvement Program included the properties known as the Tambunting Estate and the Sunog-Apog area in its priority list for a zonal improvement program. The President also designated the NHA to negotiate with the owners of the property for the acquisition of the same. This, however, did not materialize as the negotiations for the purchase of the property failed. The President issued Proc. No. 1810 declaring all sites identified by the Metro Manila local governments and approved by the Ministry of Human Settlements to be included in the ZIP upon proclamation of the President. The Tambunting Estate and the Sunog-Apog area were among the sites included. Issue: W/N the government had the right to expropriate the said areas. TAU MU Held: The power of eminent domain is inherent in every state and the provisions in the Constitution pertaining to such power only serve to limit its exercise in order to protect the individual against whose property the power is sought to be enforced. The State is not subject to

TAU MU

MANOTOK vs. CA 150 SCRA 87 (May 21, 1987)

TAU MU TAU MU

Facts: The NHA wanted to use Sumulong’s property for socialized housing for the lower and middle class. The owner contended that socialized housing is not public use because not everyone can benefit from this, only the handful of people who to be given the houses. TAU MU Held: The court held that the socialized housing is within the context of public use. Public use has acquired a more comprehensive meaning. That is whatever would result to indirect public benefit or welfare is also public use. It also ruled that it will benefit everyone in the sense that it will afect the safety, health and environment. Providing housing to these people will help in lessening the incidence of violence and problems concerning health. In the end, it will benefit everybody in a way. In short, socialize housing falls within the meaning of public use.

The Fraternal Ateneo de Davao

TAU MU TAU MU

SUMULONG vs. GUERRERRO 154 SCRA 461 (1987)

KITY

TAU MU TAU MU TAU MU TAU MU TAU MU TAU MU

CONSTITUTIONAL LAW II Order of Saint Thomas More Atty. Philip John Pojas/Atty. Rovyne G. Jumao-as, RN University College of Law

any limitation other than those imposed by the Constitution which are: First, the taking must be for a public use; Second, the payment of just compensation must be made and; Third, due process must be observed in the taking. The due process clause cannot be rendered nugatory everytime a specific decree or law orders the expropriation of somebody’s property and provides its own peculiar manner of taking the same. Neither should the courts adopt a hands-of policy just because the public use has been ordained as existing by the decree of the just compensation has been fiexed and determined beforehand by a statute. Although due process does not always necessarily demand that a proceeding be had before a court of law, it still mandates some form of proceeding wherein notice and reasonable opportunity to be heard are given to the owner to protect his property rights. There are exceptional situations when, in the exercise of the power of eminent domain, the requirement of due process may not necessarily entail judicial process. But where it is alleged that in the taking of a person’s property, his right to due process of law has been violated, the courts will have to step in and probe into such an alleged violation. A necessity must exist for the taking of private property for the proposed uses and purposes but accepted the fact that modern decisions do not call for absolute necessity. It is enough if the condemnor can show a reasonable or practical necessity, which of course, varies with the time and peculiar circumstances of each case. In the instant petitions, there is no showing whatsoever as to why the properties involved were singled out for expropriation through decrees or what necessity impelled the particular choices or selections. The provision of P.D. 1669 which allows NHA, at its sole option, to put portions of the expropriated area to commercial use in order to defray the development costs of its housing projects cannot stand constitutional scrutiny. P.D. Nos. 1669 and 1670 is violative of the petitioners' right to due process of law and, therefore, they must fail the test of constitutionality. The decrees, do not by themselves, provide for any form of hearing or procedure by which the petitioners can question the propriety of the expropriation of their properties or the reasonableness of the just compensation. The fixing of the maximum amounts of compensation and the bases thereof which are the assessed values of the properties in 1978 deprive the petitioner of the opportunity to prove a higher value because, the actual or symbolic taking of such properties occurred only in 1980 when the questioned decrees were promulgated. The decision of the government to acquire a property through eminent domain should be made known to the property owner through a formal notice wherein a hearing or a judicial proceeding is contemplated. This shall be the time of reckoning the value of the property for the purpose of just compensation. The questioned decrees transgress the petitioners'

ACADCOM 2010; Contributors: Gene Geocaniga, Jarissa Guiani, Darlene Magabilen TAU MU Page 16 of 179

right to just compensation. Having violated the due process and just compensation guarantees, P. D. Nos. 1669 and 1670 are unconstitutional and void. ESTATE vs. PHILIPPINE EXPORT 349 SCRA 240 (2000)

TAU MU TAU MU TAU MU TAU MU TAU MU TAU MU TAU MU TAU MU TAU MU

Facts: In 1977, the National Housing Authority (NHA) field complaints to expropriate sugarcane lands in Cavite. The purpose of the expropriation was the expansion of the Damariñas Resettlement Project to accommodate the squatters who were relocated from the Metropolitan Manila Area. The trial court rendered judgment ordering the expropriation, after payment of just compensation, of the subject lots which decision was affirmed by the SC in 1987. In 1992, the previous owners of the expropriated properties filed a case against NHA for their return on the ground that NHA deviated from the public purpose of the taking when it entered into a contract for the construction of low cost housing units to be sold to low-income beneficiaries. TAU MU Held: The act of NHA of entering into a contract with a real estate developer for the construction of low cost housing on the expropriated lots cannot be taken to mean as a deviation from the stated public purpose of their taking. Jurisprudence has it that the expropriation of private land for slum clearance and urban

TAU MU TAU MU

REYES vs. NHA 395 SCRA 495 (2003)

The Fraternal Ateneo de Davao

TAU MU TAU MU

Facts: Philippine Export Processing Zone (PEZA) initiated before the RTC expropriation proceedings on 3 parcels of irrigated rice land. The power of eminent domain of PEZA is contained in its original charter, PD No. 66, which allows it to expropriate any property for export processing zones, low-cost housing, or for the construction of dams, terminal facilities, structures and approaches thereto. Considering that portions of the expropriated property would be leased to banks, is it still for a “public purpose?” Held: Yes. The term “public use” has acquired a more comprehensive coverage. To the literal import of the term signifying strict use by the public has been added the broader notion of indirect public benefit or advantage. It should be pointed out that PEZA was created to be a viable commercial, industrial and investment area. The expropriation of the lot for the purpose of being leased to banks and for the construction of a terminal has the purpose of making banking and transportation facilities easily accessible to persons working at industries located at PEZA. It therefore comes as a matter of necessity to bring life to the purpose of the law, as reaffirmed by RA No. 9716. Moreover, Congress can determine the necessity of expropriating private property. In such a case, it is a question exclusively for the legislature to decide.

KITY

TAU MU TAU MU TAU MU TAU MU TAU MU TAU MU

CONSTITUTIONAL LAW II Order of Saint Thomas More Atty. Philip John Pojas/Atty. Rovyne G. Jumao-as, RN University College of Law

development is for a public purpose even if the developed area is later sold to homeowners, commercial firms, service companies and other private concerns. Moreover, the Constitution itself allows the State to undertake, for the common good, and in cooperation with the private sector, a continuing program of urban land reform and housing which will make at afordable cost decent housing to homeless citizens. It follows that the low cost housing of the NHA on the expropriated lots is consistent with the public use requirement. MANOSCA vs. CA 252 SCRA 412 (1996) Facts: Petitioners own a piece of land consisting of 492 square meters, which was later ascertained by the National Historical Institute to be the birth site of Felix Y. Manalo, the founder of the Iglesia Ni Cristo. Consequently, it was declared a national historical landmark, and the Republic instituted expropriation proceedings. Issue: Would the expropriation constitute an application of funds for the use, benefit or support of a religious entity? Held: The attempt to give some religious perspective to the case deserves little consideration, for what should be significant is the principal objective of, not the casual consequences that might follow from, the exercise of power. The purpose in setting up a marker is essentially to recognize the distinctive contribution of the late Felix Manalo to the culture of the Philippines, rather than to commemorate his founding and leadership of the Iglesia Ni Cristo. The practical reality that the greater benefit may be derived by its members than by most others could well be true but such a peculiar advantage still remains to be merely incidental and secondary in nature. Indeed that only a few would actually benefit from the expropriation of the property does not necessarily diminish the essence and character of public use. HEIRS OF MORENO vs. MACTAN CEBU 413 SCRA 502 (2003) Facts: In 1949, the National Airport Corporation as the predecessor agency of respondent MactanCebu International Airport Authority (MCIAA) wanted to acquire Lot Nos. 916 and 920 among other parcels of land for the proposed expansion of Lahug Airport. To entice the landowners to cede their properties, the government assured them that they could repurchase their lands once Lahug Airport was closed or its operations transferred to Mactan Airport. TAU MU At the end of 1991, Lahug Airport ceased operations as the Mactan Airport was opened for incoming and outgoing flights. Lot Nos. 916 and 920 which had been expropriated for the extension of Lahug Airport were not utilized. Hence, petitioners filed a complaint for reconveyance and damages to compel the repurchase of Lot Nos. 916 and 920.

ACADCOM 2010; Contributors: Gene Geocaniga, Jarissa Guiani, Darlene Magabilen TAU MU Page 17 of 179

The Fraternal Ateneo de Davao

TAU MU TAU MU TAU MU TAU MU TAU MU TAU MU TAU MU TAU MU TAU MU TAU MU TAU MU TAU MU TAU MU

Petitioners argue that Fery vs. Municipality of Cabanatuan does not apply to the case at bar since what was involved therein was the “right of reversion” and not the “right of repurchase” which they are invoking. They also diferentiate Mactan-Cebu International Airport Authority vs. CA from the instant case in that the landowners in the MCIAA case ofered inadmissible evidence to show their entitlement to a right of repurchase, while petitioners herein ofered evidence based on personal knowledge for which reason MCIAA did not object and thus waived whatever objection it might have had to the admissibility thereof. Finally, petitioners allege that their right to equal protection of the laws would be infringed if some landowners are given the right to repurchase their former properties even as they are denied the exercise of such prerogative. Held: in Fery, we declared that the government acquires only such rights in the expropriated parcels of land as may be allowed by the character of its title over the properties: “If x x x land is expropriated for a particular purpose, with the condition that when that purpose is ended or abandoned, the property shall return to its former owner, then or course, when the purpose is terminated or abandoned the former owner reacquires the property so expropriated. If x x x land is expropriated for a public street and the expropriation is granted upon condition that the city can only use it for a public street, it returns to the former owner, unless there is a statutory provision to the contrary x x x if, upon the contrary, however, the decree if expropriation gives to the entity a fee simple title, then, of course, the land becomes absolute property of the expropriator, whether it be a State, province, or municipality, and in that case the non-use does not have the effect of defeating the title acquired by the expropriation proceedings x x x” As for the public purpose of the expropriation proceeding, it cannot now be doubted. Although Mactan Airport is being constructed, it does not take away the actual usefulness and importance of the Lahug Airport: it is handling the air traffic both civilian and military. From it aircrafts fly to Mindanao and Visayas and past through it or their flights to the North and Manila. Then, no evidence was adduced to show how soon is the Mactan Airport to be placed in operation and whether the Lahug Airport will be closed immediately thereafter. It is up to the other departments of the Government to determine said matters. The Court cannot substitute its judgment for those of the said departments or agencies. In the absence of such showing, the Court will presume that the Lahug Airport will continue to be in operation. The predicament of petitioners involves a constructive trust, one that is akin to the implied trust referred to in Art. 1454 of the NCC. “If an absolute conveyance of property is made in order to secure the performance of an obligation of the grantor toward the grantee, a trust by virtue of law is established. If the fulfillment of the obligation is offered by the grantor when it becomes due, he may demand the reconveyance of the property to him.”

KITY

TAU MU TAU MU TAU MU TAU MU TAU MU TAU MU

CONSTITUTIONAL LAW II Order of Saint Thomas More Atty. Philip John Pojas/Atty. Rovyne G. Jumao-as, RN University College of Law

In the case at bar, petitioners conveyed Lot Nos. 916 and 920 to the government with the latter obliging itself to use the realties for the expansion of Lahug Airport; toiling to keep its bargain, the government can be compelled by petitioners to reconvey the parcels of land to them, otherwise, petitioners would be denied the use of their properties upon a state of afairs that was not conceived nor contemplated when the expropriation was authorized. TAU MU Hence, respondent MCIAA as representative of the State is obliged to reconvey Lot Nos. 916 and 920 to petitioners who shall hold the same subject to existing liens thereon, ie leasehold right of DPWH. HEIRS OF MORENO vs. MACTAN-CEBU 466 SCRA 288 (2005) Facts: On MFR by Mactan International Airport Authority, it was argued that the decision of the Court which granted the owner’s right of repurchase efectively overturns the ruling in Fery, which requires that for an expropriation judgment to be conditional, it must clearly spell out said condition. Held: Nothing in the Fery case bespeaks that there should be an express condition in the dispositive portion of the decision before the condemned property can be returned to its former owner after the purpose for its taking has been abandoned or ended. The indisputable certainty in the present case is that there was a prior promise by the predecessor of Mactan that the expropriated properties may be recovered by the former owners since the airport is transferred to Mactan, Cebu. In fact, the witness for Mactan testified that 15 lots were already reconveyed to their previous owners. MASIKIP vs. CITY OF PASIG 479 SCRA (2006) Facts: The City of Pasig, notified Masikip of its intention to expropriate a portion of her property to be used for the sports development and recreational activities. She replied that the intended expropriation of her property is unconstitutional, invalid, and oppressive, as the area of her lot is neither sufficient nor suitable to “provide land opportunities to deserving poor sectors of our community.” She contended that the City failed to establish a genuine necessity which justifies the condemnation of her property. While she does not dispute the intended public purpose, nonetheless, she insists that there must be a genuine necessity for the proposed use and purposes. Held: LGUs have no inherent power of eminent domain and may exercise it only when expressly authorized by statute. Section 19 of the LGC prescribes the delegation by Congress of the power of eminent domain to LGUs and lays down the parameters for its exercise. “SEC. 19. Eminent Domain. - A local government unit may, through its chief executive and acting

ACADCOM 2010; Contributors: Gene Geocaniga, Jarissa Guiani, Darlene Magabilen TAU MU Page 18 of 179

TAU MU TAU MU TAU MU TAU MU TAU MU TAU MU TAU MU TAU MU

Facts: EPZA argues that the commissioner’s report should no longer be set for hearing because under PD Nos. 76, 464, 794 and 1533, the basis for just compensation shall be the fair and current market value declared by the owner of the property sought to be expropriated or such market value as determined by the assessor, whichever is lower. Hence, there is no more need to appoint commissioners as prescribed by Rule 67 of the Rules of Court and for them to consider other highly variable factors in order to determine just compensation. Issue: Are PD Nos. 76, 464, 794 and 1533 valid and constitutional, such that in determining the just compensation of property in an expropriation case, the only basis should be its market value as declared by the owner or as determined by the assessor, whichever is lower? Held: No. the method of ascertaining just compensation under the afore-cited decrees constitutes impermissible encroachment on judicial prerogatives. It tends to render this Court

TAU MU

EPZA vs. DULAY 149 SCRA 305 (1987)

TAU MU TAU MU

JUST COMPENSATION

The Fraternal Ateneo de Davao

TAU MU TAU MU

pursuant to an ordinance, exercise the power of eminent domain for public use, or purpose, or welfare for the benefit of the poor and the landless, upon payment of just compensation, pursuant to the provisions of the Constitution and pertinent laws: Provided, however, That the power of eminent domain may not be exercised unless a valid and definite offer has been previously made to the owner, and such offer was not accepted: Provided, further, That the local government unit may immediately take possession of the property upon the filing of the expropriation proceedings and upon making a deposit with the proper court of at least fifteen percent (15%) of the fair market value of the property based on the current tax declaration of the property to be expropriated: Provided, finally, That, the amount to be paid for the expropriated property shall be determined by the proper court, based on the fair market value at the time of the taking of the property.” According to Masikip, there is already an established sports development and recreational activity center. The City does not dispute this. Evidently, there is no “genuine necessity” to justify the expropriation. Where the taking by the State of private property is done for the benefit of a small community which seeks to have its own sports and recreational facility, notwithstanding that there is such a recreational facility only a short distance away, such taking cannot be considered to be for public use. Its expropriation is not valid. The right to take private property for public purposes necessarily originates from “the necessity” and the taking must be limited to such necessity. TAU MU

KITY

TAU MU TAU MU TAU MU TAU MU TAU MU TAU MU

CONSTITUTIONAL LAW II Order of Saint Thomas More Atty. Philip John Pojas/Atty. Rovyne G. Jumao-as, RN University College of Law

inutile in a matter which under the Constitution is reserved to it for final determination. Thus, although in an expropriation proceeding the court technically would still have the power to determine the just compensation for the property, following the applicable decrees, its task would be relegated to simply stating the lower value of the property as declared either by the owner or the assessor. As a necessary consequence, it would be useless for the court to appoint commissioners under Rule 67 of the ROC. The strict application of the decrees during the proceedings would be nothing short of a mere formality or charade as the court has only to choose between the valuation of the owner and that of the assessor, and its choice is always limited to the lower of the two. The court cannot exercise its discretion or independence in determining what is just or fair. Even a grade school pupil could substitute for the judge insofar as the determination of constitutional just compensation is concerned. SANTOS vs. LAND BANK 340 SCRA 59 (2000) Facts: An RTC ordered Land Bank to pay P49M to Santos for the taking of some 77 hectares of agricultural land. It was also ordered that the compensation shall be in the manner provided under the CARL, which allows compensation in cash and in bonds. Issue: Is the mode of payment under the CARL constitutional? Held: Yes. Traditionally, the medium of payment if just compensation is money and no other. However, in land reform, we do not deal with the traditional exercise of the power of eminent domain. TAU MU This is a revolutionary kind of expropriation that does not afect only a specific property or a limited area. Accepting the theory that payment of just compensation need not always be made fully in money, we find that the proportion of cash payment is not unduly oppressive. No less important, the government financial instruments in making up the balance of the payment are negotiable at any time. ROBERN vs. QUITAIN 315 SCRA 150 (1999) Facts: Robern Development Corporation owns parcels of land in Davao City intended for a lowcost housing project. The National Power Corporation filed a complaint to expropriate them. Upon deposit of P6,121.20 with the Philippine National Bank representing the assessed value of the property, the Regional Trial Court issued a writ of possession in NPC’s favor as authorized by PD No. 42. Can the judge issue the writ without first conducting a hearing on the amount of just compensation? TAU MU Held: In Panes v. Visayas, 264 SCRA 708 (1996), we ruled that the determination of the amount of provisional deposit for the issuance of the writ of possession is a judicial function. Consequently, a

ACADCOM 2010; Contributors: Gene Geocaniga, Jarissa Guiani, Darlene Magabilen TAU MU Page 19 of 179

TAU MU TAU MU TAU MU TAU MU TAU MU TAU MU TAU MU

Facts: Respondent owns a piece of land with an area of 39,512 square meters. On October 6, 1981, with the consent of respondent provided they would be paid for the area taken after the processing of documents by the Commission on Audit, the National Irrigation Administration constructed an irrigation canal. The canal afected 24,660 square meters. Since NIA failed to pay for the property, on Dec. 10,1990, respondent filed a complaint praying that NIA be ordered to pay compensation for the property used in the canal constructed. What year should

CITY OF CEBU vs. DEDAMO 387 SCRA 754 May 7, 2002

TAU MU

ESLABAN vs. VDA DE ONORIO 360 SCRA 230 (2001)

be used as basis for determining just compensation? Held: Sec. 4, of Rule 67 of the Rules of Court states that just compensation should be determined “as of the date of the taking of the property or the filing of the complaint, whichever comes first.” The price of the land at the time of the taking, not its value after the passage of time, represents the true value to be paid as just compensation. Thus, the just compensation to be paid respondent should be determined as of its taking by the NIA in 1981, not the filing of the complaint in 1990. TAU MU

TAU MU

Facts: In 1994, the Sangguniang Panglungsod ng Mandaluyong City issued a resolution authorizing the mayor to institute expropriation proceedings over the lot of Saguitan. Saguitan filed a motion to dismiss arguing that the Local Government Code requires an ordinance, not a mere resolution. The trial court, however, denied the motion reasoning that an ordinance would only be required to appropriate funds in payment of the expropriated property once the trial court has determined its value. Is the court correct? Held: No. Under the Local Government Code, the exercise of eminent domain must comply with the following requisites: 1) an ordinance enacted by the council authorizing the chief executive; 2) the power is exercised for public use, purpose or welfare, or for the benefit of the poor and the landless; 3) payment of just compensation; and 4) a definite ofer was previously made which was not accepted by the property owner. An examination of the law shows that an ordinance is necessary to authorize the filing of the complaint with the proper court since, beginning at that point, the power of eminent domain stats to be exercised. The determination of the award of just compensation is but the last stage which cannot be arrived at without an initial finding by the court that plaintif has a right to take property. TAU MU

Ateneo de Davao

TAU MU TAU MU

HEIRS OF SUGUITAN vs. CITY 328 SCRA 137 (2000)

The Fraternal

TAU MU TAU MU

summary hearing must first be done by the judge to determine provisionally the value of the property, which full amount should be deposited with the national or provincial treasurer. However, under the 1987 Rules on Civil Procedure, it is provided that upon filing of the complain and after due notice to the defendant “the plaintif shall have the right to take or enter upon the possession of the real property involved if he deposits with the authorized government depositary an amount equivalent to the assessed value of the property for purposes of taxation to be held by such bank subject to the orders of the court.” There is therefore no need of a hearing, as the issuance of the writ of possession by the court becomes ministerial once the provisional compensation is deposited.

KITY

TAU MU TAU MU TAU MU TAU MU TAU MU TAU MU

CONSTITUTIONAL LAW II Order of Saint Thomas More Atty. Philip John Pojas/Atty. Rovyne G. Jumao-as, RN University College of Law

Facts: On Sept. 17, 1993, the City of Cebu filed expropriation proceedings against respondents who owned parcels of land which the former wanted to use for the construction of a public road. On Sept. 21, 1994, the court issued a writ of possession in favor of the City. Sec. 19 of the Local Government Code of 1991 provides that just compensation shall be determined as of the time of actual taking. However, Sec. 4, Rule 67 of the 1997 Rules of Procedure provides that just compensation shall be determined at the time of the filing of the complaint for expropriation. Which shall prevail? Held: The applicable law as to the point of reckoning for the determination of just compensation is the Local Government Code. The 1997 Rules on Civil Procedure cannot prevail over RA No. 7160, which is a substantive law. However, since the parties voluntarily agreed to be bound by the report of the commission and approved by the trial court, said valuation of the property has the force of law between the parties and should be complied with in good faith. TAU MU REPUBLIC vs. CA 383 SCRA 611 July 2, 2002 Facts: In 1969, the government instituted expropriation proceedings against tracts of land in Malolos, Bulacan, to be utilized for broadcast operation and use of radio transmitter facilities of the Voice of the Philippines. In 1979, the trial court rendered judgment condemning the property and ordering the government to pay as just compensation P6.00 per square meter, which included the 76,589 square meter subject of the dispute. Eventually, portions of the land were used for the expansion of Bulacan State University. Despite the judgment however, and despite utilization for public use by the government, as of Sept. 16, 1999, the owners of the property were not compensated. May the owners demand the return of the property? If not, may they demand compensation based on the current value of the property? Held: By final and executory judgment in the expropriation proceedings, the previous owners are not entitled to recover possession of their

ACADCOM 2010; Contributors: Gene Geocaniga, Jarissa Guiani, Darlene Magabilen TAU MU Page 20 of 179

expropriated land – which are still devoted to public use – but only to demand the fair market value. It is of no moment that the present property use difers from the original purpose of the expropriation. Surely, the government, as condemnor and owner of the property, is well within its rights to alter and decide the use, the only, limitation being that it is for public use. The compensation for the property should be computed at its market value at the time it was taken and appropriated by the State. However, the owners are entitled to an interest of 12% per annum computed from the date of the “taking” of the property, i.e. September 1969, until the due amount shall have been fully paid.

TAU MU TAU MU TAU MU TAU MU TAU MU TAU MU TAU MU

Facts: The plan to extend EDSA to Roxas Boulevard to be ultimately linked to the Cavite Coastal Road Project, originally called for the expropriation of properties along Cuneta Avenue in Pasay City. Later on, however, the Ministry of Public Highways decided to make the proposed extension pass through Fernando Rein and Del Pan Streets. Because of the protests of residents of the latter, the Commission on Human Settlements recommended the reversion to the original plan, but the Ministry argued the new route withh save the government P2 million. The government filed expropriation proceedings

TAU MU

DE KNECHT vs. BAUTISTA 100 SCRA 660 (1980)

TAU MU

JUDICIAL REVIEW

TAU MU TAU MU

Facts: On April 5, 1948 an entry of judgment was made in the expropriation case filed by the Republic against the Denzons. The judgment allowed the Republic to expropriate their property after payment of just compensation in the amount of P4,062.10. After the lapse of more than 50 years, the Republic failed to pay the amount, though it took possession of the property. May the landowner be allowed to recover? Held: Yes. While we are aware of the doctrine that “non-payment of just compensation does not entitle the landowner to recover possession, the facts of the present case do not justify its application. It bears stressing that the Republic was ordered to pay just compensation twice, the first was in the expropriation proceedings and the second in, Valdehueza. Fifty-seven years have passed since then. We cannot but construe the Republic’s failure to pay just compensation as a deliberate refusal on its part. We thus rule that the special circumstances prevailing in this case entitle respondent to recover possession of the expropriated lot from the Republic. Unless this form of swift an efective relief is granted to him, the grave injustice committed against his predecessors-in-interest, though no fault or negligence on their part, will be perpetuated.

The Fraternal Ateneo de Davao

TAU MU TAU MU

REPUBLIC vs. LIM 462 SCRA 265 (2005)

KITY

TAU MU TAU MU TAU MU TAU MU TAU MU TAU MU

CONSTITUTIONAL LAW II Order of Saint Thomas More Atty. Philip John Pojas/Atty. Rovyne G. Jumao-as, RN University College of Law

against the owners of Fernando Rein and Del Pan streets, among whom was petitioner. Held: The choice of Fernando Rein and Del Pan streets is arbitrary and should not receive judicial aprpoval. The Human Settlements Commission concluded that the cost factor is so minimal that it can be disregarded in making a choice between the two lines. The factor of functionality strongly militates against the choice of Fernando Rein and Del Pan streets, while the factor of social and economic impact bears grievously on the residents of Cuneta Avenue. While the issue would seem to boil down to a choice between people, on one hand, and progress and development, on the other, it is to be remembered that progress and development are carried out for the benefit of the people. TAU MU POWER OF TAXATION Taxes and Taxation (Definition) Taxes are the enforced proportional contributions from persons and property, levied by the State by virtue of its sovereignty, for the support of government and for all public needs. Taxation is the method by which these contributions are exacted [Cruz, Constitutional Law 2007, p.86]. Purpose The obvious, primary, and specific purpose of the power to tax is to raise revenue. However, from the earliest days of the history of the power of taxation, the power to tax has been recognized as an instrument of national economic and social policy. It has, for instance, been used as an instrument for the extermination of undesirable activities and enterprises. In the celebrated words of Justice Marshall, the power to tax involves the power to destroy [Bernas, The 1987 Philippine Constitution: A Comprehensive Reviewer, p. 279]. TAU MU

Nature The importance of taxation derives from the unavoidable obligation of the government to protect the people and extend them benefits in the form of public projects and services. In exchange for these, the people are subjected to the reciprocal duty of sharing the expenses to be incurred therefor through the payment by them of taxes. The obligation to pay taxes is not based on contract. It is a duty imposed upon the individual by the mere fact of his membership in the body politic and his enjoyment of the benefits available from such membership. Hence, except only in the case of poll taxes, nonpayment of a tax may be the subject of criminal prosecution and punishment. The accused cannot invoke the prohibition against prohibition against imprisonment for debt as taxes are not considered debts [Cruz, Constitutional Law 2007, p.87]. TAU MU

Levied

TAX to

Tax vs. License* LICENSE raise Imposed for regulatory

ACADCOM 2010; Contributors: Gene Geocaniga, Jarissa Guiani, Darlene Magabilen TAU MU Page 21 of 179

KITY

purposes only Justified under police power, and the amount of fees required is usually limited only to the cost of regulation. Exception: business that is licensed is nonuseful and is sought to be discouraged by the legislature, in which case a high license fee is required. *[Cruz, Constitutional Law 2007, p.87] Scope So pervasive is the power of taxation that it reaches even the citizen abroad and his income earned from sources outside his State. In other cases, all income earned in the taxing State, whether by citizens or aliens, and all immovable and intangible personal properties found in its territory, as well as tangible personal property owned by persons domiciled therein, are subject to its taxing power. Even shares of stock issued by a foreign corporation, but “in action” in the local State may be taxed by it, as so too are the proceeds from an insurance policy issued abroad [Cruz, Constitutional Law 2007, p.87]. TAU MU

TAU MU TAU MU TAU MU TAU MU TAU MU TAU MU TAU MU TAU MU TAU MU TAU MU TAU MU

Who may Exercise The power of taxation is inherent in the State. Primarily vested in the national legislature, it may now also be exercised by the local legislative bodies, no longer by virtue of a valid delegation as before but pursuant to a direct authority conferred by Article X, Section 5, of the Constitution, which provides that “each local government unit shall have the power to create its own sources of revenue and to levy taxes, fees and charges, subject to such guidelines and limitations as the Congress may provide, consistent with the basic policy of local autonomy.”

TAU MU TAU MU

revenues

Power to Tax and Power to Destroy The power to tax may include the power to destroy if it is used validly as an implement of the police power in discouraging and in efect ultimately prohibiting certain things or enterprises inimical to the public welfare. Thus, if massage parlors are found to be mere fronts for prostitution, they may be subjected to such onerous taxes as to practically force them to stop operating. A similar imposition, for the same purpose, may be levied upon non-useful businesses or things, like pool halls, slot machines, or idle lands. But where the power to tax is used solely for the purpose of raising revenues, the modern view is that it cannot be allowed to confiscate or destroy. If this is sought to be done, the tax may be successfully attacked as an inordinate and unconstitutional exercise of the discretion that is usually vested exclusively in the legislature in ascertaining the amount of the tax [Cruz, Constitutional Law 2007, p.88].

The Fraternal Ateneo de Davao

TAU MU TAU MU TAU MU TAU MU TAU MU TAU MU

CONSTITUTIONAL LAW II Order of Saint Thomas More Atty. Philip John Pojas/Atty. Rovyne G. Jumao-as, RN University College of Law

Not only the amount of the tax but also such other questions as whether or not to tax in the first place, or whom or what to tax, or for what purpose, are likewise subject to the discretion of the legislature. As usual, however, such discretion is not unlimited. Its exercise may be reversed in proper cases, specifically when it violates the due process and equal protection clauses or the particular restrictions on the power of taxation as prescribed in Article VI, Section 28, of the Constitution [Cruz, Constitutional Law 2007, p.88]. The President of the Philippines may also exercise the power of taxation to a limited extent when delegated tarif powers (Sec 28 (2), Article VI, of the Constitution) [Nachura, Outline Reviewer in Political Law 2006, p. 61]. TAU MU General Limit on the Power to Tax The power to tax exists for the general welfare. Hence implicit on the power is the limitation that it should be exercised only for a public purpose. “To lay, with one hand, the power of the government on the property of the citizen, and with the other to bestow it upon favored individuals to aid private enterprises and build up private fortunes, is none the less a robbery because it is done under the forms of law and is called taxation.” The power to tax ‘is an attribute of sovereignty.’ In fact, it is the strongest of all the powers of government. But for all its plenitude, the power to tax is not unconfined as there are restrictions. Adversely afecting as it does property rights, both due process and equal protections clauses of the Constitution may properly be invoked to invalidate in appropriate cases a revenue measure. If it were otherwise, there would be truth to the 1903 dictum of Chief Justice Marshall that “the power to tax involves the power to destroy.” The web of unreality spun from Justice Marshall’s famous dictum was brushed away by one stroke of Mr. Justice Holmes’ pen, thus: “The power to tax is not the power to destroy while this Court sits.” “So it is in the Philippines.” [Bernas, The 1987 Philippine Constitution: A Comprehensive Reviewer, p. 280] TAU MU Limitations on the Exercise of the Power of Taxation a. Due Process of Law Taxes will not be allowed if they are confiscatory, except where they are intended precisely for destruction as an instrument of police power [Cruz, Constitutional Law 2007, p.89]. With the legislature primarily lies the discretion to determine the nature, object, extent, coverage and situs of taxation. But where a tax measure becomes so unconscionable and unjust as to amount to confiscation of property, courts will not hesitate to strike it down, for despite all its plentitude, the power to tax cannot override constitutional prescriptions [Nachura, Outline Reviewer in Political Law 2006, p. 61]. From the procedural viewpoint, due process does not require previous notice and hearing before a

ACADCOM 2010; Contributors: Gene Geocaniga, Jarissa Guiani, Darlene Magabilen TAU MU Page 22 of 179

law prescribing fixed or specific taxes on certain articles may be enacted. But where the tax to be collected is to be based on the value of the taxable property, the taxpayer is entitled to be notified of the assessment proceedings and to be heard therein on the correct valuation to be given the property. Otherwise, the ad valorem tax may be increased to the prejudice of the owner in the ex parte appraisal to be made by the government authorities [Cruz, Constitutional Law 2007, p.90].

TAU MU TAU MU TAU MU TAU MU TAU MU TAU MU TAU MU TAU MU

d. Public Purpose To sustain a tax, it is necessary to show that the proceeds are devoted to a public purpose.

TAU MU

There is no provision in the Constitution specifically prohibiting double taxation. Our Supreme Court has not categorically held that double or multiple taxation is prohibited in our jurisdiction. Double taxation is no more than doubled taxation, as when a thing is taxed once at Php 250.00 and taxed again at another Php 250.00 while a similar thing is taxed only once at Php 500.00. “The power to tax twice,” it is said, “is as ample as to tax twice.” There is double taxation when additional taxes are laid on the same subject by the same taxing jurisdiction during the same taxing period and for the same purpose. Thus, if a person’s properties are each taxed separately and thereafter all of them are again taxed, this time collectively, by the same taxing jurisdiction for the same purpose and during the same taxing period, the second imposition would constitute double taxation [Cruz, Constitutional Law 2007, p.92].

TAU MU TAU MU

c. Double Taxation Additional taxes are laid on the same subject by the same taxing jurisdiction during the same taxing period and for the same purpose [Nachura, Outline Reviewer in Political Law 2006, p. 62].

The Fraternal Ateneo de Davao

TAU MU TAU MU

b. Equal Protection “The rule of taxation shall be uniform and equitable” (Section 28, Article VI, Constitution). Uniformity in taxation means that persons or things belonging to the same class shall be taxed at the same rate. Thus, if cigarettes are classified into local and imported, there is observance of the uniformity rule if all local cigarettes are taxed at Php 12.00 per carton, regardless of value, and all imported cigarettes are taxed at Php 20.00 per carton, also regardless of value. TAU MU This should be distinguished from equality in taxation, which simply means that the tax shall be strictly proportional to the relative value of the property. The above rules require a valid classification in the selection of the object of taxation. Higher taxes may be imposed on commercial or industrial lands than on residential lands, or on practitioners in urban centers than in rural areas. It is also provided that the rule of taxation shall be equitable. Equitable taxation connotes that taxes should be apportioned among the people according to their capacity to pay [Cruz, Constitutional Law 2007, p.90].

KITY

TAU MU TAU MU TAU MU TAU MU TAU MU TAU MU

CONSTITUTIONAL LAW II Order of Saint Thomas More Atty. Philip John Pojas/Atty. Rovyne G. Jumao-as, RN University College of Law

Revenues derived from taxes cannot be used for purely private purposes or for the exclusive benefit of private persons. Like public use in eminent domain, the phrase “public purpose” as applied to taxation is now given the broadest interpretation as to include even indirect public advantage or benefit. The mere fact that the tax will be directly enjoyed by a private individual does not make it invalid so long as some link to the public welfare is established [Cruz, Constitutional Law 2007, p.92]. TAU MU

Tax Exemptions [Cruz, Constitutional Law 2007, p.92-96] Tax exemptions are either constitutional or statutory. The constitutional exemption from taxes is provided for in Article VI, Section 28(3): “Charitable institutions, churches, and parsonages or convents appurtenant thereto, mosques, non-profit, cemeteries, and all lands, buildings, and improvements actually, directly, and exclusively used for religious, charitable or educational purposes shall be exempt from taxation.” Exemption is granted religious and charitable institutions because they give considerable assistance to the State in the improvement of the morality of the people and the care of the indigent and the handicapped. There are ministrant functions of the government which, however, it is not able to fully discharge. The above provision is intended to make it easier for these institutions to pursue these laudable objectives without the impediment of taxes that they otherwise would have to shoulder. The added justification in the case of religious institutions is the principle to give full rein to the freedom of religious profession and worship. The provision now requires that the lands, buildings or improvements be directly, actually and exclusively devoted to religious, charitable or educational purposes to be entitled to exemption. Under the 1935 Constitution, it was enough that they were at least incidentally devoted to these purposes. Statutory exemptions are granted in the discretion of the legislature. However, the Constitution provides that “no law granting any tax exemption shall be passed without the concurrence of a majority of all the Members of the Congress.” This is because tax exemptions should not be lightly extended since they will represent a loss of revenue to the government. TAU MU

LLADOC vs. CIR 14 SCRA 292 Facts: A parish priest accepted a donation of 10,000, to be used for the construction of a

ACADCOM 2010; Contributors: Gene Geocaniga, Jarissa Guiani, Darlene Magabilen TAU MU Page 23 of 179

TAU MU TAU MU TAU MU TAU MU TAU MU

Meaning of Life, Liberty and Property

TAU MU

Who are Protected It is universal in application to all persons, without regard to any diference in race, color, or nationality. Artificial persons are covered by the protection but only insofar as their property is concerned. The guarantee extends to aliens and includes the means of livelihood (Villegas vs Hui Chong) [Nachura, Outline Reviewer in Political Law 2006, p. 82].

TAU MU

Due process is a guaranty against any arbitrariness on the part of the government, whether committed by the legislature, the executive, or the judiciary. If the law itself unreasonably deprives a person of hi life or his liberty or his property, he is denied the protection of due process. Of the enjoyment of his rights is conditioned on an unreasonable requirement, due process is likewise violated. Any government act that militates against the ordinary norms of justice or fair play is considered an infraction of the greatr guaranty of due process; and this is true whether the denial involves violation merely of the procedure prescribed by the law or afects the very validity of the law itself [Cruz, Constitutional Law 2007, p.100]. TAU MU

TAU MU

Due Process of Law Definition A law which hears before it condemns, which proceeds upon inquiry and renders judgment only after trial. It is the responsiveness to the supremacy of reason, obedience to the dictates of justice (Ermita-Malate Hotel vs City of Manila). The embodiment of the sporing idea of fair play [Nachura, Outline Reviewer in Political Law 2006, p. 82].

TAU MU

Section 1. No person shall be deprived of life, liberty or property without due process of law, nor shall any person be denied the equal protection of the laws.

TAU MU TAU MU

DUE PROCESS AND EQUAL PROTECTION

The Fraternal Ateneo de Davao

TAU MU TAU MU

church. The money was spent for the purpose specified. Later, the Bureau of Internal Revenue imposed a donee’s tax on his successor, who objected, invoking the constitutional exemption of church properties from taxes. Held: The SC rejected his contention, holding that the exemption referred only to property taxes imposed on lands, buildings and improvements used for religious purposes. The tax in this case was not an ad valorem tax on the church itself but an excise tax imposed on the priest for his exercise of the privilege to accept the donation. The Commission assessed the donee’s gift tax; the assessment was not the properties themselves. Manifestly, a gift tax is not within the exemption provisions. A gift tax is not a property tax, but an excise tax imposed on the transfer of property by way of gift inter vivos, the imposition of which on property used exclusively for religious purposes does not constitute an impairment of the Constitution.

KITY

TAU MU TAU MU TAU MU TAU MU TAU MU TAU MU

CONSTITUTIONAL LAW II Order of Saint Thomas More Atty. Philip John Pojas/Atty. Rovyne G. Jumao-as, RN University College of Law

Life The constitutional protection of the right to life is not just a protection of the right to be alive or to the security of one’s limb against physical harm. The right to life is the right to a good life. The emphasis on the quality of living is found in Article II where Section 6 commands the State to promote a life of “dignity” and where Section 7 guarantees “a decent standard of living.” [Bernas, The 1987 Philippine Constitution: A Comprehensive Reviewer, p. 24] TAU MU Life includes the right of an individual to his body in its completeness, free from dismemberment, and extends to the use of God-given facilities which make life enjoyable [Justice Malcolm, Philippine Constitutional Law, p. 320] The unborn have the constitutional right to life as embodied in Section 12, Article II of the Constitution, thus: “The State recognizes the sanctity of family life and shall protect and strengthen the family as a basic autonomous social institution. It shall equally protect the life of the mother and the life of the unborn from conception. The natural and primary right and duty of parents in the rearing of the youth for civic efficiency and the development of moral character shall receive the support of the Government.” Liberty According to Mabini, “liberty is the freedom to do right and never wrong.” Liberty, as guaranteed under the due process clause, is not unbridled license; it is liberty regulated by law. A person is free to act but he may exercise his rights only in such manner as not to injure the rights of others. One’s own liberty must be enjoyed consistently with the enjoyment of a like liberty by others. In other words, the individual, as a creature of society, should be prepared to surrender part of his freedom for the benefit of the greater number in recognition of the time-honored principle of “salus populi est suprema lex.” TAU MU Thus, to illustrate, one’s freedom of expression cannot be unsed unfairly to destroy another’s reputation, or to incite rebellion, or to ofend public morals; neither may he abuse the sanctity of his home by converting it into a den of criminality or hotbed of disease; nor may he insist on selling his goods at black market prices, if they be prime necessities, to the detriment of the consuming public. TAU MU Subject only to reasonable restrictions of the law, a person is free to do as he pleases. He may marry for love or for money, pursue a profession or engage in manual labor, establish his own business or merely hire out as an employee, isolate himself from the community or mix with his neighbors, profess a religion or embrace atheism – in short, do anything that does not ofend the public welfare [Cruz, Constitutional Law 2007, p.104]. Property Property is anything what can come under the right of ownership and be the subject of contract.

ACADCOM 2010; Contributors: Gene Geocaniga, Jarissa Guiani, Darlene Magabilen TAU MU Page 24 of 179

This will include all things – real, personal, tangible and intangible – that are within the commerce of man, like lands, jewelry, automobiles, buildings, goodwill, inheritance, intellectual creations, future earnings, works of art, animals, mortgages, proceeds, etc [Cruz, Constitutional Law 2007, p105].

TAU MU TAU MU TAU MU TAU MU TAU MU TAU MU TAU MU TAU MU

Life and Property do NOT Enjoy Identical Protection from the Constitution The primacy of human rights over property rights is recognized. In the hierarchy of civil liberties, the rights of free expression and of assembly occupy a preferred position as they are essential to the preservation and vitality of our civil and political institutions. TAU MU “The superiority of these freedoms over property rights is underscored by the fact that a mere reasonable or rational relation between the means employed by the law and its object or purpose – that the law is neither arbitrary nor discriminatory nor oppressive – would suffice to validate a law which restricts or impairs property rights. On the other hand, a constitutional or valid infringement of human rights requires a more stringent criterion, namely existence of a grave and immediate danger of a substantive evil which the State has the right to prevent.” (Philippine Blooming Mills Employees Organization vs Philippine Blooming Mills, Inc.) [Bernas, The 1987 Philippine Constitution: A Comprehensive Reviewer, p. 26]. TAU MU

TAU MU

Public office is not property; but one unlawfully ousted from it may institute an action to recover the same, flowing from the de jure officer’s right to office. Indeed, the Court has recognized that while public office is not property to which one may acquire a vested right, it is nevertheless a protected right [Nachura, Outline Reviewer in Political Law 2006, p. 82].

TAU MU TAU MU

A mining license that contravenes a mandatory provision of law under which it is granted is void. Being a mere privilege, a license does not vest absolute rights in the holder. Thus, without ofending the due process and the nonimpairment clauses of the Constitution, it can be revoked by the State in the public interest. Mere privileges, such as the license to operate cockpits, are not property rights and are revocable at will [Nachura, Outline Reviewer in Political Law 2006, p. 83].

The Fraternal Ateneo de Davao

TAU MU TAU MU

Protected property includes all kinds of property found in the Civil Code. A license to operate a cockpit is not considered protected property. It is deemed merely a privilege withdrawable when the public interest require its withdrawal. A mere privilege, however, may evolve into some form of property right protected by due process, as for instance when a privilege, in this case an export quota, has been enjoyed for so long, has been subject of substantial investment and has become the source of employment for thousands (American Interfashion Corporation vs Office of the President) [Bernas, The 1987 Philippine Constitution: A Comprehensive Reviewer, p. 25].

KITY

TAU MU TAU MU TAU MU TAU MU TAU MU TAU MU

CONSTITUTIONAL LAW II Order of Saint Thomas More Atty. Philip John Pojas/Atty. Rovyne G. Jumao-as, RN University College of Law

Two Kinds of Due Process Due process has both a procedural and a substantive aspect. As a substantive requirement, it is a prohibition of arbitrary laws; because, if all that the due process clause required were proper procedure, then life, liberty, or property could be destroyed arbitrarily provided proper formalities are observed. As a procedural requirement, it relates chiefly to the mode of procedure which government agencies must follow in the enforcement and application of laws. It is a guarantee of procedural fairness. Its essence was expressed by Daniel Webster as “a laws which hears before it condemns” [Bernas, The 1987 Philippine Constitution: A Comprehensive Reviewer, p. 27]. PROCEDURAL DUE PROCESS Requisites of Civil Procedural Due Process (Judicial Due Process): a. An impartial court clothed with judicial power to hear and determine the matters before it; b. Jurisdiction properly acquired over the person of the defendant and over property which is the subject matter of the proceeding; c. Opportunity to be heard; d. Judgment rendered upon lawful hearing and based on evidence adduced. (Banco Español Filipino vs Palanca) TAU MU Impartial and Competent Court It is clear that a court afected by bias or prejudice cannot be expected to render a fair and impartial decision. As our Supreme Court has declared, every litigant is entitled to the cold neutrality of an impartial judge [Cruz, Constitutional Law 2007, p.110]. In Rivera vs CSC and in Singson vs NLRC, the Supreme Court reiterated the rule that a public officer who decided the case should not be the same person to decide it on appeal because he cannot be an impartial judge [Nachura, Outline Reviewer in Political Law 2006, p. 84]. In the Ynot case, the Supreme Court said: “the closed mind has no place in the open society. It is part of the sporting idea of fair play to hear the other side before an opinion is formed or a decision is made by those who sit in judgment…” And in Javier vs COMELEC, the imperative of impartiality was described, thus: “This court has repeatedly and consistently demanded the cold neutrality of an impartial judge as the indispensible imperative of due process. To bolster that requirement, we have held that the judge must not only be impartial but must also appear to be impartial as an added assurance to the parties that his decision will be just. The litigants are entitled to no less than that. They should be sure that when their rights are violated they can go to a judge who shall give

ACADCOM 2010; Contributors: Gene Geocaniga, Jarissa Guiani, Darlene Magabilen TAU MU Page 25 of 179

TAU MU TAU MU TAU MU TAU MU TAU MU TAU MU TAU MU TAU MU

In actions in personam, such as a complaint for recovery of a loan, jurisdiction over the defendant is acquired by the court by his voluntary appearance or through service of summons upon him. This may be efected personally, or by substituted service, or, in exceptional cases, by publication. In actions in rem or quasi in rem such as land registration proceedings or the foreclosure of a real estate mortgage, the jurisdiction of the court is derived from the power it may exercise over the property. Jurisdiction over the person is not essential, provided the relief granted by the court is limited to such as can be enforced against the property itself. Notice by publication is sufficient in these cases. TAU MU The reason is that property is always presumed to be in the possession of the owner or his agent,

TAU MU

Jurisdiction It should be emphasized that the service of summons is not only required to give the court jurisdiction over the person of the defendant but also to aford the latter the opportunity to be heard on the claim made against him. Thus, compliance with the rules regarding the service of summons is as much an issue of due process as of jurisdiction. While jurisdiction over the person of the defendant can be acquired by the service of summons, it can also be acquired by voluntary appearance before the court, which includes submission of pleadings in compliance with the order of the court or tribunal [Nachura, Outline Reviewer in Political Law 2006, p. 86].

TAU MU TAU MU

By competent court is meant one vested with jurisdiction over a case as conferred upon it by law. For example, a regional trial court is competent to try a prosecution for murder but not for violation of a municipal ordinance. The competent court in the latter case would be the metropolitan or the municipal trial court. Only the Supreme Court is competent to review a decision of the Commission on Audit, but jurisdiction over ordinary appealed cases involving only questions of fact is vested in the Court of Appeals [Cruz, Constitutional Law 2007, p.112].

The Fraternal Ateneo de Davao

TAU MU TAU MU

them justice. They must trust the judge, otherwise they will not go to him at all. They must believe in his sense of fairness, otherwise they will not seek his judgment. Without such confidence, there would be no point in invoking his action for the justice they expect. Due process is intended to insure that confidence by requirements with what Justice Frankfurter calls the rudiments of fair play. Fair play calls for equal justice. There cannot be equal justice where a suitor approaches a court already committed to the other party and with a judgment already made and waiting only to be formalized after the litigants shall have undergone the charade of a formal hearing…the judge will reach his conclusions only after the evidence is in and all the arguments are filed, on the basis of the established facts and pertinent law.” TAU MU

KITY

TAU MU TAU MU TAU MU TAU MU TAU MU TAU MU

CONSTITUTIONAL LAW II Order of Saint Thomas More Atty. Philip John Pojas/Atty. Rovyne G. Jumao-as, RN University College of Law

who may be safely held under certain conditions to know that proceedings have been instituted against it. According to Cooley, “if the owners are named in the proceedings and personal notice is provided for, it is rather from tenderness to their interests, and in order to make sure that the opportunity for a hearing shall not be lost to them, than from any necessity that the case shall assume that form [Cruz, Constitutional Law 2007, p.113]. Hearing/Opportunity to be Heard Due process is satisfied as long as the party is accorded with the opportunity to be heard. If it is not availed of, it is deemed waived or forfeited without violating the constitutional guarantee (Bautista vs CA) [Nachura, Outline Reviewer in Political Law 2006, p. 86]. Notice to a party is essential to enable it to adduce its own evidence and to meet and refute the evidence submitted by the other party. Every litigant is entitled to his day in court. He has a right to be notified of every incident of the proceeding and to be present at every stage thereof so that he may be heard by himself and counsel for the protection of his interests. As held in David vs Aquilizan, a decision rendered without a hearing is null and void ab initio and my be attacked directly or collaterally. “If it were otherwise,” the Supreme Court declared, “then the cardinal requirement that no party should be made to sufer in person or property without being given a hearing would be brushed aside. The doctrine consistently adhered to by this Court when such a question arises…is that a denial of due process suffices to cast on the official act taken by whatever branch of the government the impress of nullity.” [Cruz, Constitutional Law 2007, p.114] Not all cases require a trial-type hearing. Due process in labor cases before a Labor Arbiter is satisfied when the parties are given the opportunity to submit their position papers to which they are supposed to attach all the supporting documents or documentary evidence that would support their respective claims (Mariveles Shipyard vs CA) [Nachura, Outline Reviewer in Political Law 2006, p. 87]. In Zaldivar vs Sandiganbayan, the Supreme Court declared that “to be heard” does not only mean verbal arguments in court. One may be heard also through pleadings. Where the opportunity to be heard, either through oral arguments or pleadings, is accorded, there is no denial of procedural due process. ANG TIBAY vs. CIR 69 PHIL 635 (140) The court held in this case the requisites for administrative due process: 1. The right to a hearing which includes the right to present one’s case and submit the evidences in support thereof.

ACADCOM 2010; Contributors: Gene Geocaniga, Jarissa Guiani, Darlene Magabilen TAU MU Page 26 of 179

TAU MU TAU MU TAU MU TAU MU TAU MU TAU MU TAU MU

1. The students must be informed in writing of the nature and cause of any accusation against them. 2. They shall have the right to answer the charges against them with the assistance of counsel if desired 3. They shall be informed of the evidences against them. 4. They shall have the right to adduce evidence in their own behalf 5. The evidences must be duly considered by the investigating committee or official designated by the school authorities to hear and decide the case. The students invoked the Ang Tibay case but the court held that the applicable case is the Guzman case.

TAU MU

Important points in this case: Five requirements to satisfy due process in the discipline of students:

TAU MU

Facts: There was a hazing conducted as part of the initiation rites of the Aquila Legis Fraternity of ADM where the victim, Villa, was an applicant, and was killed as a result thereof. Students who acted as master auxiliaries were refused admittance to the school after having been found guilty of participating in hazing. The decision of the school to dismiss the students was based on the findings submitted by the Board. The accused contested that they were not given copies of the school’s rules and regulations. They were not warned on the conduct to avoid, thus, they were not aforded due process. Held: The court held that this is an exception to due process. Since they are law students, they cannot use that argument. It was presumed that they have received a copy or have diligently asked for one. If they were undergraduates, they would have been treated diferently. Hazing was not defined in the manual but is the same punishable.

TAU MU TAU MU

ADM vs. CAPULONG 222 SCRA 644 (1993)

The Fraternal Ateneo de Davao

TAU MU TAU MU

2. The tribunal must consider the evidences presented. 3. The decision must have something to support itself 4. The evidence must be substantial 5. The decision must be rendered on the evidence presented at the hearing or at least contained in the record and disclosed to the parties afected. 6. The tribunal or any of its judges must act on its own independent consideration of the law and facts of the controversy and not simply accept the views of a subordinate. 7. The body should, in all controversial questions, render its decision in such a manner that the parties to the proceeding can know the various issues involved and the reason for the decision.

KITY

TAU MU TAU MU TAU MU TAU MU TAU MU TAU MU

CONSTITUTIONAL LAW II Order of Saint Thomas More Atty. Philip John Pojas/Atty. Rovyne G. Jumao-as, RN University College of Law

They also argued that the fifth requirement was not satisfied: their evidence was not considered. The court ruled that whether or not the evidence entered the process of decision making or consideration is very hard to prove and that is why the court said that the fact that it was mentioned as having been considered is sufficient. LAO GI vs. CA 180 SCRA 756 (1989) Facts: On Sept 3, 1958, the Secretary of Justice found Filomeno Chia, Jr., alis Sia Pieng Hui, to be a Filipino Citizen as it appears that hid father is a Filipino citizen. However, on October 3, 1980, the Minister of Justice set aside his citizenship on the ground hat it was founded on fraud and misrepresentation. On March 9, 1981, a charge for deportation was filed with the CID against Lao Gi alias Filomeno Chia, Jr., his wife and children. In this case, it appears that petitioners are charged with having entered the Philippines by means of false and misleading statements or without inspection or admission by the immigration authority at a designated port of entry. Issue: Whether due process was denied from the petitioners Held: While it is not disputed that it is also within the power and authority of the commissioner to require an alien to register, such a requirement must be predicated in a positive finding that the person who is so required is an alien. In this case where the very citizenship of the petitioner is in issue, there should be a previous determination by the CID that they are aliens before the petitioners may be directed and required to register as aliens. Although a deportation proceeding does not partake of the nature of a criminal action, however, considering that it is a harsh and extraordinary administrative proceeding afecting freedom and liberty of a person, the constitutional right of such person to due process should not be denied. The charge against an alien must specify the acts or omissions complained of which must be stated in ordinary and concise language to enable a person of common understanding to know on what ground he is intended to be deported and enable the CID to pronounce a proper judgment before any charge should be filed in the CID. A preliminary investigation must be conducted to determine if there is sufficient cause to charge the respondent for deportation. Petition is GRANTED and the question order of the respondent CID is set aside. CID is directed to continue hearing the deportation case against the petitioners and thereafter, based on the evidence before it, resolve the issue of citizenship of petitioners, and if found to be aliens, to determine whether or not the petitioners should be deported and/or otherwise ordered to register as aliens. TAÑADA vs. TUVERA 146 SCRA 446 (1986)

ACADCOM 2010; Contributors: Gene Geocaniga, Jarissa Guiani, Darlene Magabilen TAU MU Page 27 of 179

The case involves the requirement that all laws must be published in order to take efect. The court stated that publication of laws is an element of due process because how can it be efective if people do not know about its existence. Laws would entail publication of all laws, including statutes of local application that are of public interest. Only regulations that are internal in nature or which regulate the personnel of an administrative agency are not required to be published.

TAU MU TAU MU TAU MU TAU MU TAU MU TAU MU TAU MU

PHILCOMSAT vs. ALCUAZ

TAU MU

Facts: Petitioner was sentenced to forced resignation without separation benefits by the Merit Systems Protection Board which was chaired by one Thelma Gaminde. He appealed to the Civil Service Commission which sustained the appealed decision. He filed a motion for reconsideration which was also denied by the CSC. However, Gaminde participated in resolving the motion for reconsideration after her appointment as CSC Commissioner. Is there a violation of due process? Held: Yes. In order that the review of the discussion of a subordinate officer might not turn out to be a farce, the reviewing officer must perforce be other than the officer whose decision is under review; otherwise there could be no diferent view or there would be no real review of the case. The decision of the reviewing officer would be a biased view; inevitably it would be the same view since being human he would not admit that he was mistaken in his first view of the case.

TAU MU

RIVERA vs. CSC 240 SCRA 3 (1995)

TAU MU TAU MU

Facts:Petitioner here was charged with violation of ordinances which requires managers of fishponds to pay operation tax. He is a lessee of a forest land converted to a fishpond. He is charged because of non-payment of tax. He admits that he has not paid said tax but alleges that the ordinances are unconstitutional or that it does not apply to him. He anchors his allegation on the vagueness of the law. The ordinance provides: “if you’re the owner or manager of a fishpond, you pay P3.00 tax. You should be taxed 3 years after the approval of the measure”. Petitioner argues that the tax is vague because there is no specific date as to its effectivity. Assuming that it is not vague, it does not apply to him because he is not the owner or manager of the fishpond but lessee. Held: The law is not vague. A law is only vague if it is “utterly vague” on its face. A vague law violates due process since it will deny people of fair notice of what conduct to avoid. A statute is only vague if people of common intelligence guess as to its meaning or would differ in the application. In the case at bar, the petitioner is included in the term “manager” because even if he is just a lessee, he still invests in it. Even if there is no specific date as to the effectivity of the tax measure, tax would accrue on a specific time, that is 3 years after the approval which is sure to happen.

The Fraternal Ateneo de Davao

TAU MU TAU MU

PEOPLE vs. NAZARIO 165 SCRA 186 (1988)

KITY

TAU MU TAU MU TAU MU TAU MU TAU MU TAU MU

CONSTITUTIONAL LAW II Order of Saint Thomas More Atty. Philip John Pojas/Atty. Rovyne G. Jumao-as, RN University College of Law

180 SCRA 218 (1989) Facts: PHILCOMSAT was given franchise to operate facilities for satellite communications. After a few years it was placed under the jurisdiction of the NTC which had the power of fixing rates. NTC ordered the petitioner to reduce its rate by 15%. Since the order was issued without notice and hearing, it was challenged as a violation of due process. Held: The court held that the order issued by NTC applies to one individual. It is quasi-judicial in nature. Hence, notice and hearing are essential for due process. If the order applies to all, it is rule-making and therefore quasi-legislative. It does not need notice and hearing. In this case, since it is only PHILCOMSAT that is afected by the order, it is entitled to notice and hearing to have due process. It is quasi-judicial. Likewise the order was based on petitioner’s financial statement. Therefore, it should have been given an opportunity to dispute the findings. RADIO vs. NTC 184 SCRA 517 (1990) Facts: In January 1984, PLDT applied to NTC approval of rates for Digital Telecommunication Service Facilities which was provisionally approved. In a hearing, petitioners were not included in the list of afected parties. Issue: Whether NTC gravely abused its discretion amounting to excess or lack of jurisdiction of issuing a provisional authorization in favor of PLDT, without prior notice to petitioners. Held: Petitioners alleged that the application is not for approval of rates but for authorization to engage in new services not covered by PLDT’s franchise and certificate of public convenience and necessity. PLDT maintains that its franchise authorization is to operate not only telephone system but transmission service facilities. Petition is devoid of merit. Well-settled is the rule that Public Service Commission is empowered to approve provisionally rates without the necessity of a prior hearing. NTC did not grant PLDT any authority to engage in any new common service. As to the required notice, it is impossible for the respondent commission to give personal notice to all parties afected, not all of them being known to it. There is no dispute that the notice was published and one of them received the notice which in turn informed the others. The petitioners have timely opposed the petition in question, lack of notice was deemed cured. ESTATE VS CA Facts: Petitioner owned a Quonset building located on a lot of the PPA. The Quonset was being used to store copra and petitioner had a lease contract over the lot with PPA. Respondent, the mayor, demolished the building on the ground that it did not conform with the zoning ordinance which located the zone for warehouse elsewhere. Held: The court held that while the zoning ordinance authorizes the removal of any property

ACADCOM 2010; Contributors: Gene Geocaniga, Jarissa Guiani, Darlene Magabilen TAU MU Page 28 of 179

TAU MU TAU MU TAU MU TAU MU TAU MU

SUMMARY DISMISSAL vs. TORCITA 330 SCRA 153 (2000)

TAU MU

Facts: Mollaneda was a DECS Division Superintendent in Davao City charged before the Civil Service Commission with grave misconduct and conduct grossly prejudicial to the service arising from an act of sexual harassment. The Commission assigned a hearing officer to conduct the hearing including the taking of the testimony of witnesses. Thereafter, based on the recommendation of the hearing officer, the Commission dismissed Mollaneda from the service. Considering that the testimony was not actually heard by the Commission, was the right of Mollaneda to due process violated? Held: No. Due process of law or the requirements of fair hearing does not require that the actual taking of testimony be before the same officer who will make the decision in the case. As long as a party is not deprived of his right to present his own case and submit evidence in support thereof, and the decision is supported by the evidence in the record, there is no question that the requirements of due process and fair trial are fully met. In short, there is no abnegation of responsibility on the part of the tribunal concerned as the actual decision remains with and is made by it.

TAU MU

MOLLANEDA vs. UMACOB 358 SCRA 537 (2001)

TAU MU

Facts: Respondents were public school teachers who were dismissed by the Secretary of Education for participating in the walk-outs and strikes in 1990. The Magna Carta provides that a member or a representative of any local or national teacher’s organization must be included in the investigating committee. This was not the case. Held: Violation of due process – impartiality

TAU MU TAU MU

FABELLA vs. CA 282 SCRA 256 (1997)

The Fraternal Ateneo de Davao

TAU MU TAU MU

which does not conform to it. It should not be interpreted as authorizing the summary removal of the Quonset building. If it does, it would be a contravention of the requirements of due process. Violation of an ordinance does not empower the mayor to avail of extrajudicial remedies. On the contrary the LGC imposes upon him the duty to institute judicial proceedings for violation of ordinance. The authority to abate nuisances without judicial proceedings applies to nuisances per se. While the Sangguniang Bayan may provide for the abatement of a nuisance per se when it is not, the nuisance can only be so adjudged by a judicial determination. Petitioner ws entitled to an impartial hearing to determine whether the Quonset building was a nuisance per se.

KITY

TAU MU TAU MU TAU MU TAU MU TAU MU TAU MU

CONSTITUTIONAL LAW II Order of Saint Thomas More Atty. Philip John Pojas/Atty. Rovyne G. Jumao-as, RN University College of Law

Facts: Respondent policeman was charged before the Summary Dismissal Board with 7 counts of Abuse of Authority, 3 counts of Conduct Unbecoming of a Police Officer, 1 Grave Threats and 1 Illegal Search in connection with a single incident. During the pre-trial, the 12 cases were consolidated into one major complaint of “conduct unbecoming of a police officer.” After hearing, however, the charge was dismissed for insufficiency of evidence. Instead, the Board found him guilty only of Simple Irregularity in the Performance of Duty for taking alcoholic drinks during the incident in question and sentenced him to 20 days suspension. Did the Board violate respondent’s right to due process? Held: Yes, while the definition of “conduct unbecoming of a police officer”, the more serious ofense, is broad and almost all-encompassing, a finding of guilt for an ofense, no matter how light, for which one is not properly charged and tried cannot be countenanced without violation the rudimentary requirements of due process. None of the 12 administrative cases made reference to the specific act of being drunk while in the performance of official duty. There is also no indication in the summary dismissal proceedings that respondent was also being charged with breach of internal discipline consisting of taking alcoholic drinks while in the performance of his duties. Respondent was entitled to know what was he being charged of so that he could traverse the accusation and squarely adduce evidence in his defense. SECRETARY vs. LANTION 343 SCRA 377 (2000)

TAU MU

Facts: The Department of Justice received from the Department of Foreign Afairs U.S. Note verbale No. 0522 requesting for the extradition of Mark Jimenez. Together with the request was a Grand Jury Indictment and Warrant of Arrest issued by the United States District Court for the Southern District of Florida for tax evasion, wire fraud, conspiracy, giving false statements and illegal campaign contributions to President Clinton. The Secretary immediately formed a panel to conduct a technical evaluation of the request. Meanwhile, Jimenez demanded a copy of the documents and asked for ample time to comment of the request. The DOJ refused contending that the documents included grand jury proceedings which are covered by secrecy rules and may only be released upon orders of the United States Court. The Philippine Supreme Court initially ruled that as part of due process Jimenez was entitled to the documents as well as to oppose the request even before the petition reaches the court. Hence, the Motion for Reconsideration. Is Jimenez entitled to the documents as a requirement of due process? Held: No. there is no provision in the RP-US Extradition Treaty of PD No. 1069 which gives an extraditee the right to demand from the Justice Secretary copies of the extradition request from the United States government and its supporting documents and to comment thereon while the

ACADCOM 2010; Contributors: Gene Geocaniga, Jarissa Guiani, Darlene Magabilen TAU MU Page 29 of 179

TAU MU TAU MU TAU MU TAU MU TAU MU TAU MU TAU MU

Facts: Sangguniang Bayan member Nicolas Montesclaros was 72 years old widower when he married Milagros on July 10, 1983. He retired on

TAU MU

GSIS vs. MONTESCLAROS 434 SCRA 441 (2004)

TAU MU

Facts: President Macapagal-Arroyo issued EO 156 on Dec. 12, 2002, prohibiting the importation of all types of used motor vehicles into the country. Importers of said vehicles challenged the order on the ground that it was issued without notice and hearing. Is the executive order valid? Held: Yes. As in the enactment of laws, the general rule is that, the promulgation of administrative issuances requires [no] previous notice and hearing, the only exception being where that legislature itself requires it and mandates that the regulation shall be based on certain facts as determined at an appropriate investigation. This exception pertains to the issuances of legislative rules as distinguished from interpretative rules which give no real consequences more than what the law itself has already prescribed; and are designed merely to provide guidelines to the law which the administrative agency is in charge of enforcing. A legislative rule, on the other hand, is in the nature of subordinate legislation, crafted to implement a primary legislation. In this case, EO 156 is obviously a legislative rule as it seeks to implement or execute primary legislative enactments intended to protect the domestic industry by imposing a ban on the importation of a specified product not previously subject to such prohibition. The due process requirements in the issuance thereof are embodied in Section 401 of the Tarif and Customs Code, which essentially mandate the conduct of investigation and public hearings before the regulatory measure or importation ban may be issued. But since respondents did not raise the issue, the presumption is that said executive issuance duly complied with the procedures and limitations imposed by law. SUBSTANTIVE DUE PROCESS

TAU MU TAU MU

EXECUTIVE vs. SOUTHWING 482 SCRA 673 (2006)

The Fraternal Ateneo de Davao

TAU MU TAU MU

request is still undergoing evaluation. The court cannot write a provision in the treaty giving private respondent that right where there is none. An equally compelling factor is that other countries with similar extradition treaties with the Philippines have expressed the same interpretation adopted by the Philippines and US governments. It is not an international practice to aford a potential extraditee with a copy of the extradition papers during the evaluation stage. An extradition proceeding is not a criminal proceeding to determine his guilt or innocence which will call into operation all the rights of an accused under the Bill of Rights. Besides, under the Treaty, he is aforded opportunity to meet the evidence against him once the petition is filed in court.

KITY

TAU MU TAU MU TAU MU TAU MU TAU MU TAU MU

CONSTITUTIONAL LAW II Order of Saint Thomas More Atty. Philip John Pojas/Atty. Rovyne G. Jumao-as, RN University College of Law

February 17, 1985 and got his retirement benefits fro the GSIS. When he died seven years later, GSIS denied the claim for survivorship pension of Milagros because under PD 1146 the surviving spouse is not qualified if the marriage with the pensioner was contracted within 3 years before the pensioner qualified for the pension. Does the law violate the equal protection clause? Held: Yes. A statue based on reasonable classification does not violate the equal protection clause provided it satisfies the following: 1. It must rest on a substantial distinction 2. It must be germane to the purpose of the law 3. It must not be limited to existing conditions only 4. It must apply equally to all embers of the same class The above provision discriminates against the dependent spouse who contracts marriage within 3 years befre the petitioner qualified for the pension. Even if the dependent souse married the pensioner more than three years before the pensioner’s death, the dependent spouse would not receive the pension if the marriage took place within three years before the pensioner qualified for the pension. If the purpose of the prohibition is to prevent “deathbed marriages”, there is no reason why the 3-year prohibition is reckoned from the date the pensioner qualified for the pension and not from the date he died. The classification does not rest on substantial distinction. It lumps all those marriages contracted within three years before the pensioner qualified for the pension as having contracted within three years before the pensioner qualified for the pension as having been contracted primarily for financial convenience. CHAVEZ vs. COMELEC 437 SCRA 415 (2004) Comelec resolution n. 6520, sec. 32, states: All propaganda materials such as posters, streamers, stickers or paintings on walls and other materials showing the picture, image or name of a person, and all advertisements on print, in radio or on television showing the image or mentioning the name of a person, who subsequent to the placement or display thereof becomes a candidate for public office shall be immediately removed by said candidate and radio station, print media or television within 3 days after the afectivity of these implementing rules. Issue: is the resolution a valid exercise of police power? Held: Yes. To determine the validity of a police power measure, two questions must be asked: 1. Does the interest of the public in general, as distinguished fron those of a particular class, require the exercise of police power? 2. Are the means employed reasonably necessary for the accomplishment of the purpose and not unduly oppressive upon individuals? A close examination of the assailed provision reveals that its primary objectives are to prohibit premature campaigning and to level the playing

ACADCOM 2010; Contributors: Gene Geocaniga, Jarissa Guiani, Darlene Magabilen TAU MU Page 30 of 179

field for candidates of public office, to equalize the situation of rich candidates and poorer candidates by preventing the former from enjoying undue advantage in exposure and publicity. The latter is valid reason for the exercise of police power. CHAVEZ vs. ROMULO 431 SCRA 430 (2004)

TAU MU TAU MU TAU MU TAU MU TAU MU TAU MU

Facts: The law concerns the phasing out of dilapidated taxicabs. The lawful subject here is public welfare. The lawful method is to phase out old taxis to ensure public safety. Taxicab owners and operators challenged this by saying that it afects their property rights. Held: The court held that the law is not unduly oppressive, therefore valid. Six years cut-of time is sufficient for the owners to have substantial return of their investment.

TAU MU

TAXICAB VS BOT

TAU MU

Held: The ordinance regulating motels is not a violation of due process as this consists merely a manifestation of police power measure to safeguard public morals. Similarly, in another case (Ericta vs. City), the prohibition to check in for more than 24 hours in a motel is valid exercise of police power, the purpose being to promote good morals.

TAU MU TAU MU

ERMITA vs. CITY MAYOR 20 SCRA 849 (1967)

The Fraternal Ateneo de Davao

TAU MU TAU MU

Facts: On January 31, 2003, the Chief of the Philippine National Police issued the Guidelines in the Implementation of the Ban on the Carrying of Firearms Outside of Residence (PTCFOR). Among others, the guidelines suspended the issuance of permits to carry firearms outside of one’s residence. Does the Constitution protect the right to bear arms? Does the ban on carrying firearms constitute deprivation of property without due process? Held: The right to bear arms is a mere statutory privilege, not a constitutional right. The Philippine Constitution contains no provision similar to the Second Amendment. Similarly, a license to permit authorizing a person to enjoy certain privileges is neither a property nor a property right. Just like ordinary licenses in other regulatory fields, a license to carry firearm may be revoked anytime. It does not confer an absolute right, but only personal privilege to be exercised under existing restrictions, ad such as may thereafter be reasonably imposed. Even if it is assumed that a PTCFOR constitutes a property right protected by the Constitution, the same cannot be considered as absolute to be placed beyond the reach of the State’s police power. All property in the State is held subject to its general regulations, necessary to the common good and general welfare.

KITY

TAU MU TAU MU TAU MU TAU MU TAU MU TAU MU

CONSTITUTIONAL LAW II Order of Saint Thomas More Atty. Philip John Pojas/Atty. Rovyne G. Jumao-as, RN University College of Law

TOMAS VELASCO vs. HON. ANTONIO J. VILLEGAS Facts: This is an appeal from an order of the lower court dismissing a suit for declaratory relief challenging the constitutionality based on Ordinance No. 4964 of the City of Manila, the contention being that it amounts to a deprivation of property of petitioners-appellants of their means of livelihood without due process of law. The assailed ordinance is worded thus: "It shall be prohibited for any operator of any barber shop to conduct the business of massaging customers or other persons in any adjacent room or rooms of said barber shop, or in any room or rooms within the same building where the barber shop is located as long as the operator of the barber shop and the room where massaging is conducted is the same person." The issue in the case is whether or not the ordinance is unconstitutional. Held: Even if such were not the case, the attack against the validity cannot succeed. As pointed out in the brief of respondents-appellees, it is a police power measure. The objectives behind its enactment are: "(1) To be able to impose payment of the license fee for engaging in the business of massage clinic under Ordinance No. 3659 as amended by Ordinance 4767, an entirely diferent measure than the ordinance regulating the business of barbershops and, (2) in order to forestall possible immorality which might grow out of the construction of separate rooms for massage of customers." 3 This Court has been most liberal in sustaining ordinances based on the general welfare clause. As far back as U.S. v. Salaveria, 4 a 1918 decision, this Court through Justice Malcolm made clear the significance and scope of such a clause, which "delegates in statutory form the police power to a municipality. As above stated, this clause has been given wide application by municipal authorities and has in its relation to the particular circumstances of the case been liberally construed by the courts. Such, it is well to really is the progressive view of Philippine jurisprudence." 5 As it was then, so it has continued to be. 6 There is no showing, therefore, of the unconstitutionality of such ordinance. WHEREFORE, the appealed order of the lower court is affirmed. No costs. CITY OF MANILA vs. LAGUIO, JR. 455 SCRA 308 (2005)

TAU MU

Facts: On March 30, 1993, the City of Manila approved an ordinance prohibiting and person from operating in the Ermita-Malate area “any business providing certain forms of amusement, entertainment, services and facilities where women are used as tools in entertainment,” including sauna parlors, beerhouses, discoteques, cabarets, dance halls, motels and inns. Those already engaged in said business were given three months to wind up their operations or transfer elsewhere. Issue: Is the ordinance a valid exercise of police power?

ACADCOM 2010; Contributors: Gene Geocaniga, Jarissa Guiani, Darlene Magabilen TAU MU Page 31 of 179

TAU MU TAU MU TAU MU TAU MU TAU MU TAU MU

Facts: This concerns the Generic Act which was quite an issue in the medical world. The Act requires doctors to prescribe medicine using generic names and not to use brand names, which was the common practice before the passage of the law. Doctors contended that the law burdened their freedom to contract. The lawful subject is health. The Supreme Court also cited some constitutional policies wherein the State will promote health and make it afordable to the people. Held: The court held that it is too hard to challenge the presumption of validity. And since it has constitutional basis, the more it would be difficult to challenge. With regards the doctor’s

TAU MU

DEL ROSARIO vs. BENGZON 180 SCRA 521 (1989)

TAU MU

Held: The court held that the fee paid to enter movie houses represents property rights. The theater owners should have the right to sell their tickets in whatever way they want to. Interference on the right to sell tickets was not validated. As to #2 and #3, the court ruled them to be valid because they afect public safety which fell within the scope of police power. What is being avoided is the overcrowding of movie houses. They comply with Substantive Due Process.

TAU MU

Facts: The ordinance: 1) reduced by ½ the fee for children aged 7-12 years old to enter into a movie house; 2) prohibited theater owners from selling tickets beyond the seating capacity; 3) prohibited owners from selling a ticket to two persons. The lawful subject used as basis is general welfare. What is burdened is the property right of the theater owners.

TAU MU TAU MU

BALACUIT vs. CFI 163 SCRA 182 (1988)

The Fraternal Ateneo de Davao

TAU MU TAU MU

Held: The promotion of morals is undoubtedly one of the fundamental duties of the City of Manila. However, the worthy aim of fostering public morals and the eradication of the community’s social ills can be achieved through means less restrictive of private rights. It can be attained by reasonable restrictions rather than by absolute prohibition. The closing down and transfer of businesses or their conversion into businesses “allowed” under the ordinance have no reasonable relation to the accomplishment of its purposes. Otherwise stated, the prohibition of the enumerated establishments will not per se protect and promote the social and moral welfare of the community; it will not in itself eradicate the alluded social ills of prostitution, adultery, fornication nor will it arrest the spread of sexual disease in Manila. The enumerated establishments are lawful pursuits which are not per se ofensive to the moral welfare of the community. The City Council instead should regulate human conduct that occurs inside the establishments.

KITY

TAU MU TAU MU TAU MU TAU MU TAU MU TAU MU

CONSTITUTIONAL LAW II Order of Saint Thomas More Atty. Philip John Pojas/Atty. Rovyne G. Jumao-as, RN University College of Law

argument, the court said that there is no contract between the doctor and the patient. In fact, the patient can refuse to buy the medicine prescribed by his doctor. Therefore the freedom of contract is not involved here. In fact, there seems to be no case wherein the doctors sued his patient for not following his prescription. DEPARTMENT vs. SAN DIEGO 180 SCRA 533 (1989) Facts: San Diego challenged the NMAT rule that one cannot take a medical course after failing the test three times. The lawful subject here is health. The rational relation is that the medical profession is intimately related to life and health and is impressed with public interest. Restriction of the medical profession is related to health (because doctors promote health). What is being burdened here according to San Diego is the right to education. Held: The court held that the right to education is not unduly burdened because if a person fails the NMAT three times, it seems that he is not fit to be in the medical profession at all.

YNOT VS IAC Facts: Pres. Marcos promulgated an EO which prohibited the inter-provincial movement of carabaos and carabeefs. Anyone caught violating the law will have his carabao and carabeef confiscated. The purpose of the law is to prevent indiscriminate slaughter of carabaos so that they could be preserved for agricultural use by the farmers. The lawful subject is general welfare since the Philippines is an agricultural country. The farmers need them for farming instead of using machineries. (The country was in an energy crisis) Held: The court held that there was a lawful subject which is general welfare. However, the method chosen by the government has no logical connection with the purpose of the law. Prohibiting the inter-provincial transfer of carabaos would not prevent their indiscriminate slaughter. The carabaos can still be killed anywhere without even transferring them. CORONA VS. UNITED HARBOR PILOTS ASSOCIATION OF THE PHIL Facts: Respondents were holders of a pilot license. An order was issued by Dayan, general manager of the Phil Ports Authority, which provided that license previously issued is efective only until Dec 31, 1992, subject to a yearly renewal after a rigid evaluation of performance. This was questioned by the pilots because it deprives them of their property without due process. Held: On procedural due process – no violation

ACADCOM 2010; Contributors: Gene Geocaniga, Jarissa Guiani, Darlene Magabilen TAU MU Page 32 of 179



Notice and hearing are essential only when an administrative body exercises quasi-judicial function not when it exercises quasi-legislative function such as the issuance of rules and regulation.

On substantive due process – there is violation  The fact that renewal of licenses are subject to a rigid evaluation violates right to property because it would mean that your license may not be renewed if you do not pass the evaluation.

TAU MU TAU MU TAU MU TAU MU TAU MU TAU MU TAU MU

Held: The scope of the function of MMDA as an administrative, coordinating and policy-setting body has been settled in Metropolitan Manila Development Authority (MMDA) v. Bel-Air Village Association, Inc.41 In that case, the Court stressed clearly, the scope of the MMDAs function is limited to the delivery of the seven (7) basic services. One of these is transport and traffic management which includes the formulation and monitoring of policies, standards and projects to rationalize the existing transport operations, infrastructure requirements, the use of thoroughfares and promotion of the safe movement of persons and goods. It also covers the mass transport system and the institution of a system of road regulation, the administration of all traffic enforcement operations, traffic engineering services and traffic education programs, including the institution of a single ticketing system in Metro Manila for traffic violations. Under this service, the MMDA is expressly authorized to "to set the policies concerning traffic" and "coordinate and regulate

TAU MU

Issues: the issues were narrowed down to whether 1) the MMDA�s power to regulate traffic in Metro Manila included the power to direct provincial bus operators to abandon and close their duly established and existing bus terminals in order to conduct business in a common terminal; (2) the E.O. is consistent with the Public Service Act and the Constitution.

TAU MU

The first assailed Order of September 8, 2005,2 which resolved a motion for reconsideration filed by herein respondents, declared Executive Order (E.O.) No. 179, hereafter referred to as the E.O., "unconstitutional as it constitutes an unreasonable exercise of police power." The second assailed Order of November 23, 20053 denied petitioner’s motion for reconsideration.

TAU MU TAU MU

The present petition for review on certiorari, rooted in the traffic congestion problem, questions the authority of the Metropolitan Manila Development Authority (MMDA) to order the closure of provincial bus terminals along Epifanio de los Santos Avenue (EDSA) and major thoroughfares of Metro Manila.

The Fraternal Ateneo de Davao

TAU MU TAU MU

THE METROPOLITAN MANILA DEVELOPMENT AUTHORITY VS. VIRON TRANSPORTATION CO., Inc.

KITY

TAU MU TAU MU TAU MU TAU MU TAU MU TAU MU

CONSTITUTIONAL LAW II Order of Saint Thomas More Atty. Philip John Pojas/Atty. Rovyne G. Jumao-as, RN University College of Law

the implementation of all traffic management programs." In addition, the MMDA may install and administer a single ticketing system," fix, impose and collect fines and penalties for all traffic violations. Even then, for reasons which bear reiteration, the MMDA cannot order the closure of respondents� terminals not only because no authority to implement the Project has been granted nor legislative or police power been delegated to it, but also because the elimination of the terminals does not satisfy the standards of a valid police power measure. No matter how noble the intentions of the MMDA may be then, any plan, strategy or project which it is not authorized to implement cannot pass muster In the second issue the court ruled that the establishment, as well as the maintenance of vehicle parking areas or passenger terminals, is generally considered a necessary service to be provided by provincial bus operators like respondents, hence, the investments they have poured into the acquisition or lease of suitable terminal sites. Eliminating the terminals would thus run counter to the provisions of the Public Service Act. Finally, an order for the closure of respondents� terminals is not in line with the provisions of the Public Service Act. Romeo Gerochi vs. Department of Energy Facts: Petitioners Romeo P. Gerochi, Katulong Ng Bayan (KB), and Environmentalist Consumers Network, Inc. (ECN) (petitioners), come before this Court in this original action praying that Section 34 of Republic Act (RA) 9136, otherwise known as the “Electric Power Industry Reform Act of 2001” (EPIRA), imposing the Universal Charge, and Rule 18 of the Rules and Regulations (IRR) which seeks to implement the said imposition, be declared unconstitutional. Petitioners also pray that the Universal Charge imposed upon the consumers be refunded and that a preliminary injunction and/or temporary restraining order (TRO) be issued directing the respondents to refrain from implementing, charging, and collecting the said charge. The universal charge shall be a non-bypassable charge which shall be passed on and collected from all end-users on a monthly basis by the distribution utilities. Issues : Whether or not, the Universal Charge imposed under Sec. 34 of the EPIRA is a tax; and Whether or not there is undue delegation of legislative power to tax on the part of the ERC. Ruling: Evidently, the establishment and maintenance of the Special Trust Fund, under the last paragraph of Section 34, R.A. No. 9136, is well within the pervasive and non-waivable power and responsibility of the government to secure the physical and economic survival and wellbeing of the community, that comprehensive sovereign authority we designate as the police power of the State. This feature of the Universal Charge further boosts the position that the same

ACADCOM 2010; Contributors: Gene Geocaniga, Jarissa Guiani, Darlene Magabilen TAU MU Page 33 of 179

TAU MU TAU MU TAU MU TAU MU TAU MU TAU MU TAU MU TAU MU

There is substantial distinction between aliens and Filipinos since alien’s loyalty to the country is transitory in nature which is motivated only by some kind of personal interest. The purpose of the law is to prevent aliens from engaging in business in the Philippines. It is observed that there is alien dominance in retail trade. That is the reason why the law intends to give it back to the Filipinos. It also applies to all members of the class, except to Americans, since they are covered by the Parity Amendment found in the 1935 Constitution. This provision authorizes Americans

TAU MU

ICHIONG VS. HERNANDEZ

TAU MU TAU MU

EQUAL PROTECTION

The Fraternal Ateneo de Davao

TAU MU TAU MU

is an exaction imposed primarily in pursuit of the State's police objectives. The STF reasonably serves and assures the attainment and perpetuity of the purposes for which the Universal Charge is imposed, i.e., to ensure the viability of the country's electric power industry. Ergo, Universal Charge is considered a tax. The second issue governs the principle of separation of powers ordains that each of the three branches of government has exclusive cognizance of and is supreme in matters falling within its own constitutionally allocated sphere. A logical corollary to the doctrine of separation of powers is the principle of non-delegation of powers In the face of the increasing complexity of modern life, delegation of legislative power to various specialized administrative agencies is allowed as an exception to this principle. All that is required for the valid exercise of this power of subordinate legislation is that the regulation be germane to the objects and purposes of the law and that the regulation be not in contradiction to, but in conformity with, the standards prescribed by the law. These requirements are denominated as the completeness test and the sufficient standard test. Under the first test, the law must be complete in all its terms and conditions when it leaves the legislature such that when it reaches the delegate, the only thing he will have to do is to enforce it. The second test mandates adequate guidelines or limitations in the law to determine the boundaries of the delegate's authority and prevent the delegation from running riot. The Court finds that the EPIRA, read and appreciated in its entirety, in relation to Sec. 34 thereof, is complete in all its essential terms and conditions, and that it contains sufficient standards. Finally, every law has in its favor the presumption of constitutionality, and to justify its nullification, there must be a clear and unequivocal breach of the Constitution and not one that is doubtful, speculative, or argumentative. Indubitably, petitioners failed to overcome this presumption in favor of the EPIRA. We find no clear violation of the Constitution which would warrant a pronouncement that Sec. 34 of the EPIRA and Rule 18 of its IRR are unconstitutional and void.

KITY

TAU MU TAU MU TAU MU TAU MU TAU MU TAU MU

CONSTITUTIONAL LAW II Order of Saint Thomas More Atty. Philip John Pojas/Atty. Rovyne G. Jumao-as, RN University College of Law

to engage in all trade in the country. The classification also has relevance to the lawful subject and method. PASEI VS DRILON What was questioned was the order by the DOLE which prohibited temporarily the sending of female domestic abroad. The law was challenged as violative of the equal protection clause since it singled out domestic helpers who are female only. Held: The SC ruled that there is valid classification since it is the females who experience the various abuses abroad. The female domestic workers are those who live with their employers, unlike entertainers, and it is their houses where most abuses occur. They are very susceptible to maltreatment. The ban is germane to the purpose of the law which is to protect them. The law applies to all members of the class. It will apply as long as there are female domestic workers abroad. NUÑEZ VS SANDIGANBAYAN Nuñez claimed that the PD minimizes the constitutional rights of the public officers. What the law imposes is that public officials are to be tried by the Sandiganbayan for cases concerning their position or office. If an appeal is made by the official, it goes straight to the SC unlike ordinary citizens, whose cases can still be appealed in the CA. Nuñez claimed that the law is discriminatory since public officials have only one chance to appeal (certiorari), while others have more remedies. It should be noted that the SC only deals with appeals with questions of law. Held: The court held that the law did not violate the equal protection clause. There is a substantial distinction since public officers occupy a position diferent from others: Public office is a public trust. It is also germane to the purpose of the law. It can also apply inefinitely and it applies to all members of the class. ORMOC SUGAR COMPANY VS TREASURER An ordinance was passed imposing tax specifically on sugar produced by Ormoc Sugar Co. It violated the equal protection clause because there is a possibility that somebody might put up a new sugar company. In efect, the law will still apply singly to Ormoc Sugar Company because it was specifically sated so. Held: The ordinance is a violation of the equal protection clause. The law was limited to existing conditions only. Although Ormoc Sugar was the only sugar miller at that time, the ordinance should also cover other future sugar millers. The ordinance should have stated that all sugar mills in the municipality should be taxed so that anybody who comes in and starts the same

ACADCOM 2010; Contributors: Gene Geocaniga, Jarissa Guiani, Darlene Magabilen TAU MU Page 34 of 179

business can now be covered by the ordinance. There was no valid classification. The court held that the ordinance violated the equal protection clause. It did not satisfy requirement #3. The law was only limited to existing conditions. Although Ormoc Sugar was the only sugar miller at the time, the ordinance should also cover for other future sugar millers. Presently, the law does not cover other future millers. VILLEGAS VS. HIU CHIONG

TAU MU TAU MU TAU MU TAU MU TAU MU

Petitioner is a policeman charged with murder and attempted murder. Upon acceptance of the case, the trial court suspended him until the case is terminated by virtue of the provision of RA 6975 sec. 47 (LGC). If you are a policeman

TAU MU

HIMAGAN VS. PEOPLE

TAU MU

Held: The court ruled in favor of Chua, the law violates the equal protection clause. However, the court did not declare the law null and void for being unconstitutional. Usually, it will. The court merely explained the law in another manner, it injected contractual workers as included in the law. There is no substantial distinction between a co-terminous employee and a casual employee when in fact the former should enjoy better treatment than the casuals since they are in plantilla.

TAU MU

In 1988, in order to bring about the reorganization in the government, an early retirement law was enacted. The law covered regular, temporary casual and emergency employees who have rendered at least 2 years of consecutive service. Chua, who was hired and rehired 4 times, has worked with the government continuously for 15 years as a contractual employee. And she applied for early retirement. She was refused the benefits of the law because under existing civil service law she was classified as a contractual, and her tenure is co-terminous with the project. She was not considered as included in the law. She contested this being violative of the equal protection clause having no substantial distinction among casual temporary, emergency and contractual workers.

TAU MU

CHUA VS. CIVIL SERVICE COMMISSION

TAU MU TAU MU

Held: The court held that it violated the equal protection clause because it did not indicate as to the particular class of aliens who are required to pay for working permits. The ordinance was applicable to all, whether the alien is a lowly worker or a highly paid executive or a rich businessman.

The Fraternal Ateneo de Davao

TAU MU TAU MU

An ordinance was passed imposing a P50.00 working fee for permits to aliens who wish to work in Manila. This was a revenue raising program of the City of Manila.

KITY

TAU MU TAU MU TAU MU TAU MU TAU MU TAU MU

CONSTITUTIONAL LAW II Order of Saint Thomas More Atty. Philip John Pojas/Atty. Rovyne G. Jumao-as, RN University College of Law

charged with an ofense whose penalty is 6yrs and 1 day or more, you will be suspended from the service until the termination of the case. According to the petitioner, the 90-day suspension under the Civil Service Law should apply because allegedly RA 6975 unduly prejudices members of the PNP described above. Held: Petitioners allegation is misplaced because the Civil Service Law applies to administrative cases. Sec 47 of LGC, a later enactment, repealed this part of the CSL. Besides on the aspect of equal protection, there is a valid distinction between members of the PNP and other accused because PNP members charged with serious ofenses might influence the victims and witnesses. Police officers carry weapon and the badge of the law which can be used to harass witnesses. Members of the PNP are diferent from other accused by reason of the position they occupy. ALMONTE V. VASQUEZ Petitioners allege that the procedure which allows the Office of the Ombudsman to investigate anonymous complaints violates the equal protection clause. They point out that in all forums and tribunals, aggrieved parties can only hale respondents via verified complaints or sworn statements with their identities fully disclosed. Held: There can be no objection to the procedure because the Constitution itself authorizes it. Art. XI, Sec. 12. of the Constitution requires the Ombudsman and his deputies to act promptly on complaints “filed in any form or manner” against public officials or government employees. In addition, the Office of the Ombudsman is diferent from other agencies because those subject to its jurisdiction are public officials, who, through official pressure and influence, can quash, delay or dismiss investigations held against them. TIU V. COURT OF APPEALS Pursuant to R.A. No. 7227, the President issued A.O. No. 97-A grading tax and duty incentives only to businesses and residents within the “secured area” of the Subic Special Economic Zone. Does the administrative order violate the equal protection clause? Held: No. The classification rests on a substantial distinction which is germane to the purpose law. R.A. No. 7227 aims primarily to accelerate the conversion of military reservations into productive uses. Obviously, the “lands covered under the 1947 Military Base Agreement” are its object. It is this specific area which the government intends to transform and develop from its status quo ante as an abandoned naval facility into a self-sustaining industrial and commercial zone, particularly for big foreign and local investors to use as

ACADCOM 2010; Contributors: Gene Geocaniga, Jarissa Guiani, Darlene Magabilen TAU MU Page 35 of 179

operational bases. The bias for big investors may be justified because they are the ones who can pour huge investments to spur economic growth. INTERNATIONAL V. QUISUMBING

TAU MU TAU MU

LACSON V. EXECUTIVE Chief Superintendent Lacson of the Philippine National Police was charged as an accessory in a multiple murder case filed before the Sandiganbayan. Under R.A No. 7975, said court has jurisdiction where one of the” principal accused” has the rank of Chief Superintendent or higher. Since Lacson was charged as a mere accessory, he moved to quash the information for lack of jurisdiction. While the motion was pending, the Congress enacted R.A. No. 8249 which deleted the word “principal”. The amendment was made to apply to all pending cases over which trial had not begun. Is the law discriminatory? Held: No. the classification between those pending cases involving public officials and those whose trials have not commended as of the approval of the law rests on substantial distinction that makes real diferences. In the first instance, evidence against them were not yet presented, whereas in the latter parties had already submitted their respective proofs. Thus, accuse cannot claim that the law placed them under a diferent category form those similarly situated as they are. DE GUZMAN V. COMELEC Sec 44 of RA no. 8189, otherwise known as “The Voter’s Registration Act” states: No Election Officer shall hold office in a particular city or municipality for more than 4 years. Any election officer who, either at the time of the approval of this Act or subsequent thereto, has served for at least 4 years in a particular city or municipality shall automatically be reassigned by the Commission to a new station outside the original congressional district. Does it violate the equal protection clause?

TAU MU TAU MU TAU MU TAU MU TAU MU

Held: never has the call of a particular duty lifted a prisoner into a diferent classification from those others who are validity restrained by law. The performance of a legitimate and even essential duties by public officers has never been an excuse to free a person in prison. After all , accused is only one of 250 members of the House of Representatives, which can continue to function without him. Election to the position of Congressman is not a reasonable classification in criminal law enforcement. The functions and duties of the office are not substantial distinctions which remove him from the class of prisoners restricted in their liberty of movement.

TAU MU

Accused is a full- fledged member of Congress who is now confined at the national penitentiary while his convictions for statutory rape on two counts acts of lasciviousness are pending appeal. Accused filed a motion asking that he be allowed to fully discharge the duties of a Congressman, including attendance at legislative sessions and committee meetings despite his having been convicted of a nonbailable ofense. Does being an elective official result in a substantial distinction that allows a diferent treatment?

TAU MU

PEOPLE V. JALOSLOS

TAU MU TAU MU

Held: There is no reasonable distinction between the services rendered by foreign-hires and local hires. Persons who work with substantially equal qualifications, efort and responsibility, under similar conditions, should be paid similar salaries. If an employer accords employees the same position and rank, the presumption is that these employees perform equal work. There is no evidence that foreign-hires perform 25% more efficiently than local hires. Both groups have similar functions and responsibilities, which they perform under similar working conditions. The “dislocation factor” and their limited tenure cannot serve as valid basis for the distinction in salary rate. They are adequately compensated by certain benefits such as housing, transportation, taxes and home leave travel allowance.

The Fraternal Ateneo de Davao

TAU MU TAU MU

International School is a domestic educational institution established for dependents of foreign diplomatic personnel. As much, it is authorized to employ aliens and these personnel are exempt from employment laws. Foreign-hires receive benefits not accorded to local hires including housing, transportation, taxes as well as a 25% higher salary rates to ofset the “dislocation factor.” The labor union is now challenging the salary rate diference on the ground that it is discriminatory.

KITY

TAU MU TAU MU TAU MU TAU MU TAU MU TAU MU

CONSTITUTIONAL LAW II Order of Saint Thomas More Atty. Philip John Pojas/Atty. Rovyne G. Jumao-as, RN University College of Law

Held: The singling out of election officers in order to “ensure the impartiality of election officials by preventing them form developing familiarity with the people of their place of assignment” does not violate the equal protection clause. It can be discerned that the legislature thought the noble purpose of the law would be served by breaking an important link in the chain of corruption than by breaking up each and every link thereof. Election officers are the highest officials or authorized representatives of the COMELEC in the city or municipality. It is safe to say that without the complicity of such officials, large scale anomalies in the registration of voters can hardly be carried out. ELENA P. DYCAICO vs SOCIAL SECURITY SYSTEM

ACADCOM 2010; Contributors: Gene Geocaniga, Jarissa Guiani, Darlene Magabilen TAU MU Page 36 of 179

TAU MU TAU MU TAU MU TAU MU TAU MU TAU MU TAU MU TAU MU TAU MU

Ruling: The classification of dependent spouses on the basis of whether their respective marriages to the SSS member were contracted prior to or after the latter’s retirement for the purpose of entitlement to survivor’s pension does not rest on real and substantial distinctions. It is arbitrary and discriminatory. It is too sweeping because the proviso “as of the date of his retirement,” which efectively disqualifies the dependent spouses whose respective marriages to the retired SSS member were contracted after the latter’s retirement as primary beneficiaries, unfairly lumps all these marriages as sham relationships or were contracted solely for the purpose of acquiring benefits accruing upon the death of the other spouse. The proviso thus unduly prejudices the rights of the legal surviving spouse, like the petitioner, and defeats the avowed policy of the law “to provide meaningful protection to members and their beneficiaries against the hazards of disability, sickness, maternity, old age, death, and other contingencies resulting in loss of income or financial burden.” In the petitioner’s case, for example, she asserted that when she and Bonifacio got married in 1997, it was merely to legalize their relationship and not to commit

TAU MU TAU MU

Issue: Whether or not Rep. Act 8282 is violative of equal protection and due process claus and the petitioner qualified to the benefits.

The Fraternal Ateneo de Davao

TAU MU TAU MU

Facts: Bonifacio S. Dycaico became a member of the SSS on January 24, 1980. In his selfemployed data record (SSS Form RS-1), he named the petitioner, Elena P. Dycaico, and their eight children as his beneficiaries. At that time, Bonifacio and Elena lived together as husband and wife without the benefit of marriage. In June 1989, Bonifacio was considered retired and began receiving his monthly pension from the SSS. He continued to receive the monthly pension until he passed away on June 19, 1997. A few months prior to his death, however, Bonifacio married the petitioner on January 6, 1997. Shortly after Bonifacio’s death, the petitioner filed with the SSS an application for survivor’s pension. Her application, however, was denied on the ground that under Section 12B(d) of Republic Act (Rep. Act) No. 8282 or the Social Security Law she could not be considered a primary beneficiary of Bonifacio as of the date of his retirement. The said proviso reads: Sec. 12-B. Retirement Benefits. – (d) Upon the death of the retired member, his primary beneficiaries as of the date of his retirement shall be entitled to receive the monthly pension. … Applying this proviso, the petitioner was informed that the records show that the member [referring to Bonifacio] was considered retired on June 5, 1989 and monthly pension was cancelled upon our receipt of a report on his death on June 19, 1997. In your death claim application, submitted marriage contract with the deceased member shows that you were married in 1997 or after his retirement date; hence, you could not be considered his primary beneficiary.

KITY

TAU MU TAU MU TAU MU TAU MU TAU MU TAU MU

CONSTITUTIONAL LAW II Order of Saint Thomas More Atty. Philip John Pojas/Atty. Rovyne G. Jumao-as, RN University College of Law

fraud. This claim is quite believable. After all, they had been living together since 1980 and, in fact, during that time their eldest child was already twenty-four (24) years old. However, the petitioner was not given any opportunity to prove her claim that she was Bonifacio’s bona fide legal spouse as she was automatically disqualified from being considered as his primary beneficiary. In efect, the petitioner was deprived of the survivor’s benefits, a property interest, accruing from the death of Bonifacio without any opportunity to be heard. Finally, the Court concedes that the petitioner did not raise the issue of the validity of the proviso “as of the date of his retirement” in Section 12-B(d) of Rep. Act No. 8282. The rule is that the Court does not decide questions of a constitutional nature unless absolutely necessary to a decision of the case.[29] However, the question of the constitutionality of the proviso is absolutely necessary for the proper resolution of the present case. Accordingly, the Court required the parties to present their arguments on this issue and proceeded to pass upon the same in the exercise of its equity jurisdiction and in order to render substantial justice to the petitioner who, presumably in her advanced age by now, deserves to receive forthwith the survivor’s pension accruing upon the death of her husband. JAMES MIRASOL vs DEPARTMENT OF PUBLIC WORKS AND HIGHWAYS The facts: The petition sought the declaration of nullity of the following administrative issuances for being inconsistent with the provisions of Republic Act 2000, entitled “Limited Access Highway Act” enacted in 1957: 1. Previously, pursuant to its mandate under R.A. 2000, DPWH issued on June 25, 1998 Department Order (DO) No. 215 declaring the Manila-Cavite (Coastal Road) Toll Expressway as limited access facilities. 2. Accordingly, petitioners filed an Amended Petition on February 8, 2001 wherein petitioners sought the declaration of nullity of the aforesaid administrative issuances. Moreover, petitioners prayed for the issuance of a temporary restraining order and/or preliminary injunction to prevent the enforcement of the total ban on motorcycles along the entire breadth of North and South Luzon Expressways and the Manila-Cavite (Coastal Road) Toll Expressway under DO 215. 3. And AO1 which limited access highways, AO1 States that it is unlawful for any person or group of persons to: Drive any bicycle, tricycle, pedicab, motorcycle or any vehicle (not motorized); 4. On July 18, 2001, the DPWH acting thru the TRB, issued Department Order No. 123 allowing motorcycles with engine displacement of 400 cubic centimeters inside limited access facilities (toll ways).

ACADCOM 2010; Contributors: Gene Geocaniga, Jarissa Guiani, Darlene Magabilen TAU MU Page 37 of 179

Issues: WHETHER DO 74 DO 215 AND THE TRB REGULATIONS CONTRAVENE RA 2000; AND WHETHER AO 1 AND DO 123 ARE UNCONSTITUTIONAL. Ruling: First Issue

The Fraternal Ateneo de Davao

TAU MU TAU MU TAU MU TAU MU TAU MU TAU MU TAU MU TAU MU TAU MU TAU MU TAU MU TAU MU TAU MU

DPWH issued DO 74 and DO 215 declaring certain expressways as limited access facilities on 5 April 1993 and 25 June 1998, respectively. Later, the TRB, under the DPWH, issued the Revised Rules and Regulations on Limited Access Facilities. However, on 23 July 1979, long before these department orders and regulations were issued, the Ministry of Public Works, Transportation and Communications was divided into two agencies – the Ministry of Public Works and the Ministry of Transportation and Communications – by virtue of EO 546. The question is, which of these two agencies is now authorized to regulate, restrict, or prohibit access to limited access facilities. Even under Executive Order No. 125 (EO 125) and Executive Order No. 125-A (EO 125-A), which further reorganized the DOTC, the authority to administer and enforce all laws, rules and regulations relative to transportation is clearly with the DOTC. Thus, DO 74 and DO 215 are void because the DPWH has no authority to declare certain expressways as limited access facilities. Under the law, it is the DOTC which is authorized to administer and enforce all laws, rules and regulations in the field of transportation and to regulate related activities. Since the DPWH has no authority to regulate activities relative to transportation, the TRB cannot derive its power from the DPWH to issue regulations governing limited access facilities. The DPWH cannot delegate a power or function which it does not possess in the first place. Since DO 74 and DO 215 are void, it follows that the rules implementing them are likewise void. On the second issue DO 123 is unconstitutional on the ground that it violates the equal protection clause by allowing only motorcycles with at least 400 cubic centimeters engine displacement to use the toll ways. The RTC reasoned that the creation of a distinction within the class of motorcycles was not based on real diferences. The RTC’s Decision dated 10 March 2003 declared DO 123 unconstitutional on the ground that it violates the equal protection clause by allowing only motorcycles with at least 400 cubic centimeters engine displacement to use the toll ways. The RTC reasoned that the creation of a distinction within the class of motorcycles was not based on real diferences. We need not pass upon the constitutionality of the classification of motorcycles under DO 123. As previously discussed, the DPWH has no authority to regulate limited access highways since EO 546 has

KITY

TAU MU TAU MU TAU MU TAU MU TAU MU TAU MU

CONSTITUTIONAL LAW II Order of Saint Thomas More Atty. Philip John Pojas/Atty. Rovyne G. Jumao-as, RN University College of Law

devolved this function to the DOTC. Thus, DO 123 is void for want of authority of the DPWH to promulgate it. On the other hand, the assailed portion of AO 1 states it is unlawful for any person or group of persons to drive any bicycle, tricycle, pedicab, motorcycle or any vehicle (not motorized). They also claim that AO 1 introduces an unreasonable classification by singling-out motorcycles from other motorized modes of transport. Finally, petitioners argue that AO 1 violates their right to travel. The SC ruled on the negative. A toll way is not an ordinary road. As a facility designed to promote the fastest access to certain destinations, its use, operation, and maintenance require close regulation. Public interest and safety require the imposition of certain restrictions on toll ways that do not apply to ordinary roads. As a special kind of road, it is but reasonable that not all forms of transport could use it. The right to travel does not mean the right to choose any vehicle in traversing a toll way. The right to travel refers to the right to move from one place to another. Petitioners can traverse the toll way any time they choose using private or public four-wheeled vehicles. Petitioners are not denied the right to move from Point A to Point B along the toll way. Petitioners are free to access the toll way, much as the rest of the public can. The mode by which petitioners wish to travel pertains to the manner of using the toll way, a subject that can be validly limited by regulation. Finally, petitioners assert that their possession of a driver’s license from the Land Transportation Office (LTO) and the fact that their vehicles are registered with that office entitle them to use all kinds of roads in the country. Again, petitioners are mistaken. There exists no absolute right to drive. On the contrary, this privilege, is heavily regulated. Only a qualified group is allowed to drive motor vehicles: those who pass the tests administered by the LTO. A driver’s license issued by the LTO merely allows one to drive a particular mode of transport. It is not a license to drive or operate any form of transportation on any type of road. Vehicle registration in the LTO on the other hand merely signifies the roadworthiness of a vehicle. This does not preclude the government from prescribing which roads are accessible to certain vehicles. Therefore, Only AO 1 is constitutional, the rest otherwise. 2nd LT. SALVADOR PARREÑO vs COMMISSION ON AUDIT Facts: Salvador Parreño (petitioner) served in the Armed Forces of the Philippines (AFP) for 32 years. On 5 January 1982, petitioner retired from the Philippine Constabulary with the rank of 2nd Lieutenant. Petitioner availed, and received payment, of a lump sum pension equivalent to three years pay. In 1985, petitioner started

ACADCOM 2010; Contributors: Gene Geocaniga, Jarissa Guiani, Darlene Magabilen TAU MU Page 38 of 179

receiving his monthly pension amounting to P13,680.

TAU MU TAU MU TAU MU TAU MU TAU MU TAU MU TAU MU

There is compliance with all these conditions. There is a substantial diference between retirees who are citizens of the Philippines and retirees who lost their Filipino citizenship by naturalization in another country, such as petitioner in the case before us. The constitutional right of the state to require all citizens to render personal and military service necessarily includes not only private citizens but also citizens who have retired from military service. A retiree who had lost his Filipino citizenship already renounced his allegiance to the state. Thus, he may no longer be compelled by the state to render compulsory military service when the need arises. Petitioner’s loss of Filipino

TAU MU

The constitutional right to equal protection of the laws is not absolute but is subject to reasonable classification. To be reasonable, the classification (a) must be based on substantial distinctions which make real diferences; (b) must be germane to the purpose of the law; (c) must not be limited to existing conditions only; and (d) must apply equally to each member of the class.

TAU MU

Ruling: The jurisdiction of the COA over money claims against the government does not include the power to rule on the constitutionality or validity of laws. The 1987 Constitution vests the power of judicial review or the power to declare unconstitutional a law, treaty, international or executive agreement, presidential decree, order, instruction, ordinance, or regulation in this Court and in all Regional Trial Courts. Petitioner’s money claim essentially involved the constitutionality of Section 27 of PD 1638, as amended. Hence, the COA did not commit grave abuse of discretion in dismissing petitioner’s money claim. Taking into consideration the exclusivity of the jurisdiction of the branches of government the COA holds no jurisdiction over the case. On the second issue

TAU MU TAU MU

Issues: Whether or not there is substantial distinction between retirees who are citizens of the Philippines and retirees who lost there Filipino citizenship by naturalization in another country. And whether or not COA ha jurisdiction to rule on the constitutionality of sec 27 of PD 1638 as amended.

The Fraternal Ateneo de Davao

TAU MU TAU MU

Petitioner migrated to Hawaii and became a naturalized American citizen. In January 2001, the AFP stopped petitioner’s monthly pension in accordance with Section 27 of Presidential Decree No. 1638 (PD 1638), as amended by Presidential Decree No. 1650. Section 27 of PD 1638, as amended, provides that a retiree who loses his Filipino citizenship shall be removed from the retired list and his retirement benefits terminated upon loss of Filipino citizenship. Petitioner requested for reconsideration but the Judge Advocate General of the AFP denied the request. Petitioner filed a claim before the COA for the continuance of his monthly pension.

KITY

TAU MU TAU MU TAU MU TAU MU TAU MU TAU MU

CONSTITUTIONAL LAW II Order of Saint Thomas More Atty. Philip John Pojas/Atty. Rovyne G. Jumao-as, RN University College of Law

citizenship constitutes a substantial distinction that distinguishes him from other retirees who retain their Filipino citizenship. If the groupings are characterized by substantial distinctions that make real diferences, one class may be treated and regulated diferently from another. Petitioner will be entitled to receive his monthly pension should he reacquire his Filipino citizenship since he will again be entitled to the benefits and privileges of Filipino citizenship reckoned from the time of his reacquisition of Filipino citizenship. There is no legal obstacle to the resumption of his retirement benefits from the time he complies again with the condition of the law, that is, he can receive his retirement benefits provided he is a Filipino citizen. Therefore, the petitioner cannot get hi benefits not until he returns to being a Filipino citizen. ARREST WARRANT WEBB VS. DE LEON The NBI filed a complaint before the DOJ for the so called Vizconde massacre indicting Webb and five others. The DOJ formed a panel of prosecutors to conduct preliminary investigation, finding that probable cause exist, information was filed before the RTC of Paranaque under judge Escano. It was judge DE Leon, however, who issued the WA. De Leon was the pairing judge of Esacano. Escano then voluntarily inhibited himself from the case because he had formerly worked with the NBI. The case was reraffled and landed on the sala of judge Tolentino. Tolentino then issued another WA. Petitioners question the issuance of WA because there was allegedly no preliminary investigation conducted and there are inconsistencies on the affidavit of the witness. Held: There is no requirement for a preliminary investigation since personal review of the documents is sufficient. Inconsistencies in the affidavit is immaterial. When the judge determines probable cause, what is required of him is to determine probability not certainty of the guilt. If by all probability they have to be held for trial then WA should be issued. Determination of probability is diferent from determination of guilt. Upon determination of probable caue, guilt or non-guilt should not be the issue. PANGANDAMAN VS. CESAR There was a shooting incident which killed 5 people. After the incident and after preliminary investigation, the Lanao judge issued a warrant for the arrest of 14 individuals and 50 John Does. Not one of the 50 can be identified by the witnesses, but it seems that the number was about the number of people who participated in the shooting. ISSUE: Is a John Doe warrant valid?

ACADCOM 2010; Contributors: Gene Geocaniga, Jarissa Guiani, Darlene Magabilen TAU MU Page 39 of 179

Held: According to the court, a John Doe warrant is void. The Constitution requires that there must be specific description or designation of the person to be arrested. John Does warrants are general warrants. PEOPLE VS. VELOSO

ISSUE: Is this a violation of the Constitution?

TAU MU TAU MU

Held: The court said that the judge need not personally conduct the investigation because it will duly burdensome on the part of the judges if they will be required to do so. They will not have enough time to conduct the trial, which is their main responsibility.

TAU MU

This is the famous libel case filed by former Pres. Aquino against Luis Beltran. Beltran questioned the issuance of the warrant. He said the judge did not “personally examine” the witnesses, which is unconstitutional. In an arrest warrant, the judge may rely on the evidences. He need not call the witnesses.

TAU MU

SOLIVEN VS. MAKASIAR

TAU MU TAU MU

Held: According to the court, this is not a violation of the Constitution because it describes the John Doe. The police can also identify him by the following designation written in the warrant who is the owner of the establishment.

LIM VS. FELIX

TAU MU TAU MU TAU MU

Held: The court ruled that the warrant is void. Before the fiscal can conclude that there is

TAU MU

ISSUE: Whether the judge can issue a warrant based on the certificate alone. Can it be considered valid?

TAU MU

The killing of Congressman Espinosa happened in Masbate. The fiscal conducted a preliminary investigation to find out if there was probable cause to hold the accused for trial. And he found out that there was. During the pendency of the case before the RTC of Masbate, the SC approved a petition for a change in venue to Makati. The judge, on the certificate alone, which was issued by the prosecutor, determined that there was probable cause. The accused challenged the validity of the warrant. He argued that the judge should make a personal determination as to the existence of probable cause to issue the warrant.

The Fraternal Ateneo de Davao

TAU MU TAU MU

A search warrant was issued for an establishment designated as the “Parliamentary Club” which was believed to be a gambling den. The search warrant identified the owner. They also recovered gambling paraphernalia hidden in his pocket. The accused questioned the warrant being a John Doe warrant. There was not exact description of the person to be arrested.

KITY

TAU MU TAU MU TAU MU TAU MU TAU MU TAU MU

CONSTITUTIONAL LAW II Order of Saint Thomas More Atty. Philip John Pojas/Atty. Rovyne G. Jumao-as, RN University College of Law

probable cause, he should first examine all the evidences, documents presented to him, and all of those should be attached on the certificate stating that there is sufficient ground that the accused should be put on trial. In this case, there was nothing attached to the certificate stating that there is sufficient ground that the accused should be put on trial. In this case, there was nothing attached to the certificate. The Constitution underscores that the judge should determine if there is probable cause basing on all the facts and evidences, and not on the certificate alone. In the case of arrest warrant, the judge need not call the witnesses for questioning. He may rely on the record submitted to him by the fiscal. But he must not rely solely on the resolution or certification of the fiscal. The resolution must be supported by other evidence which led to the filing of the case. In the case of a search warrant, the judge must personally examine the witnesses himself to determine probable cause. COJUANGCO V. SANDIGANBAYAN The Ombudsman filed a criminal case for violation of Sec.3 (e) of P.A. No. 3019 against Cojuangco. On the basis of the Resolution of the Panel of Investigators of the Office of the Ombudsman and the Memorandum of the Office of the Special Prosecutor denying the existence of a prejudicial question which would warrant the suspension of the criminal case, the Sandiganbayan issued the warrant of arrest even without the supporting evidence. Is this proper? Held: No. The Sandiganbayan committed grave abuse of discretion when it issued the warrant of arrest without the benefit of the records or evidence supporting the prosecutor’s finding of probable cause. The court failed to abide by the constitutional mandate of personally determining the existence of probable cause before issuing a warrant of arrest. The two documents before the court were the product of somebody else’s determination, insufficient to support a finding of probable cause by the Sandiganbayan. However, by posting bail and filing various motions, accused has waived the defects that attended the issuance of the warrant of arrest. There was voluntary submission to the court’s jurisdiction. ABDULA V. GUIANI In a murder case, accused filed a petition for certiorari and prohibition claiming that the warrant of arrest issued against them was null and void because the judge “did not personally examine the evidence nor did he call the complainant and the witnesses in the face of their incredible accounts.” The judge in his Comment stated that there was “no reason for [him] to doubt the validity of the certification made by the Assistant Prosecutor that a preliminary investigation was conducted and that probable

ACADCOM 2010; Contributors: Gene Geocaniga, Jarissa Guiani, Darlene Magabilen TAU MU Page 40 of 179

cause was found to exist as against those charged in the Information filed,” so that he issued the warrant. Is the warrant valid?

TAU MU TAU MU TAU MU TAU MU TAU MU TAU MU

The Government of the United States of America, through the Department of Justice, filed with the Regional Trial Court a petition for the extradition of Mark Jimenez who was facing various criminal charges in Florida. Petitioner prayed fro his immediate arrest, and the judge set it for hearing. In extradition proceedings, is the potential extraditee entitled to notice and “hearing” before a warrant for his arrest can issue?

TAU MU

GOVERNMENT V. PURGANAN

TAU MU

Held: No. It is the exclusive and personal responsibility of the judge to satisfy himself as to the existence of probable cause before issuing an arrest warrant. To this end he must personally evaluate the report and the supporting documents submitted by the prosecutor regarding the existence of probable cause. We have ruled that a judge fails in his bounden duty if he relies merely on the certification of the investigation officer as to the existence of probable cause making him administratively liable. We can do no less in this case where respondent issued the subject warrant of arrest without even such certification to rely upon, and worse, merely at the instance of the clerk who mechanically typed the warrant for his signature.

TAU MU

Upon the filing of a complaint for libel directly with the Regional Trial Court, the judge issued a warrant for the arrest of accused. It appears that no preliminary investigation was conducted, nor an information prepared, by the Office of the Prosecutor. Asked to explain his action, the judge said that he signed the warrant typed by his Criminal Docket Clerk, who under the Check List for Criminal Cases which the judge prepared for her, was supposed to verify from the records whether an information had already been filed. Should the judge be absolved?

TAU MU TAU MU

TALINGDAN V. EDUARTE

The Fraternal Ateneo de Davao

TAU MU TAU MU

Held: No. The statement is an admission that the judge relied solely and completely on the certification made by the prosecutor that probable cause exists as against those charged in the information, and issued the challenged warrant of arrest. The Constitution commands the judge to personally determine probable cause in the issuance of warrants of arrest. To be sure it cannot be determined before hand how cursory or exhaustive the examination of the records should be. The extent depends on the exercise of sound discretion as the circumstances of the case require. But he cannot adopt the judgment of the prosecutor regarding the existence of probable cause on his own without abdicating his duty under the Constitution.

KITY

TAU MU TAU MU TAU MU TAU MU TAU MU TAU MU

CONSTITUTIONAL LAW II Order of Saint Thomas More Atty. Philip John Pojas/Atty. Rovyne G. Jumao-as, RN University College of Law

Held: No. PD No. 1069, the Extradition Law uses the word “immediate” to qualify the arrest of accused. Arrest subsequent to a hearing can no longer be considered immediate. Immediately upon receipt of the petition, the judge shall make a prima facie finding whether the petition is sufficient in form and substance, whether it complies with the Extradition Law, and whether the person sought is extraditable. If the judge is convinced, he issues a warrant for the arrest of the person summons him to appear at scheduled hearings. Even Sec. 2 of Art. III of the Constitution does not require a notice or a hearing before the issuance of a warrant of arrest. What it requires is for the judge to determine probable cause by examination of complainants and the witnesses they may produce. There is no requirement to notify and hear the accused before the issuance of warrants of arrest. SEARCH WARRANT SILVA VS. PRESIDING JUDGE The deposition of the witness in support of the application for search warrants proceeded as follows: Q: Do you personally know M/Sgt. Ranulfo Villamor Jr., the applicant of a search warrant? A: Yes sir. Q: Do you have personal knowledge that the said premises subject of the ofense stated above and other proceeds or fruit of the ofense used or obtained are intended to be used as means of committing the ofense? A: Yes sir. Q: Do you know personally the person(s) who have the property in his/their possession and control? A: Yes sir. Q: How did you know all their(sic) things? A: Through discreet surveillance Held: The court held that the abuse deposition did not only contain long questions but it was also very broad. The questions propounded to the witness were not probing but were merely routinary. The deposition was already mimeographed and all that the witnesses had to do was to fill in their answers on the blanks provided. Asking of leading questions to the deposition and conducting of an examination in a general manner would satisfy the requirements for issuance of a valid search warrant. CORRO VS. LISING Search of the Philippine Times with search warrant based on two affidavits filed by the witnesses  Affidavit of Col. Castillo submitted to the judge contained: We found that the said publication in fact commenced distrust and hatred against the government of the Philippines and its duly

ACADCOM 2010; Contributors: Gene Geocaniga, Jarissa Guiani, Darlene Magabilen TAU MU Page 41 of 179

constituted authorities defined and penalized under Art. 142 of the RPC as amended by PD1835.  The other witness, Lt. Ignacio stated in his affidavit: The said periodical published by Corro contains article leading to cite distrust and hatred for the authority of the Republic of the Philippines or any of the duly concerned authorities. The judge issued the search warrant based on the 2 affidavits. This was challenged before the SC arguing that there was no probable cause.

TAU MU TAU MU

PRUDENTE VS. DAYRIT

The Fraternal Ateneo de Davao

TAU MU TAU MU

Held: The court held that the above statements do not amount to probable cause. The statements do not contain facts for conclusions of law. The statements do not contain facts for conclusions of law. The statements are only conclusions determined by the witnesses and not by the judge.

KITY

TAU MU TAU MU TAU MU TAU MU TAU MU TAU MU

CONSTITUTIONAL LAW II Order of Saint Thomas More Atty. Philip John Pojas/Atty. Rovyne G. Jumao-as, RN University College of Law

TAU MU

Held 1: In the first issue, the Court ruled that this is not a general search warrant. It appears that if the general place follows the particular place, there is no violation of the Constitution as long as the place is under the control of the same person. It would be diferent if the search is to be conducted on a condo building, you cannot search anymore the other condo units which are not owned by the same person. Usually a search warrant must be specific of the place to search and the object to be seized. This is to prevent the peace officers from exercising discretion as to what are to be seized. Otherwise, he can seize anything he wants which are void and unconstitutional. Held 2: As to the 2nd issue, the court ruled that the warrant is void because the judge did not ask searching questions, but leading questions. It would seem that the judge knows more about the alleged crime than the witness. In addition, was found that the witness did not hve personal knowledge of the facts regarding the crime than the witness. In addition, it was found that the witness did not have personal knowledge of the facts regarding the crime. His affidavit contained statements like “…I have been informed and have good reason to believe…”, “…with regards to the

TAU MU

TAU MU

During evacuation in Leyte, Prudente allowed the evacuees to be housed on the Polytechnic University. The police applied for a search warrant alleging the existence of firearms in the school. The judge commanded to make an immediate search anytime in the day or night of the premises of Polytechnic Univ. of the Philippines, more particularly in the office of the Dept. of Science and Tactics at the ground floor and other rooms at the second floor. Lawyers of Prudente got hold of a copy of the examination of the witness by the judge. They discovered that the judge did not ask searching questions, but only leading questions in which the witness only answered: yes sir.

statement…”, “…the undersigned has verified the report…” therefore acquiring personal knowledge, the court held that nothing in the verification was made. It did not satisfy the requirement of searching questions. COLUMBIA VS. FLORES The judge issued a search warrant for violation of the Decree on Protection of Intellectual Property against FGT Video Network, a licensed video tape distributor. In addition to video tapes, posters, and journal, it is also ordered the seizure of: Xxx c.) television sets, video cassette recorders, rewinders, tape head cleaners, accessories, equipment and other machines and paraphernalia or materials used or intended to be used in the unlawful sale, lease, distribution, or possession for purpose of sale, lease, distribution, circulation or public exhibition of the abovementioned pirated video tapes. Held: The court ruled that these articles and appliances are generally connected with, or related to a legitimate business not necessarily involving piracy of intellectual property or infringement of copyright laws. Hence, including these articles without specification and/or particularly that they were really instruments in violating the Anti-Piracy Law makes the search warrant too general which could result in the confiscation of all items found in any video store.

TAU MU

COLUMBIA V. CA 261 SCRA 144 (1996)

TAU MU

The search warrant issued for violation of Presidential Decree No. 49 directed the seizure, among others, of pirated video tapes. Considering that there are as many or ofenses of infringement as there are rights protected in the various movie titles involved, does the warrant violate the rule that a search warrant must be issued only in connection with one specific ofense?

TAU MU TAU MU TAU MU

Held: No. The search warrant itself indicates it was issued for violation of Sec. 56, PD No. 49, as amended only. The specifications therein merely refer to the titles of the copyrighted motion pictures belonging to private complainants which defendants were in possession for sale or lease in violation of PD No. 49. That there were several counts of the ofense of copy right infringement and the search warrant uncovered several contraband items in the form of pirated video tapes in not to be confused with the number of ofenses charged.

TAU MU

COLUMBIA PICTURES V. CA 262 SCRA 219 (1996)

ACADCOM 2010; Contributors: Gene Geocaniga, Jarissa Guiani, Darlene Magabilen TAU MU Page 42 of 179

Held: Judges can issue warrants efective outside of their jurisdiction because the Rules of Court does not prohibit it. Sec. 3 of BP Blg. 129 states that “all other processes” “may be served anywhere in the Philippines.” A search warrant is a judicial process. This ruling applies when the issuance is necessitated and justified by compelling consideration of urgency, subject, time and place. However, if a case is pending, the court where such case is pending has the primary jurisdiction to issue a warrant, except under extreme and compelling circumstances.

An RTC judge of Kaloocan issued a search warrant for alleged violation of PD 1866. He directed the search of a house in Quezon City which is outside his territorial jurisdiction. The validity of the search is now questioned.

SALAZAR VS. ACHACOSO In line with the power granted by Art. 38 of the Labor Code, the Secretary issued a warrant to conduct a search in a particular property, in connection with the crime of illegal recruitment.

TAU MU

Held: The court held that the PD which was later incorporated in to the Labor Code as Art. 38 was unconstitutional. Under the Constitution, only judges can issue search warrants.

TAU MU

PICOP V. ASUNCION

TAU MU

PEOPLE VS. DICHOSO

TAU MU

TAU MU

Held: No. The above items could not be anymore specified as the circumstances will allow since they are all used or intended to be used in the unlawful sale or lease of pirated tapes. [But see, Columbia v. Flores, 223 SCRA 761 (1993), where another division of the Supreme Court voided a similarly-worded search warrant as a general warrant.]

TAU MU TAU MU

The court issued a search warrant for violation of the Dangerous Drugs Act. It directed the peace officer to search and seize the following: a.)marijuana, b.)shabu, c.)and paraphernalia. The accused challenged the constitutionality of the warrant on the basis that it covers three articles, making it a general search warrant. He claimed that the warrant failed to specify the thing to be seized.

ILANO V. CA

TAU MU

For violation of Sec. 56 of the Decree on the Protection of Intellectual Property, the judge issued a search warrant for the seizure of: (c) Television sets, video cassettes records, rewinders, tape head cleaners, accessories, equipment and other machines and paraphernalia on material used or intended to be used in the unlawful sale, lease, distribution, or possession for purpose of sale, lease, distribution, circulation or public exhibition of the abovementioned pirated video tapes which he is keeping and concealing in the premises abovedescribed. Is the warrant a general warrant?

Held: The court ruled that this is not a general search warrant. The ofenses involved, or the objects subject for seizure belong to the same class. Therefore, the officer does not need one warrant for each item.

TAU MU

COLUMBIA PICTURES V. CA supra

Ateneo de Davao

TAU MU TAU MU

Held: The ruling in 20th Century Fox has no retroactive application. The lower court could not have expected more evidence from the applicant other that what the law and jurisprudence required at that time with respect to finding of probable cause. In addition, the Court did not rule therein that representation of the master tapes is an absolute requirement for a search warrant to issue. It should serve only as a guide post when there is doubt as to the nexus between the master tapes and the pirated copies. It does not rule out the use of testimonial, documentary or other evidence tending to establish probable cause.

The Fraternal

TAU MU TAU MU

On July 28, 1986, the judge issued a search warrant for Violation of the Decree on the Protection of Intellectual Property ordering the seizure of pirated video tapes, without requiring the production of the master tapes. In August 19, 1988, the Supreme Court voided a search warrant in 20th Century v. CA, 162 SCRA 655 (1988), on the ground of lack of probable cause because the master tapes of the alleged pirated tapes were not shown to the lower court. Is the search warrant valid?

KITY

TAU MU TAU MU TAU MU TAU MU TAU MU TAU MU

CONSTITUTIONAL LAW II Order of Saint Thomas More Atty. Philip John Pojas/Atty. Rovyne G. Jumao-as, RN University College of Law

The Philippine National Police (PNP) applied for a search warrant in Quezon City for the search of Paper Industries Corporation of the Philippines (PICOP) in Bislig, Surigao del Sur. The search was for the seizure of 70 Armalite rifles and other assorted firearms. The applicant did not allege before the trial court that PICOP has no license to possess the firearms nor attached a certification from the Firearms and Explosives Unit on the lack of license. Upon application by the Philippines National Police (PNP), the judge ordered the search and seizure of firearms in possession of the Paper Industries Corporation of the Philippines “located at PICOP Compound, Barangay Tabon, Bislig, Surigao del Sur.” Issues: (1) Was there probable cause for the issuance of the warrant? ; (2) Is the description of the place to be searched sufficient? Held: First Issue: There was no probable cause for the issuance of the search warrant. The facts and circumstances that would show probable

ACADCOM 2010; Contributors: Gene Geocaniga, Jarissa Guiani, Darlene Magabilen TAU MU Page 43 of 179

Are the warrants valid?

TAU MU TAU MU TAU MU

KENNETH ROY V. TAYPAN

TAU MU

Held: Yes. The law does not require that the things to be seized must be described in precise and minute detail as to leave no room for doubt on the part of the searching authorities. Otherwise, it will be virtually impossible for the applicants to obtain a warrant as they would not know exactly what kind of things they are looking for. Since the element of time is very crucial in criminal cases, the efort and time spent in researching the details to be embodied in the warrant would render the purpose of the search nugatory. Consequently, the above descriptions could be adjudged in substantial compliance with the requirements of law.

TAU MU

“Chop-chop

TAU MU

Search Warrant No. 90-14: vehicles and other spare parts.”

TAU MU

Search Warrant No 90-12: “Unlicensed firearms of various calibers and ammunitions for the said firearms.”

TAU MU

The judge issued 3 search warrants describing the items to be seized as follows: Search Warrant No. 90-11: “Unlicensed radio communications equipment such as transmitters, transceivers, handsets, scanners, monitoring devices and others.”

TAU MU TAU MU

KHO V. MAKALINTAL

The Fraternal Ateneo de Davao

TAU MU TAU MU

cause must be the best evidence that could be obtained under the circumstances. The introduction of such evidence is necessary in cases where the issue is the existence of the negative ingredient of the ofense charged --- for instance, the absence of a license required by law, as in the present case --- and such evidence is within the knowledge and control of the applicant who could easily produce the same. But if the best evidence could not be secured at the time of the application, the applicant must show a justifiable reason therefore during the examination by the judge. 2nd issue: No. The warrant failed to describe the place with particularity. Is simply authorizes a search of “the aforementioned premises,” but it did not specify such premises. It identifies only one place, and that is the PICOP Compound. The compound, however, is made up to 200 offices/buildings, 15 plants, 84 staf houses, 1 airstrip, 3 piers/wharves, 23 warehouses, 6 POL depots/quick service outlets and some 300 miscellaneous, all of which are spread out over one hundred fifty-five hectares. Obviously, the warrant gives the police officers unbridled and thus illegal authority to search all the structures found inside the PICOP Compound.

KITY

TAU MU TAU MU TAU MU TAU MU TAU MU TAU MU

CONSTITUTIONAL LAW II Order of Saint Thomas More Atty. Philip John Pojas/Atty. Rovyne G. Jumao-as, RN University College of Law

Upon application by the National Bureau of Investigation, the Regional Trial Court of Cebu, Branch 12, ordered the seizure of several pieces of furniture from petitioner. The ofense was for violation of Art. 189 of the Revised Penal Code on unfair competition involving design patents. Does RTC, Branch 12, have the authority to issue the warrant considering that it was not designated as a special court for Intellectual Property Rights? In applying for a search warrant must the applicant execute a certificate of nonforum shopping? Held: A search warrant is merely a process issued by the court in the exercise of its ancillary jurisdiction and not a criminal action which it may entertain pursuant to its original jurisdiction. The authority to issue search warrants is inherent in all courts and may be efected outside their territorial jurisdiction. AO No. 113-95 merely specified which court could try and decide cases involving intellectual property rights. It did not vest exclusive jurisdiction with regard to all matters in any one court. Jurisdiction is conferred upon courts by substantive law, not by an administrative order. Nor is a certificate of nonforum shopping necessary. It is required only for initiatory pleadings, not “applications.” UY V. BIR For the crime of selling cartons of sardines without issuing receipts in violation of Sec. 253 of the National Internal Revenue Code, the Regional Trial Court issued a Search Warrant against petitioner at Unifish Packing Corporation. In the caption of the Search Warrant, his address was indicated as “Hernan Cortes St., Cebu City,” while the body of warrant states the address as “Hernan Cortes St., Mandaue City. Is there a violation of the constitutional requirement that there be a particular description of the place to be search? Held: No. The Rule is that a description of a place to be searched is sufficient if the officer with the warrant can, with reasonable efort, ascertain and identify the place intended and distinguish it from other places in the community. In this case, it was not shown that a street similarly named Hernan Cortes could be found in Cebu City. Nor was it established that the enforcing officers had any difficulty in locating the premises described in the warrant. That the warrant, therefore, inconsistently identified the city where the premises to be searched is, is not a defect that would spell the warrant’s invalidation. UY V. BIR supra

TAU MU

For violation of Sec. 253 of the National Internal Revenue Code, the RTC issued 2 Search Warrants, the second apparently to correct the inconsistencies in the first. The first warrant was issued solely against “Uy Chin Ho alias Frank Uy,”

ACADCOM 2010; Contributors: Gene Geocaniga, Jarissa Guiani, Darlene Magabilen TAU MU Page 44 of 179

and the second against “Uy Chin Ho alias Frank Uy, and Unifish Packing Corporation.” Is there a defect in the description of the place to be searched?

TAU MU TAU MU TAU MU

The warrant for the ofense of selling cartons of sardines without issuing receipts in violation of Sec. 253 of the National Internal Revenue Code issued by the Regional Trial Court described the objects to be seized as follows: 1. “Multiple sets of Books of Accounts, Ledgers, Journals, Provisional & Official Receipts; 2. Production Record Books/Inventory Lists; 3. Unregistered Delivery Receipts; 4. Unregistered Purchase & Sales Invoice; 5. Sales Records, Job Orders.” Are the objects described with particularity?

TAU MU TAU MU

UY V. BIR supra.

The Fraternal Ateneo de Davao

TAU MU TAU MU

Held: No. The Constitution does not require the warrant to name the person who occupies the named premises. Where the warrant is issued for the search of a specifically described premises only and not for the search of a person, the failure to name the owner or occupant of such property in the warrant does not invalidate the warrant; and where the name of the owner of the premises is incorrectly inserted, it is not a fatal defect if the legal description of the premises is otherwise correct so that no discretion is left to the officer making the search as to the place to be searched. Since in this case the warrant was issued not for the search of the persons owning the premises but only a search of the premises occupied by them, the search could not be declared unlawful because of the inconsistencies in stating their names.

KITY

TAU MU TAU MU TAU MU TAU MU TAU MU TAU MU

CONSTITUTIONAL LAW II Order of Saint Thomas More Atty. Philip John Pojas/Atty. Rovyne G. Jumao-as, RN University College of Law

TAU MU

PEOPLE V. SALANGUIT

TAU MU

TAU MU

Held: Most of the items listed fail to meet the test of particularity, especially since the judge was furnished with the photocopies of the documents sought to be seized. The use of generic term or a general description is acceptable only when a more specific description of the things is unavailable. The use of terms like “multiple sets of books of account ledgers… journals, cash register books…provisional & official receipts… stock delivery receipts” and “unregistered purchase & sales invoices”, they need not be specified as it is not possible to do so precisely because they are unregistered. The general description of most of the documents listed in the warrant does not render the entire warrant void. Insofar as the warrant authorized the seizure of unregistered delivery receipts and unregistered purchase and sales invoices, the warrant remain valid. The search warrant is severable, and those items not particularly described may be cut of without destroying the whole warrant. Accordingly, the items not particularly described ought to be returned to petitioner.

The Search Warrant issued by the judge states: It appearing to the satisfaction of the undersigned after examining under oath SR. INSP. Rodolfo V. Aguilar, PNP and his witness SPO1 Edmund M. Badua, PNP, that there is probable cause to believe that Robert Salanguit has in his possession and control in his premises Binhagan St., San Jose, Quezon City as shown in Annex “A”, the properties to wit: UNDETERMINED QUANTITY OF SHABU AND DRUG PARAPHERNALIA.” Considering that during the depositiontaking, no witness testified on anything about drug paraphernalia, should the warrant be nullified for having been issued without probable cause? Held: No. The fact that there was no probable cause to support the application for the seizure of drug paraphernalia does not warrant the conclusion that the search warrant is void. This fact would be material only if drug paraphernalia were seized by the police. But none was taken by virtue of the search warrant. If at all, therefore, the search warrant is void only insofar as it authorized the seizure of drug paraphernalia, but it is valid as to the seizure of shabu about which evidence was presented showing probable cause as to its existence. It would be a drastic remedy indeed if a warrant, which was issued on probable cause and particularly describing the items to be seized on the basis thereof, is to be invalidated in to because the judge erred in authorizing a search for other items not supported by evidence. PEOPLE V. SALANGUIT

TAU MU

The Search Warrant issued by the judge ordered the search of the residence of Robert Salanguit at “Binhagan St., San Jose, Quezon City as shown in Annex “A.” Among the documents in the records of the applications for the warrant are: 1) the application itself which stated that the premises to be searched was located in between No. 7 and 11 of Binhagan St.; 2) deposition of witness which describe the premises as “a house without a number located at Binhagan St.”; and 3) the pencil sketch of the location of the premises to be searched. In addition, the leader of the police learn who conducted the search resides in the neighborhood. Did the warrant satisfy the requirement of particularity of description of the place to be searched?

TAU MU TAU MU

Held: Yes. The rule is that a description of the place to be searched is sufficient if the officer with the warrant can, with reasonable efort, ascertain and identify the place intended to be searched. For example, a search warrant authorized a search of Apartment No. 3 of a building at 83 Peasant St., Malborough, Massachusettss. As is turned out, there were 5 apartments in the basement and 6 apartments on both the ground and top floors and that there was an Apartment No. 3 on each floor. However, the description was made determinate by a reference to the affidavit supporting the warrant that the

ACADCOM 2010; Contributors: Gene Geocaniga, Jarissa Guiani, Darlene Magabilen TAU MU Page 45 of 179

apartment was occupied by Morris Ferrante of 83 Pleasant St., Malboro, Mass. In this case, the location of Salanguit’s house being indicated by the evidence on record, there can be no doubt that the warrant described the place to be search with sufficient particularity. AL-GHOUL V. CA 364 SCRA 363 (2001)

TAU MU TAU MU TAU MU TAU MU

A team from the Police Assistance and Reaction Against Crime (PARAC) of the DILG was

TAU MU

Facts:

TAU MU

Case: Quelnan vs. People (526 SCRA 353)

TAU MU

Held: There was no specific ofense mentioned. There is a need to mention only one specific ofense. Otherwise, it will be considered a general warrant.

TAU MU

Note: This case was also in the exclusionary rule. Stated in the warrant: The above items are subject to the ofense, stolen or embezzled, or intended to be used as a means to commit ofenses violating CB laws, tarifs and customs laws, The Internal Revenue Code, and the RPC.

TAU MU

STONEHILL VS. DIOKNO (discussed by Ms. Jumao-as – General Warrant)

TAU MU TAU MU

Held: Yes. The articles seized are of the same kind and nature as those items enumerated in the search warrant. The items ordered seized in the warrant were described with specificity. The nature of the items ordered to be seized did not require a technical description. Moreover, the law does not require that the things to be seized must be described in precise and minute details as to leave no room for doubt, otherwise, it would be virtually impossible for the applicants to obtain a search warrant as they would not know exactly what kind of things they are looking for. Once describe, however, the articles subject of seizure need not be so invariant as to require absolute concordance between those seized and those described in the warrant. Substantial similarity of those articles described as a class or species would suffice.

The Fraternal Ateneo de Davao

TAU MU TAU MU

The judge ordered the search of Apartment No. 2 and the seizure of the following: 1 .45 caliber pistol; 1 5.56 M16 Rifle with ammunitions; 1 9MM pistol with ammunitions; 3 boxes of explosives; 10 sticks of dynamite; 30 pieces of blasting caps. The police, however, found and seized the following: 2 M-16 rifles with 2 magazines and 20 ammunitions; 1 bar demolition charge; 1 caliber Pistol, with magazine of Caliber .45 and 3 .45 ammunitions; 1 22 caliber handgun with 5 ammunitions nitrate; 22 detonating cords; ½ and ¼ pound of TNT; 1 timer alarm clock; and 2 batteries 9 volts. Are the objects admissible?

KITY

TAU MU TAU MU TAU MU TAU MU TAU MU TAU MU

CONSTITUTIONAL LAW II Order of Saint Thomas More Atty. Philip John Pojas/Atty. Rovyne G. Jumao-as, RN University College of Law

formed to implement a search warrant. The team proceeded to the CityLand Condo in Makati and upon arrival, they went to the Security Office of the said building to seek assistance in serving the warrant and they were accompanied to Unit 615. The team presented the search warrant to petitioner and the police operatives searched the unit. In the presence of the petitioner, the team started searching the place and eventually found on top of the bedroom table 3 pcs. of transparent plastic sachets containing white crystalline substances later confirmed by the NBI as shabu, plastic tubings, weighing scales, an improvised burner and empty transparent plastic sachets. Thereafter, the team prepared a receipt of the properties seized and an Affidavit of Orderly Search allegedly signed by petitioner in their presence and that of security officer of the building. The team also went to Unit 418 of the same building to serve the warrant and search the place. The police operatives did not find any occupant in the room. The following day, petitioner was charged for violation of Sec. 16, Article III of R.A. 6245. In his defense, petitioner testified that he is the registered owner of Unit 615 of CityLand Condo which he leased to Sung Kok Lee. He also said that at the time the police operatives went to the said unit, he was supposed to collect payment of rental from Lee. He heard somebody knocking in the door when he opened it. The police operatives asked for a certain Bernard Kim and he introduced himself as the owner of the condo unit. The police operatives then searched the house and he was forced to sign some documents at gunpoint. Issue:

WON the search warrant was validly implemented. Ruling: Section 4, Rule 126 of the Revised Rules of Criminal Procedure provides for the requisites for the issuance of search warrant which states that: a search warrant shall not issue except upon probable cause in connection with one specific ofense to be determined personally by the judge after examination under oath or affirmation of the complaint and the witness he may produce, and particularly describing the place to be searched and the things to be seized which may be anywhere in the Philippines. Nowhere in said rule or any other provision in the Revised Rules of Criminal Procedure is it required that the search warrant must name the person who occupies the described premises. A cursory reading of the search warrant reveals that the police officers were ordered to make an immediate search of the premises mentioned and to seize and take possession of shabu. Furthermore, they were directed to bring “persons to be dealt with as the law may direct.” While petitioner may not be the person subject of the search, the fact that he was caught in flagrante delicto necessitated his valid warrantless arrest. Therefore, the fact that petitioner’s name was not indicated in the search warrant is immaterial.

ACADCOM 2010; Contributors: Gene Geocaniga, Jarissa Guiani, Darlene Magabilen TAU MU Page 46 of 179

Case: People vs. Francisco (387 SCRA 569) Facts:

TAU MU TAU MU TAU MU TAU MU TAU MU TAU MU TAU MU TAU MU

Ruling: The basic guarantee to the protection of the privacy and sanctity of a person, his home and his possessions against unreasonable intrusions of the State is articulated in Section 2, Article III of the Constitution. For the validity of the search warrant, the Constitution requires that there be a particular description of the “place to be searched and the persons or things to be seized.” The rule is that a description of the place to be searched is sufficient if the officer with the warrant can, with reasonable efort, ascertain and identify the place intended and distinguish it from other places in the community. Any designation or description known to the locality that leads the officer unerringly to it satisfies the constitutional requirement. The requisites for the issuance of a valid search warrant are: (1) probable cause is present;

TAU MU

WON the search conducted by the police officers at the accused-appellant’s residence was reasonable.

TAU MU TAU MU

Issue:

The Fraternal Ateneo de Davao

TAU MU TAU MU

Verona and his live-in girlfriend Francisco, were placed under surveillance after the police confirmed that they were engaged in selling shabu. SPO2 Teneros and SPO4 San Juan applied for a search warrant to authorize them to search the premises at 122 M. Hizon St., Caloocan City. Attached to such application was the AfterSurveillance Report that one of Verona’s runners in the illegal drugs operations, allegedly sought the assistance of SPO4 Teneros for the arrest of Verona. According to Francisco, who was then 9 months pregnant at the time the police operatives suddenly entered her bedroom, while she was resting inside their bedroom at 120 M. Hizon St., Caloocan City, she heard a loud bang downstairs and 8 policemen entered and conducted a search for an hour. She inquired about their identities but it was only at the police station where she found out that the team of searchers was led Teneros. The police team enforced the warrant and seized the following: (1) 1 pyrex salad set wrapped in a plastic containing white crysthalline substance or shabu with markings by the undersigned inside the house of subject’s residence weighing 230 grams of shabu: (2) several plastics in diferent sizes: (3) 2 roll of strip aluminum foil: (4) 5 tooter water pipe and 2 improvised burner: (5) 2 measuring weight in shabu: (6) 2 motorola cellular phones; (7) 1 monitoring device with cord: and (8) Php22, 990. Consequently, Francisco was charged with violation of RA 6425 (Dangerous Drug Act). Francisco then filed a motion to quash the search warrant asserting that she and her live-in partner Verona had been leasing an apartment at 120 M. Hizon St., District 2 Caloocan City. Upon arraignment, she pleaded not guilty.

KITY

TAU MU TAU MU TAU MU TAU MU TAU MU TAU MU

CONSTITUTIONAL LAW II Order of Saint Thomas More Atty. Philip John Pojas/Atty. Rovyne G. Jumao-as, RN University College of Law

(2) such presence is determined personally by the judge: (3) the complainant and the witnesses he or she may produce are personally examined by the judge, in writing, and under oath or affirmation: (4) the applicant and the witnesses testify on the facts personally known to them: and (5) the warrant specifically describes the place to be searched and the things to be seized. The absence of any of these requisites will cause the downright nullification of the search warrants. The warrants will always be construed strictly without, however, going the full length of requiring technical accuracy. No presumptions of regularity are to be invoked in aid of the process when an officer undertakes to justify it. In the case at bar, the trial court conducted an ocular inspection of the area. It turned out that No. 122 M. Hizon St., District 2 Caloocan City was a concrete two-storey residential building with steel-barred windows and a terrace. It was owned by certain Mr. Joseph Ching. The house bore no house number. He house marked No. 122 M. Hizon St., District 2 Caloocan City was actually two houses away from Francisco’s house at 120 M. Hizon St., District 2 Caloocan City. The particularity of the place described is essential in the issuance of search warrants to avoid the exercise by the enforcing officers of discretion. The controlling subject of search warrants is the place indicated in the warrant itself and not the place identified by the police. Hence, Francisco was acquitted. Case: Prudente vs. Dayrit and PP Facts: Dimagmaliw, chief of ISAD, filed an application for the issuance of a search warrant for violation of PD 1866 against Prudente. Dimagmaliw alleged that Prudente has in his control or possession of firearms, explosives, hand grenades and ammunitions which are illegally possessed or intended to be used as the means of committing an ofense which Prudente is keeping and concealing at the premises of PUP in the Office of the Department Military Science and Tactics and Office of the President. In support of his application, the OIC of the Intelligence Section of ISAD, executed a “Deposition of Witness” which made Dayrit to issue the search warrant on the same day. Thereafter, the search warrant was enforced. A member searching team alleged that he found in the drawer of the cabinet inside the wash room of Prudente a bulging brown envelope with 3 live fragmentation hand grenades separately wrapped with old newspapers, classified as 1 pc. M33 fragmentation hand grenade, 1 pc. M26 fragmentation hand grenade and 1 pc. PRB-423 fragmentation hand grenade. Prudente moved to quash the search warrant claiming that the complainant’s lone witness had no personal knowledge of the facts which formed the basis for the issuance of the search warrant, that the examination of the said witness was not in the form of searching questions and answers, the search warrant was a general warrant for the

ACADCOM 2010; Contributors: Gene Geocaniga, Jarissa Guiani, Darlene Magabilen TAU MU Page 47 of 179

reason that it did not particularly describe the place to be searched and that it failed to charge one specific ofense and the search warrant was issued in violation of Circular No. 19 of the SC in that the complainant failed to allege under oath that the issuance of the search warrant on that certain day was urgent. Issue: WON the search warrant was valid.

TAU MU TAU MU TAU MU TAU MU TAU MU TAU MU TAU MU TAU MU

SONY filed a complaint against Supergreen with the NBI for the reproduction and distribution of counterfeit “Playstation” game software, consoles and accessories in violation of Sony

TAU MU

Facts:

TAU MU TAU MU

Case: SONY Computer Entertainment, Inc. vs. Supergreen Incorporated

The Fraternal Ateneo de Davao

TAU MU TAU MU

Ruling: For a valid search warrant to issue, there must be a probable cause, which is to be determined personally by the judge, after examination under oath or affirmation of the complainant and the witnesses he may produce, and particularly describing the place to be searched and the persons or things to be seized. The probable cause must be in connection with one specific ofense, and the judge must, before issuing the warrant, personally examine in the form of searching questions and answers, in writing and under oath, the complainant and any witness he may produce, on facts personally known to them and attach to the record their own statements together with any affidavits submitted. The probable cause for a valid search warrant has been defined “as such facts and circumstances which would lead a reasonably discreet and prudent man to believe that an ofense has been committed and that objects sought in connection with the ofense are in the place sought to be searched.” This probable cause must be shown to be within the personal knowledge of the complainant or the witnesses he may produce and not based on mere hearsay. In the case at bar, the evidence failed to show the existence of probable cause to justify issuance of the search warrant. The SC also notes post facto that the search in the question yielded, no armalites, handguns, pistols, assorted weapons or ammunitions as stated in the application for search warrant, the supporting deposition, and the search warrant itself. Only 3 live fragmentation hand grenades were found in the searched premises of the PUP, according to the affidavit of an alleged member of the searching party. The SC also avails of the decision to reiterate the strict requirements for determination of “probable cause” in the valid issuance of a search warrant. The requirements are stringent but the purpose is to assure the constitutional right of the individual against unreasonable search and seizure shall remain both meaningful and efective. Hence, the petition for certiorari to annul and set aside the order of Dayrit was granted.

KITY

TAU MU TAU MU TAU MU TAU MU TAU MU TAU MU

CONSTITUTIONAL LAW II Order of Saint Thomas More Atty. Philip John Pojas/Atty. Rovyne G. Jumao-as, RN University College of Law

Computer’s Intellectual Property Rights. Thus, NBI applied with the RTC of Manila, Branch 1, for warrants to search respondent’s premises in Paranaque and Cavite. RTC issued search warrants covering Supergreen’s premises at Trece-Tanza Road, Purok 7, Brgy. De Ocampo, Trece Martires City, Cavite and at Room 302, 3rd Floor Chateau de Baie Condo, 149 Roxas Blvd. corner Airport Road, Paranaque City. NBI simultaneously served the search warrants on the subject premises and seized a replicating machine and several units of counterfeit “Playstation” consoles, joy pads, housing, labels and game software. Thus, Supergreen filed a motion to quash the search warrants on the ground that the search warrant failed to particularly describe the properties to be seized. Issue: WON the quashal of search warrants were valid in this case. Ruling: In the case at bar, Supergreen’s premises in Cavite, within the Fourth Judicial Region, is definitely beyond the territorial jurisdiction of the RTC of Manila in the NCR. Thus, the RTC of Manila does not have the authority to issue a search warrant for ofenses committed in Cavite. However, SC agreed with Sony that this case involves a transitory continuing ofense of unfair competition under Section 168 of RA 8293. Supergreen’s imitation of the general appearance of petitioner’s goods was done allegedly in Cavite. It sold the goods allegedly in Mandaluyong. The alleged acts would constitute a transitory or continuing ofense. Thus, under Section 2 (b) of Rule 126, Section 168 of RA 8293 and Article 189 (1) of the RPC, Sony may apply for a search warrant in any court where any element of the alleged ofense was committed, including any courts within NCR. Hence, the petition for review sought by Sony was granted. Case: Yao Sr. vs. PP Facts: Petitioners are incorporators and officers of MASAGANA, an entity engaged in refilling, sale and distribution of LPG products. PR Petron and Shell are two of the largest bulk suppliers and producers of LPG in the Phils. Their LPG products are sold under the marks of “GASUL” and “SHELLANE” respectively. NBI filed two applications for search warrants against petitioners and other occupants of MASAGANA compound located at Governor’s Drive, Brgy. Lapidario, Trece Martires, Cavite City for alleged violation of RA 8293, otherwise knows as “The Intellectual Property Code of the Philippines.” The 2 applications for search warrants uniformly alleged that per info, belief and personal verification of Oblanca (the NBI agent who applied for search warrants), the petitioners are actually producing, selling, ofering for sale and/or distributing LPG products using steel cylinders owned by, and bearing the tradenames, trademarks and devices of Petron and Shell,

ACADCOM 2010; Contributors: Gene Geocaniga, Jarissa Guiani, Darlene Magabilen TAU MU Page 48 of 179

TAU MU TAU MU TAU MU TAU MU TAU MU TAU MU TAU MU TAU MU TAU MU TAU MU TAU MU

Ruling: Article III, Section 2 of the Constitution states the requirements before a search warrant may be validly issues such as: (1) probable cause is present; (2) such presence is determined personally by the judge: (3) the complainant and the witnesses he or she may produce are personally examined by the judge, in writing, and under oath or affirmation: (4) the applicant and the witnesses testify on the facts personally known to them: and (5) the warrant specifically describes the place to be searched and the things to be seized. A search warrant can be issued only upon finding a of probable cause. Probable cause for search warrant means such facts and circumstances which would lead a reasonably discreet and prudent man to believe that an ofense has been committed and that the objects sought in connection with the ofense are in the place to be searched. The facts and circumstances being referred thereto pertain to facts, data or info personally known to the applicant and the witnesses he may present. The applicant or his witnesses must have personal knowledge of the circumstances surrounding the commission of the ofense being complained of. “Reliable Info” is insufficient. Mere affidavits are not enough, and the judge must depose in writing the complainant and his witnesses. In the case at bar, documentary and object evidence is that Oblanca and his witness have personal knowledge of the fact that petitioners, through MASAGANA, have been using the cylinders bearing the marks GASUL and SHELLANE without permission from Petron and Shell which is a probable case for trademark infringement. Both Oblanca and his witness were clear and insistent that they were the very same persons who monitored the activities of

The Fraternal Ateneo de Davao

TAU MU TAU MU

without authority and in violation of the rights of the said entities. The search warrants commanded any peace officer to make an immediate search of the MASAGANA compound and to seize the all empty or filled LPG cylinder tanks/containers bearing the tradename “SHELL” or “SHELLANE” and the trademarks and other devices owned by Shell including sales invoices as well as Petron’s all empty or filled LPG cylinder tanks/containers bearing the tradename “GASUL” and all other devices owned by Petron. Petitioners filed a motion to quash the search warrants on the following grounds: (1) that there is no probable cause for the issuance of search warrant and the conditions for search warrant were not complied with: (2) that NBI agent Oblanca and his witness do not have any authority to apply for a search warrant: (3) that the place to be searched was not specified in the search warrant: and (4) the search warrant is characterized as a general warrant as the items to be seized as mentioned in the search warrant are being used in the conduct of the lawful business and the same are not being used in refilling Shellane and Gasul LPG’s. Issue: WON the search warrant was valid.

KITY

TAU MU TAU MU TAU MU TAU MU TAU MU TAU MU

CONSTITUTIONAL LAW II Order of Saint Thomas More Atty. Philip John Pojas/Atty. Rovyne G. Jumao-as, RN University College of Law

MASAGANA: that they conducted test-buy thereon: and that in order to avoid suspicion, they used diferent names during the test-buys. They also personally witnessed the refilling of LPG cylinders bearing the marks GASUL and SHELLANE inside the MASAGANA refilling plant station and the deliveries of these refilled containers to some outlets using mini trucks. Indeed, the aforesaid facts and circumstances are sufficient to establish probable cause. It should be borne in mind that the determination of the probable cause does not call for the application of the rules and standards of proof that a judgment of conviction requires after trial on the merits. As the term implies, probable cause is concerned with probability, not absolute or even moral certainty. The standards of judgment are those of a reasonably prudent man, not the exacting calibrations of a judge after a full blown trial. The facts that Oblanca and his witness used diferent names in the purchase receipts do not negate personal knowledge on their part. It is common practice of the law enforcers such as NBI agents during covert investigations to use diferent names in order to conceal their true identities. This is reasonable and understandable so as not to endanger the life of the undercover agents and to facilitate the lawful arrest or apprehension of suspected violators of the law. Oblanca is a member of the AntiOrganized Crime Division of NBI and it does not abrogate his authority to apply for search warrant. The applicant law enforcer must be a member of a division that is assigned or related to the subject crime or ofense before the application of the search warrant may be acted upon. A description of the place to be searched is sufficient if the officer with the warrant can, with reasonable efort, ascertain and identify the place intended and distinguish it from other places in the community. Any description or designation known to the locality that points out to the place to the exclusion of others, and on inquiry leads the officers unerringly to it satisfies the constitutional requirement. Moreover, in the determination of whether a search warrant describes the premises to be searched with sufficient particularity, it has been held that the executing officer’s prior knowledge as to the place intended in the warrant is relevant. This would seem to be especially true where the executing officer is the affiant on whose affidavit the warrant had been issued, and when he knows that the judge who issued the warrant intended the compound described in the affidavit. WARRANTLESS ARREST UMIL VS. RAMOS This is a consolidation of about 8 or 9 cases. What are important are the cases of Nazareno and Dural. Dural Case: On Jan. 31, 1998, there were two CAPCOM soldiers who were killed by Dural in a shootout in

ACADCOM 2010; Contributors: Gene Geocaniga, Jarissa Guiani, Darlene Magabilen TAU MU Page 49 of 179

Quezon City. On Feb. 1, Dural was arrested based on information that he was a member of the sparrow unit. He was arrested in a hospital while undergoing treatment. Only a few hours had lapsed. Nazareno Case: Nazareno was charged with the killing of a person which took place on Dec. 14, 1998. On Dec. 28, he was arrested. 14 days had lapsed. The arrest of Nazareno was based on the statement by an arrested suspect who implicated Nazareno.

TAU MU TAU MU TAU MU TAU MU TAU MU TAU MU TAU MU

At about midnight of May 29, the accused shot and stabbed and agent of the NARCOM to death. At around 7pm of May 30, Manlulu was arrested by policemen based on information given by an eyewitness. The police did not have a warrant of arrest.

TAU MU

PEOPLE VS. MANLULU

TAU MU

NOTE: There seems to be an inconsistency here because the personal knowledge given here in the Umil case is derived from investigation and “personal knowledge of probable cause”. Note also that it has been observed that personal knowledge in non-political cases is usually derived from personal sensed perception. Also, the 14 days that passed in the Nazareno case still considered that a crime was just “freshly committed.”

TAU MU TAU MU

Held: The court ruled the arrests as valid. In the Dural case, the police got the identity of the killer at about 5am. The arrest was made by 7am. The court said that the crime had just been committed , only 2 hours and 20m minutes had passed after learning of the suspect’s identity. The authorities acted immediately after having the knowledge. In this case, the court ruled that “personal knowledge” can be made up of “personal knowledge of probable cause”. It was not personal knowledge of facts anymore. Personal knowledge of probable cause can be made up of 2: 1. That there were reasonable grounds of suspicion based on actual facts. Basis: i. There was a confidential information which led to the arrest of the accused ii. There was actually a crime that happened: 2 soldiers were killed in the shootout. iii. There was actually a wounded person, who happened to be the accused, being treated in the hospital. 2. That the police must be acting in good faith when conducting the arrest. There is always a presumption that the police are performing their duties regularly.

The Fraternal Ateneo de Davao

TAU MU TAU MU

ISSUE: In the 2 cases, have the crime just freshly been committed? Did the officers have personal knowledge of the facts to the crime to constitute the arrest as valid? (Take note that there is a significant diference in the time element)

KITY

TAU MU TAU MU TAU MU TAU MU TAU MU TAU MU

CONSTITUTIONAL LAW II Order of Saint Thomas More Atty. Philip John Pojas/Atty. Rovyne G. Jumao-as, RN University College of Law

ISSUE: Can the State argue that the crime has just been committed, considering that 19 hours had lapsed? RULING: According to the court you cannot arrest a person after 19 hours without a valid warrant of arrest. There was also no personal knowledge since the info was from a witness. PEOPLE V. ALAVARIO The victim, a house leper, was allegedly raped by her employer on Jan 22,23,25,27 & 28, 1993. On the last day she called her sister by phone who reported to the police. The police went to the address and when the victim opened the door and saw the police, she pointed the accused as the rapist. Held: The arrest is legal. The arresting officers had personal knowledge of facts as culled from the information supplied by the victim herself who pointed to accused as the man who raped her at the time of his arrest. PEOPLE V. SINOC At 6:00 o’clock in the morning of Sept. 20, 1991, a pajero was stopped by armed men at Claver, Surigao del Norte. The occupants were driven to Barobo, Surigao del Norte, bound, brought to a coconut grove and gunned down. At 7:00 o’clock in the morning of Sept. 21, the police of Monkayo, Davao, learned from an informer that the pajero was brought to Bliss Housing Project in the town. Proceeding to the place, they saw the pajero and were told by an apartment owner that the person driving it would return. By 10:30 o’clock that morning, he was back carrying the key to the pajero. The police arrested him. Held: The arrest is legal. Even if warrantless, an arrest may be made when an ofense has in fact been committed and the arresting officer has personal knowledge of facts indicating that the person to be arrested has committed it. There is no question that the police were aware that some twelve [?] hours earlier an ofense had been committed. They received a report about the pajero and went to the place to recover it. Accused was identified by the apartment owner, he was moving to the pajero and had the key to it at the time of his arrest so that his link to the crime was palpable. PEOPLE V. JAYSON At 10:00 o’clock in the evening of March 16, 1991, the police received a radio message directing them to report to Bonifacio St., Davao City at Ihaw-Ihaw where there was a shooting incident. They rushed to the scene and there they saw the victim sprawled on the ground. Bystanders present pointed to the accused as the one who shot the victim. He was walking away

ACADCOM 2010; Contributors: Gene Geocaniga, Jarissa Guiani, Darlene Magabilen TAU MU Page 50 of 179

but was only about ten meters from Ihaw-Ihaw when they arrested him and seized his .38 caliber revolver.

CUDIA V. CA

TAU MU TAU MU TAU MU TAU MU TAU MU TAU MU TAU MU

On July 2, 1991, Rolito Go shot Maguan, a student. As a result of the investigation, the police immediately identified Go as the killer. The police conducted a manhunt for the arrest of Go. On July 8, Rolito Go, after reading in the newspaper that there was manhunt for him went to the police station while being accompanied by his two lawyers. It turned out that the witnesses to the shooting were also there. The witness

TAU MU

PEOPLE VS. GO

TAU MU

Held: Yes. It has been ruled that “personal knowledge of facts” in arrests without a warrant must be based upon probable cause, an actual belief or reasonable grounds of suspicion. In this case, there was an initial report to the police concerning the robbery. A radio dispatch was then given to the arresting officers, who proceeded to the place to verify. They met up with the complainants who initiated the report about the robbery. Upon the officer’s invitation, both mother and daughter boarded the mobile unit to join the search. The accused was spotted in the vicinity. Based on the reported statements of the complainants, he was identified as a logical suspect in the ofense committed. Moreover, at the time the police called out to accused, he was in the act of drawing his gun. Actual possession of an unlicensed firearm, which accused attempted to draw out, amounts to committing an ofense in the presence of the arresting officers.

TAU MU TAU MU

While the police were on patrol one evening aboard a vehicle, they received a radio call to proceed to a place where a hold-up had taken place. There, the victims said that the hold-uppers had fled so that the police requested them to board the patrol vehicles to facilitate the search. The victims described the color of the clothing of the 2 perpetrators. They later came upon 2 men walking whom the victims identified as the culprits. As one of the 2 attempted to pull something from his waist, a policeman pointed his firearm and frisked the suspect. A gun was taken from him. Was the arrest of accused valid?

The Fraternal Ateneo de Davao

TAU MU TAU MU

Held: A warrantless arrest is valid when the ofense has in fact just been committed and the arresting officer has personal knowledge of facts indicating that the person to be arrested has committed it. In this case, the policemen summoned to the crime scene saw the victim. Accused was pointed to them only moments after the shooting and he had not gone very far. The arresting officers thus acted on the basis of personal knowledge of the death of the victim and of facts indication that accused was the assistant.

KITY

TAU MU TAU MU TAU MU TAU MU TAU MU TAU MU

CONSTITUTIONAL LAW II Order of Saint Thomas More Atty. Philip John Pojas/Atty. Rovyne G. Jumao-as, RN University College of Law

pointed to him as the killer which prompted the police to arrest him on the spot. ISSUE: Whether Rolito Go was validly arrested. Held: Applying the principles of the law, the crime has not just been committed since a few days had already lapsed. The police also had no personal knowledge of the fact that he committed the crime. Their knowledge was just based on the statement of the witnesses. To say that the crime has just been committed, there must be immediacy of the time which is from the commission of the crime up to the arrest. Personal knowledge of facts means that his knowledge must be derived from his own personal sensed perception, not knowledge derived from investigation, nor from witnesses, nor from any secondary sources. PEOPLE VS GERENTE At 2pm of April 30, 1995, Clarito Blace was killed. At 4pm, the police learned of the killing and after which they immediately went to the hospital only to find out that the victim was pronounce DOA. The police went to the crime scene where they found a piece of wood with blood stains and a hollow block. They were able to interview an eyewitness to the incident who identified the accused as the killer. The police went to the house of the accused where they found him sleeping, and they arrested him. ISSUE: Was the arrest valid? Did the police have personal knowledge of the facts? Held: The court ruled that the arrest was valid. The crime has just been freshly committed. The police had personal knowledge because they saw the victim dead in the hospital and they also saw the instruments of the crime. (the wood and the hollow block). The police had also interviewed the eyewitness. PEOPLE V. OLIVARES In the morning of Dec. 26, the body of Mr. Sy was found by his employees. On Dec. 28, through the police focused on Arellano as a suspect. They visited him and he readily admitted participation in the crime. They invited him to the police station and he revealed the place where the Sanyo cassettes and a wristwatch owned by the victim were hidden. The police detained Arellano. Was the arrest valid? Held: No. At the time accused was apprehended, two days had already lapsed since the discovery of the crime. Accused had not just committed a criminal act. Neither was he caught in flagrante delicto or had escaped from confinement. Probably aware of the illegality of the arrest, the arresting officers testified that accused was merely invited to the police station.

ACADCOM 2010; Contributors: Gene Geocaniga, Jarissa Guiani, Darlene Magabilen TAU MU Page 51 of 179

Such invitation, however, when construed in he light of the circumstances is actually in the nature of an arrest designed for the purpose of conducting an interrogation. Mere invitation is covered by the proscription on warrantless arrests because it is intended for no other reason than to conduct an investigation. Thus by virtue of Sec. 3, Art. IV, of the Constitution, any evidence obtained, including the things allegedly stolen by accused which were taken by the police from the place of the illegal arrest, cannot be used in evidence. PEOPLE V. DEL ROSARIO

TAU MU TAU MU TAU MU TAU MU TAU MU TAU MU

Held: No. Neneth was not caught red-handed during the buy-bust operation to give ground for her arrest. Neither could it be justified under the theory that the arresting officers had personal knowledge that she had just committed an ofense. “Personal knowledge” of facts in arrest must be based on “probable cause” which means actual belief or reasonable ground of suspicion. The grounds of suspicion are reasonable when

TAU MU

In the course of the buy-bust operation, the police handed P1,600.00 to Doria who agreed to get the marijuana. When he returned an hour later with the marijuana, the police arrested him. He told the police that the P1,600.00 was with his co-accused Neneth. The police went to the house of Neneth and arrested her. Was the arrest legal?

TAU MU

PEOPLE V. DORIA supra.

TAU MU

In this case, the arrest of the accused did not comply with these requirements since the arrest came a day after the consummation of the crime. Likewise, the arresting officers had no personal knowledge of facts indicating that the person to be arrested had committed the ofense since they were not actual witnesses to the crime.

TAU MU TAU MU

Held: No. Under Sec. 5, para (b), Rule 113, of the Rules of Court, a warrantless arrest can be efected when: 1) an ofense had just been committed; and 2) the person making the arrest has personal knowledge of facts indicating that the person to be arrested had committed it. Hence, there must be a large measure of immediacy between the time the ofense was committed and the time of the arrest. Aside from the sense of immediacy, it is also mandatory that the person making the arrest must have personal knowledge of certain facts indicating that the person to be taken into custody had committed the crime.

The Fraternal Ateneo de Davao

TAU MU TAU MU

After the robbers snatched the bag of a woman and killed her, they boarded the tricycle of accused that 5:00 o’clock in the evening. Another tricycle driver recognized accused and reported to the police. Before lunch the following day, accused was arrested. Was the arrest valid?

KITY

TAU MU TAU MU TAU MU TAU MU TAU MU TAU MU

CONSTITUTIONAL LAW II Order of Saint Thomas More Atty. Philip John Pojas/Atty. Rovyne G. Jumao-as, RN University College of Law

the suspicion that the person to be arrested is probably guilty of committing the ofense is based on actual facts, i.e., supported by circumstances sufficiently strong in themselves to create the probable cause of guilt of the person to be arrested. A reasonable suspicion therefore must be founded on probable cause, coupled with good faith on the part of the person making the arrest. In this case, Neneth was arrested on the basis of the alleged identification by Doria. Doria did not point to her as associate in the drug business, but as the person with whom he left the money. Doria may have left the money in Neneth’s house with or without the latter’s knowledge. Save for Doria’s word, the police had no reasonable ground to believe that Neneth was engaged in drug business. PEOPLE V. BOLASA Tipped by an anonymous caller that a man and woman were repacking prohibited drugs at a certain house in Valenzuela, Metro Manila, the police went to the place. When they reached the house accompanied by their informer, they peeped inside through a window and saw a man and a woman repacking suspected marijuana. They entered the house, confiscated the items and arrested accused. Are the search and seizure and arrest valid? Held: No. The arrest was illegal. First, the arresting officers had no personal knowledge that at the time of the arrest accused had just committed, were committing or were about to commit an ofense. Second, the arresting officers had no personal knowledge that a crime was committed, nor did they have any reasonable ground to believe that accused committed it. The search was also illegal. Since the arrest was illegal, the search was not incident to a lawful arrest. Likewise, it was not also a search of evidence in plain view since the marijuana was not inadvertently discovered. Now was it a case of search of a moving vehicle, a consented search, customs search, stop and frisk or one done under exigent and emergency circumstances. PEOPLE V. POSADAS On December 8, 1994, a student of the University of the Philippines was killed in a rumble between fraternities. On December 12, on the basis of identification by 2 eyewitnesses, agents of the National Bureau of Investigation attempted to arrest several students. The University President as well as the counsel of the students objected to the arrest on the ground that there was no warrant of arrest. For this, they were charged with violation of PD No. 1829 or for obstructing the apprehension of criminal ofenders. Are they liable?

ACADCOM 2010; Contributors: Gene Geocaniga, Jarissa Guiani, Darlene Magabilen TAU MU Page 52 of 179

Held: No. The attempted arrest of the students could not be validly made without a warrant. First, the NBI agents tried to make the arrest 4 days after the commission of the crime when the law states that for a valid warrantless arrest it is required that “an ofense has in fact just been committed.” Second, they had no personal knowledge of any fact that might indicate that the suspects were guilty of a crime. What they had was the supposed positive identification of alleged eye-witnesses, which is insufficient to justify the arrest without a warrant. Indeed, at the time of the killing, these agents where nowhere near the scene of the crime.

TAU MU TAU MU TAU MU TAU MU TAU MU TAU MU

Nolasco and another companion was arrested while aon board on the jeepney at around 11:30AM at the intersection of Mayon & Margal St. At around 12:00nn of the same day, there was also a search conducted on the premises of 239-B Mayon St., which was believed to be the residence of the two. There was no mention of the distance where the two incidents happened. When they were arrested, there was only one warrant of arrest for one person. The

TAU MU

NOLASCO VS. PANO

TAU MU

INCIDENT TO ARREST:

TAU MU

Held: No. For a warrantless arrest to be valid, it is required that the ofender has just committed an ofense and the arresting officer had personal knowledge of facts indicating that the person arrested has committed it. In this case, the first requisite is satisfied because the arrest of accused was efected shortly after the victim was killed. But they had no probable cause to believe that accused committed the crime. Their knowledge of facts and circumstances from which they inferred that accused was guilty was based entirely on what they had been told by others: to wit, and report of the killing, the information that he was at the café with the victim, and the description given by the tricycle driver. At the time he was arrested, he was not doing anything overtly criminal. WARRANTLESS SEARCHES

TAU MU TAU MU

After being informed that a person has been shot, the police send a team to investigate. They saw the victim slumped on his tricycle parked near the road. After talking to a waitress and a tricycle driver who gave them the description of the person last seen with the victim, the police went to the house of accused. They knocked and when accused opened the door, they noticed that he fitted the description. Entering the house, they found a “bloodied” Hanes T-shirt placed over a divider. When the police picked it up, two spent .38 caliber shells fell from it. They then brought him to the café where the waitress identified him as the person she saw drinking beer with the victim. Was the arrest valid?

The Fraternal Ateneo de Davao

TAU MU TAU MU

PEOPLE V. CUBCUBIN

KITY

TAU MU TAU MU TAU MU TAU MU TAU MU TAU MU

CONSTITUTIONAL LAW II Order of Saint Thomas More Atty. Philip John Pojas/Atty. Rovyne G. Jumao-as, RN University College of Law

operatives also had a search warrant but it turned out that the warrant was void. ISSUE: whether the subsequent search conducted can be justified as an incident to a lawful arrest by virtue of an arrest warrant. Held: In the first case, the court ruled that the warrantless search conducted was valid. According to the court, it was incident to a lawful arrest because the search was conducted within the general vicinity of the place where the arrest had taken place – this is as to the element of place. As to the element of time, the 30 min gap was still allowable. (This was based on the old doctrine of incident to a lawful arrest). In the motion for reconsideration, the court already declared this doctrine illegal. The court ruled that a search incident to a lawful arrest is only valid if it is conducted on: (1) the person of the accused; and (2) the premises within his immediate control – that is to search him of weapons and evidences of the crime. With regards to time, the search should be contemporaneous with the arrest. Meaning, the search should be conducted simultaneously with the arrest. (Immediately after or during) PEOPLE V. LUA Policemen conducted a buy-bust operation to entrap accused who was then standing outside the door of his house. After the sale was consummated, they arrested him. When asked by the police, accused informed them that the rest of the marijuana was inside his house. Accompanied by the accused they went in and found a brick dried marijuana. Is it admissible in evidence? Held: No. While the arrest was lawful, the search made inside the house of the accused became unlawful since the police operatives were not armed with a search warrant. Such search cannot fall under “search made incidental to a lawful arrest” the same being limited to body search and to the point within reach or control of the person arrested, or that which may furnish him with the means of committing violence or of escaping. PEOPLE VS. MUSA The police conducted a “buy-bust operation”, an instance where in a warrantless arrest is considered as legal (Selling marijuana is a valid ground for a policeman to make an arrest, even without a warrant). The incident took place in the sala of the accused. The policeman used marked money during the operation. While they were looking for the money, they found a plastic bag in the kitchen which after opening, contained marijuana. ISSUE: Can the arresting officers raise the issue that this was incident to a lawful arrest? Since the

ACADCOM 2010; Contributors: Gene Geocaniga, Jarissa Guiani, Darlene Magabilen TAU MU Page 53 of 179

accused was a dealer of marijuana. Therefore the marijuana found should be admissible as evidence. Held: The search was not incident to a lawful arrest because the marijuana was not obtained in the person of the accused nor in the place within his immediate control. It would be valid if Musa were in the kitchen when the bag was found. ESPANO V. CA

was passing by, the policeman called for assistance from his precinct so that a team was sent to the area. Subsequently, a “trisikad” carrying accused passed by. The policemen aboard a vehicle overtook it and ordered one of the accused to open the bag he was carrying. The bag contained marijuana. Was the search incident to a lawful arrest? Held: No. “Reliable information” alone, absent any overt act indicative of a felonious enterprise in the presence or within the view of the arresting officers, are not sufficient to constitute probable cause that would justify an in “flagrant delicto” arrest. In this case, accused manifested no outward indication that would justify their arrest. In holding a bag on board a “trisikad”, accused could not be said to be committing an ofense. Apart from the fact that they were pointed to by an informer, accused could not be the subject of any suspicion, reasonable or otherwise. Since the arrest was illegal, the search conducted on their person is likewise illegal. Consequently, the marijuana seized could not be admitted in evidence. Pp vs. Molina – there was an information that a certain man was carrying shabu. Search was made and found were illegal drugs. There was no overt act, however, indicative of a felonious act. HELD: The search was invalid since there was no valid arrest. CONSENTED SEARCH: LOPEZ VS. COMMISSIONER The NBI searched a room in Skylark Hotel, which was occupied by Veloso. The NBI did not possess a warrant at that time. When they knocked and entered the room, there was a woman inside who introduced herself as the wife of Veloso. When the agents stated their purpose, the woman gave them consent to search the place. In efect, they confiscated the illegal documents and firearms, which they used as evidence in court against Veloso. Veloso contested this because the woman in the room had no right to give the consent for the policeman to enter the room.

TAU MU

Held: The court ruled that the consent given by the woman is sufficient to validate the search. It would be awkward if the agents would require the woman to produce her marriage license to prove his identity. On the other hand, this has been modified by the next case.

TAU MU

TAU MU

PEOPLE VS. DAMASO

TAU MU TAU MU

After receiving an information that a drug pusher, whom he put under surveillance earlier,

TAU MU

PEOPLE V. MOLINA

TAU MU

Held: No. For a search incident to a lawful arrest to be valid, the search must have been conducted at about the time of the arrest or immediately thereafter and only at the place where the suspect was arrested, or the premises or surrounding under his immediate control. In this case, accused was arrested in the act of delivering shabu outside Unit 122. Moreover, Unit 122 was not even his residence but that of his girlfriend and that he was merely a sojourner there. Hence, it can hardly be said that the inner portion of the home constituted a permissible area within his reach or immediate control to justify a warrantless search.

TAU MU

As soon as accepted stepped and handed the “shabu” to a buyer in a buy bust operation just outside the door of Unit 122 of Roxas Seafront Garden in Pasay City, the police arrested him. The police then proceeded to search Unit 122 and at the second floor found plastic bags containing 5,578.8 grams of shabu. Unit 122 belonged to the girlfriend of accused. Was the search incident to a lawful arrest valid?

TAU MU

PEOPLE V. CHE CHUN TING

TAU MU TAU MU

Held: Accused was caught in flagranti handling something to a buyer. He was in the act of committing an ofense so that his arrest was lawful, and the 2 plastic bags of marijuana are admissible in evidence, being fruits of the crime. It was search incident to a lawful arrest. However, the ten plastic bags of marijuana found in his residence are not admissible because the policemen were not armed with a warrant. At the time of his arrest, his house was beyond his reach and control.

The Fraternal Ateneo de Davao

TAU MU TAU MU

To confirm reports of drug pushing, policemen stationed themselves in a street corner in Metro Manila. They saw accused selling something to another person. When the buyer left, they approached accused, frisked him and found 2 tea bags of marijuana. When asked if he had more, he replied that he had them in his house. They went to his house and found 10 more plastic tea bags of marijuana.

KITY

TAU MU TAU MU TAU MU TAU MU TAU MU TAU MU

CONSTITUTIONAL LAW II Order of Saint Thomas More Atty. Philip John Pojas/Atty. Rovyne G. Jumao-as, RN University College of Law

The PC conducted a search on the house rented by the accused, who was believed to be a member of the NPA. The PC had no search warrant. That time, only the helper was present. She gave them the consent to enter and search the house and rooms wherein they obtained firearms. The accused challenged the admissibility of the evidences.

ACADCOM 2010; Contributors: Gene Geocaniga, Jarissa Guiani, Darlene Magabilen TAU MU Page 54 of 179

Held: The court stated that the evidences are inadmissible. Only the person whose rights will be invaded can give the consent to a search. Only Damaso can waive his rights. Consent can also be given by a person authorized to do so. Such authorization must be expressly given by the owner to a caretaker. VEROY VS. LAYAGUE

TAU MU TAU MU TAU MU TAU MU

Based on a tip supplied by an informant, the police conducted a surveillance of the residence and saw 2 tall marijuana plants in the backyard of accused in San Carlos City. On July 12, 1995, the police applied for a warrant in Bacolod City but the judge told them that he had no territorial jurisdiction over the matter. They

TAU MU

PEOPLE V. COMPACION

TAU MU

Held: There was no probable cause for the search. The radio communications from Gen. Nazareno pertained to cases of robbery and holdups, not drug pushing. While the acts of the accused might have annoyed the police, or caused them to suspect that something was amiss, it does not constitute probable cause to conduct an extensive search. However, as accused expressly consented to the search, the marijuana is admissible in evidence under the principle of waiver.

TAU MU

The police received an order form Gen. Nazareno to monitor strategic places in the City and barangays of Manila. While thus patrolling on a police car, they noticed a taxi with two passengers. One was beside the driver while the other was at the back seat. The two bowed their heads and slouched on their seats refusing to look at the police. Their suspicion aroused, the police stopped the taxi. They asked permission to search the vehicle and the occupants agreed. In the course of search, the found marijuana inside a plastic bag.

TAU MU

PEOPLE V. LACERNA

TAU MU

Held: The court held that the firearms were not admissible as evidence since the warrant was for the search of rebels, not firearms. There was a prior justification for the intrusion. On the other hand, the police did not come across the guns inadvertently since they were inside the drawer.

TAU MU TAU MU

ISSUE: Are the firearms admissible in court, even if the warrant was for the search of rebels?

The Fraternal Ateneo de Davao

TAU MU TAU MU

Here was a report that there were rebels in the house of Veroy. During that time, Veroy was out of town. To conduct the search, the police asked the permission from Veroy, through a long distance call to search the house for rebels. In efect of the search, they were able to recover firearms, but there were no rebels.

KITY

TAU MU TAU MU TAU MU TAU MU TAU MU TAU MU

CONSTITUTIONAL LAW II Order of Saint Thomas More Atty. Philip John Pojas/Atty. Rovyne G. Jumao-as, RN University College of Law

went to San Carlos City but the judge there told them to go back the following day, it was already night-time. At about 1:30 in the morning of the following day, without a search warrant, the police arrived at the house of accused who let them into the gate without objection. The police found the marijuana. Was there a waiver of the right against warrantless searches? Held: No. While the right against unreasonable searches and seizures may be waived expressly or impliedly, such waiver must be made voluntarily, knowingly and intelligently. The acts of the accused in allowing the police to enter the premises and his consequent silence during the search should not be construed as voluntary submission or and implied acquiescence especially so when members of the raiding team were numerous. His implied consent, if any, could not have been more than mere passive conformity given under coercive circumstances, and is, thus, no consent at all. Consequently, his lack of objection is not tantamount to waiver of a constitutional right. PLAIN VIEW: PEOPLE VS. EVARISTO The police while on patrol heard gunfires. They saw Rosillo in the act of firing his gun in the air. When they tried to approach him, Rosillo ran away and entered a house. The police followed him. When they came to the house, they asked Evaristo, the owner, about the suspect’s whereabouts. Evaristo replied that Rosillo was no longer in the house. Anyway, the police were able to get Evaristo’s consent to conduct a search of the house. It turned out that the house was full of firearms. ISSUE: Are the evidences recovered admissible in court? Held: The court ruled that they are. The police had prior justification for the intrusion since they were given consent by Evaristo. They also came across the guns inadvertently, and the illegality of the objects are readily apparent. PEOPLE V. QUE MING KHAN The police received an information that a van with plate No. UPN 595 which was being used in the transport of shabu was seen at the vicinity of barangay Holy Spirit, Quezon City. Three teams were dispatched to the area, one of which spotted the van, which hit a 7 year old boy. The van stopped and the owner got of to bring the boy to a hospital. A police officer approached the van to apprehend the driver for reckless imprudence. As he stood near the van, he saw through the lightly tinted window several sacks. One sack was open and he noticed white plastic bags containing crystalline substance, which turned out to be shabu. Is the shabu admissible in evidence?

ACADCOM 2010; Contributors: Gene Geocaniga, Jarissa Guiani, Darlene Magabilen TAU MU Page 55 of 179

Held: Yes. Objects falling in plain view of an officer who has a right to be in a position to have that view are subject to seizure even without a search warrant. The plain view doctrine applies when (a) the law enforcement officer has a prior justification for an intrusion or is in a position form which he can view a particular area; (b) the discovery of the evidence in plain view is inadvertent; and (c) the illegality of the object is immediately apparent. Clearly, the prohibited substance was in plain view of the police officer who was in a position to be near the van at the time. The substance is therefore not the product of an illegal search.

TAU MU TAU MU TAU MU TAU MU

Held: No. Seizure is limited to those items particularly described in the warrant . In this case, the firearm was not found inadvertently and in plain view. It was found as a result of a meticulous search in the kitchen. This firearm was not mentioned in the search warrant. Hence, the seizure was illegal. True that as an exception, the police may seize without a warrant illegally possessed firearm or any contraband inadvertently found in plain view. However, the seizure in plain view applies only where the police officer is not searching for evidence against the accused, but inadvertently comes across an incriminating object, which is not the case here.

TAU MU

The judge issued a search warrant authorizing the search and seizure of a .45 caliber pistol at the residence of accused. In the course of the search, during which they found the gun which turned out to be licensed, the policemen also seized a .22 caliber revolver, not described in the warrant, from his daughter’s bedroom. Is the .22 caliber revolver admissible in evidence?

TAU MU

DEL ROSARIO V. PEOPLE

TAU MU

NOTE: The possession of parts of a gun will also hold a person liable under the crime of illegal possession of firearms.

TAU MU

Held: Basing on the Musa case, they should be held as admissible in court. But according to the court, they are not. The authority given by the court was only to obtain the shabu and paraphernalia. They had no authority to get the firearms and ammunitions. The warrant specifically defines the object to be searched. It would seem that there should be some modification in the requisites of search in plain view.

TAU MU

ISSUE: Are these admissible in court?

TAU MU TAU MU

The policemen had a search warrant for the seizure of shabu and their paraphernalia. When the police entered the house, they found a gun and a cup filled with bullets on top of the television set.

The Fraternal Ateneo de Davao

TAU MU TAU MU

PEOPLE VS. DEL ROSARIO

KITY

TAU MU TAU MU TAU MU TAU MU TAU MU TAU MU

CONSTITUTIONAL LAW II Order of Saint Thomas More Atty. Philip John Pojas/Atty. Rovyne G. Jumao-as, RN University College of Law

PEOPLE V. SALANGUIT The Search Warrant issued by the judge ordered the seizure of “UNDETERMINED QUANTITY OF SHABU AND DRUG PARAPHERNALIA.” During the search, however, in addition to the shabu, the police found and seized 2 bricks of dried marijuana leaves weighing 1,254 grams. Is the marijuana admissible in evidence? Held: No. While police officers with a search warrant may seize contraband items in plain view even if not described in the warrant, the “plain view doctrine” cannot apply here. The doctrine requires (1) prior justification for the intrusion; (2) inadvertent discovery; and (3) immediate apparent illegality. Because the police knew that the shabu was in the cabinet, it is reasonable to assume that they found it first. Once the valid portion of the warrant has been execute, the “plain view” doctrine can no longer serve as valid basis for admitting the other items subsequently found. In addition, the marijuana bricks were wrapped in newsprint. Its illegality, therefore, was not immediately apparent to justify its seizure. PEOPLE V. DORIA, In the course of the buy bust operation, the police handed P1,600.00 to Doria who proceeded to get the marijuana. When he returned an hour later with the marijuana, the police arrested him. He told the police that he left the buy bust money to Neneth and the police went to the latter’s house. Standing by the door, they saw a carton box under the dining table. The box was partially open and revealed something wrapped in plastic. As the plastic looked similar to the one they seized from Doria, the police opened it and found marijuana. Was the marijuana seized in plain view? Held: No. For the plain view doctrine to apply, the following requisites must concur: 1) the officer has prior justification for the intrusion or is in a position from which he can view a particular area; 2) the discovery of the evidence in plain view must be inadvertent; and 3) it is immediately apparent to the officer that the item in contraband or otherwise subject to seizure. In this case, it was not immediately apparent to the police that the box contained marijuana. The plastic wrapper was not colorless and transparent as to manifest its contents to the viewer. Each of ten bricks of marijuana in the box was individually wrapped in old newspapers and place inside plastic bags --- white, pink or blue in color. The seizure of the marijuana therefore violated the Constitution. PEOPLE V. ELAMPARO After the buy bust the police arrest Spencer, who was able to free himself and run

ACADCOM 2010; Contributors: Gene Geocaniga, Jarissa Guiani, Darlene Magabilen TAU MU Page 56 of 179

inside the house of appellant. The police pursued Spence inside the house and found appellant repacking 5 bricks of marijuana on top of a table. They arrested appellant and confiscated the marijuana. Is the marijuana admissible in evidence?

TAU MU TAU MU TAU MU TAU MU TAU MU TAU MU

While a policeman was urinating at a fence behind a bamboo school, he saw a garden of about 70 square meters with corn and camote tops. Concealed by the corn, however, were marijuana plants. The policeman asked from a storekeeper nearby as to who owned the garden and he was told that it was accused. He reported to the Chief of Police who dispatched a team of policemen an hour later. Going straight to the house of accused, they asked him to bring the team to the backyard garden which was just 5

TAU MU

PEOPLE V. PASUDAG

TAU MU

Held: No. Considering that the informant had revealed the name of accused as well as the place were the marijuana was planted and the police had at least one day to obtain a search warrant, they had no reason not to obtain one. The plain view doctrine cannot apply. The seizure of evidence in plain view applies where the police inadvertently came across the object. In this case, the police team was dispatch precisely to search the prohibited flora. It must also be noted that they first had to “look around the area” before they could spot the illegal plants. Patently, the seized marijuana plants were not “immediately apparent.” Thus, the plain view doctrine cannot be made to apply.

TAU MU

At about 10:15 o’clock in the morning, the police were informed of the presence of a marijuana plantation. At 5:00 o’clock in the morning the following day, a police team, accompanied by the informer, left for the site were the marijuana plants were grown. After a 3hour uphill trek from the nearest barangay road, they arrived at the place and found accused in his nipa hut. They looked around the unfenced “kaingin” and saw 7 five-foot high marijuana plants planted approximately 25 meters from his hut. Are the marijuana plants admissible in evidence?

TAU MU TAU MU

PEOPLE V. VALDEZ

The Fraternal Ateneo de Davao

TAU MU TAU MU

Held: Yes. The case falls under the plain view doctrine. Under the said doctrine, the following requisites must concur; 1) the law enforcer has a prior justification for the intrusion; 2) the discovery of the evidence in plain view is inadvertent, and 3) the illegality of the object is immediately apparent. In this case, the policemen were justified in running after Spencer and entering the house without a search warrant for they were not in the heels of a fleeing criminal. They also caught appellant in flagrante delicto repacking the marijuana bricks which were in full view on top of the table.

KITY

TAU MU TAU MU TAU MU TAU MU TAU MU TAU MU

CONSTITUTIONAL LAW II Order of Saint Thomas More Atty. Philip John Pojas/Atty. Rovyne G. Jumao-as, RN University College of Law

meters away. The police team uprooted the 7 fully grown marijuana plants. Was the search and seizure valid? Held: No. The marijuana plants were seized in violation of the rights against unreasonable searches and seizures. The policeman had ample time to secure a warrant. He knew who owned the garden. He was acquainted with the marijuana plants and immediately recognized them. Time was not of the essence to uproot and confiscate the plants. They were three months old and there was no sufficient reason to believe that they would be uprooted on that same day. PEOPLE VS. MUSA This was a buy-bust operation. Information was obtained that a certain Musa was selling marijuana. A test-buy was conducted and true enough marijuana was brought the following day. A buy-bust was arranged. Sgt Ani posed as a prospective buyer. After Musa gave him marijuana and after getting the marked P20.00 bill, Musa was arrested. In the living room, they could not find the P20.00 bill. Hence, they proceeded to the other parts of the house and found in the kitchen a plastic bag which when opened contained dried marijuana leaves. Held: No valid search. Although it was a valid arrest, the subsequent search in the kitchen was not. It cannot be considered as a surrounding within his immediate control. The prosecution’s contention that it was evidence in plain view is untenable because the evidence of illegality is not apparent. They have to open the plastic bag to know what was contained therein. The marijuana there in the plastic bag is inadmissible in evidence. PEOPLE VS. MUSA (as discussed by Ms. Jumao-as under plain view) - (People vs. Musa, supra) Held: The court ruled that this was not a search in plain view. The police did not come across the evidences inadvertently. There was prior justification for the intrusion but it is limited only within the immediate vicinity. Also, the illegality was not readily apparent since it was inside a plastic bag. STOP AND FRISK: POSADAS VS. CA Posadas was carrying a buri bag. When he was stopped by the police, he tried to run away but he failed to escape. The police found guns and ammunitions inside his bag. The police reasoned that he was acting suspiciously and that he was acting suspiciously and that is why they searched him.

ACADCOM 2010; Contributors: Gene Geocaniga, Jarissa Guiani, Darlene Magabilen TAU MU Page 57 of 179

Held: The court ruled that the search was valid. There was probable cause(acting suspiciously) for the police to conduct the search. The suspect was looking side-to-side and he tried to flee when the police stopped him. PEOPLE VS. MENGOTE

TAU MU TAU MU TAU MU TAU MU TAU MU TAU MU TAU MU

Held: There was a valid stop-and-frisk search. “Where a police officer observes an unusual conduct which leads him reasonably to conclude in the light of his experience that criminal activity may be afoot and that the person with whom he is dealing may be armed and presently dangerous, where in the course of investigating this behavior he identified himself as a policeman and makes reasonable inquiries, and where nothing in the initial stage of the encounter serves to dispel his reasonable fear for his own or other’s safety, he is entitled for the protection of himself and others in the area to conduct a carefully limited search of the outer clothing of such persons in an attempt to discover weapons which might be used to assault him.” [Terry b. Ohio]. In this case, the policemen, based on the

TAU MU

At 2:00 o’clock in the afternoon, policemen form Caloocan were conducting a surveillance in response to a report that drug addicts were roaming in front of Caloocan Cemetery. They chanced upon a male person observed to have reddish eyes and walking in a swaying manner as though he was high on drugs. When he tried to avoid the policemen, they approached him and introduced themselves. They asked him what he was holding but he tried to resist. He later showed the wallet he as holding and the police found crushed marijuana residue inside.

TAU MU

MANALILI VS. CA

TAU MU TAU MU

Held: The court ruled that the firearm was not admissible as evidence. The search was not valid since there was no probable cause. When the court tried to look at the facts how the police arrived to the conclusion that the suspects were acting suspiciously, they found it insufficient. Looking side-to-side or holding one’s abdomen does not necessarily constitute acting suspiciously. As to the illegal possession of firearm, the court ruled that the police had no personal knowledge that the suspect was actually carrying a gun at that time. Subsequent discovery of the firearm will not cure the illegality of the search.

The Fraternal Ateneo de Davao

TAU MU TAU MU

The police received a phone call that there were three suspiciously looking men at a street corner in Tondo. The police found 2 men looking side-to-side, and one had his hand on his abdomen. When the police approached them, the two tried to flee. The police caught up with them and they were searched. The police found a .38 caliber pistol with 6 live ammunitions.

KITY

TAU MU TAU MU TAU MU TAU MU TAU MU TAU MU

CONSTITUTIONAL LAW II Order of Saint Thomas More Atty. Philip John Pojas/Atty. Rovyne G. Jumao-as, RN University College of Law

circumstances, had sufficient reason to stop accused and investigate if he was high on drugs. PEOPLE V. ENCINADA At 4:00 o’clock in the afternoon of May 20, 1992, the police received an intelligence report that appellant was arriving in Surigao City aboard M/V Sweet Pearly in the morning of May 21. The following day, as accused was disembarking, the police watched him. He walked briskly down the gangplank carrying 2 plastic baby chairs. He took a motorella which was stopped by the police who then searched the chairs where they found 88 grams of marijuana leaves. Held: the search was illegal. Accused was not committing a crime in the presence of the policemen. He was not acting suspiciously when he disembarked from the ship or while he rode the motorella. Moreover, the lawmen did not have personal knowledge of facts indicating that the person to be arrested had committed an ofense. The search cannot be said to be incidental to a lawful arrest. Raw intelligence information is not a sufficient ground for a warrantless arrest. Nor can the silence of accused be taken as consent to the warrantless search. The police cannot also claim that there was no time to obtain a warrant since Administrative Circular No. 19, Series of 1987, allows application for search warrants even on Sundays and holidays or after office hours. PEOPLE V. CHUA HO SAN In response to reports of smuggling, the Chief of Police of Bacnotan, La Union, ordered the patrolling of coastlines. One past midnight, they were informed of an unfamiliar speedboat about to dock. They came to the place in time to see a male passenger disembark carrying a strawbag. They accosted him when he changed direction and tried to run. The man turned out to be a Taiwanese who spoke no English and his bag contained 28.7 kilos of shabu. Was the search lawful? Held: No. There was no probable cause for the search. There was no report that a foreigner would disembark bearing prohibited drug on that date. The fact that the vessel that ferried him looked unusual does not suggest that he was perpetrating an ofense. True, accused entered Philippine territory without a visa but that was not obvious to the police. What was clear was that accused was alighting from a speedboat, walking casually towards the road and carrying a strawbag. These acts did not convey that he illegally entered Philippine shores. Nor were these manifestations of an on-going felonious activity. MOVING VEHICLES: VALMONTE VS. DE VILLA

ACADCOM 2010; Contributors: Gene Geocaniga, Jarissa Guiani, Darlene Magabilen TAU MU Page 58 of 179

PEOPLE VS. MALMSTEADT

TAU MU TAU MU TAU MU

There were several people including Exala riding a jeepney and they were stopped in a checkpoint. While the police were conducting a routine check and asking questions, they noticed a black leather bag with budging sides. An officer asked what the contents of the bag were but nobody responded. The police noticed the demeanor of the accused hanged. They become suspiciously quiet and nervous as if they were

TAU MU

PEOPLE VS. EXALA

Held: According to the court, the evidence is admissible. There was probable cause since the accused were acting suspiciously. They did not seem to be acting normally which gave the police a reason to an exclusive search. Likewise, there were no protests and objections regarding the search. It was an implied waiver of the right against unreasonable searches and seizures. ANIAG VS. COMELEC

Held: In searches made of moving vehicle, extensive search can only be done if there is existence of probable cause. In this case, the SC ruled that there was no probable cause to conduct a more extensive search. The driver was not acting suspiciously. He did not become fidgety. The guns were even hidden. They could not be seen by mere visual search. They have to open the trunk and the bag for the guns to be apparent. PEOPLE VS. CLAUDIO

TAU MU

Held: According to the court, the evidence is admissible. There was probable cause based on the report which also gave a description of the suspect.

TAU MU

The NARCOM received a report that a person is transporting marijuana. The report described the person as a woman, 5’2” in height, about 25 years of age, with short curly hair. The agents put up check points and conducted extensive searches on the buses. In one of the buses, they saw a woman matching the description sitting quietly at the rear, with a bag on her lap. When they searched the bag, they found marijuana.

TAU MU

PEOPLE VS. BAGISTA

COMELEC issued resolutions for implementation of the gun ban during the election period in preparation for the coming elections as well as the authority for the COMELEC to establish checkpoints. Consequently, the Congress sergeant-at-arms ordered the return of 2 firearms issued to Aniag. In compliance, Aniag ordered his driver to pick up the guns at his residence and deliver the same to the sergeant-at-arms. PNP established a checkpoint 30 meters from the Batasan Complex, so when the car passed by, it was flagged down. The car was inspected and extensively searched. The trunk was opened and a bag was found containing guns neatly packed in their cases. The driver was apprehended and a case was filed for violation of the resolution and PD 1866.

TAU MU

Held: The court held that the search was legal. 1.) There was probable cause based on the report 2.) The accused acted suspiciously since he refused to give his passport. 3.) There was a bulge on his waist. The search was also valid since this was a search of moving vehicles. Requiring a warrant gives the suspect time to flee.

trying to connect something. When the bag was opened, the police found it to contain marijuana.

TAU MU

ISSUE: Is the marijuana admissible in court?

Ateneo de Davao

TAU MU TAU MU

The police received a report that a foreigner from the Sagada will be riding a bus carrying marijuana. The police immediately put up check points and inspected the buses coming from that direction. During the checks, they saw a Caucasian matching the description. The police noticed a bulge on his waist. When they asked for his passport, he did not comply. They then ordered him to turn of the pouch on his waist. The police found it to contain marijuana.

The Fraternal

TAU MU TAU MU

The government put up check points in Manila because some problems in peace and order. Valmonte sought to dismantle these check points for being unconstitutional. The police conducted arbitrary searches on the vehicles and the passengers without a valid warrant. Held: The court upheld the constitutionality of the search because removing it will hamper the police from performing some of their functions.

KITY

TAU MU TAU MU TAU MU TAU MU TAU MU TAU MU

CONSTITUTIONAL LAW II Order of Saint Thomas More Atty. Philip John Pojas/Atty. Rovyne G. Jumao-as, RN University College of Law

There was a police officer who aboard a bus bound for Olongapo. While he was riding, there was a woman who boarded the bus and seated in front of the officer. However, the woman placed her bag at the rear seat of the bus she sat. This act of the woman raised the suspicion of the policeman. At the next station, when the woman went down to the bus maybe just to relieve herself in a CR. The policeman went to the bag and placed his hand inside the bag. When he smelled his fingers, it smelled like marijuana. He did not immediately arrest the woman. At the next bus stop, the policeman unboarded the bus at the same time he also invited the woman to the police station. In short he arrested the woman. In the station they searched the bag and they found marijuana. ISSUE: Is the marijuana admissible based on the facts presented?

ACADCOM 2010; Contributors: Gene Geocaniga, Jarissa Guiani, Darlene Magabilen TAU MU Page 59 of 179

Held: In search of moving vehicle, you can only conduct extensive search if there is probable cause. According to the S, the evidence is admissible since the search was done in the police station. The police had reason to do the search and arrest because he already knew that she is already committing a crime, she was already carrying marijuana. The court ruled that it was a search incident to a lawful arrest. PEOPLE V. QUE

TAU MU TAU MU TAU MU TAU MU TAU MU

PEOPLE V. GONZALES

TAU MU

Held: Yes. The police officer, faced with an on the spot information from the asset, had to respond quickly to the call of duty. There was no time to obtain a warrant. It must be noted that the target of the pursuit was jut the “thin Ilocano person with a green bag” and no other. Evidently, there was definite information of the identity of the person engaged in transporting marijuana at a particular time and place. The police officer had probable cause to stop the buses in view of the information. He likewise had probable cause to search accused since he fitted the description. As the warrantless search was valid, the marijuana taken is admissible.

TAU MU

While the policeman was waiting for a ride, he was approached by an informer and told that an Ilocano person was going to transport marijuana. The informer described the suspect as thin and carrying a green bag. At a certain place, the policeman and the asset stopped two buses and on the second, the policeman saw accused who matched the description. When searched, the bag accused carried yielded marijuana. Is the marijuana admissible?

TAU MU

PEOPLE V. VALDEZ

TAU MU TAU MU

Held: Yes. There was probable cause to justify the extensive search of appellant’s truck even without a warrant. There was a reliable information which included the plate number of the truck. The truck the police apprehended matched the description of the truck given by the informant. Accused admitted to the police that there were sawn lumber. Finally, accused could not present the lumber’s supporting documents when asked by the police.

The Fraternal Ateneo de Davao

TAU MU TAU MU

The Task Force on Illegal Logging received an information that a ten-wheeler truck bearing plate number PAD-548 loaded with illegality cut lumber would pass through. Two weeks later, when policemen went on patrol, they saw the truck which they stopped. The truck owner aboard admitted that the truck was carrying sawn lumber inserted in between coconut slabs. An examination of the cargo confirmed his statement. Was there a valid search?

KITY

TAU MU TAU MU TAU MU TAU MU TAU MU TAU MU

CONSTITUTIONAL LAW II Order of Saint Thomas More Atty. Philip John Pojas/Atty. Rovyne G. Jumao-as, RN University College of Law

The police received an information tat a woman with long hair, wearing “maong” pants and jacket, and Ray Ban sunglasses would be transporting marijuana along the national highway. According to the information, the woman would bring a black traveling bag and would ride a “trisikad”. Based on this information, policemen were set to the area. They found the woman, ordered her to open the black bag, which she refused. The police opened it and found marijuana. Is it admissible evidence? Held: Yes. The arrest was legal and the search of her bag justified. The police had no time to obtain a warrant since they were tipped of only in the evening and the contraband would be transported early in the morning of the following day. The law enforcers had a definite target of the arrest. There was a description of the identity of the person engaged in transporting prohibited drugs at a particular place and time. Accordingly, when they saw accused who fitted the description they had probable cause to apprehend her. SEARCH IN THE CUSTOM LAWS:

ENFORCEMENT

OF

PAPA VS. MAGO Customs Authorities received information that there were misdeclared or undervalued goods which were to be released in the port of Manila. This prompted to put up surveillance. They saw two trucks presumably full of goods and which they immediately pursued. Without the benefit of a search warrant, they seized the items loaded in the truck. ISSUE: Is the search permissible? Is it legal? Held: According to the court, it was a search under Tarifs And Customs Code and it can be done without a search warrant provided there is probable cause. The court provided several requisites where you can conduct warrantless searches under the Tarifs and Customs Code. HIZON V. CA Members of the Maritime Command, in response to reports that illegal fishing activities were taking place in the coastal waters of Palawan, boarded F/B Robinson, a fishing vessel. The following day, after having placed the vessel under guar, they obtained five fish samples from the catch of the fishermen. A laboratory examination of the sample showed that it was positive of sodium cyanide, a poisonous substance. Was the warrantless search lawful?

TAU MU

Held: Search and seizure without search warrant of vessels for violation of custom laws have been traditionally regarded as an exception to the constitutional requirement of a search warrant. It is rooted on the recognition that a vessel can be quickly moved out of the locality or jurisdiction in

ACADCOM 2010; Contributors: Gene Geocaniga, Jarissa Guiani, Darlene Magabilen TAU MU Page 60 of 179

which the search warrant of vessels before their search and seizure can be validly afected. The same exception ought to apply to seizure of fishing vessels and boats breaching fishery laws. These vessels are normally powered by highspeed motors that enable them to elude arresting ships of government authorities enforcing fishery laws. BUREAU OF CUSTOMS V. OGARIO

TAU MU TAU MU TAU MU TAU MU TAU MU

Held: According to the court, they will allow searches without the warrant because it was an abnormal time and the police do not have the opportunity to secure a search warrant. In this case, the judge even testified that his court was not always open because of the tension existing

TAU MU

ISSUE: Whether the evidence obtained is admissible considering the search was without a warrant.

TAU MU

Accused was charged with illegal possession of firearms which was obtained from a warrantless search. The search was conducted at the height of the coup attempt in 1989. The authorities were conducting searches and surveillance as counter-measures to stop the activities of the rebel forces in one of the searches, the officers were able to obtain firearms from the accused.

TAU MU

PEOPLE VS. DE GRACIA

TAU MU

SEARCH DURING EXIGENCY:

TAU MU TAU MU

Held: No. Regional Trial Court are devoid of any competence to pass upon the validity of seizure and forfeiture proceedings in the Bureau of Customs. The Collector of Customs sitting in a seizure and forfeiture proceedings has the exclusive jurisdiction to hear and determine all questions touching on the seizure of dutiable goods. The question on whether probable cause exists for the seizure of the rice is not for the RTC to determine. The customs authority do not have to prove to the satisfaction of the court that the articles on board a vessel were imported from abroad before they may exercise the power to efect customs searches, seizures or arrest. Actions of the collector of customs are appealable to the Commissioner of Customs, whose decision in turn is subject to the exclusive appellate jurisdiction of the Court of Tax Appeals.

The Fraternal Ateneo de Davao

TAU MU TAU MU

The District Collector of Customs of Cebu issued a Warrant of Seizure and Detention of 25,000 sacks of rice on the basis of a report that the rice had been illegally imported. Forfeiture proceedings were subsequently started in the custom’s office in Cebu. Upon complaint, however, the Regional Trial Court of Cebu ordered the return of he rice to the consignee. Does the Regional Trial Court have the power to quash the warrant of seizure issued by the collector of customs on the ground of lack of probable cause in its issuance?

KITY

TAU MU TAU MU TAU MU TAU MU TAU MU TAU MU

CONSTITUTIONAL LAW II Order of Saint Thomas More Atty. Philip John Pojas/Atty. Rovyne G. Jumao-as, RN University College of Law

at that time. But this is just a very exceptional circumstance.

SEARCH BY PRIVATE PERSONS PEOPLE VS. MARTI Petitioner here was trying to ship several boxes outside of the country through a cargo forwarder. The wife of the cargo forwarder received the boxes. As a matter of procedure, she wanted to inspect the boxes. Petitioner informed her that the boxes contained gloves, books and cigars so the wife did not insist on inspecting the boxes. Before the box were actually shipped as a matter of procedure, the husband conducted a final inspection. He noticed that one of the boxes emitted an unusual smell. So, he got something inside wrapped in a plastic and had it examined at the NBI laboratory. It turned out positive for prohibited drugs. The NBI agents went with the forwarder and he opened the boxes. Found were marijuana. Petitioner seeks that the evidence be excluded. Held: The search is valid. Search by private individual is allowed provided it is without the interference of the agents of the State. Since it was a Standard Operating Procedure (SOP) for the proprietor to inspect objects to be shipped, his search for the boxes was not illegal and since he was the one who opened the boxes and who showed the contents to the agents, the agents did not interfere in the search. EXCLUSIONARY RULE STONEHILL VS. DIOKNO The court issued several warrants, 42 in all, to search the houses and business addresses of Stonehill and of the members of his board. Stonehill was known to engage in various illegal activities. Unfortunately, all the warrants were declared null and void for the reason that they were general warrants. The warrant stated that the objects to seize were to be used as evidence for violations of the Tarif Laws, Internal Revenue Laws, Central Bank Laws, RPC, and etc. Held: The court ruled that evidences obtained from Stonehill’s residences cannot be held admissible in any proceeding against him. The court however, declared that evidences obtained from his offices can be used against him. The reason for this is section 2 can only be invoked by the person whose rights have been invaded. Evidences obtained from the corporation, being a diferent person and entity from Stonehill, can be held admissible in court. Section 3. (1) The privacy of communication and correspondence shall be inviolable except upon

ACADCOM 2010; Contributors: Gene Geocaniga, Jarissa Guiani, Darlene Magabilen TAU MU Page 61 of 179

PRIVACY OF COMMUNICATION AND CORRESPONDENCE  rules out eavesdropping on private conversations through the use of electronic gadgets and thus covers w/ its protection even intangible things --so tangible and intangible things  EXCEPTIONS: upon lawful order of the court

Prohibited acts: 1. to secretly overhear; 2. intercept 3. communicate any part thereof. Who are prohibited: Any person not being authorized by ALL the parties to the communication, including the parties to the communication..

1. The law does not distinguish between a party to the private communication or a third person. Hence, both a party and a third person could be held liable under R.A. 4200 if they commit any of the prohibited acts under R.A. 4200 (Ramirez v. Ca) 2. The use of a telephone extension to overhear a private conversation is not a violation of R.A. 4200 because it is not similar to any of the prohibited devices under the law. Also, a telephone extension is not purposely installed for the purpose of secretly intercepting or recording private communication.

TAU MU TAU MU TAU MU TAU MU TAU MU

Types of communication protected: Tangible and Intangible objects

Objects covered: private communication or spoken word

TAU MU

EXCLUSIONARY RULE  evidence obtained in violation of this section shall be inadmissible for any purpose in any proceeding

Indeed the documents and papers in question are inadmissible in evidence. The constitutional injunction declaring “the privacy of communication and correspondence [to be] inviolable” is no less applicable simply because it is the wife (who thinks herself aggrieved by her husband’s infidelity) who is the party against whom the constitutional provision is to be enforced. The only exception to the prohibition in the Constitution is if there is a “lawful order [from a] court or when public safety or order requires otherwise, as prescribed by law.” Any violation of this provision renders the evidence obtained inadmissible “for any purpose in any proceeding. RA 4200 – Anti-Wiretapping Act

TAU MU

Letters and sealed packages in the mails may be examined only as to their external appearance and weight and may not be opened except in accordance with the constitutional requirements of lawful search and seizures.  Listening to phone conversation via an extension phone is not wire tapping. It is not protected and prohibited by law.

Zulueta vs. CA – This is more of an exceptional ruling because the bill of rights is being invoked against a private individual.

TAU MU



TAU MU TAU MU

INTRUSION WITHOUT COURT ORDER  the intrusion may be ordered by an executive officer, under authority of law (there must be a law w/c justifies the act of the executive officer), if he believes that public order or safety so requires.  PUBLIC ORDER and SAFETY concern the security of human lives. Liberty and property against the activities of invaders, insurrectionists and rebels.  this should be resorted to only in extreme cases

The Fraternal Ateneo de Davao

TAU MU TAU MU

INTRUSION UPON LAWFUL COURT ORDER  there must be probable cause  if the subject of the intrusion is WRITTEN CORRESPONDENCE the same must be particularly described as to its: a. contents, b. the persons whose communication is to be intercepted, c. the identity of the ofense/s sought to be prevented, and d. the period of the authorized intrusion  if intrusion is done through WIRE-TAPS it would be unreasonable to require a description of the communication’s contents but the following must still be particularly described: a. the persons whose communication is to be intercepted, b. the identity of the ofense/s sought to be prevented, and c. the period of the authorized intrusion when public safety or order requires otherwise, as may be provided by law (Correlate this with Section 1 on whether or not the law is a valid exercise of police power)

KITY

TAU MU TAU MU TAU MU TAU MU TAU MU TAU MU

CONSTITUTIONAL LAW II Order of Saint Thomas More Atty. Philip John Pojas/Atty. Rovyne G. Jumao-as, RN University College of Law lawful order of the court, or when public safety or order requires otherwise as prescribed by law. (2) Any evidence obtained in violation of this or the preceding section shall be inadmissible for any purpose in any proceedings.

Notes:

The law refers to a "tap" of a wire or cable or the use of a "device or arrangement" for the purpose of secretly overhearing, intercepting, or recording the communication. There must be either a physical interruption through a wiretap or the deliberate installation of a device or arrangement in order to overhear, intercept, or record the spoken words. (Gaanan v. IAC, 145 SCRA 112) RIGHT TO PRIVACY In the case of OPLE vs. Torres (1998); Pres. Ramos issued Administrative Order 308, this is the adoption of the national computerized identification. Ople argued that it is a violation of a constitutional right – the right to privacy. In this case the SC enlarged the scope of Section 3.

TAU MU

Even though it is not specifically stated, even though there is no specific provision in the Constitution which protects a person's right to privacy, according to the SC, the Bill of Rights has several provisions that aim to protect privacy (indirectly) like sec. 1, 2, 3, 6, 8, 17. These provisions indirectly protect the right to privacy and it includes sometimes a shadow provision over one's right to privacy in general. So, therefore, the right to privacy is a

ACADCOM 2010; Contributors: Gene Geocaniga, Jarissa Guiani, Darlene Magabilen TAU MU Page 62 of 179

Since the right to privacy is a fundamental right, the Gov't has the burden of proof to show that: (Balancing of Interest test) (1.) the restriction to the right to privacy is justified by a compelling government interest, and that (2.)the restriction must be narrowly drawn to protect that compelling valid interest.

CASES: PRIVACY OF COMMUNICATION

TAU MU TAU MU TAU MU TAU MU TAU MU

Issue: WON the opening and reading of the detainee’s letter violated their right to privacy of communication.

TAU MU

Facts: Some 321 armed soldiers, led by the now detained junior officers (herein petitioners), entered and took control of the Oakwood Premier Luxury Apartments. The soldiers disarmed the security officers of Oakwood and planted explosive devices in its immediate surroundings. The junior officers publicly renounced their support for the administration and called for the resignation of PGMA and several cabinet members. The soldiers voluntarily surrendered to the authorities after several negotiations with government emissaries and returned to their barracks. Thereafter, the government prosecutors filed Information for coup d’etat against the soldiers involved in the Oakwood incident. Gen. Abaya issued a directive to all Major Service Commanders to take into custody the military personnel under their command who took part in the Oakwood incident except the detained junior officers who were to remain under the custody of ISAFP. In this case, petitioners filed a petition for habeas corpus with the Supreme Court. The Court issued a Writ of Habeas Corpus directing respondents to make a return of the writ and to appear and produce the persons of the detainees before the Court of Appeals. Petitioners, while being detained, alleged that the officials of the ISAFP Detention Center violated their right to privacy of communication when the ISAFP officials opened and read the personal letters of Trillanes and Capt. Milo Maestrecampo. Hence, this case.

TAU MU TAU MU

IN RE: PETITION FOR HABEAS CORPUS OF CAPT. ALEJANO, et. al. 468 SCRA 185

The Fraternal Ateneo de Davao

TAU MU TAU MU

Exclusionary rule: Any evidence obtained shall be inadmissible for any purpose in any proceeding. However, in the absence of governmental interference, the protection against unreasonable search and seizure cannot be extended to acts committed by private individuals. (People v. Martin)

KITY

TAU MU TAU MU TAU MU TAU MU TAU MU TAU MU

CONSTITUTIONAL LAW II Order of Saint Thomas More Atty. Philip John Pojas/Atty. Rovyne G. Jumao-as, RN University College of Law fundamental right and it prevails over other rights.

TAU MU

Ruling: American cases recognize that the unmonitored use of pre-trial detainees’ non-privileged mail poses a genuine threat to jail security. Hence, when a detainee places his letter in an envelope for non-privileged mail, the detainee knowingly exposes his letter to possible inspection by jail officials. A pretrial detainee has no reasonable expectation of privacy for his incoming mail. However, incoming mail from lawyers of inmates enjoys limited protection such that prison officials can open and inspect the mail for contraband but could not read the contents without violating the inmates’ right to correspond with his lawyer. The inspection of privileged mail is limited to physical contraband and not to verbal contraband. Basing the decision in US jurisprudence, the SC held that the opening and reading of the detainees’ letters did not violate the detainees’ right to privacy of communication. The letters were not in a sealed ACADCOM 2010; Contributors: Gene Geocaniga, Jarissa TAU MU Page 63 of 179

envelope. The inspection of the folded letters is a valid measure as it serves the same purpose as the opening of sealed letters for the inspection of contraband. The letters were not confidential letters between the detainees and their lawyers. The petitioner who received the letters from detainees Trillanes and Maestrecampo was merely acting as the detainees’ personal courier and not as their counsel when he received the letters for mailing. In the present case, since the letters were not confidential communication between the detainees and their lawyers, the officials of the ISAFP Detention Center could read the letters. If the letters are marked confidential communication between the detainees and their lawyers, the detention officials should not read the letters but only open the envelopes for inspection in the presence of the detainees. A law is required before an executive officer could intrude on a citizen’s privacy rights is a guarantee that is available only to the public at large but not to persons who are detained or imprisoned. The right to privacy of those detained is subject to Section 4 of RA 7438, as well as to the limitations inherent in lawful detention or imprisonment. By the very fact of their detention, pre-trial detainees and convicted prisoners have a diminished expectation of privacy rights. The detainees in the present case are junior officers accused of leading 300 soldiers in committing coup d’etat, a crime punishable with reclusion perpetua. The junior officers are not ordinary detainees but visible leaders of the Oakwood incident involving an armed takeover of a civilian building in the heart of the financial district of the country. As members of the military armed forces, the detainees are subject to the Articles of War. Moreover, the junior officers are detained with other high-risk persons from the Abu Sayyaf and the NPA. Thus, the military custodian must give a wider range of deference in implementing the regulations in the ISAFP Detention Center. The military custodian is in a better position to know the security risks involved in detaining the junior officers, together with the suspected Abu Sayyaf and NPA members. Since the appropriate regulations depend largely on the security risks involved, the people should defer to the regulations adopted by the military custodian in the absence of patent arbitrariness. The ruling in this case, however, does not foreclose the right of detainees and convicted prisoners from petitioning the courts for the redress of grievances. Regulations and conditions in detention and prison facilities that violate the Constitutional rights of the detainees and prisoners will be reviewed by the courts on a case-by-case basis. The courts could aford injunctive relief or damages to the detainees and prisoners subjected to arbitrary and inhumane conditions. However, habeas corpus is not the proper mode to question conditions of confinement. The writ of habeas corpus will only lie if what is challenged is the fact or duration of confinement.

OPLE vs. TORRES July 23, 1998

TAU MU TAU MU

Facts: The petition at bar is a commendable efort on the part of Senator Ople to prevent the shrinking of the right to privacy, which the revered Mr. Justice Brandeis considered as "the most comprehensive of rights and the right most valued by civilized men." Petitioner Ople prays the invalidation of Administrative Order No. 308 entitled "Adoption of a National Computerized Identification Reference System" on two important constitutional grounds, viz: one, it is a usurpation of Guiani, Darlene Magabilen

Issue: WON AO 308 violates citizen’s right to privacy of communication.

The Fraternal Ateneo de Davao

TAU MU TAU MU TAU MU TAU MU TAU MU TAU MU TAU MU TAU MU TAU MU TAU MU TAU MU

Ruling: The SC held that AO 308 cannot pass constitutional muster as an administrative legislation because facially it violates the right to privacy. The essence of privacy is the "right to be let alone." The right of privacy is recognized and enshrined in several provisions of the Constitution. The right to privacy is a fundamental right guaranteed by the Constitution, hence, it is the burden of government to show that A.O. No. 308 is justified by some compelling state interest and that it is narrowly drawn. A.O. No. 308 is predicated on two considerations: (1) the need to provide our citizens and foreigners with the facility to conveniently transact business with basic service and social security providers and other government instrumentalities and (2) the need to reduce, if not totally eradicate, fraudulent transactions and misrepresentations by persons seeking basic services. It is debatable whether these interests are compelling enough to warrant the issuance of A.O. No. 308. The heart of A.O. No. 308 lies in its Section 4 which provides for a Population Reference Number (PRN) as a "common reference number to establish a linkage among concerned agencies" through the use of "Biometrics Technology" (biological encoding is finger scanning) and "computer application designs." But what is not arguable is the broadness, the vagueness, the overbreadth of A.O. No. 308 which if implemented will put our people's right to privacy in clear and present danger. A.O. No. 308 falls short of assuring that personal information which will be gathered about our people will only be processed for unequivocally specified purposes. The lack of proper safeguards in this regard of A.O. No. 308 may interfere with the individual's liberty of abode and travel by enabling authorities to track down his movement; it may also enable unscrupulous persons to access confidential information and circumvent the right against selfincrimination; it may pave the way for "fishing expeditions" by government authorities and evade the right against unreasonable searches and seizures. The possibilities of abuse and misuse of the PRN, biometrics and computer technology are accentuated when we consider that the individual lacks control over what can be read or placed on his ID, much less verify the correctness of the data encoded. They threaten the very abuses that the Bill of Rights seeks to prevent. The right to privacy is one of the most threatened rights of man living in a mass society. The threats emanate from various sources-governments, journalists, employers, social scientists, etc. In the case at bar, the threat comes from the executive branch of government which by issuing A.O. No. 308 pressures the people to surrender their privacy by giving information about themselves on the pretext that it will facilitate delivery of basic services. Given the record-keeping power of the computer, only the indiferent will fail to perceive the danger that A.O. No. 308 gives the government the power to compile a devastating dossier against unsuspecting citizens. It is timely to take note of the well-worded warning of Kalvin, Jr., "the disturbing result could be that everyone will live burdened by an unerasable record of his past and his limitations. In a way, the threat is that because of its record-keeping, the society will have lost its benign capacity to forget." Oblivious to this counsel, the dissents still say we should not be too quick in labelling the right to privacy as a fundamental right.

KITY

TAU MU TAU MU TAU MU TAU MU TAU MU TAU MU

CONSTITUTIONAL LAW II Order of Saint Thomas More Atty. Philip John Pojas/Atty. Rovyne G. Jumao-as, RN University College of Law the power of Congress to legislate, and two, it impermissibly intrudes on our citizenry's protected zone of privacy.

We close with the statement that the right to privacy was not engraved in our Constitution for flattery. KMU vs. DIRECTOR-GENERAL 487 SCRA 623 Facts: Petitioners allege that, EO 420 which is REQUIRING ALL GOVERNMENT AGENCIES AND GOVERNMENT-OWNED AND CONTROLLED CORPORATIONS TO STREAMLINE AND HARMONIZE THEIR IDENTIFICATION (ID) SYSTEMS, AND AUTHORIZING FOR SUCH PURPOSE THE DIRECTORGENERAL, NATIONAL ECONOMIC AND DEVELOPMENT AUTHORITY TO IMPLEMENT THE SAME, AND FOR OTHER PURPOSES, is unconstitutional because it constitutes usurpation of legislative functions by the executive branch of the government. Furthermore, they allege that EO 420 infringes on the citizen’s right to privacy because it allows access to personal confidential data without the owner’s consent. Issue: WON EO 420 infringes citizen’s right to privacy. Ruling: EO 420 shows no constitutional infirmity because it even narrowly limits the data that can be collected, recorded and shown compared to the existing ID systems of government entities. EO 420 further provides strict safeguards to protect the confidentiality of the data collected, in contrast to the prior ID systems which are bereft of strict administrative safeguards. The right to privacy does not bar the adoption of reasonable ID systems by government entities. Some one hundred countries have compulsory national ID systems. Even with EO 420, the Philippines will still fall under the countries that do not have compulsory national ID systems but allow only sectoral cards for social security, health services, and other specific purposes. Without a reliable ID system, government entities like GSIS, SSS, Philhealth, and LTO cannot perform efectively and efficiently their mandated functions under existing laws. Without a reliable ID system, GSIS, SSS, Philhealth and similar government entities stand to sufer substantial losses arising from false names and identities. The integrity of the LTO’s licensing system will sufer in the absence of a reliable ID system. In the case at bar, petitioners have not shown how EO 420 will violate their right to privacy. Petitioners cannot show such violation by a mere facial examination of EO 420 because EO 420 narrowly draws the data collection, recording and exhibition while prescribing comprehensive safeguards. EO 420 applies only to government entities that already maintain ID systems and issue ID cards pursuant to their regular functions under existing laws. EO 420 does not grant such government entities any power that they do not already possess under existing laws. Obviously, a national ID card system requires legislation because it creates a new national data collection and card issuance system where none existed before. Thus, EO 420 does not establish a national ID system but makes the existing sectoral card systems of government entities like GSIS, SSS, Philhealth and LTO less costly, more efficient, reliable and user-friendly to the public. IN RE: SABIO 504 SCRA 714

TAU MU

Facts: Senator Miriam Santiago introduced Senate Res. No. 455, “directing an inquiry in aid of legislation on the anomalous losses incurred by the Philippines Overseas Telecommunications Corporation (POTC), Philippine Communications Satellite Corporation (PHILCOMSAT), and PHILCOMSAT Holdings Corporation (PHC) due to the alleged improprieties in their operations by their respective Board of ACADCOM 2010; Contributors: Gene Geocaniga, Jarissa Guiani, Darlene Magabilen TAU MU Page 64 of 179

TAU MU

The Fraternal Ateneo de Davao

TAU MU TAU MU TAU MU TAU MU TAU MU

Issue: WON the petitioner’s right to privacy was infringed.

KITY

TAU MU TAU MU TAU MU TAU MU TAU MU TAU MU

CONSTITUTIONAL LAW II Order of Saint Thomas More Atty. Philip John Pojas/Atty. Rovyne G. Jumao-as, RN University College of Law Directors.” Hence, a public meeting to deliberate on Senate Res. No. 455 was conducted. Chairman Sabio of PCGG was invited to be one of the resource persons in the public meeting jointly conducted by Committee on Government Corporations and Public Enterprises and Committee on Public Services. However, he declined the invitation because of a prior commitment. Senator Gordon issued a Subpoena Ad Testificandum, requiring Chairman Sabio and PCGG Commissioners to appear in the public hearing and testify on what they know relative to the matters specified in Senate Res. No. 455. Similar subpoena were issued against the directors and officers of Philcomsat Holdings Corporation. Again, Chairman Sabio refused to appear. Hence, Senator Gordon issued an order to arrest Chairman Sabio and PCGG Commisioners for contempt of the Senate. He was later on arrested in his office. With this, he filed a petition for writ of habeas corpus against the Senate Committee on Government Corporations and Public Enterprises and Committee on Public Services. PHILCOMSAT filed a petition for certiorari and prohibition against the Senate Committees on Government Corporations and Public Enterprises and Public Services. PHILCOMSAT and its directors alleged that the subpoena issued to them violated their right to privacy and against self-incrimination.

TAU MU

Ruling: In evaluating a claim for violation of the right to privacy, the court must determine whether a person has exhibited a reasonable expectation of privacy and, if so, whether that expectation has been violated by unreasonable government intrusion. Applying this determination to these cases, the important inquiries are: first, did the directors and officers of Philcomsat Holdings Corporation exhibit a reasonable expectation of privacy?; and second, did the government violate such expectation? The answers are in the negative. Petitioners were invited in the Senate’s public hearing to deliberate on Senate Res. No. 455, particularly “on the anomalous losses incurred by the Philippine Overseas Telecommunications Corporation (POTC), Philippine Communications Satellite Corporation (PHILCOMSAT), and Philcomsat Holdings Corporations (PHC) due to the alleged improprieties in the operations by their respective board of directors.” Obviously, the inquiry focuses on petitioners’ acts committed in the discharge of their duties as officers and directors of the said corporations, particularly Philcomsat Holdings Corporation. Consequently, they have no reasonable expectation of privacy over matters involving their offices in a corporation where the government has interest. Certainly, such matters are of public concern and over which the people have the right to information. This goes to show that the right to privacy is not absolute where there is an overriding compelling state interest. In Valmonte v. Belmonte, the Court remarked that as public figures, the Members of the former Batasang Pambansa enjoy a more limited right to privacy as compared to ordinary individuals, and their actions are subject to closer scrutiny. Taking this into consideration, the Court ruled that the right of the people to access information on matters of public concern prevails over the right to privacy of financial transactions. Under the present circumstances, the alleged anomalies in the PHILCOMSAT, PHC and POTC, ranging in millions of pesos, and the conspiratorial participation of the PCGG and its officials are compelling reasons for the Senate to exact vital information from the directors and officers of Philcomsat Holdings Corporations, as ACADCOM 2010; Contributors: Gene Geocaniga, Jarissa TAU MU Page 65 of 179

well as from Chairman Sabio and his Commissioners to aid it in crafting the necessary legislation to prevent corruption and formulate remedial measures and policy determination regarding PCGG’s efficacy. There being no reasonable expectation of privacy on the part of those directors and officers over the subject covered by Senate Res. No. 455, it follows that their right to privacy has not been violated by respondent Senate Committees.

SJS vs. DANGEROUS DRUGS BOARD November 3, 2008

TAU MU

TAU MU TAU MU TAU MU

Facts: The constitutionality of Section 36 of Republic Act No. (RA) 9165, otherwise known as the Comprehensive Dangerous Drugs Act of 2002, insofar as it requires mandatory drug testing of candidates for public office, students of secondary and tertiary schools, officers and employees of public and private offices, and persons charged before the prosecutor’s office with certain ofenses, among other personalities, is put in issue. , Petitioner Social Justice Society, a registered political party, seeks to prohibit the Dangerous Drugs Board (DDB) and the Philippine Drug Enforcement Agency (PDEA) from enforcing paragraphs (c), (d), (f), and (g) of Sec. 36 of RA 9165 on the ground that they infirm the constitution. For one, the provisions constitute undue delegation of legislative power when they give unbridled discretion to schools and employers to determine the manner of drug testing. For another, the provisions trench in the equal protection clause inasmuch as they can be used to harass a student or an employee deemed undesirable. And for a third, a person’s constitutional right against unreasonable searches is also breached by said provisions. The drug test prescribed under Sec. 36(c), (d), and (f) of RA 9165 for secondary and tertiary level students and public and private employees, while mandatory, is a random and suspicionless arrangement. The objective is to stamp out illegal drug and safeguard in the process “the well being of [the] citizenry, particularly the youth, from the harmful efects of dangerous drugs.” This statutory purpose can be achieved via the pursuit by the state of “an intensive and unrelenting campaign against the trafficking and use of dangerous drugs through an integrated system of planning, implementation and enforcement of anti-drug abuse policies, programs and projects.” The primary legislative intent is not criminal prosecution, as those found positive for illegal drug use as a result of this random testing are not necessarily treated as criminals. They may even be exempt from criminal liability should the illegal drug user consent to undergo rehabilitation. Issue: WON the right to privacy has been violated.

TAU MU TAU MU TAU MU

Ruling: The right to privacy has been accorded recognition in this jurisdiction as a facet of the right protected by the guarantee against unreasonable search and seizure under Sec. 2, Art. III of the Constitution. But while the right to privacy has long come into its own, this case appears to be the first time that the validity of a state-decreed search or intrusion through the medium of mandatory random drug testing among students and employees is, in this jurisdiction, made the focal point. Thus, the issue tendered in these proceedings is veritably one of first impression. In the case at bar, the Court is of the view and so holds that the provisions of RA 9165 requiring mandatory, random, and suspicionless drug testing of students are constitutional. Indeed, it is within the Guiani, Darlene Magabilen

TAU MU TAU MU TAU MU



These rights are preferred rights. (Philippine Blooming Mills case) These rights are not absolute. They are always subject to:

1. State regulation – premised on the need to protect society from the injurious exercise of such freedom. 2. Police power of the State – premised on the need to promote public welfare, safety, morals and national security. Examples of State regulation: 1. Seditious speeches are not covered 2. Slanderous/libelous words can be penalized 3. Exclusion from the mails of newspapers containing seditious articles is valid 4. A moviegoer has no right to yell “fire” inside the movie house. Rights included: 3. Speech and Expression 4. Assembly and petition 5. Press 6. Religion 7. To form association Freedom of Speech and Expression is the right to freely utter whatever the citizen may please, and to be protected from any responsibility for doing so. Freedom of the Press the liberty to discuss publicly and publish truthfully any matter of public interest without previous censorship from the government. Freedom of Assembly is the right to assemble peacefully for the consultation and discussion of matters of public interest and to petition the government for redress of grievances.  To protect and preserve the right of the people to information and matters of public concern;  To enable every citizen to bring the Government and persons in authority to the bar of public opinion;  To encourage free and general discussion of matters of public concern. Scope of Protection (what are protected) : political speeches, self or artistic expressions, symbolic speeches. One cannot be punished for doing these things) Protected speech includes every form of expression, whether oral, written, tape or disc recorded. It includes motion pictures as well as what is known as symbolic speech such as the wearing of an armband as a symbol of protest. Peaceful picketing has also been included within the meaning of speech.

TAU MU TAU MU

freedom are not confined only to those that are sympathetic of acceptable to the majority.  It permits the articulation of even the unorthodox view, that it be hostile to or derided by others, or “induces a condition of unrest, creates dissatisfaction w/ conditions as they are, or even stirs people to anger”.  The freedom to speak includes the right to be silent. It also includes the right to an audience, in the sense that the state cannot prohibit the people from hearing what a person has to say, whatever be the quality of his thoughts. The right to listen also includes the right NOT to listen.



Purposes :

TAU MU

 The ideas that may be expressed under this

TAU MU

SCOPE:

TAU MU

FREEDOM OF EXPRESSION is available only insofar as it is exercised for the discussion of matters afecting the public interest. Purely private matters do not come w/i the guaranty.

TAU MU



TAU MU TAU MU

Section 4. No law shall be passed abridging the freedom of speech, of expression, or of the press, or of the right of the people peaceably to assemble and petition the government for redress of grievances.

The Fraternal Ateneo de Davao

TAU MU TAU MU

Just as in the case of secondary and tertiary level students, the mandatory but random drug test prescribed by Sec. 36 of RA 9165 for officers and employees of public and private offices is justifiable, albeit not exactly for the same reason. The Court notes in this regard that petitioner SJS, other than saying that “subjecting almost everybody to drug testing, without probable cause, is unreasonable, an unwarranted intrusion of the individual right to privacy,” has failed to show how the mandatory, random, and suspicionless drug testing under Sec. 36(c) and (d) of RA 9165 violates the right to privacy and constitutes unlawful and/or unconsented search under Art. III, Secs. 1 and 2 of the Constitution. The essence of privacy is the right to be left alone. In context, the right to privacy means the right to be free from unwarranted exploitation of one’s person or from intrusion into one’s private activities in such a way as to cause humiliation to a person’s ordinary sensibilities. The right to privacy yields to certain paramount rights of the public and defers to the state’s exercise of police power. Thus, taking into account the foregoing factors, i.e., the reduced expectation of privacy on the part of the employees, the compelling state concern likely to be met by the search, and the well-defined limits set forth in the law to properly guide authorities in the conduct of the random testing, the SC held that the challenged drug test requirement is, under the limited context of the case, reasonable and, ergo, constitutional.

KITY

TAU MU TAU MU TAU MU TAU MU TAU MU TAU MU

CONSTITUTIONAL LAW II Order of Saint Thomas More Atty. Philip John Pojas/Atty. Rovyne G. Jumao-as, RN University College of Law prerogative of educational institutions to require, as a condition for admission, compliance with reasonable school rules and regulations and policies. To be sure, the right to enroll is not absolute; it is subject to fair, reasonable, and equitable requirements. The Court can take judicial notice of the proliferation of prohibited drugs in the country that threatens the well-being of the people, particularly the youth and school children who usually end up as victims. Accordingly, and until a more efective method is conceptualized and put in motion, a random drug testing of students in secondary and tertiary schools is not only acceptable but may even be necessary if the safety and interest of the student population, doubtless a legitimate concern of the government, are to be promoted and protected.

ACADCOM 2010; Contributors: Gene Geocaniga, Jarissa Guiani, Darlene Magabilen TAU MU Page 66 of 179

CONSTITUTIONAL LAW II Order of Saint Thomas More Atty. Philip John Pojas/Atty. Rovyne G. Jumao-as, RN University College of Law

1. 2. 3. 4.

ACTS NOT COVERED seditious speeches libelous speeches obscene speeches/express ion contemptu ous speeches

TAU MU TAU MU Freedom exercise

of be







TAU MU TAU MU TAU MU

 whether the words used in such circumstances are of such nature as to create a clear and present danger that they will bring about the substantive evils that the State has a right to prevent  the danger created must not only be clear and present but also traceable to the ideas expressed --- unless this nexus is established, the individual may not be held accountable  JUSTICE FERNANDO:

TAU MU

2. CLEAR AND PRESENT DANGER – It is a “working principle” that: (Before the utterance can be punished)

TAU MU



TAU MU



This test has been abandoned by the court already. It is only used when there purpose to ignite strife or sedition. You cannot use it on anything else. It is only used when the crime committed is sedition. it is not necessary that some definite or immediate act or force, violence, or unlawfulness be advocated in general terms it is also not necessary that the language used be reasonably calculated to incite persons to acts of force, violence or unlawfulness it is SUFFICIENT if the natural tendency and probable efect of the utterance be to bring about the substantive evil w/c the legislative body seeks to prevent a person could be punished for his ideas even if they only tended to create the evil sought to be prevented --- it is not necessary to actually create the evil; mere tendency toward the evil is enough

TAU MU



TAU MU

1. DANGEROUS TENDENCY TEST - if the word spoken creates a dangerous tendency which the state has the right to prevent.

TAU MU

MODES OF EXPRESSION: 1. language – oral or written 2. symbolism Tests on the Restriction of Expression: (When can the restricted)

clear seems to point to a causal connection w/ the danger arising from the utterance questioned present refers to the time element --- it must be imminent and immediate danger the danger must not only be probable but very likely inevitable [Case: GONZALES vs. COMELEC (27 SCRA 835) Notes: a. A substantive evil must be extremely serious, and b. Degree of imminence extremely high  Furthermore, the danger created must have a causal relation to the expression, and the evil which the state has the right to suppress. (Gonzales vs. Comelec)  Used when the suit is between a private individual and the government. Requisites :

TAU MU TAU MU

1. Political speech 2. self/artistic expression 3. commercial speech a. speech must not be false, misleading or proposing an illegal activity b. governmental interest sought to be served by the regulation must be substantial c. the regulation must directly advance the government’s interest d. regulation must not be overbroad 4. scientific information 5. symbolic speeches 6. picketing

The Fraternal Ateneo de Davao

TAU MU TAU MU TAU MU TAU MU TAU MU TAU MU

ACTS PROTECTED

KITY

a. the word used must is of such nature that would create a danger that would bring about a substantive evil that the state has the right to prevent; b. the substantive evil that will be brought about by such expression must be extremely serious and clear. (There must be a causal relation between the act sought to be prevented and the evil that will be brought about by the utterance) c. And it must be present, not mere tendency. It probability of occurrence is inevitable and apparent. Case: Zaldivar vs. Gonzales - there doesn't need to be an immediate catastrophe, the evil sought to be avoided here was the obstruction of justice, the derogation of the justice system and the diminishing of the standard of professional conduct. It is actually a danger if you degrade the justice system. 3. BALANCING OF INTEREST. - Principle which requires the court to consider the circumstances of each particular case and thereafter it shall settle the issue of which right demands greater protection. The court must weigh and balance two or more conflicting social interests. It involves many considerations but in the end, the court will uphold what should be considered as the most important interest.  





It is used when there are two or more rights from the Bill of Rights being invoked by private individuals. When particular conduct is regulated in the interest of public order, and the regulation results in the indirect, conditional, partial abridgment of speech, the duty of the courts is to determine w/c of the 2 conflicting interests demands greater protection under the particular circumstances presented if in a given situation it should appear that there is urgent necessity for protecting the national. security against improvident exercise of freedom of expression, the right must yield BUT if no special justification for its curtailment exists, it must prevail FLAW: it in efect allows the courts to decide that this freedom may not be enforced unless they believe it is reasonable to do so

Case: Lagunzad vs. Sotto (right to privacy is superior to the freedom of expression)

ACADCOM 2010; Contributors: Gene Geocaniga, Jarissa Guiani, Darlene Magabilen TAU MU Page 67 of 179

LAGUNSAD vs. SOTTO



authority pre-ferred

is

BALANCE-OFINTEREST TEST 

resolves the issue in the light of the peculiar circumstances obtai-ning in each case

These tests are also applicable to other preferred freedoms: 1) freedom of association, 2) right of assembly and petition, and 3) freedom of religion.

TAU MU TAU MU

 official/governmental restrictions on the press and other forms of expression in advance of actual publication or dissemination  in cases of prior restraint the rule that all exercise of police power bears the presumption of constitutionality does not apply (so this is an exception)

TAU MU

b.

When one is restrained from expressing himself, no one knows what that person is going to say; The value of the presumption against the State is that one has no evidence for conviction.

TAU MU

a.

TAU MU

REASONS for the presumption of unconstitutionality:

TAU MU

Note: Generally, when the state exercises police power it bears the presumption of constitutionality. However, in cases of prior restraint, the presumption of constitutionality is not applicable. The government has the burden of proof in proving the constitutionality of the law. (Pass the Clear and present danger test)

TAU MU

Prohibitions under Section 4/ Forms of Curtailment of Freedom of Speech: 1. PRIOR RESTRAINT It means official governmental restrictions on the press or other forms of expression in advance of actual publication or dissemination; curtailment of the freedom of expression made through restrictions in advance of actual publication or dissemination. This means that the exercise of such freedom depends upon the prior approval of the government.

TAU MU



DANGEROUS TENDENCY TEST

TAU MU

CLEAR and PRESENT DANGER TEST  liberty is pre-ferred  a question of proximity and degree

TAU MU TAU MU

Ruling: The court held that the right to privacy of the Padilla family should prevail. One consideration was that the movie intruded into the private life of the Padilla family. Another was that the movie was partly “fictionalized”. Furthermore, Lagunsad also agreed to enter into a contract with the Padillas.

The Fraternal Ateneo de Davao

TAU MU TAU MU

Facts: Lagunsad was a movie producer. He entered into a contract with the heirs of Moises Padilla for the production of the movie “The MOises Padilla Story”. After the movie earned profits, Lagunsad refused to pay the royalty to the heirs of Padilla, saying that the contract is void because it violated the freedom of expression. According to him, the life of Padilla is a matter of public interest and he should be allowed to produce the movie without the obligation to pay the heirs. Padilla was already considered a national hero or a folklore legend of the Ilonggos. The conflict to be resolved here is between the right of free expression against the right to privacy.

KITY

TAU MU TAU MU TAU MU TAU MU TAU MU TAU MU

CONSTITUTIONAL LAW II Order of Saint Thomas More Atty. Philip John Pojas/Atty. Rovyne G. Jumao-as, RN University College of Law

 EXCEPTIONS (the exercise of police power bears a presumption of constitutionality --- exception to the exception so that the rule applies): a. when a nation is at war b. when the COMELEC exercises its power to regulate the mass media for election purposes [Art. IX C Sec. 4; NPC vs. COMELEC (207 SCRA 1)]  FORMS: a. censorship b. closures c. court injunctions d. system of issuance of permits and licenses

Q: What is CENSORHIP? A: CENSORSHIP conditions the exercise of freedom of expression upon prior approval of the government. It need not partake of total suppression; even restriction of circulation is unconstitutional. Examples/forms of prior restraint 1. movie censorship 2. judicial prior restraint = court injunction against publication 3. issuances of licenses and permits, taxes based on gross receipts for the privilege of engaging in the business of advertising in any newspaper 4. flat license fees for the privilege of selling religious books 5. Closures EXCEPTION TO THE EXCEPTION (when prior restraint in unconstitutional and burden of proof is on the person restrained. In other words, there is a presumption of constitutionality): a.) IN TIMES OF WAR -- Ex. Government can prevent publication about the number/locations of its troops (Near v. Minnesota, 238 US 697) b.) ART. 9 (C), SEC. 4, -- when the comelec exercises its power to regulate mass media for election purposes. (take Note, for election purposes only) c.) OBSCENE PUBLICATIONS. d.) CONTENT NEUTRAL- does not restrict the content of the speech, or the content of your publication, but the time, the manner or the place. The restriction is not imposed on the content of the speech and publication, but only on the time and manner of expression. There is no presumption of unconstitutionality. What is only required is substantial government interest for the restriction to be accepted. There is no need for the clear and present danger test ex. preventing people from writing graffiti on the wall. As opposed to CONTENT BASED - restriction of the content of the speech, what the person is going to say or write ex. preventing people from writing about GMA on the wall; there is a presumption of unconstitutionality and the test to be applied is the clear and present danger test) Note: Just because it is content neutral and there is no presumption of unconstitutionality, this doesn't mean that okay na sya. It still has to pass a test . What kind of test should be used if the restriction is content neutral? The O’Brien Test (SWS vs. COMELEC case): A. if it (restriction) is within constitutional powers of the government;

ACADCOM 2010; Contributors: Gene Geocaniga, Jarissa Guiani, Darlene Magabilen TAU MU Page 68 of 179

the

TAU MU TAU MU TAU MU

The general rule in criticism of official conduct is characterized by liberality. The official acts, and even the private life of a public servant are legitimate subjects of public comment. People have a right to scrutinize and commend or condemn the conduct of their chosen representatives in government.  As long as the people’s comments are made in good faith and w/ justifiable ends, they are insulated from prosecution or damage suits for defamation even if such views are found to be inaccurate or erroneous.

TAU MU



TAU MU

CRITICISM OF OFFICIAL CONDUCT

TAU MU

Q: Does this right cover ideas ofensive to the public order or decency or reputation of persons entitled to protection by the state? A: NO.

TAU MU

Q: Is this right absolute? A: NO! Freedom of expression is not absolute. It is subject to the police power and may be properly regulated in the interest of the public.

TAU MU

 Freedom of speech includes freedom after speech.

TAU MU

Coverage of valid subsequent punishment Criminal prosecution for:  sedition  libel  obscenity  citation for contempt  damages *Unprotected speech

TAU MU TAU MU

2. SUBSEQUENT PUNISHMENT  a restraint that comes AFTER the exercise of the freedom of speech, press or expression.  Punishing the expression that has already come out. What the Constitution protects is the right to be able to express one's ideas without being punished for it. However, it doesn't mean that anything that one says, one can escape punishment for that by invoking the freedom of expression since this freedom is not absolute. (Correlate with unprotected speech)  Freedom from any punishment as a consequence of, or in connection with a speech, utterance or writing. (This is the guarantee of the constitution)  In order to make the freedom of expression clause more meaningful, there must be an assurance that, after making any utterance or publication, the author will not be subjected to any form of punishment.

The Fraternal Ateneo de Davao

TAU MU TAU MU

Examples of Content neutral restrictions: 1. When the Comelec exercises its power to regulate mass media for election purposes under Art 9-C se. 4 in relation to Sec. 11 of RA 6646. (Osmena vs. comelec) 2. Preventing the conducting of exit polls right after the elections. (ABS-CBN vs. Comelec – here, even if it is a content neutral provision it still violates the freedom of expression as holding of exit polls does not really undermine the result of the election. The comelec resolution is overbroad. ) 3. Preventing the publication of surveys before an election. SWS vs. Comelec

KITY

TAU MU TAU MU TAU MU TAU MU TAU MU TAU MU

CONSTITUTIONAL LAW II Order of Saint Thomas More Atty. Philip John Pojas/Atty. Rovyne G. Jumao-as, RN University College of Law B. if furthers an important or substantial government interest; C. if the substantial government interest is unrelated to the suppression of the expression; D. if the incidental restriction is no greater than is essential to the furtherance of the substantial interest.



A private individual may still be the subject of public comment even if he is not a public official or at least a public figure, as long as he is involved in a public issue.  If a matter is a subject of general/public interest, it cannot suddenly become less so because a private individual did not ‘voluntarily’ choose to become involved.  The public’s primary interest is in the event; the public focus is on the conduct of the participant and the content, efect and significance of the conduct, NOT the participant’s prior anonymity or notoriety. CONTEMPT 

Public comment is proscribed pending litigation on the ground that it would interfere w/ the administration of justice.  Newspaper publications tending to impede, obstruct, embarrass or influence the courts in the administering justice in a pending suit/proceeding constitutes criminal contempt, summarily punishable by the courts. The rule is otherwise if the case is ended [ROSENBLOOM vs. METROMEDIA]. 

This section is subordinate to the authority, integrity and independence of the judiciary and the proper administration of justice.  For expression to constitute contempt, the danger must cause a serious imminent threat to the administration of justice.  Only publication made in the pendency of a case constituted contempt.  Where the questioned publication was alleged to be contemptuous of the SC or its Justices, publication was declared contemptuous. But in every case where an inferior court or its judge was the target, absolution followed.  The freedom of expression under Sec. 4 is subordinate to the authority, integrity, independence of the judiciary in the proper administration of justice. Q: Who has the power to declare a person in contempt? A: The courts – this is an inherent power even w/o any express grant in the Constitution because it is part of the exercise of judicial power Congress – can pass a law as to which agency has the power to cite a person in contempt. LIBELOUS SPEECH  are by no means essential to the exposition of ideas and are of such scant social value that any benefit w/c may be derived from them is outweighed by the social interest of public order  NOT protected speech  there are no tests A. FAIR COMMENT (U.S. Rule). These are statements of OPINION, not of fact, and are not considered actionable, even if the words used are neither mild nor temperate. What is important is that the opinion is the true and honest opinion of the person. The statements are not used to attack personalities but to give one’s opinion on decisions and actions. B. OPINIONS. With respect to public personalities (politicians, actors, anyone with a connection to a newsworthy event), opinions can be aired regarding their public actuations. Comment on their private lives, if not germane to their public personae, are not protected.

ACADCOM 2010; Contributors: Gene Geocaniga, Jarissa Guiani, Darlene Magabilen TAU MU Page 69 of 179

(1) He must prove that the charge is false, (2)That the one making the charge knows that it is false and (3) That there was utter disregard as to whether or not the charge is really false.



TAU MU TAU MU TAU MU

OBSECENTITY  TESTS of OBSCENITY: 1. whether the average person, applying contemporary community standards, would find that the work, taken as a whole, appeals to prurient interest 2. whether the work depicts/describes, in a patently ofensive way, sexual conduct specifically defined by the applicable law whether the work, taken as a whole, lacks serious literary, artistic, political or scientific value

TAU MU



TAU MU



if the statement concerns matters that are essentially of his private life are protected by law --Sec. 4 is NOT a defense if the statement concerns matters connected w/ his official acts, OR his mental, moral or physical fitness to be in office --- issue is constitutional statements that are purely destructive that afect his function as an official are restricted BUT as a public official it is expected that you may be criticized

TAU MU



Libelous Statements vs. PUBLIC INDIVIDUALS

TAU MU TAU MU



Libelous Statements vs. PRIVATE INDIVIDUALS almost always punishable because a person has a right to his reputation and integrity freedom of expression under Sec. 4 is NOT a defense

The Fraternal Ateneo de Davao

TAU MU TAU MU

Note: If the person being attacked is a public official, and what is being said is in relation to his public function, then the freedom of expression is stronger even if the statement is slanderous or libelous. Borjal case expanded the Vasquez decision stating that not only public officials or government employees have to prove these things, but also someone who is a public figure.

KITY

TAU MU TAU MU TAU MU TAU MU TAU MU TAU MU

CONSTITUTIONAL LAW II Order of Saint Thomas More Atty. Philip John Pojas/Atty. Rovyne G. Jumao-as, RN University College of Law Note: LIBEL/SLANDER - a crime; as a general rule, the burden of proof is on the prosecution. In libel, generally, proof of truth is not a defense. But if it is against the public official, proof of truth is a defense. When the libel is against a public official, it is incumbent on the public official to prove certain things (3 things).

v. Proper action should be filed under Art. 201 of the RPC. vi. Conviction is subject to appeal. Principles: 1. Movies/Tv programs- admin officials such as MTCRB have the power to determine what is obscene or not (Iglesia vs. CA) – Classification only, not prohibition, ie R-18 2. Printed material – it is the court who will determine if it is obscene or not. 3. Obscene materials cannot be considered as nuisance per se 4. Procedure to be taken by authorities when there are alleged obscene materials: a.Obtain a search warrant b.It must be the judge, and not the policemen, who will determine what is obscene and what is not c. The warrant can only be granted by the judge if a clear and present danger of a substantive evil. CASES: RESTRICTION ON EXPRESSION OSMENA vs. COMELEC 288 SCRA 447 Facts: This is a petition for prohibition, seeking a reexamination of the validity of Sec. 11(b) of R.A. No. 6646, the Electoral Reforms Law of 1987, which prohibits mass media from selling or giving free of charge print space or air time for campaign or other political purposes, except to the Commission on Elections. Petitioner Osmeña is candidate for President of the Philippines, while petitioner Pablo P. Garcia is governor of Cebu Province, seeking reelection. They contend that events after the ruling in National Press Club v. Commission on Elections “have called into question the validity of the very premises of that decision.” In NPC vs. COMELEC, the SC upheld upheld the validity of Sec. 11(b) of R.A. No. 6646 against claims that it abridged freedom of speech and of the press. In urging a reexamination of that ruling, petitioners claim that experience in the last five years since the decision in that case has shown the “undesirable efects” of the law because “the ban on political advertising has not only failed to level the playing field, but actually worked to the grave disadvantage of the poor candidates” by depriving them of a medium which they can aford to pay for while their more affluent rivals can always resort to other means of reaching voters like airplanes, boats, rallies, parades, and handbills. Issue: WON freedom of expression and of the press was abridged by Sec. 11(b) of RA 6646.

TAU MU

Ruling: The SC, in applying the O’Brien test in this case, held that Sec. 11(b) of R.A. No. 6646 is a valid exercise of the power of the State to regulate media of communication or information for the purpose of ensuring equal opportunity, time and space for political campaigns; that the regulation is unrelated to the suppression of speech; that any restriction on freedom  In determining WON an expression is obscene, of expression is only incidental and no more than is the courts should apply the standard of the community necessary to achieve the purpose of promoting in w/c the material is being tested equality. Procedure for seizure of allegedly obscene The Court is just as profoundly aware that publications discussion of public issues and debate on the i. Authorities must apply for issuance of search qualifications of candidates in an election are essential warrant. to the proper functioning of the government ii. Court must be convinced that the materials established by our Constitution. But it is precisely with are obscene. Apply clear and present danger test. this awareness that the SC think democratic eforts at iii. Judge will determine whether they are in fact reform should be seen for what they are: genuine “obscene”. eforts to enhance the political process rather than iv. Judge will issue a search warrant. infringements on freedom of expression. The statutory ACADCOM 2010; Contributors: Gene Geocaniga, Jarissa Guiani, Darlene Magabilen TAU MU Page 70 of 179

TAU MU

TAU MU

TAU MU TAU MU TAU MU TAU MU TAU MU TAU MU TAU MU TAU MU

Facts: Press Secretary Bunye told reporters that the opposition was planning to destabilize the administration by releasing an audiotape of a mobile phone conversation allegedly between the PGMA and a high-ranking official of the COMELEC. The conversation was audiotaped allegedly through wire-tapping. Later, in a Malacañang press briefing, Secretary Bunye produced two versions of the tape, one supposedly the complete version, and the other, a spliced, “doctored” or altered version, which would suggest that the President had instructed the COMELEC official to manipulate the election results in the President’s favor. It seems that Secretary Bunye admitted that the voice was that of President Arroyo, but subsequently made a retraction. Counsel of ERAP subsequently released an alleged authentic tape recording of the wiretap. Included in the tapes were purported conversations of the President, the First Gentleman Jose Miguel Arroyo, COMELEC Commissioner Garcillano, and the late Senator Barbers. Respondent DOJ Secretary Raul Gonzales warned reporters that those who had copies of the compact disc (CD) and those broadcasting or publishing its contents could be held liable under the Anti-Wiretapping Act. These persons included Secretary Bunye and Atty. Paguia, counsel of ERAP. He also stated that persons possessing or airing said tapes were committing a continuing ofense, subject to arrest by anybody who had personal knowledge if the crime was committed or was being committed in their presence. Secretary Gonzales ordered the NBI to go after media organizations “found to have caused the spread, the playing and the printing of the contents of a tape” of an alleged wiretapped conversation involving the President about fixing votes in the 2004 national elections. Thereafter, the NTC issued a press release which states that: NTC GIVES FAIR WARNING TO RADIO AND TELEVISION OWNERS/OPERATORS TO OBSERVE ANTI-WIRETAPPING LAW AND PERTINENT CIRCULARS ON PROGRAM STANDARDS. Thereafter, NTC held a dialogue with the Board of Directors of the Kapisanan ng mga Brodkaster sa Pilipinas (KBP). NTC allegedly assured the KBP that the press release did not violate the constitutional freedom of speech, of expression, and of the press, and the right to information. Petitioner Chavez filed a petition against respondents Secretary Gonzales and the NTC alleging that the acts of respondents are violations of the freedom of expression and of the press, and the right of the people to information on matters of public concern. Respondents denied that the acts transgress the Constitution, and questioned petitioner’s legal standing to file the petition.

The Fraternal Ateneo de Davao

TAU MU TAU MU

CHAVEZ vs. GONZALES 555 SCRA 441

KITY

TAU MU TAU MU TAU MU TAU MU TAU MU TAU MU

CONSTITUTIONAL LAW II Order of Saint Thomas More Atty. Philip John Pojas/Atty. Rovyne G. Jumao-as, RN University College of Law provision involved in this case is part of the reform measures adopted in 1987 in the aftermath of EDSA. A reform-minded Congress passed bills which were consolidated into what is now R.A No. 6646 with near unanimity. The House of Representatives, of which petitioner Pablo P. Garcia was a distinguished member, voted 96 to 1 (Rep. Eduardo Pilapil) in favor, while the Senate approved it 19-0. In the book The Irony of Free Speech by Owen Fiss, it speaks of “a truth that is full of irony and contradiction: that the state can be both an enemy and a friend of speech; that it can do terrible things to undermine democracy but some wonderful things to enhance it as well.” Thus, the SC held R.A. No. 6646, Sec. 11 (b) to be such a democracy-enhancing measure.

Issue: WON the mere press statements of the Secretary of Justice and of the NTC in question constitute a form of content-based prior restraint that has transgressed the constitutionally protected freedoms of speech, of expression and of the press. Ruling: The SC held that it is not decisive that the press statements made by respondents were not reduced in or followed up with formal orders or circulars. It is sufficient that the press statements were made by respondents while in the exercise of their official functions. Undoubtedly, respondent Gonzales made his statements as Secretary of Justice, while the NTC issued its statement as the regulatory body of media. Any act done, such as a speech uttered, for and on behalf of the government in an official capacity is covered by the rule on prior restraint. The concept of an “act” does not limit itself to acts already converted to a formal order or official circular. Otherwise, the non formalization of an act into an official order or circular will result in the easy circumvention of the prohibition on prior restraint. The press statements at bar are acts that should be struck down as they constitute impermissible forms of prior restraints on the right to free speech and press. There is enough evidence of chilling efect of the complained acts on record. The warnings given to media came from no less the NTC, a regulatory agency that can cancel the Certificate of Authority of the radio and broadcast media. They also came from the Secretary of Justice, the alter ego of the Executive, who wields the awesome power to prosecute those perceived to be violating the laws of the land. After the warnings, the KBP inexplicably joined the NTC in issuing an ambivalent Joint Press Statement. After the warnings, petitioner Chavez was left alone to fight this battle for freedom of speech and of the press. This silence on the sidelines on the part of some media practitioners is too deafening to be the subject of misinterpretation. The constitutional imperative to strike down unconstitutional acts should always be exercised with care and in light of the distinct facts of each case. For there are no hard and fast rules when it comes to slippery constitutional questions and the limits and construct of relative freedoms are never set in stone, issues revolving on their construct must be decided on a case to case basis, always based on the peculiar shapes and shadows of each case. But in cases where the challenged acts are patent invasions of a constitutionally protected right, the SC must be swift in striking them down as nullities per se. A blow too soon struck for freedom is preferred than a blow too late. ABS-CBN vs. COMELEC 323 SCRA 811

TAU MU

Facts: This case stems from a petition assailing Commission on Elections (Comelec) en banc Resolution No. 98-1419. In the said Resolution, the poll body "RESOLVED to approve the issuance of a restraining order to stop ABS-CBN or any other groups, its agents or representatives from conducting such exit survey and to authorize the Honorable Chairman to issue the same." The Resolution was issued by the Comelec allegedly upon "information from a reliable source that ABS-CBN (Lopez Group) has prepared a project, with PR groups, to conduct radio-TV coverage of the elections and to make an exit survey of the vote during the elections for national officials particularly for President and Vice President, results of which shall be broadcast immediately." The electoral body believed that such project might conflict with the official Comelec count, as well as the unofficial quick count of the National Movement for Free Elections (Namfrel). It also noted ACADCOM 2010; Contributors: Gene Geocaniga, Jarissa Guiani, Darlene Magabilen TAU MU Page 71 of 179

TAU MU

TAU MU

Issue: WON the act of the COMELEC in restraining petitioner from holding exit polls and the nationwide reporting of their results violated freedoms of speech and of the press.

The Fraternal Ateneo de Davao

TAU MU TAU MU TAU MU TAU MU TAU MU TAU MU TAU MU TAU MU TAU MU TAU MU TAU MU

Ruling: The freedom of expression is a means of assuring individual self-fulfillment, of attaining the truth, of securing participation by the people in social and political decision-making, and of maintaining the balance between stability and change. It represents a profound commitment to the principle that debates on public issues should be uninhibited, robust, and wide open. It means more than the right to approve existing political beliefs or economic arrangements, to lend support to official measures, or to take refuge in the existing climate of opinion on any matter of public consequence. The Court has always ruled in favor of the freedom of expression, and any restriction is treated an exemption. The power to exercise prior restraint is not to be presumed; rather the presumption is against its validity. And it is respondent's burden to overthrow such presumption. Any act that restrains speech should be greeted with furrowed brows, so it has been said. To justify a restriction, the promotion of a substantial government interest must be clearly shown. Thus, "A government regulation is sufficiently justified if it is within the constitutional power of the government, if it furthers an important or substantial government interest; if the governmental interest is unrelated to the suppression of free expression; and if the incidental restriction on alleged First Amendment freedoms is no greater than is essential to the furtherance of that interest." Hence, even though the government's purposes are legitimate and substantial, they cannot be pursued by means that broadly, stifle fundamental personal liberties, when the end can be more narrowly achieved. The freedoms of speech and of the press should all the more be upheld when what is sought to be curtailed is the dissemination of information meant to add meaning to the equally vital right of sufrage. The Court cannot support any ruling or order "the efect of which would be to nullify so vital a constitutional right as free speech." When faced with borderline situations in which the freedom of a candidate or a party to speak or the freedom of the electorate to know is invoked against actions allegedly made to assure clean and free elections, this Court shall lean in favor of freedom. For in the ultimate analysis, the freedom of the citizen and the State's power to regulate should not be antagonistic. There can be no free and honest elections if, in the eforts to maintain them, the freedom to speak and the right to know are unduly curtailed. In order to justify a restriction of the people's freedoms of speech and of the press, the state's responsibility of ensuring orderly voting must far outweigh them. These freedoms have additional importance, because exit polls generate important research data which may be used to study influencing factors and trends in voting behavior. An absolute prohibition would thus be unreasonably restrictive, because it efectively prevents the use of exit poll data

KITY

TAU MU TAU MU TAU MU TAU MU TAU MU TAU MU

CONSTITUTIONAL LAW II Order of Saint Thomas More Atty. Philip John Pojas/Atty. Rovyne G. Jumao-as, RN University College of Law that it had not authorized or deputized Petitioner ABSCBN to undertake the exit survey. Thereafter, the Supreme Court issued the Temporary Restraining Order prayed for by petitioner and directed the Comelec to cease and desist, until further orders, from implementing the assailed Resolution or the restraining order issued pursuant thereto, if any. In fact, the exit polls were actually conducted and reported by media without any difficulty or problem.

not only for election-day projections, but also for longterm research. The Comelec's concern with the possible noncommunicative efect of exit polls -- disorder and confusion in the voting centers -- does not justify a total ban on them. Undoubtedly, the assailed Comelec Resolution is too broad, since its application is without qualification as to whether the polling is disruptive or not. There is no showing, however, that exit polls or the means to interview voters cause chaos in voting centers. Neither has any evidence been presented proving that the presence of exit poll reporters near an election precinct tends to create disorder or confuse the voters. Moreover, the prohibition incidentally prevents the collection of exit poll data and their use for any purpose. The valuable information and ideas that could be derived from them, based on the voters' answers to the survey questions will forever remain unknown and unexplored. Unless the ban is restrained, candidates, researchers, social scientists and the electorate in general would be deprived of studies on the impact of current events and of election-day and other factors on voters' choices. The absolute ban imposed by the Comelec cannot, therefore, be justified. It does not leave open any alternative channel of communication to gather the type of information obtained through exit polling. SWS INC. vs. COMELEC 357 SCRA 497 Facts: Petitioner Social Weather Stations is a private non-stock, non-profit social research institution conducting surveys in various fields, including economics, politics, demography, and social development, and thereafter processing, analyzing, and publicly reporting the results thereof. On the other hand, petitioner Kamahalan Publishing Corporation publishes the Manila Standard, a newspaper of general circulation, which features newsworthy items of information including election surveys. Petitioners brought this action for prohibition to enjoin the Commission on Elections from enforcing Sec. 5.4 of R.A. No. 9006, the Fair Elections Act. Petitioner SWS states that it wishes to conduct an election survey throughout the period of the elections both at the national and local levels and release to the media the results of such survey as well as publish them directly. Petitioner Kamahalan Publishing Corporation, on the other hand, states that it intends to publish election survey results up to the last day of the elections. Petitioners argue that the restriction on the publication of election survey results constitutes a prior restraint on the exercise of freedom of speech without any clear and present danger to justify such restraint. Respondent Commission on Elections justifies the restrictions in Sec. 5.4 of R.A. No. 9006 as necessary to prevent the manipulation and corruption of the electoral process by unscrupulous and erroneous surveys just before the election. Issue: WON Section 5.4 of RA 9006 violated the freedoms of speech, of expression and of the press.

TAU MU

Ruling: The SC held that Sec. 5.4 of R.A. No. 9006 constitutes an unconstitutional abridgment of freedom of speech, expression, and the press. Sec. 5.4 lays a prior restraint on freedom of speech, expression, and the press by prohibiting the publication of election survey results afecting candidates within the prescribed periods of fifteen days immediately preceding a national election and seven days before a local election. Because of the preferred status of the constitutional rights of speech, expression, and the press, such a measure is vitiated by a weighty presumption of invalidity. Indeed, “any system of prior ACADCOM 2010; Contributors: Gene Geocaniga, Jarissa Guiani, Darlene Magabilen TAU MU Page 72 of 179

TAU MU

TAU MU TAU MU TAU MU

BORJAL vs. CA 301 SCRA 1

TAU MU

Ruling: Yes. Under Art. 361 of the Revised Penal Code, when the libelous statement is made against a public official with respect to the discharge of his official duties and functions, truth is a defense. For that matter, even if the defamatory statement is false, no liability can attach if it relates to official conduct unless the public official proves that the statement was made with actual malice --- that is, with the knowledge that it was false or with reckless disregard of whether it was false or not. In this case, the prosecution failed to prove not only that the charges made were false, but also that Vasquez made them with knowledge of their falsity or with reckless disregard or whether they were false or not. A rule placing on the accused the burden of proving the truth of allegations of official misconduct and/or good motives and justifiable ends in making the allegation would infringe on the constitutionally guaranteed freedom of expression. If the person being attacked is a public official, and what is being said is in relation to his public function, then the freedom of expression is stronger even if the statement is slanderous or libelous.

TAU MU

Issue: WON placing on the accused the burden of proving the truth of the allegations would infringe freedom of expression.

TAU MU TAU MU

Facts: Vasquez was a spokesman of a group of squatters. During an interview which was published in a newspaper, he charged that the Barangay Captain connived with officials of the National Housing Authority to grab their land, and that the Barangay Captain was involved in illegal gambling and theft of chicken.

The Fraternal Ateneo de Davao

TAU MU TAU MU

VASQUEZ vs. CA 314 SCRA 460

KITY

TAU MU TAU MU TAU MU TAU MU TAU MU TAU MU

CONSTITUTIONAL LAW II Order of Saint Thomas More Atty. Philip John Pojas/Atty. Rovyne G. Jumao-as, RN University College of Law restraints of expression comes to this Court bearing a heavy presumption against its constitutional validity. The Government ‘thus carries a heavy burden of showing justification for the enforcement of such restraint.’” There is thus a reversal of the normal presumption of validity that inheres in every legislation. Hence, the SC ruled that Sec. 5.4 is invalid because (1) it imposes a prior restraint on the freedom of expression, (2) it is a direct and total suppression of a category of expression even though such suppression is only for a limited period, and (3) the governmental interest sought to be promoted can be achieved by means other than the suppression of freedom of expression.

TAU MU TAU MU

Facts: Respondent, a private citizen, was Executive Director of the First National Conference on Land Transportation (ENOLT). The Conference, to be participated in by the private sector and government agencies, was intended to find ways to solve the transportation crises, and to draft an omnibus bill to be presented to Congress that would embody tong-term transportation policy. In several issues of the Philippine Star, petitioner implicated respondent in some anomalous activities. Hence, the latter filed a case for libel. Is respondent a “public figure” so as to shift to him the burden of proving actual malice? Issue: WON the implications made by petitioner is within the protection of freedoms of expresson and of speech. Ruling: Yes. In the crime of libel, as a general rule, the burden of proof is on the prosecution. In libel, generally, proof of truth is not a defense. But if it is against the public official, proof of truth is a defense. ACADCOM 2010; Contributors: Gene Geocaniga, Jarissa TAU MU Page 73 of 179

When the libel is against a public official, it is incumbent on the public official to prove certain things such as: (1) He must prove that the charge is false; (2)That the one making the charge knows that it is false; and (3) That there was utter disregard as to whether or not the charge is really false. As held in the case of Vasquez vs. CA, if the person being attacked is a public official, and what is being said is in relation to his public function, then the freedom of expression is stronger even if the statement is slanderous or libelous. This case expanded the decision of in Vasquez by stating that not only public officials or government employees have to prove these things, but also someone who is a public figure. Respondent is deemed a public figure within the purview of the New York Times ruling, being “a person who, by his accomplishments, fame, mode of living, or by adopting a profession or calling which gives the public a legitimate interest in his doings, his afairs and his character, had become a public personage.” The FNCLT was an undertaking infused with public interest. It was promoted as a joint project of the government and the private sector, and organized by top management officials and prominent businessmen. For this reason, it attracted media mileage and drew public attention. As its Executive Director and spokesman, private respondent consequently assumed the status of a public figure. IN RE: PUBLISHED ALLEGED THREATS AGAINST MEMBERS OF THE COURT IN THE PLUNDER LAW CASE HURLED BY ATTY. LEONARD DE VERA 385 SCRA 285 Facts: The court En Banc issued a Resolution directing respondent Atty. Leonard De Vera to explain why he should not be cited for indirect contempt of court for uttering some allegedly contemptuous statements in relation to the case involving the constitutionality of the Plunder Law (Republic Act No. 7080) which was then a pending resolution. The statements allegedly uttered by De Vera are the following: (1) Erap camp blamed for oust-Badoy maneuvers. De Vera asked the Supreme Court to dispel rumors that it would vote in favor of a petition filed by Estrada’s lawyers to declare the plunder law unconstitutional for its supposed vagueness. De Vera said he and his group were “greatly disturbed” by the rumors from Supreme Court insiders. His group said that they were afraid that the Estrada camp’s efort to coerce, bribe, or influence the justices ---considering that it has a P500 million slush fund from the aborted power grab that May-will most likely result in pro-Estrada decision declaring the Plunder Law either unconstitutional or vague; (2) SC under pressure from Erap pals, foes. He said that people are getting dangerously passionate and emotionally charged. He voiced his concern that a decision by the high tribunal rendering the plunder law unconstitutional would trigger mass actions, probably more massive than those that led to People Power II. De Vera warned of a crisis far worse than the “jueteng” scandal that led to People Power II if the rumor turned out to be true. He added that, people wouldn’t just swallow any Supreme Court decision that is basically wrong. Sovereignty must prevail. Issue: WON the statements made by De Vera is within the protection of freedom of expression.

TAU MU TAU MU

Ruling: Respondent cannot justify his contemptuous statements--asking the Court to dispel rumors that it would declare the Plunder Law unconstitutional, and stating that a decision declaring it as such was basically wrong and would not be accepted by the people—as utterances protected by his right to freedom of speech. Guiani, Darlene Magabilen

TAU MU TAU MU TAU MU TAU MU TAU MU

Ruling (1): The court held that the Freedom of Expression protects not only citizens of the Philippines but also foreigners in our country. Sec.4 also extends to commercial media. Even if they did it for profit, they are also protected. The reason is that most media is privately owned and operates for profit. To prohibit them would render Sec. 4 useless. Nobody can say anything anymore. Issue (2): WON it violated Enrile’s right to privacy

TAU MU

Issue (1): WON Sec. 4 also protects foreigners

TAU MU TAU MU

Facts: Hal McElroy, an Australian film-maker, wanted to join the People’s Power Revolution in a movie entitled “The 4-day Revolution”. The movie was a mixture of fiction and history to that in addition to a love story, prominent personalities, like Enrile, had to be portrayed. While the production was in progress, Enrile obtained a court injunction to stop it. He argued that the film violated his right to privacy. TAU MU

The Fraternal Ateneo de Davao

TAU MU TAU MU

AYER vs. CAPULONG

KITY

TAU MU TAU MU TAU MU TAU MU TAU MU TAU MU

CONSTITUTIONAL LAW II Order of Saint Thomas More Atty. Philip John Pojas/Atty. Rovyne G. Jumao-as, RN University College of Law Indeed, freedom of speech includes the right to know and discuss judicial proceedings, but such right does not cover statements aimed at undermining the Court’s integrity and authority, and interfering with the administration of justice. Freedom of speech is not absolute, and must occasionally be balanced with the requirements of equally important public interests, such as the maintenance of the integrity of the courts and orderly functioning of the administration of justice. Thus, the making of contemptuous statements directed against the Court is not an exercise of free speech; rather, it is an abuse of such right. Unwarranted attacks on the dignity of the courts cannot be disguised as free speech, for the exercise of said right cannot be used to impair the independence and efficiency of courts or public respect therefor and confidence therein. It is a traditional conviction of civilized society everywhere that courts should be immune from every extraneous influence as they resolve the issues presented before them. Respondent’s utterances pressuring the Court to rule in favor of the constitutionality of the Plunder Law or risk another series of mass actions by the public cannot be construed as falling within the ambit of constitutionallyprotected speech, because such statements are not fair criticisms of any decision of the Court, but obviously are threats made against it to force the Court to decide the issue in a particular manner, or risk earning the ire of the public. Such statements show disrespect not only for the Court but also for the judicial system as a whole, tend to promote distrust and undermine public confidence in the judiciary, by creating the impression that the Court cannot be trusted to resolve cases impartially and violate the right of the parties to have their case tried fairly by an independent tribunal, uninfluenced by public clamor and other extraneous influences.

beforehand because of the preferred character of the Right of Freedom of Speech and of Expression. And while production was still in progress, no one knew whether the final outcome would pose a clear and present danger. There should have been no prior restraint because there was no basis yet. NOTE: Private individuals have more rights than public figures. The decision did not, however, define what a “public figure” is. The right to privacy comes out of the shadows of the other rights in the Constitution. There is no textual grant of the Right of Privacy found in the Constitution. ASSEMBLY AND PETITION 

FREEDOM OF ASSEMBLY is the right of the people to assemble peaceably for consultation and discussion of matters of public concern.  This right is important to freedom of expression because public issues are better resolved after an exchange of views among citizens meeting w/ each other for the purpose.  Public meeting is an efective forum for the ventilation of ideas afecting common welfare. The size of these gatherings is often a dependable gauge of the people’s support, or lack of it, for particular causes or candidates, and a barometer also of the political climate in general.  The right to assemble is NOT subject to previous restraint or censorship.  If the assembly is intended to be held in a public place, a permit for the use of such place, and not for the assembly itself, may be required.  The power of local officials in this regard is only one of regulation and not of prohibition. They cannot altogether bar the use of public places for lawful assemblies; the most they can do is indicate the time and conditions for their use. PROCEDURE to be followed before holding a RALLY: 1. Apply for a permit to hold a rally; 2. The application must contain the date, time and place of the intended rally; 3. The permit is for the PLACE; 4. The authorities can modify the particulars if there is CLEAR AND PRESENT DANGER; 5. If the rally is to be held on a private property, the consent of the owner is sufficient. A permit is not necessary. a. in the campus of a GOCC-operated educational institution b. in a freedom park – to be established in every municipality and city  The permit must be filed w/ the Mayor’s Office at least 5 days before the activity. The application must be acted on by the said office w/i 2 days from the filing, otherwise it will be deemed granted by the Mayor.  Denial of the application may be justified upon clear and convincing evidence that the activity will create a clear and present danger to public order, safety, convenience, morals or health.  Action on the application shall be communicated w/I 24 hours to the applicant who may appeal the same to the appropriate court.  The test of the lawfulness of an assembly is the purpose for w/c it is held, regardless of the auspices under w/c it is organized. Untoward incidents during the assembly do not make it unlawful.  The law prohibits law enforcers from interfering w/ a lawful assembly but permits them to detail a contingent under a responsible commander @ least 100 meters away in case it becomes necessary to maintain order.

TAU MU

Ruling (2): As to the contention of Enrile that it violated his right to privacy. The SC looked at the particular circumstances and did not apply any formula to decide on the issue of which shall prevail: right to privacy or the freedom of expression. The court ruled that the events that were portrayed were of public interest and Enrile is also a public figure. And because of this, the SC is constrained to rule that his Right to Privacy shall give way to Freedom of Expression. Only the balancing of interest was used by the SC in deciding(during this time, Enrile was a Senator). What is protected by the Right to Privacy is unwarranted publicity and wrongful publicizing of private afairs. The trial judge should not have issued an injunction ACADCOM 2010; Contributors: Gene Geocaniga, Jarissa Guiani, Darlene Magabilen TAU MU Page 74 of 179

TAU MU

TAU MU

TAU MU

iii. If the public authority is of the view that there is an imminent and grave danger of a substantive evil, the applicants must be heard on the matter.

TAU MU

ii. The application should be filed ahead of time to enable the public official concerned to appraise whether there are valid objections to the grant of the permit or to its grant, but in another public place. The grant or refusal should be based on the application of the Clear and Present Danger Test.

TAU MU

Rules on assembly in public places: i. Applicant should inform the licensing authority of the date, the public place where and the time when the assembly will take place.

TAU MU

Note: Intelligence information is not enough to pass the Clear and Present Danger Test.

TAU MU TAU MU

Q: Can the authorities deny a permit to assembly? A: YES, the freedom of assembly is not absolute. As a general rule, No. But the authorities can deny, but it must pass again the clear and Present Danger Test. NOTE: The standards for allowable impairment of speech and press also apply to the right of assembly and petition.

The Fraternal Ateneo de Davao

TAU MU TAU MU

Q: Is the right of assembly and petition subject to any prior restraint? A: NO. You cannot prevent people if they want to assemble. The exercise should not be made to depend upon the issuance of any permit. It should not be prohibited, but only regulated. It may be subject to regulation but not prohibition by the State as to when and where it should be held.

KITY

TAU MU TAU MU TAU MU TAU MU TAU MU TAU MU

CONSTITUTIONAL LAW II Order of Saint Thomas More Atty. Philip John Pojas/Atty. Rovyne G. Jumao-as, RN University College of Law Q: Can there be demonstrations in the vicinity of the courts? A: AM 98-7-02 SC provides that: Demonstrators, picketers, rallyists and all other similar persons are enjoined from holding any activity on the sidewalks and streets adjacent to, in front of, or w/i a radius of 200 meters from, the outer boundary of the Supreme Court Building, any Hall of Justice, and any other building that houses at least 1 court sala. Such activities unquestionably interrupt and hamper the working conditions in the salas, offices and chambers of courts.

a concerted and unauthorized stoppage of or absence from work carried out for essentially economic reasons. The exercise of any constitutional liberty must be within reasonable limits so as not to prejudice the public welfare. In this case, by staging mass protests or regular school days, abandoning their classes and refusing to go back even after they had been ordered to do so, the school teachers committed acts prejudicial to the best interest of the service. The holding in Philippine Blooming Mills that free expression and assembly are superior to property rights cannot apply. What is involved here is the education of the youth which must, at the very least, be equated with the freedom of assembly and petition for redress of grievances. BAYAN vs. ERMITA 488 SCRA 226 Facts: Petitioners come in three groups. The first petitioners, Bayan, allege that they are citizens and taxpayers of the Philippines and that their rights as organizations and individuals were violated when the rally they participated in was violently dispersed by policemen implementing Batas Pambansa 880 (The Public Assembly Act of 1985). The second group consists of 26 individual petitioners, Jess del Prado, et al., who allege that they were injured, arrested and detained when a peaceful mass action was pre-empted and violently dispersed by the police. They further assert that a group they participated in marched to Malacañang to protest issuances of the Palace which, they claim, put the country under an “undeclared” martial rule, and the protest was likewise dispersed violently and many among them were arrested and sufered injuries. The third group, Kilusang Mayo Uno or KMU, allege that they conduct peaceful mass actions and that their rights as organizations and those of their individual members as citizens, specifically the right to peaceful assembly, are afected by Batas Pambansa No. 880 and the policy of “Calibrated Preemptive Response” (CPR) being followed to implement it. Issue: WON BP 880 is a curtailment of the right to peaceable assembly and petition for redress of grievances.

TAU MU

Ruling: The right to peaceably assemble and petition for redress of grievances is, together with freedom of speech, of expression, and of the press, a right that enjoys primacy in the realm of constitutional protection. For these rights constitute the very basis of iv. The decision of the public authority, a functional democratic polity, without which all the whether favorable or adverse, must be transmitted to other rights would be meaningless and unprotected. the applicants at the earliest opportunity so that they A fair and impartial reading of B.P. No. 880 thus may, if they so desire, have recourse to the proper readily shows that it refers to all kinds of public judicial authority. assemblies that would use public places. The reference to “lawful cause” does not make it contentCASES: ASSEMBLY AND PETITION based because assemblies really have to be for lawful causes; otherwise they would not be “peaceable” and DELA CRUZ vs. CA entitled to protection. Neither are the words “opinion,” 305 SCRA 303 “protesting” and “influencing” in the definition of public assembly content based, since they can refer to Facts: Petitioners were public school teachers who any subject. The words “petitioning the government participated in “mass actions” to dramatize their for redress of grievances” come from the wording of demands for certain benefits granted by statutes. For the Constitution, so its use cannot be avoided. Finally, not attending to their classes, they were dismissed by maximum tolerance is for the protection and benefit of the Secretary of Education on charge of grave all rallyists and is independent of the content of the misconduct, gross neglect of duty and violation of the expressions in the rally. Civil Service Law and Rules and Regulations. TAU MU Furthermore, the permit can only be denied on the ground of clear and present danger to public order, Issue: Did the dismissal violate their right to petition public safety, public convenience, public morals or and assembly? public health. Not every expression of opinion is a public assembly. The law refers to “rally, Ruling: No. The “mass actions” by the public school demonstration, march, parade, procession or any other teachers amounted to a strike constituting, as they did, form of mass or concerted action held in a public ACADCOM 2010; Contributors: Gene Geocaniga, Jarissa Guiani, Darlene Magabilen TAU MU Page 75 of 179

TAU MU

TAU MU

TAU MU

TAU MU

TAU MU TAU MU TAU MU TAU MU

RELIGION  any specific system of belief, worship, conduct, etc., often involving a code of ethics and philosophy.  a profession of faith to an active power that binds and elevates man to his Creator [AGLIPAY vs. RUIZ (64 Phil. 201)]  embraces matters of faith and dogma, as well as doubt, agnosticism and atheism

TAU MU

Section 5. No law shall be made respecting an establishment of religion, or prohibiting the free exercise thereof. The free exercise and enjoyment of religious profession and worship, without discrimination or preference, shall forever be allowed. No religious test shall be required for the exercise of civil or political rights.

TAU MU

NOTE: However if the permit granted allows the rally to be held only in a particular place, there is a possibility that the place designated may be far and therefore impossible for other people to heat the group’s grievances. The State is not allowed to do such.

1. Art. VI Sec. 29; 2. Art. II Sec. 6.  MATTERS PROHIBITED: 1. The state cannot organize a church; 2. The state cannot promote one religion; 3. The sate cannot impose taxes to support religious activity; 4. The state cannot participate in the afairs of religious organizations.

TAU MU

NOTE: Students do not lose their constitutional rights once they enter the school grounds. However, they still need to obtain a permit from the school authorities. Schools can still deny permit if there is Clear and Present Danger.



TAU MU

Ruling: Although the case became moot and academic, the court nevertheless decided to rule on the merits of the constitutional issues raised and held that the penalty imposed was out of proportion to the misdeed. The decision emphasized that the students did not “shed their constitutional rights to freedom of speech or expression at the schoolhouse gate,” although these rights were not unlimited.

Ateneo de Davao

TAU MU TAU MU

Facts: Several student leaders were suspended for one year when they held a demonstration in the premises of a university outside the area permitted by the school authorities which disrupted classes and disturbed the work of the administrative personnel. The SC issued a temporary restraining order that in efect permitted the students to re-enroll and finish their studies.

The Fraternal

TAU MU TAU MU

MALABANAN vs. RAMENTO

KITY

TAU MU TAU MU TAU MU TAU MU TAU MU TAU MU

CONSTITUTIONAL LAW II Order of Saint Thomas More Atty. Philip John Pojas/Atty. Rovyne G. Jumao-as, RN University College of Law place.” So it does not cover any and all kinds of gatherings. Neither is the law overbroad. It regulates the exercise of the right to peaceful assembly and petition only to the extent needed to avoid a clear and present danger of the substantive evils Congress has the right to prevent. There is, likewise, no prior restraint, since the content of the speech is not relevant to the regulation. Hence, B.P. No. 880 cannot be condemned as unconstitutional; it does not curtail or unduly restrict freedoms; it merely regulates the use of public places as to the time, place and manner of assemblies. Moreover, the Court goes even one step further in safeguarding liberty by ordering the local governments to designate specific freedom parks as provided under B.P. No. 880. If no such parks are so identified in accordance with Section 15 of the law, all public parks and plazas of the municipality or city concerned shall in efect be deemed freedom parks; no prior permit of whatever kind shall be required to hold an assembly therein. The only requirement will be written notices to the police and the mayor’s office to allow proper coordination and orderly activities.

It is not only the state that is prohibited from interfering in purely ecclesiastical afairs; the Church is likewise barred from meddling in purely secular matters because a union of Church and State tends to destroy government and to degrade religion. It  The wall between Church and State is not one of hostility. The state recognizes the beneficent influence of religion in the enrichment of the nation’s life. NON ESTABLISHMENT CLAUSE  This clause prevents the state from passing laws w/c

aid one religion, or all religions, or prefer one religion over another. The state cannot establish or sponsor an official religion.  Related provisions:

 The purpose of this clause is to allow each religion

to compete freely w/ each other based on their own merit and w/o the state’s patronage  EXCEPTIONS to the Non-Establishment Clause: 1. Art. VI Sec. 28 (3) – tax exemption of religious properties 2. Art VI Sec. 29 (2) – payment of public funds to priests in the AFP, any penal institution, gov’t. orphanage, or leprosarium 3. Art. XIV Sec. 3 (3) – allows religious instructions in public schools under certain conditions Distinction between the clauses (School District v. Schempp, 374 US 203) 1. The non-establishment clause does not depend upon any showing of direct governmental compulsion. It is violated by the enactment of laws which establish an official religion whether those laws operate directly to coerce non-observing individuals or not. The test of compliance with the non-establishment clause can be stated as follows: What are the purposes and primary efect of the enactment? If either is the advancement or inhibition of religion, the law violates the nonestablishment clause. Thus, in order for a law to comply with the non-establishment clause, two requisites must be met. First, it has a secular legislative purpose. Second, its primary efect neither advances nor inhibits religion. 2. The free exercise of religion clause withdraws from legislative power the exertion of any restraint on the free exercise of religion. In order to show a violation of this clause, the person afected must show the coercive efect of the legislation as it operates against him in the practice of his religion. While the freedom to believe (non-establishment) is absolute, the moment such belief flows over into action, it becomes subject to government regulation.

TAU MU

Q: Can a statute be passed w/o infringing the nonestablishment clause? A: YES. There is no violation if (TESTS ON PERMISSIBLE AID TO RELIGION/LEMON TEST) [LEMON vs. KURTZMAN (403 US 602)]: 1. the statute has a secular legislative purpose; 2. its principal or primary efect is one that neither advances nor inhibits religion; and

ACADCOM 2010; Contributors: Gene Geocaniga, Jarissa Guiani, Darlene Magabilen TAU MU Page 76 of 179



Freedom of religion includes freedom FROM religion; the right to worship includes the right NOT to worship.



TAU MU TAU MU TAU MU TAU MU TAU MU TAU MU

Facts: Petitioner inherited a piece of land. When the parcel of land was ascertained to have been the birth site of Felix Y. Manalo, the founder of INC, it passed a resolution declaring the land to be a national historical landmark. The resolution was approved by the Minister of Education, Culture and Sports. The opinion of the Secretary of Justice was asked on the legality of the measure and he explained: “According to your guidelines, national landmarks are places or objects that are associated with an event, achievement, characteristic or modification that makes a turning point or stage in Philippine history. Thus, the birth site of the founder of the INC, who admittedly had made contributions to Philippine history and culture, has been declared as a national landmark. It has been held that places invested with unusual historical interest are a public use for which the power of eminent domain may be authorized.” Petitioners moved to dismiss the complaint on the main thesis that the intended expropriation was not for a public purpose and, incidentally, that the act would constitute an

TAU MU

MANOSCA vs. CA 252 SCRA 414 (1996)

TAU MU

CASES: NON-ESTABLISHMENT CLAUSE

TAU MU

The Lemon Test applied in the case:  This does not seem to have a secular purpose since the religious factor cannot be separated from the social factor. (This is still considered questionable)  Also the Bario Councils seem to be really participating in a “Religious Activity”. Although it was argued that the purpose of the fiesta was to “relieve the monotony” of the people. It is not convincing. It will have a principal efect of validating a religion. This is a form of participation of a government entity in a religious afair.  It does not have excessive government entanglement since the image is kept by the religious leader. If it was kept by the priest, this will be an excessive entanglement. The image was purchased with private funds.

TAU MU TAU MU

Payment of public funds is prohibited to ecclesiastics only “as such” w/c means that they may be paid IF they serve the gov’t. in a non-ecclesiastical capacity.  The purchase of religious image by the barangay council with private funds raised from voluntary contributions did not violate the Constitution. The fiesta is a socio-religious afair. It is considered part of the Filipino tradition. Purchase of the image does not support any religion, but a socio-religious tradition as ruled by the Court. [GARCES vs. ESTENZO]

The Fraternal Ateneo de Davao

TAU MU TAU MU

Q: If a religious sect enjoys indirect benefit from governmental action, is there a violation of Sec. 5? A: Any benefit indirectly enjoyed by a religious institution, as long as such benefit was only incidental to a legitimate secular objective, would not violate the prohibition. The use of public property for religious purposes when the religious character of such use is merely incidental to a temporary use w/c is available indiscriminately to the public in general [AGLIPAY vs. RUIZ, supra].

KITY

TAU MU TAU MU TAU MU TAU MU TAU MU TAU MU

CONSTITUTIONAL LAW II Order of Saint Thomas More Atty. Philip John Pojas/Atty. Rovyne G. Jumao-as, RN University College of Law 3. it does not foster an excessive government. entanglement w/ religion.

application of public funds, directly or indirectly, for the use, benefit or support of INC, a religious entity. Issue: WON the members of INC.

resolution

gives

preference

to

Ruling: Eminent domain, also often referred to as expropriation and, with less frequency, as condemnation, is like police power and taxation, an inherent power of sovereignty. It need not be clothed with any constitutional gear to exist. Instead, provisions in our constitution on the subject are meant more to regulate, rather than to grant, the exercise of power. The only direct constitutional qualification is that “private property shall not be taken for public use without just compensation,” to provide a safeguard against possible abuse and so to protect as well the individual against those whose property the power is sought to be enforced. The term “public use”, not having been otherwise defined by the Constitution, must be considered in its general concept of meeting a public need or a public exigency. The validity of the exercise of the power of eminent domain for traditional purposes is beyond question: it is not at all to be said, however, that public use should thereby be restricted to such traditional uses. The idea that “public use” is strictly limited to clear cases of “use by the public” has long been discarded. This attempt to give some religious perspective to the case deserves little consideration, for what should be significant is the principal objective of, not the casual consequences that might follow from, the exercise of the power. The purpose in setting up the marker is essentially to recognize the distinctive contribution of Felix Manalo to the culture of the Philippines, rather than to commemorate his founding and leadership of INC. The practical reality that greater benefit may be derived by members of the INC than by most others could well be true but such a peculiar advantage still remains to be merely incidental and secondary in nature. Indeed, that only a few actually benefit from the expropriation of property does not necessarily diminish the essence and character of public use. AGLIPAY vs. RUIZ FACTS: The government appropriated P60T for design of new stamps which commemorated the 33 rd Eucharistic Congress in Manila. The design was the Map of the Philippines. Monsignor Aglipay petitioned to stop the making and sale of the stamps. He reasoned that this was a violation of Section 5. RULING: The court ruled that the purpose of the stamp was to attract tourist to the Philippines, not for the purpose of promoting religion. The benefiting by the Catholic Church was only incidental to the main purpose or the principal efect. It was not intended to be the primary beneficiary. The stamp emphasized Manila (as shown by the design), not the event or a particular religion. INTRAMURAL RELIGIOUS DISPUTES They are disputes within the church. The general rule when there is an intramural dispute is that the State cannot interfere because it will be a violation of the Free exercise clause. When can the state interfere? (As a general rule, when the matter is not ecclesiastical in nature as in Taruc vs. Dela Cruz) 1. When the problem is not intramural. Even if it is intramural, when the problem has something to do

ACADCOM 2010; Contributors: Gene Geocaniga, Jarissa Guiani, Darlene Magabilen TAU MU Page 77 of 179

When there is a conflict between two factions in the church, who gets the property? The property will go to the faction which agrees to the original doctrine. However, if there is no dispute over doctrine, but only a typical division, the property will go to whoever is the majority or whoever is the duly constituted authority 2. When the church authorities acts outside the scope of his authority or contrary to its own rules.

TAU MU TAU MU

Q: What is an ECCLESIASTICAL AFFAIR? A: It is one that concerns doctrine, creed or form of worship of the church, or the adoption and enforcement w/i a religious association of needful laws and regulations for the government of the membership and the power of excluding from such associations those that are deemed unworthy of membership.

TAU MU

Intramural Disputes re: religious dogma and other matters of faith are outside the jurisdiction of the secular authorities.  Whatever dogma is adopted by a religious group cannot be binding upon the state if it contravenes its valid laws.  Where the civil right depends upon some matter pertaining to ecclesiastical afairs, the civil tribunal tries the civil right and nothing more [ARCHBISHOP OF MANILA Case (51 Phil. 417)].

TAU MU



TAU MU TAU MU

What are the matters of church which the court cannot interfere with? a. matters of faith; b. matters of doctrine; c. forms of worship, unless it is against the law; d. law and rules of customs within the church

Q: What about those considered as PURELY SECULAR MATTERS? A: These are matters that have no relation whatsoever w/ the practice of faith, worship or doctrines of the church.

TAU MU

CASE: INTRAMURAL DISPUTE

TAU MU

AUSTRIA vs. NLRC 312 SCRA 410 (1999)

TAU MU TAU MU

Facts: Private respondent is a religious corporation duly organized and existing under Philippine laws. Petitioner is a pastor of the SDA until 1991, when his services are terminated. Due to the assistance of Austria in collecting Pastor Rodrigo’s debt, the latter harbored ill-feelings against petitioner. He also had a quarrel with another pastor and was asked to answer for non-remittance of church collection. Petitioner received a letter of dismissal. NLRC ordered his reinstatement but was vacated after appeal. TAU MU

The Fraternal Ateneo de Davao

TAU MU TAU MU

Austria vs. NLRC- there was an employer-employee relationship, so the State can interfere. It was not a purely ecclesiastical issue since he was not expelled from the church but only released from employment as a pastor. Accdg. to the SC, an ecclesiastical matter is one that concerns doctrine, creed or form of worship of the church, or adoption or enforcement within a religious association of regulation and government of the membership of the church.

KITY

TAU MU TAU MU TAU MU TAU MU TAU MU TAU MU

CONSTITUTIONAL LAW II Order of Saint Thomas More Atty. Philip John Pojas/Atty. Rovyne G. Jumao-as, RN University College of Law with the law. For example, when a property right is involved. So, when there is a dispute over a property right within the church, the State can interfere.

Ruling: The principle of separation of church and state finds no application in this case. This case does not concern an ecclesiastical or purely religious afair as to bar the state from taking cognizance of the same. An ecclesiastical afair is one that concerns doctrine, creed, or form of worship of the church, or the adoption and enforcement within religious association of needful laws and regulations for the government of the membership, and the power of excluding from such association those deemed unworthy of membership. What is involved us the relationship of the church as an employer and the minister as an employee. The SDA cannot hide behind the mantle of protection of the doctrine of separation of church and state. Petitioner was terminated from service without just or lawful cause. Having been illegally dismissed, petitioner is entitled to reinstatement to his former position. TARUC vs. DELA CRUZ 453 SCRA 123 Facts: Petitioners were lay members of the Philippine Independent Church (PIC). Respondents Porfirio de la Cruz and Rustom Florano were the bishop and parish priest, respectively. Petitioners, led by Dominador Taruc, clamored for the transfer of Fr. Florano to another parish but Bishop de la Cruz denied their request. It appears from the records that the family of Fr. Florano’s wife belonged to a political party opposed to petitioner Taruc’s, thus the animosity between the two factions with Fr. Florano being identified with his wife’s political camp. Bishop de la Cruz, however, found this too flimsy a reason for transferring Fr. Florano to another parish. Meanwhile, hostility among the members of the PIC worsened when petitioner Taruc tried to organize an open mass to be celebrated by a certain Fr. Renato Z. Ambong during the town fiesta of Socorro. When Taruc informed Bishop de la Cruz of his plan, the Bishop tried to dissuade him from pushing through with it because Fr. Ambong was not a member of the clergy of the diocese of Surigao and his credentials as a parish priest were in doubt. The Bishop also appealed to petitioner Taruc to refrain from committing acts inimical and prejudicial to the best interests of the PIC. He likewise advised petitioners to air their complaints before the higher authorities of PIC if they believed they had valid grievances against him, the parish priest, the laws and canons of the PIC. Bishop de la Cruz, however, failed to stop Taruc from carrying out his plans. Hence, Bishop de la Cruz declared petitioners expelled/excommunicated from the Philippine Independent Church. Petitioners appealed to the Obispo Maximo and sought reconsideration of the above decision. In his letter to Bishop de la Cruz, the Obispo Maximo opined that Fr. Florano should step down voluntarily to avert the hostility and enmity among the members of the PIC parish in Socorro. In the meantime, Bishop de la Cruz was reassigned to the diocese of Odmoczan and was replaced by Bishop Timbang. Like his predecessor, Bishop Timbang did not find a valid reason for transferring Fr. Florano to another parish. He issued a circular denying petitioners’ persistent clamor for the transfer/re-assignment of Fr. Florano. Petitioners were informed of such denial but they continued to celebrate mass and hold other religious activities through Fr. Ambong who had been restrained from performing any priestly functions in the PIC parish of Socorro. Because of the order of expulsion/excommunication, petitioners filed a complaint for damages with preliminary injunction against Bishop de la Cruz.

TAU MU

Issue: WON the courts have jurisdiction to hear a case involving the expulsion/excommunication of members of a religious institution ACADCOM 2010; Contributors: Gene Geocaniga, Jarissa Guiani, Darlene Magabilen TAU MU Page 78 of 179 Issue: WON NLRC has jurisdiction to try and decide the complaint.

TAU MU TAU MU TAU MU

The reason to this is because there are no statutory, constitutional, or legal standards apply.

TAU MU

Matters that the Court will not inquire into: 1) Matters of faith, e.g. 3 persons in one God 2) Doctrine, e.g. Virgin Mary is Mother of God 3) Forms of Worship, e.g. Priests faces people 4) Church Law or Rule, e.g. confessions, etc. 5) Customs, e.g. pouring water over the head.

TAU MU

This rule was not followed in this case. This applies only to big churches with external leadership. In this case, there was no external leadership involved but only the bishops themselves. Since there is no external leader who can decide as to who is the legitimate authority, the basis could be to find out who was validly elected or removed.

TAU MU

Q: What if there is no quarrel about the doctrines (so schism), only physical division? A: (1) Whoever constitutes the majority and (2) Whoever is the duly constituted authority.

TAU MU

Q: In case of a division of a church, who gets the property? (Like when a church separates as when there are doctrinal diferences) A: The property will go to the faction which adheres to the original doctrines.

TAU MU

Ruling: The Court ruled that the Ordinance 2925 violated freedom of religion. The government cannot impose some form of tax on somebody for the exercise of his religion. American Bible was a missionary organization which sells bibles as part of propagating their faith. It was only making a small profit from the sale of bibles. If they were making a lot of profit, then that would be the time that the government should impose a tax on them since it would not anymore be part of “propagating the faith.”

TAU MU

Facts: American Bible was a religious corporation, selling bibles and other religious articles. They were required to obtain a license and pay the corresponding fee for being engaged in the sale of merchandise.

TAU MU TAU MU

AMERICAN BIBLE vs. CITY OF MANILA 101 PHIL. 386 (1957)

The Fraternal Ateneo de Davao

TAU MU TAU MU

Ruling: The SC held that the Church and the State are separate and distinct from each other. The expulsion/excommunication of members of a religious institution/organization is a matter best left to the discretion of the officials, and the laws and canons, of said institution/organization. It is not for the courts to exercise control over church authorities in the performance of their discretionary and official functions. Rather, it is for the members of religious institutions/organizations to conform to just church regulations. In the leading case of Fonacier v. Court of Appeals, the SC enunciated the doctrine that in disputes involving religious institutions or organizations, there is one area which the Court should not touch: doctrinal and disciplinary diferences. Thus, the amendments of the constitution, restatement of articles of religion and abandonment of faith or abjuration alleged by appellant, having to do with faith, practice, doctrine, form of worship, ecclesiastical law, custom and rule of a church and having reference to the power of excluding from the church those allegedly unworthy of membership, are unquestionably ecclesiastical matters which are outside the province of the civil courts.

KITY

TAU MU TAU MU TAU MU TAU MU TAU MU TAU MU

CONSTITUTIONAL LAW II Order of Saint Thomas More Atty. Philip John Pojas/Atty. Rovyne G. Jumao-as, RN University College of Law

 The basis of this clause is the respect for the inviolability of the human conscience.  The free exercise of religious profession/calling is superior to contract rights. In case of conflict, the latter, must yield to the former [VICTORIANO vs. ELIZALDE ROPE WORKERS UNION (235 SCRA 197)].  It is only where unavoidably necessary to prevent an immediate and grave danger to security and welfare of the community that infringement of religious freedom may be justified, and only to the smallest extent necessary to avoid the danger [VICTORIANO vs. ELIZALDE ROPE WORKERS UNION (235 SCRA 197)].  Freedom of religion implies respect for every creed. No one is privileged to characterize the actuation of its adherents in a derogatory sense [INK vs. Gironella (106 SCRA 1)].  The exercise of religious freedom can be regulated by the state when it will bring about the clear and present danger of some substantive evil w/c the state is duty bound to prevent --- serious detriment to the more overriding interest of public health, public morals, or public welfare.

TWO ASPECTS OF RELIGIOUS PROFESSION AND WORSHIP: 1. FREEDOM TO BELIEVE  absolute as long as the belief is confined w/i the realm of thought 2. FREEDOM TO ACT ON ONE’S BELIEF  is subject to regulation where the belief is translated into external acts that afect the public welfare  where the individual externalizes his beliefs in acts/ omissions that afect the public, his freedom to do becomes subject to state authority a. the state cannot compel a person to do something w/c his religion prohibits b. the state cannot prohibit a person from doing something w/c his religion commands 

Religious freedom can be enjoyed only w/ a proper regard for the rights of others.  The police power can be exercised to prevent religious practices inimical to society even if such practices are pursued out of sincere religious conviction and not merely for the purpose of evading the reasonable requirements/prohibitions of law.  If the exercise of religious belief clashes w/ the established institution of society and w/ the law, then the former must yield and give way to the latter.  As long as it can be shown that the exercise of the right does not impair public welfare, the attempt of the state to regulate/ prohibit such right would be unconstitutional.  The constitutional guaranty of FREE EXERCISE and enjoyment of religious profession and worship carries w/ it the right to disseminate religious information [AMERICAN BIBLE SOCIETY vs. CITY OF MANILA (101 Phil. 386].  The test to determine w/c shall prevail between religious freedom and the powers of the state is the TEST OF REASONABLENESS.  The constitutional PROHIBITION AGAINST RELIGIOUS TESTS is aimed against clandestine attempts on the part of the gov’t. to prevent a person from exercising his civil or political rights because of his religious beliefs.  To allow religious tests would have the efect of formal or practical establishment of particular religious faiths... w/ consequent burdens imposed on the free exercise of the faiths of non-favored believers.

FREE EXERCISE CLAUSE Q: Who decides whether something is religious or not? ACADCOM 2010; Contributors: Gene Geocaniga, Jarissa Guiani, Darlene Magabilen TAU MU Page 79 of 179

The freedom to act in accordance with one's belief has two parts: the affirmative aspect and the negative aspect. The affirmative aspect means the state cannot compel the person to do something which his religion prohibits. The negative aspect means the state cannot prevent a person from doing what his religion commands. 

TAU MU TAU MU TAU MU

Facts: Respondent Iglesia ng Dios kay Cristo Jesus, Haligi at Suhay ng Katotohanan (Church of God in Jesus Christ, the Pillar and Ground of Truth), is a non-stock religious society or corporation. In 1976, one Eliseo Soriano and several other members of respondentcorporation disassociated themselves from the latter and succeeded in registering a new non-stock religious society or corporation named, Haligi ng Dios kay Kristo Hess, Haligi at Saligan ng Katotohan in 1977. Respondent-corporation filed with the SEC a petition to compel Haligi ng Dios kay Kristo Hess, Haligi at Saligan ng Katotohan to change its corporate name to another name that is not similar or identical to any name already used by a corporation, partnership or association registered with the Commission. Petitioner is compelled to change its corporate name and be barred from using the same or similar name on the ground that the same causes confusion among their members as well as the public. SEC rendered a decision ordering petitioner to change its name. The CA rendered the assailed decision and affirmed the decision of SEC En Banc.

TAU MU

ANG MGA KAANIB vs. IGLESIA 372 SCRA 172 (2001)

TAU MU

CASES: FREEDOM TO EXERCISE CLAUSE

TAU MU

NO RELIGIOUS TEST CLAUSE - one does not have to pass a religious test to be able to exercise one's civil or political right. Since the 1987 Constitution came out, there is no more religious test. A priest can run for a public office.

TAU MU TAU MU

NOTE: Tolentino vs. Sec. case vs. American Bible Society: The removal of the TAX exemption is unconstitutional because this is an encroachment on the freedom of religion. But the imposition of VAT is not unconstitutional because the VAT here is not imposed on sale, bartering or exchange of goods. It is not a license tax. If it is a license tax, if you need to pay for something in order to practice a certain thing it is unconstitutional. But you don't need the VAT to practice your religion, it is merely incidental.

The Fraternal Ateneo de Davao

TAU MU TAU MU

The application of the compelling state interest test could result to three situations of accommodation: First, mandatory accommodation would result if the Court finds that accommodation is required by the Free Exercise Clause. Second, if the Court finds that the State may, but is not required to, accommodate religious interests, permissive accommodation results. Finally, if the Court finds that that establishment concerns prevail over potential accommodation interests, then it must rule that the accommodation is prohibited. 

KITY

TAU MU TAU MU TAU MU TAU MU TAU MU TAU MU

CONSTITUTIONAL LAW II Order of Saint Thomas More Atty. Philip John Pojas/Atty. Rovyne G. Jumao-as, RN University College of Law A: 1. GERONA Case - court 2. EBRALINAG Case – it seems that it was still the court 3. Justice CRUZ – it should not be left to the court because it is something w/c is intensely personal.

Ruling: The additional words “Ang Mga Kaanib” and “Sa Bansang Pilipinas Inc.” in petitioner’s name are, , as correctly observed by the SEC, merely descriptive of and also referring to the members or kaanib, of respondent who are likewise residing in the Philippines. These words can hardly serve as an efective diferentiating medium necessary to avoid confusion or difficulty in distinguishing petitioner from respondent. This is especially so, since both petitioner and respondent corporations are using the same acronyms --- H.S.K, not to mention the fact that both are espousing religious beliefs and operating in the same place. The fact that there are other non-stock religious societies or corporations using the names Church of the Living God, Inc., Church of God Jesus Christ the Son of God the Head, Church of God in Christ and By the Holy Spirit and other similar names, is of no consequence, it does not authorize the use by petitioner of the essential and distinguishing feature of respondent’s registered and protected corporate name. Ordering petitioner to change its corporate name is not a violation of its constitutionally guaranteed right to religious freedom. In so doing, the SEC merely compelled petitioner to abide by one of the SEC guidelines in the approval of partnership and corporate names, namely its undertaking to manifest its willingness to change its corporate name in the event that another person, firm or entity has a acquired a prior right to the use of the said firm name or one deceptively or confusingly similar to it. IDCP, INC. vs. OFFICE 405 SCRA 497 (2003) Facts: Petitioner, a corporation that operates under DSWD, is a non-governmental organization that extends voluntary services to the Filipino people, especially to Muslim communities. Among the functions petitioner carries out is to conduct seminars, orient manufacturers. In 2001, respondent Office of the Executive Secretary issued EO 46 creating the Philippine Halal Certification Scheme and designating respondent Office on Muslim Afairs to oversee its implementation. Under the EO, respondent OMA has the exclusive authority to issue halal certificates and perform other related regulatory activities. Petitioner contends that the subject EO violates the constitutional provision on the separation of Church and State and that it is unconstitutional for the government to formulate policies and guidelines on the halal certification scheme because said scheme is a function only religious organizations, entity or scholars can lawfully and validly perform for the Muslims. TAU MU Issue: WON the EO is violates the constitutional provision as to freedom of religion.

TAU MU

Ruling: The SC grants the petition. OMA deals with the societal, legal, political and economic concerns of the Muslim community as a “national cultural community” and not as a religious group. Thus, bearing in mind the constitutional barrier between the Church and State, the latter must make sure that OMA does not intrude into purely religious matters lest it violate the nonestablishment clause and the “free exercise of religion” provision found in Article 3, section 5 of the 1987 Constitution. Freedom of religion was accorded preferred status, well aware that it is “designed to protect the broadcast possible liberty of conscience, to allow each man to believe as his conscience directs, to profess his beliefs and to alive as he believes he ought to live, consistent with the liberty of others and with the common good.” Without doubt, classifying a food Issue: WON SEC’s order to petitioner to change its product as halal is a religious function because the corporate name violated its right to religious freedom. standards used are drawn from the Quar’an and Islamic beliefs. By giving OMA the exclusive power to ACADCOM 2010; Contributors: Gene Geocaniga, Jarissa Guiani, Darlene Magabilen TAU MU Page 80 of 179

TAU MU

TAU MU

KITY

The Fraternal Ateneo de Davao

TAU MU TAU MU TAU MU TAU MU TAU MU TAU MU

CONSTITUTIONAL LAW II Order of Saint Thomas More Atty. Philip John Pojas/Atty. Rovyne G. Jumao-as, RN University College of Law classify food products as halal, EO 46 encroached on the religious freedom of Muslim organizations like herein petitioner to interpret for Filipino Muslim what food products are fit for Muslim consumption. Also, by arrogating to itself the task of issuing halal certifications, the State has in efect forced Muslims to accept its own interpretation of the Qur’an and Sunnah on halal food. Only the prevention of an immediate and grave danger to the security and welfare of the community can justify the infringement of religious freedom. If the government fails to show the seriousness and immediacy of the threat, State intrusion is constitutionally unacceptable. In a society with a democratic framework like ours, the State must minimize its interference with the afairs of its citizens and instead allow them to exercise reasonable freedom of personal and religious activity. There is no compelling justification for the government to deprive Muslim organizations, like herein petitioner, of their religious right to classify a product as halal, even on the premise that the health of Muslim Filipinos can be efectively protected by assigning to OMA the exclusive power to issue halal certifications. The protection and promotion of the Muslim Filipinos’ right to health are already provided for in existing laws and ministered to by government agencies charged with ensuring that food products relased in the market are fit for human consumption, properly labeled and safe. Unlike EO 46, these laws do not encroach on the religious freedom of Muslims. With these regularity bodies given detailed functions on how to screen and check the quality and safety of food products, the perceived danger against the health of Muslim and non-Muslim Filipinos alike is totally avoided. The halal certifications issued by petitioner and similar organizations come forward as the official religious approval of a food product fit for Muslim consumption.

three witnesses. At the time Escritor executed her pledge, her husband was still alive but living with another woman. Luciano was likewise married at that time, but had been separated in fact from his wife. During her testimony, Escritor volunteered to present members of her congregation to confirm the truthfulness of their “Declarations of Pledging Faithfulness,” but Judge Caoibes deemed it unnecessary and considered her identification of her signature and the signature of Quilapio sufficient authentication of the documents. Issue: WON respondent’s right to religious freedom was abridged when she was held administratively liable for having illicit relations.

TAU MU TAU MU

Ruling: The Declaration of Pledging of Faithfulness executed by the respondent and her mate greatly afect the administrative liability of respondent. Jehovah’s Witnesses admit and recognize the supremacy of the proper public authorities in the marriage arrangement. Freedom of choice guarantees the liberty of the religious conscience and prohibits any degree of compulsion or burden, whether direct or indirect, in the practice of one’s religion. The Free Exercise Clause principally guarantees voluntarism, although the Establishment Clause also assures voluntarism by placing the burden of the advancement of religious groups on their intrinsic merits and not on the support of the state. In interpreting the Free Exercise Clause, the realm of belief poses no difficulty. The realm of belief and creed is infinite and limitless bounded only by one’s imagination and thought. So is the freedom of belief, including religious belief, limitless and without bounds. Court has ruled that government employees engaged in illicit relations are guilty of “disgraceful and immoral conduct” for which he/she may be held ESTRADA vs. ESCRITOR administratively liable. Respondent Escritor does not 408 SCRA 1 (2003) claim that there is error in the settled jurisprudence that an illicit relation constitutes disgraceful and Facts: Alejandro Estrada wrote to Judge Caoibes immoral conduct for which a government employee is requesting for an investigation of rumors that held liable. Nor is there an allegation that the norms of respondent Soledad Escritor, court interpreter in said morality with respect to illicit relations have shifted court, is living with a man not her husband. They towards leniency from the time these precedent cases allegedly have a child of eighteen to twenty years old. were decided. The Court finds that there is no such He filed the charge against Escritor as he believes that error or shift, thus the SC find no reason to deviate she is committing an immoral act that tarnishes the from these rulings that such illicit relationship image of the court, thus she should not be allowed to constitutes “disgraceful and immoral conduct” remain employed therein as it might appear that the punishable under the Civil Service Law. Respondent court condones her act. having admitted the alleged immoral conduct, she, like Judge Caoibes referred the letter to Escritor the respondents in the above-cited cases, could be who stated that “there is no truth as to the veracity of held administratively liable. However, there is a the allegation” and challenged Estrada to “appear in distinguishing factor that sets the case at bar apart the open and prove his allegation in the proper forum.” from the cited precedents, i.e., as a defense, Judge Caoibes set a preliminary conference. Escritor respondent invokes religious freedom since her moved for the inhibition of Judge Caoibes from hearing religion, the Jehovah’s Witnesses, has, after thorough her case to avoid suspicion and bias as she previously investigation, allowed her conjugal arrangement with filed an administrative complaint against him and said Luciano based on the church’s religious beliefs and case was still pending in the Office of the Court practices. Administrator (OCA). Escritor’s motion was denied. There is no doubt that choosing between The preliminary conference proceeded with both keeping her employment and abandoning her religious Estrada and Escritor in attendance. Respondent belief and practice and family on the one hand, and Escritor testified that when she entered the judiciary in giving up her employment and keeping her religious 1999, she was already a widow, her husband having practice and family on the other hand, puts a burden died in 1998. She admitted that she has been living on her free exercise of religion. The burden on with Luciano without the benefit of marriage for twenty respondent in the case at bar is even greater as the years and that they have a son. But as a member of price she has to pay for her employment is not only her the religious sect known as the Jehovah’s Witnesses religious precept but also her family which, by the and the Watch Tower and Bible Tract Society, their Declaration Pledging Faithfulness, stands “honorable conjugal arrangement is in conformity with their before God and men.”However, even if the Court religious beliefs. In fact, after ten years of living deems sufficient respondent’s evidence on the together, she executed a “Declaration of Pledging sincerity of her religious belief and its centrality in her Faithfulness.” faith, the case at bar cannot still be decided using the Escritor’s partner, Luciano, executed a similar “compelling state interest” test. The case at bar is one pledge. Both pledges were executed and signed by of first impression, thus the parties were not aware ACADCOM 2010; Contributors: Gene Geocaniga, Jarissa Guiani, Darlene Magabilen TAU MU Page 81 of 179

TAU MU TAU MU

TAU MU

TAU MU

TAU MU

TAU MU

TAU MU

TAU MU

TAU MU

TAU MU

TAU MU

TAU MU TAU MU TAU MU TAU MU TAU MU TAU MU TAU MU

Ruling: The Constitution adheres to the benevolent neutrality approach that gives room for accommodation of religious exercises as required by the Free Exercise Clause. Thus, in arguing that respondent should be held administratively liable as the arrangement she had was “illegal per se because, by universally recognized standards, it is inherently or by its very nature bad, improper, immoral and contrary to good conscience,” the Solicitor General failed to appreciate that benevolent neutrality could allow for accommodation of morality based on religion, provided it does not ofend compelling state interests. Finally, even assuming that the OSG has proved a compelling state interest, it has to further demonstrate that the state has used the least intrusive means possible so that the free exercise is not infringed any more than

TAU MU

Issue: WON the compelling state interest test must be applied.

TAU MU

Facts: Respondent Soledad Escritor once again stands before the Court invoking her religious freedom and her Jehovah God in a bid to save her family – united without the benefit of legal marriage - and livelihood. The State, on the other hand, seeks to wield its power to regulate her behavior and protect its interest in marriage and family and the integrity of the courts where respondent is an employee.

TAU MU TAU MU

ESTRADA vs. ESCRITOR 492 SCRA 1 (2006)

The Fraternal Ateneo de Davao

TAU MU TAU MU

Dispositive portion: The case is REMANDED to the Office of the Court Administrator. The Solicitor General is ordered to intervene in the case where it will be given the opportunity (a) to examine the sincerity and centrality of respondent’s claimed religious belief and practice; (b) to present evidence on the state’s “compelling interest” to override respondent’s religious belief and practice; and (c) to show that the means the state adopts in pursuing its interest is the least restrictive to respondent’s religious freedom.

KITY

TAU MU TAU MU TAU MU TAU MU TAU MU TAU MU

CONSTITUTIONAL LAW II Order of Saint Thomas More Atty. Philip John Pojas/Atty. Rovyne G. Jumao-as, RN University College of Law of the burdens of proof they should discharge in the Court’s use of the “compelling state interest” test. , it is inappropriate for the complainant, a private person, to present evidence on the compelling interest of the state. The burden of evidence should be discharged by the proper agency of the government which is the Office of the Solicitor General. To properly settle the issue in the case at bar, the government should be given the opportunity to demonstrate the compelling state interest it seeks to uphold in opposing the respondent’s stance that her conjugal arrangement is not immoral and punishable as it comes within the scope of free exercise protection. Should the Court prohibit and punish her conduct where it is protected by the Free Exercise Clause, the Court’s action would be an unconstitutional encroachment of her right to religious freedom. The SC cannot therefore simply take a passing look at respondent’s claim of religious freedom, but must instead apply the “compelling state interest” test. The government must be heard on the issue as it has not been given an opportunity to discharge its burden of demonstrating the state’s compelling interest which can override respondent’s religious belief and practice. To repeat, this is a case of first impression where we are applying the “compelling state interest” test in a case involving purely religious conduct. The careful application of the test is indispensable as how we will decide the case will make a decisive diference in the life of the respondent who stands not only before the Court but before her Jehovah God.

necessary to achieve the legitimate goal of the state, i.e., it has chosen a way to achieve its legitimate state end that imposes as little as possible on religious liberties. Again, the Solicitor General utterly failed to prove this element of the test. Other than the two documents ofered as cited above which established the sincerity of respondent’s religious belief and the fact that the agreement was an internal arrangement within respondent’s congregation, no iota of evidence was ofered. In fact, the records are bereft of even a feeble attempt to procure any such evidence to show that the means the state adopted in pursuing this compelling interest is the least restrictive to respondent’s religious freedom. Thus, the SC held that in this particular case and under these distinct circumstances, respondent Escritor’s conjugal arrangement cannot be penalized as she has made out a case for exemption from the law based on her fundamental right to freedom of religion. The Court recognizes that state interests must be upheld in order that freedoms - including religious freedom - may be enjoyed. In the area of religious exercise as a preferred freedom, however, man stands accountable to an authority higher than the state, and so the state interest sought to be upheld must be so compelling that its violation will erode the very fabric of the state that will also protect the freedom. In the absence of a showing that such state interest exists, man must be allowed to subscribe to the Infinite. RE: REQUEST OF MUSLIM EMPLOYEES IN THE DIFFERENT COURTS IN ILIGAN CITY (RE: OFFICE HOURS) 477 SCRA 648 Facts: Several Muslim employees in the diferent courts in the said city requested to Judge Salazar that they be allowed to enjoy certain privileges during the Holy Month of Ramadhan. Judge Salazar expressed his conformity with the first request, i.e., allowing them to hold office from 7:30 a.m. to 3:30 p.m. without any break during the month of Ramadan. However, he expressed some misgivings about the second request, i.e., excusing them from work from 10:00 a.m. to 2:00 p.m. every Friday during the entire calendar year. In support of their requests, the Muslim employees invoke PD 291 as amended by P.D. No. 322. The avowed purpose of P.D. No. 291 was to reinforce national unity by recognizing Muslim holidays and making them part of our national holidays. Issue: WON the freedom to exercise one’s religion was abridged in this case. Ruling: The Court recognizes that the observance of Ramadan and the Friday Muslim Prayer Day is integral to the Islamic faith. However, while the observance of Ramadan and allowing the Muslim employees in the Judiciary to hold flexible office hours from 7:30 a.m. to 3:30 p.m. without any break during the month of Ramadan finds support in Section 3 (a) of P.D. No. 291, as amended by P.D. No. 322, there is no such basis to excuse them from work from 10:00 a.m. to 2:00 p.m. every Friday, the Muslim Prayer Day, during the entire calendar year. On the other hand, the need of the State to prescribe government office hours as well as to enforce them uniformly to all civil servants, Christians and Muslims alike, cannot be disregarded. Underlying Section 5, Rule XVII of the Omnibus Rules Implementing Book V of E.O. No. 292 is the interest of the general public to be assured of continuous government service during office hours every Monday through Friday. The said rule enjoins all civil servants, of whatever religious denomination, to render public

ACADCOM 2010; Contributors: Gene Geocaniga, Jarissa Guiani, Darlene Magabilen TAU MU Page 82 of 179

TAU MU TAU MU TAU MU

Ruling: The Court ruled that there was no clear and present danger posed if they were exempted from the requirement. There was no danger that they will produce citizens who have no love of country if the government allowed a religious exemption. The danger predicted in the Gerona case has come to pass. The Jehovah’s Witnesses is a small group and it will not shake our part of the world if we exempt them from requirement. If your religion is against the saluting of the flag, then you are entitled to it because of the free exercise clause. There is no clear and present danger. You are only required to stand and this afects only a very small group.

TAU MU TAU MU TAU MU TAU MU TAU MU

Section 6. The liberty of abode and of changing the same within the limits prescribed by law shall not be impaired except upon lawful order of the court. Neither shall the right to travel be impaired except in the interest of national

TAU MU

Ruling: The Court ruled that the Ordinance 2925 violated freedom of religion. The government cannot impose some form of tax on somebody for the exercise of his religion. American Bible was a missionary organization which sells bibles as part of propagating their faith. It was only making a small profit from the sale of bibles. If they were making a lot of profit, then that would be the time that the government should impose a tax on them since it would not anymore be part of “propagating the faith.”

TAU MU

Facts: American Bible was a religious corporation, selling bibles and other religious articles. They were required to obtain a license and pay the corresponding fee for being engaged in the sale of merchandise.

TAU MU

Facts: There were students, all members of the Jehovah’s Witnesses, who were expelled for refusing to salute the flag, sing the national anthem and recite the oath of allegiance. Their religion prohibited them from doing so.

AMERICAN BIBLE vs. CITY OF MANILA 101 PHIL. 386 (1957)

The Fraternal Ateneo de Davao

TAU MU TAU MU

EBRALINAG vs. SECRETARY 219 SCAR 256 (1993)

KITY

TAU MU TAU MU TAU MU TAU MU TAU MU TAU MU

CONSTITUTIONAL LAW II Order of Saint Thomas More Atty. Philip John Pojas/Atty. Rovyne G. Jumao-as, RN University College of Law service of no less than eight hours a day or forty (40) hours a week. To allow the Muslim employees in the Judiciary to be excused from work from 10:00 a.m. to 2:00 p.m. every Friday (Muslim Prayer Day) during the entire calendar year would mean a diminution of the prescribed government working hours. For then, they would be rendering service twelve (12) hours less than that required by the civil service rules for each month. Further, this would encourage other religious denominations to request for similar treatment. The performance of religious practices, whether by the Muslim employees or those belonging to other religious denominations, should not prejudice the courts and the public. Indeed, the exercise of religious freedom does not exempt anyone from compliance with reasonable requirements of the law, including civil service laws. Hence, the SC GRANTED the request to allow the Muslim employees in the Judiciary to hold office hours from 7:30 a.m. to 3:30 p.m. without break during the month of Ramadan pursuant to Section 3 (a) of Presidential Decree No. 291, as amended by Presidential Decree No. 322.

security, public safety or public health, as may be provided by law. RIGHTS GUARANTEED UNDER SECTION 6: 1. Freedom to choose and change one’s place of abode (Liberty of abode) - the right of a person to have his home, or to maintain or change his home or dwelling in whatever place he has chosen. LIMITATION: it must be within the limits prescribed by law. You cannot move to the public plaza. 2. Freedom to travel within the country and outside - he right given to a person to go wherever he pleases without interference from anyone. FREEDOM OF MOVEMENT:  The purpose of the guaranty is to further emphasize the individual’s liberty as safeguarded in general terms by the due process clause. Q: What are the liberties guaranteed under this clause? A: The liberties under this clause include the a. right to choose one’s residence, b. to leave it whenever he pleases, and c. to travel wherever he wills. 

Right: Liberty of abode Manner of curtailment: Lawful order of the court and within the limits prescribed by law. 

Right: Right to travel Manner of curtailment: May be curtailed even by administrative officers (ex. passport officers) in the interest of national security, public safety, or public health, as may be provided by law. General Rule: The right to travel shall not be impaired. Exceptions: When can the right to travel be restricted? Who can validly restrict this right? 1. Congress, but on the grounds of (a) national security, (b) public safety, and (c) public health. 2. The Court.-- It is an inherent power of the court. The limitation of the right to travel it need not be grounded on the grounds of national security, public safety, and public health. It is inherent for the proper administration of justice, even if there is no express grant from the constitution. The guarantee that the accused will appear for trial and the issuance of the bail bond serves as the lawful order for the refusal of the right to travel. 3. Administrative and Executive officials – i.e. Bureau of Immigration if there is a law. (Note: Implementation of the law enacted by the Congress, which again, must be based on the 3 grounds in number 1)  The right to travel and the liberty of abode are distinct from the right to return to one’s country, as shown by the fact that the Declaration of Human Rights and the Covenant on Human Rights have separate guarantees for these. Hence, the right to return to one’s country is not covered by the specific right to travel and liberty of abode. [MARCOS vs. MANGLAPUS]  The right to travel can be impaired even w/o lawful court order BUT the executive officer must not be armed w/ arbitrary discretion to impose limitations. He can impose limits only on the basis of national security, public safety, or public health and as may be provided by law. 

IMPAIRMENT may be subject to judicial review.

ACADCOM 2010; Contributors: Gene Geocaniga, Jarissa Guiani, Darlene Magabilen TAU MU Page 83 of 179

MEASURE OF ALLOWABLE IMPAIRMENT: a. necessity in the interest of a.national security, b.public safety, or c. public health; and d.explicit provisions of statutory law or the Rules of Court. 

TAU MU TAU MU TAU MU TAU MU TAU MU

Facts: Petitioners filed a Petition for Declaratory Judgment with Application for Temporary Restraining Order and Injunction which sought the declaration of nullity of the administrative issuances for being inconsistent with the provisions of Republic Act 2000, entitled "Limited Access Highway Act" enacted in 1957 such as (a) DPWH Administrative Order No. 1, Series of 1968; (b) DPWH Department Order No. 74, Series of 1993; and, (c) Art. II, Sec. 3(a) of the Revised Rules on Limited Access Facilities promulgated in 199[8] by the DPWH thru the Toll Regulatory Board (TRB). Previously, pursuant to its mandate under R.A. 2000, DPWH issued DO No. 215 declaring the Manila-Cavite (Coastal Road) Toll Expressway as limited access facilities. Accordingly, petitioners filed an Amended Petition wherein petitioners sought the declaration of nullity of the aforesaid administrative issuances. Moreover, petitioners prayed for the issuance of a temporary restraining order and/or preliminary injunction to prevent the enforcement of the total ban on motorcycles along the entire breadth of North and South Luzon Expressways and the Manila-Cavite (Coastal Road) Toll Expressway under DO 215. Thereafter, the trial court, after due hearing, issued an order granting petitioners’ application for preliminary injunction. The DPWH acting thru the TRB, issued Department Order No. 123 allowing motorcycles with engine displacement of 400 cubic centimeters inside limited access facilities (toll ways).

TAU MU

MIRASOL vs. DPWH 490 SCRA 318 (2006)

TAU MU

CASES: RIGHT TO TRAVEL/LIBERTY OF ABODE

TAU MU TAU MU

Q: Can the accused be granted the right to travel? A: YES if 1. he can show to the court the URGENCY to travel 2. in the application, he must state the DURATION of his travel 3. he must obtain the CONSENT OF HIS SURETY.

The Fraternal Ateneo de Davao

TAU MU TAU MU

This right should not be construed as delimiting the inherent power of the Courts to use all means necessary to carry their orders into efect in criminal cases pending before them.  While this right covers both the right to exit from and entry into the country, aliens cannot claim the same right because every sovereign nation has the inherent power essential to self-preservation to forbid the entrance of foreigners w/i its dominion, or to admit them only in such cases and upon such conditions as it may see fit to prescribe [EKIU vs. US (42 US 651)].  Norms for admission of aliens into the country are political matters virtually beyond the reach of judicial review.

KITY

TAU MU TAU MU TAU MU TAU MU TAU MU TAU MU

CONSTITUTIONAL LAW II Order of Saint Thomas More Atty. Philip John Pojas/Atty. Rovyne G. Jumao-as, RN University College of Law  Thus a person facing criminal charges may be restrained from leaving the country or, if abroad, compelled to return [ZEMEL vs. RUSK (381 US 1)].

Ruling: A toll way is not an ordinary road. As a facility designed to promote the fastest access to certain destinations, its use, operation, and maintenance require close regulation. Public interest and safety require the imposition of certain restrictions on toll ways that do not apply to ordinary roads. As a special kind of road, it is but reasonable that not all forms of transport could use it. The right to travel does not mean the right to choose any vehicle in traversing a toll way. The right to travel refers to the right to move from one place to another. Petitioners can traverse the toll way any time they choose using private or public four-wheeled vehicles. Petitioners are not denied the right to move from Point A to Point B along the toll way. Petitioners are free to access the toll way, much as the rest of the public can. The mode by which petitioners wish to travel pertains to the manner of using the toll way, a subject that can be validly limited by regulation. Petitioners themselves admit that alternative routes are available to them. Their complaint is that these routes are not the safest and most convenient. Even if their claim is true, it hardly qualifies as an undue curtailment of their freedom of movement and travel. The right to travel does not entitle a person to the best form of transport or to the most convenient route to his destination. The obstructions found in normal streets, which petitioners complain of (i.e., potholes, manholes, construction barriers, etc.), are not sufered by them alone. Moreover, contrary to what the petitioner’s claim that their possession of a driver’s license from the LTO and the fact that their vehicles are registered with that office entitle them to use all kinds of roads in the country, the SC held that there exists no absolute right to drive because this privilege is heavily regulated. Only a qualified group is allowed to drive motor vehicles: those who pass the tests administered by the LTO. A driver’s license issued by the LTO merely allows one to drive a particular mode of transport. It is not a license to drive or operate any form of transportation on any type of road. Vehicle registration in the LTO on the other hand merely signifies the roadworthiness of a vehicle. This does not preclude the government from prescribing which roads are accessible to certain vehicles. YAP Jr. vs. CA, PEOPLE OF THE PHILIPPINES 358 SCRA 564 (2001) Facts: For misappropriating amounts, petitioner was convicted of estafa. The RTC denied the motion to fix bail for his provisional liberty under the cash bond he had filed earlier in the proceedings. CA granted the said motion but with conditions that a certification/guaranty from the Mayor of the place of his residence that he is a resident of the area and that he will remain to be a resident therein until final judgment is rendered or in case he transfers residence, it must be prior notice to the court, and that a hold departure order be issued against and that he surrender his passport. Issue: WON petitioner’s constitutional liberty of abode and travel in imposing the other conditions for the grant of bail was unduly restricted.

TAU MU

Ruling: Petitioner contests the condition that he secures such certification or guaranty from the Mayor, claiming that the same violates his liberty of abode and travel. Notably, petitioner does not question the holddeparture order which prevents him from leaving the Philippines unless expressly permitted by the court which issued the order. The right to change abode and Issue: WON the prohibition on the use of motorcycles travel within the Philippines are not absolute rights. in toll ways unduly deprive the petitioners of their right The order of the Ca releasing petitioner on bail to travel. constitutes such lawful order as contemplated by the provision in the Constitution. It is simply consistent ACADCOM 2010; Contributors: Gene Geocaniga, Jarissa Guiani, Darlene Magabilen TAU MU Page 84 of 179

TAU MU

MANOTOC vs. CA 142 SCRA 149 (1987)

TAU MU TAU MU TAU MU

This right to return is part of those generally accepted principles.

TAU MU

Ruling 1: The Court held that the right to travel means only the right to leave. It does not include the right to return to the country. These are two distinct rights. The right to return is not protected by the Bill of Rights but is protected by the Philippine Law by virtue of the Principle of Incorporation. The principle of incorporation states that generally accepted principles of international law is part of the law of the land.

TAU MU

Facts: Former President Marcos sought to compel the Secretary of Foreign Afairs to issue him travel documents so that he could return to the Philippines from his exile in Hawaii. He argued that the decision of then Pres. Cory Aquino to bar his return violates his right to travel as guaranteed by the Bill of Rights. Note: There was no court order, nor was there a law limiting his right to travel. TAU MU

TAU MU

MARCOS vs. MANGLAPUS 177 SCRA 668 (1989)

TAU MU

Ruling: The Court held that a court has the power to restrict the right to travel, with or without an express grant from the Constitution if there is a criminal case before it. This is necessary for the efficient administration of justice. The power is inherent in the courts. This is an implied power in the administration of justice. If the courts would have no right to limit the right to travel, the administration of justice would not be efective.

TAU MU

Facts: Silverio was charged with a criminal case and he posted bail. Two years later, the government filed a motion to cancel his passport because he could not be arraigned as he was abroad every time the arraignment was set. The trial court granted the motion. He now contends that a court can only restrict the right to travel but only on the ground of national security, public safety or public health.

TAU MU

SILVERIO vs. CA 195 SCRA 760 (1991)

TAU MU TAU MU

Ruling: The Court held that since he was on bail, the Court can validly restrict his right to travel. The bail is supposed to be used as guarantee that he will appear on trial. If he leaved for the U.S., the Court can no longer have jurisdiction over him. The conditions of the bail bond serves as the lawful order for the refusal of the right to travel. However, this is not absolute. The Court may allow those out on bail to travel according to the court’s discretion.

The Fraternal Ateneo de Davao

TAU MU TAU MU

Facts: Manotoc was facing several charges of estafa and he was out on bail. During the pendency of the case, he filed a motion in court to allow him to travel to the US in connection with some business transactions. The prosecutor opposed his motion. He went to the SC and contended that the denial violated his right to travel.

KITY

TAU MU TAU MU TAU MU TAU MU TAU MU TAU MU

CONSTITUTIONAL LAW II Order of Saint Thomas More Atty. Philip John Pojas/Atty. Rovyne G. Jumao-as, RN University College of Law with the nature and function of a bail bond, which is to ensure that petitioner will make himself available at all times whenever the Court requires his presence. Besides, a closer look at the questioned condition will show that petitioner is not prevented from changing abode; he is merely required to inform the court in case he does so.

 

If the right to return is protected by the Constitution, it can only be restricted by law. In this case, there was no law. If the right to return is not protected by the Bill of Rights but is protected by the Bill of Rights but is protected by the Philippine Law, who can restrict it?

Ruling 2: The Court ruled that there is no more need of a law. the executive can limit it as part of the President’s implied powers (Doctrine of Residual Powers). This was how the SC justified the power of Pres. Aquino in barring the return of former Pres. Marcos. Note: Inherent power of the executive: (1) The power to do anything not prohibited by the Constitution; and (2) The power to do anything not prohibited by law. Section 7. The right of the people to information on matters of public concern shall be recognized. Access to official records, and to documents and papers pertaining to official acts, transactions, or decisions, as well as to government research data used as basis for policy development, shall be aforded the citizen, subject to such limitations as may be provided by law. Pricnciple of Transparency and full disclosure in Government:  Article. 2, Section 28: Subject to reasonable conditions prescribed by law, the State adopts and implements a policy of full public disclosure of all its transactions involving public interest. Rights guaranteed under Section 7: 1. Right to information on matters of public concern The right only involves matters of public concern. So, when one invokes this right, you have to consider first whether what the person wants to find out is a matter of public concern. 2. Right of access to official records and to documents and papers pertaining to official acts, transactions or decisions, as well also government research data used as basis for policy development Q: What may be the subject of the right of access? A: 1. official records 2. documents and papers pertaining to official acts, transactions, or decisions 3. government research data used as basis for policy development Q: Can the right to access be limited? A: Yes, as may be provided by law. Limitations: As may be provided by law; the custodians of the record can regulate the manner of access to the documents, such as time and place. But the custodian cannot deny one access because they are public documents. Persons entitled to the above rights: The first right is guaranteed to anyone, but the second right is reserved only to citizens of the Philippines. But if the alien wants to access records, say there is a criminal record involving him, he can access such records. Discretion of government: The government has discretion with respect to the authority to determine what matters are of public concern and the authority to determine the manner of access to them.

ACADCOM 2010; Contributors: Gene Geocaniga, Jarissa Guiani, Darlene Magabilen TAU MU Page 85 of 179

CONSTITUTIONAL LAW II Order of Saint Thomas More Atty. Philip John Pojas/Atty. Rovyne G. Jumao-as, RN University College of Law

KITY

Ateneo de Davao

Matters of public concern: It is a broad spectrum of subjects w/c the public may want to know, either because these directly afect their lives or simply because such matters arouse the interest of an ordinary citizen. 

of

the

TAU MU TAU MU TAU MU TAU MU TAU MU

By “government research data” what is meant are findings of government-funded research.

TAU MU



TAU MU

1. NATIONAL SECURITY MATTERS and INTELLIGENCE INFORMATION These include state secrets re: military, diplomatic and other nat’l. security, and information on inter-gov’t. exchanges prior to the conclusion of treaties and executive agreements. Where there is no need to protect state secrets, the privilege to withhold documents and other information may not be invoked, provided that they are examined “in strict confidence” and given “scrupulous protection”. 2. TRADE SECRETS and BANKING TRANSACTIONS This is in pursuance to the Intellectual Property Code (RA 8293) and the Secrecy of Bank Deposits Act (RA 1405). Exception: if there is a case filed against a person and there is a lawful order of the court) 3. INTER-GOVERNMENT EXCHANGES PRIOR TO THE CONCLUSION OF TREATIES AND EXECUTIVE AGREEMENTS 4. DIPLOMATIC CORRESPONDENCE 5. CLOSED DOOR CABINET MEETINGS 6. EXECUTIVE SESSIONS OF EITHER HOUSE OF CONGRESS 7. INTERNAL DELIBERATIONS OF THE SUPREME COURT 8. CRIMINAL MATTERS 9. ETHICAL STANDARDS ACT – prohibits public officials and employees from divulging “confidential or classified information” officially known to them by reason of their office which is NOT made available to the public

TAU MU

the right

TAU MU

on

TAU MU TAU MU

Note: While access to official records may NOT be prohibited, it may be regulated by statutory law or by the inherent power of an officer to control his office and the records under his custody... to exercise some discretion as to the manner in w/c persons desiring to inspect, examine, or copy the record may exercise their rights

TAU MU TAU MU

This is also related to Art. II Sec. 28 (Policy of Disclosure) and the Principle of Transparency under PIL.  Access to information on matters of public concern is essential to the proper exercise of freedom of expression on such matters.  The citizen has a right to know what is going on in the country and in his government so he can express his views thereon knowledgeably and intelligently.  The purpose of these rights is to prevent abuse of government power.

TAU MU TAU MU TAU MU TAU MU TAU MU TAU MU

Note: This right is recognized but this does not mean that the government agency who is obliged to supply the information should supply the documents. The constitution only guarantees access to the documents, and not to compel the government to provide the copies to any person exercising such right.

Recognized restrictions people to information:

The Fraternal



In the exercise of the courts’ discretion that access to court records may be permitted subject to the supervisory and protective powers of the court,, the following issues may be relevant: (1) whether parties have interest in privacy; (2) whether information is being sought for legitimate purpose or for improper purpose; (3) whether there is threat of particularly serious embarrassment to party, (4) whether information is important to public health and safety; (5) whether sharing of information among litigants would promote fairness and efficiency; (6) whether party benefiting from confidentiality order is public entity or official; and (7) whether case involves issues important to the public. [Case: HILADO vs. REYES 496 SCRA 282] Remedies of persons unlawfully denied of the right to information: a. Mandamus b. Damages under Articles 19 and 27 of the Civil Code Note: [CHAVEZ vs. PCGG] - Does the right to information include the right to access to the terms of Government negotiations prior to the conclusion of the negotiation? A: The public has the right to access to this information, even the negotiation proper the public has the right to that kind of information -- not only the contract or the finished product. However, there should be a distinction between proposal and inter-agency communication. Inter-agency communications or the process of formulating proposals is not included in this right. Note: Just distinguish between inter-agency communications – if the discussion is still within the office, this is still not covered by the right to information; but once they have agreed that this (option) will be our proposal, then that is already covered by the right to information (Chavez vs. PEA) Note: The right to information, however, does not extend to matters recognized as privileged information under the separation of powers. The right does not also apply to information on military and diplomatic secrets, information afecting national security, and information on investigations of crimes by law enforcement agencies before the prosecution of the accused, which courts have long recognized as confidential. The right may also be subject to other limitations that Congress may impose by law. (Chavez vs. PEA, 384 SCRA 152) Valmonte v. Belmonte, Jr.: “An essential element of these freedoms is to keep open a continuing dialogue or process of communication between the government and the people. It is in the interest of the State that the channels for free political discussion be maintained to the end that the government may perceive and be responsive to the people’s will. Yet, this open dialogue can be efective only to the extent that the citizenry is informed and thus able to formulate its will intelligently. Only when the participants in the discussion are aware of the issues and have access to information relating thereto can such bear fruit.” CASES: RIGHT TO INFORMATION ECHEGARAY vs. SECRETARY 297 SCRA 754 (October 12, 1998)

ACADCOM 2010; Contributors: Gene Geocaniga, Jarissa Guiani, Darlene Magabilen TAU MU Page 86 of 179

The Fraternal Ateneo de Davao

TAU MU TAU MU TAU MU TAU MU TAU MU TAU MU

Issue: WON the petitioner was deprived of his right to information of the execution procedure by a convict through lethal injection.

KITY

TAU MU TAU MU TAU MU TAU MU TAU MU TAU MU

CONSTITUTIONAL LAW II Order of Saint Thomas More Atty. Philip John Pojas/Atty. Rovyne G. Jumao-as, RN University College of Law Facts: The SC affirmed the conviction of petitioner Echegaray for the crime of rape of the 10 year-old daughter of his common-law spouse and the imposition upon him of the death penalty for the said crime. Petitioner subsequently filed a Motion for Reconsideration raising mainly factual issues, and on its heels, a Supplemental Motion for Reconsideration raising for the first time the issue of the constitutionality of Republic Act No. 7659 (the death penalty law) and the imposition of the death penalty for the crime of rape. In the meantime, Congress had seen it fit to change the mode of execution of the death penalty from electrocution to lethal injection, and passed Republic Act No. 8177, An Act Designating Death By Lethal Injection As The Method Of carrying Out Capital Punishment, Amending For The Purpose Article 81 Of The Revised Penal Code, As Amended By Section 24 Of Republic Act 7659. Pursuant to the provisions of said law, the Secretary of Justice promulgated the Rules and Regulations to Implement Republic Act No. 8177 (Implementing Rules) and directed the Director of the Bureau of Corrections to prepare the Lethal Injection Manual. Petitioner filed a Petition for Prohibition, Injunction and/or Temporary Restraining Order to enjoin respondents Secretary of Justice and Director of the Bureau of Prisons from carrying out the execution by lethal injection of petitioner under R.A. No. 8177 and its implementing rules as these are unconstitutional and void. Thereafter, petitioner filed a Motion for Leave of Court to Amend and Supplement Petition with the Amended and Supplemental Petition attached thereto, invoking the additional ground of violation of equal protection.

TAU MU

Ruling: Yes. The SC held that Section 19 fails to provide for review and approval of the Lethal Injection Manual by the Secretary of Justice, and unjustifiably makes the manual confidential, hence unavailable to interested parties including the accused/convict and counsel. Section 19 of the Rules and Regulations to Implement Republic Act No. 8177 provides that: SEC. 19. EXECUTION PROCEDURE. - Details of the procedure prior to, during and after administering the lethal injection shall be set forth in a manual to be prepared by the Director. The manual shall contain details of, among others, the sequence of events before and after execution; procedures in setting up the intravenous line; the administration of the lethal drugs; the pronouncement of death; and the removal of the intravenous system. xxx Said manual shall be confidential and its distribution shall be limited to authorized prison personnel. Thus, the Supreme Court finds in the first paragraph of Section 19 of the implementing rules a veritable vacuum. The Secretary of Justice has practically abdicated the power to promulgate the manual on the execution procedure to the Director of the Bureau of Corrections, by not providing for a mode of review and approval thereof. Being a mere constituent unit of the Department of Justice, the Bureau of Corrections could not promulgate a manual that would not bear the imprimatur of the administrative superior, the Secretary of Justice as the rule-making authority under R.A. No. 8177. Such apparent abdication of departmental responsibility renders the said paragraph invalid. As to the second paragraph of section 19, the Court finds the requirement of confidentiality of the contents of the manual even with respect to the convict unduly suppressive. It sees no legal impediment for the convict, should he so desire, to ACADCOM 2010; Contributors: Gene Geocaniga, Jarissa TAU MU Page 87 of 179

obtain a copy of the manual. The contents of the manual are matters of public concern "which the public may want to know, either because these directly afect their lives, or simply because such matters naturally arouse the interest of an ordinary citizen." The incorporation in the Constitution of a guarantee of access to information of public concern is recognition of the essentiality of the free flow of ideas and information in a democracy. In the same way that free discussion enables members of society to cope with the exigencies of their time, access to information of general interest aids the people in democratic decisionmaking by giving them a better perspective of the vital issues confronting the nation. CHAVEZ vs. PCGG 299 SCRA 744 (1998) Facts: Petitioner, invoking his constitutional right to information and the correlative duty of the state to disclose publicly all its transactions involving the national interest, demands that respondents make public any and all negotiations and agreements pertaining to PCGG’s task of recovering the Marcoses’ ill-gotten wealth. He claims that any compromise on the alleged billions of ill-gotten wealth involves an issue of “paramount public interest,” since it has a “debilitating efect on the country’s economy” that would be greatly prejudicial to the national interest of the Filipino people. Hence, the people in general have a right to know the transactions or deals being contrived and efected by the government. Respondents, on the other hand, do not deny forging a compromise agreement with the Marcos heirs. They claim that petitioner’s action is premature because there is no showing that he asked the PCGG to disclose the negotiations and the Agreements. And even if he has, PCGG may not yet be compelled to make any disclosure since the proposed terms and conditions of the agreements have not become efective and binding. Issue: WON the right to information includes access to the terms of government negotiations prior to their consummation or conclusion.

TAU MU TAU MU TAU MU TAU MU TAU MU TAU MU

Ruling: The petition is imbued with merit. Access to public documents and records is a public right and the real parties in interest are the people themselves. The instant petition is anchored on the right of the people to information and access to official records, documents and papers – a right guaranteed under Section 7, Article 3 of the 1987 Constitution. Because of the satisfaction of the two basis requisites laid down by decisional law to sustain petitioner’s legal standing, i.e. (1) the enforcement of a public right, (2) espoused by a Filipino citizen, the Court rules that the petition at bar, should be allowed. There are no specific laws prescribing the exact limitations within which the right may be exercised or the correlative state duty may be obliged. The following are some of the recognized restrictions: (1) national security matters and intelligence information, (2) trade secrets and banking transactions, (3) criminal matters, and (4) other confidential information. It is for the courts to determine on a case by case basis whether the matter at issue is of interest or importance, as it relates to or afects the public. The recovery of the Marcoses’ alleged ill-gotten wealth is a matter of public concern and imbued with public interest. It is incumbent upon the PCGG and its officers, as well as other government representatives to disclose sufficient public information on any proposed settlement they have decided to take up with the ostensible owners and holders of ill-gotten wealth. Guiani, Darlene Magabilen

CHAVEZ vs. PEA 384 SCRA 152

KITY

The Fraternal Ateneo de Davao

TAU MU TAU MU TAU MU TAU MU TAU MU TAU MU

CONSTITUTIONAL LAW II Order of Saint Thomas More Atty. Philip John Pojas/Atty. Rovyne G. Jumao-as, RN University College of Law

Issue: WON the constitutional right to information includes official information on on-going negotiations before a final agreement.

TAU MU TAU MU

Ruling: Before the consummation of the contract, PEA must, on its own and without demand from anyone, Facts: The government, through the Commissioner of disclose to the public matters relating to the disposition Public Highways, signed a contract with the of its property. These include the size, location, Construction and Development Corporation of the technical description and nature of the property being Philippines to reclaim certain foreshore and ofshore disposed of, the terms and conditions of the areas of Manila Bay. The contract also included the disposition, the parties qualified to bid, the minimum construction of Phases I and II of the Manila-Cavite price and similar information. PEA must prepare all Coastal Road. CDCP obligated itself to carry out all the these data and disclose them to the public at the start works in consideration of fifty percent of the total of the disposition process, long before the reclaimed land. In 1977, then President Marcos issued consummation of the contract, because the Presidential Decree No. 1084 creating PEA. PD No. Government Auditing Code requires public bidding. If 1084 tasked PEA “to reclaim land, including foreshore PEA fails to make this disclosure, any citizen can and submerged areas,” and “to develop, improve, demand from PEA this information at any time during acquire, x x x lease and sell any and all kinds of lands.” the bidding process. On the same date, then President Marcos issued Information, however, on on-going evaluation Presidential Decree No. 1085 transferring to PEA the or review of bids or proposals being undertaken by the “lands reclaimed in the foreshore and ofshore of the bidding or review committee is not immediately Manila Bay” under the Manila-Cavite Coastal Road and accessible under the right to information. While the Reclamation Project. evaluation or review is still on-going, there are no In 1981, then President Marcos issued a “official acts, transactions, or decisions” on the bids or memorandum directing PEA to amend its contract with proposals. However, once the committee makes its CDCP. In 1988, President Aquino issued Special Patent official recommendation, there arises a “definite No. 3517, granting and transferring to PEA “the parcels proposition” on the part of the government. From this of land so reclaimed under the Manila-Cavite Coastal moment, the public’s right to information attaches, and Road and Reclamation Project containing a total area of any citizen can access all the non-proprietary 1,915,894 square meters.” Subsequently, the Register information leading to such definite proposition. of Deeds of the Municipality of Parañaque issued A consummated contract is not a requirement Transfer Certificates of Title in the name of PEA, for the exercise of the right to information. Otherwise, covering the three reclaimed islands known as the the people can never exercise the right if no contract is “Freedom Islands.” In 1995, PEA entered into a Joint consummated, and if one is consummated, it may be Venture Agreement with AMARI, a private corporation, too late for the public to expose its defects. Requiring a to develop the Freedom Islands. The JVA also required consummated contract will keep the public in the dark the reclamation of an additional 250 hectares of until the contract, which may be grossly submerged areas surrounding these islands to disadvantageous to the government or even illegal, complete the configuration in the Master Development becomes a fait accompli. This negates the State Plan of the Southern Reclamation Project-MCCRRP. PEA policy of full transparency on matters of public and AMARI entered into the JVA through negotiation concern, a situation which the framers of the without public bidding. The Board of Directors of PEA, Constitution could not have intended. Such a in its Resolution No. 1245, confirmed the JVA. requirement will prevent the citizenry from Thereafter, JVA was approved. participating in the public discussion of any proposed In 1996, then Senate President Maceda contract, efectively truncating a basic right enshrined delivered a privilege speech in the Senate and in the Bill of Rights. The Court can allow neither an denounced the JVA as the “grandmother of all scams.” emasculation of a constitutional right, nor a retreat by The Senate Committees reported the results of their the State of its avowed “policy of full disclosure of all investigation in Senate Committee Report No. 560 and its transactions involving public interest.” among the conclusions of their report are: (1) the The information that petitioner may access on reclaimed lands PEA seeks to transfer to AMARI under the renegotiation of the JVA includes evaluation the JVA are lands of the public domain which the reports, recommendations, legal and expert opinions, government has not classified as alienable lands and minutes of meetings, terms of reference and other therefore PEA cannot alienate these lands; (2) the documents attached to such reports or minutes, all certificates of title covering the Freedom Islands are relating to the JVA. However, the right to information thus void, and (3) the JVA itself is illegal. does not compel PEA to prepare lists, abstracts, Hence, petitioner, as a taxpayer, filed the summaries and the like relating to the renegotiation of instant Petition for Mandamus with Prayer for the the JVA. The right only afords access to records, Issuance of a Writ of Preliminary Injunction and documents and papers, which means the opportunity Temporary Restraining Order. Petitioner contends the to inspect and copy them. One who exercises the right government stands to lose billions of pesos in the sale must copy the records, documents and papers at his by PEA of the reclaimed lands to AMARI. Petitioner expense. The exercise of the right is also subject to prays that PEA publicly disclose the terms of any reasonable regulations to protect the integrity of the renegotiation of the JVA, invoking Section 28, Article II, public records and to minimize disruption to and Section 7, Article III, of the 1987 Constitution on government operations, like rules specifying when and the right of the people to information on matters of how to conduct the inspection and copying. public concern. Petitioner assails the sale to AMARI of There is no claim by PEA that the information lands of the public domain as a blatant violation of demanded by petitioner is privileged information Section 3, Article XII of the 1987 Constitution rooted in the separation of powers. The information prohibiting the sale of alienable lands of the public does not cover Presidential conversations, domain to private corporations. Finally, petitioner correspondences, or discussions during closed-door asserts that he seeks to enjoin the loss of billions of Cabinet meetings which, like internal deliberations of pesos in properties of the State that are of public the Supreme Court and other collegiate courts, or dominion. executive sessions of either house of Congress, are recognized as confidential. This kind of information ACADCOM 2010; Contributors: Gene Geocaniga, Jarissa Guiani, Darlene Magabilen TAU MU Page 88 of 179

TAU MU TAU MU

TAU MU

TAU MU

TAU MU

TAU MU

TAU MU

TAU MU

TAU MU

TAU MU

TAU MU

Note: The right covers three categories of information which are “matters of public concern,” namely: (1) official records; (2) documents and papers pertaining to official acts, transactions and decisions; and (3) government research data used in formulating policies. The first category refers to any document that is part of the public records in the custody of government agencies or officials. The second category refers to documents and papers recording, evidencing, establishing, confirming, supporting, justifying or explaining official acts, transactions or decisions of government agencies or officials. The third category refers to research data, whether raw, collated or processed, owned by the government and used in formulating government policies. The State policy of full transparency in all transactions involving public interest reinforces the people’s right to information on matters of public concern. This State policy is expressed in Section 28, Article II of the Constitution, thus: Subject to reasonable conditions prescribed by law, the State adopts and implements a policy of full public disclosure of all its transactions involving public interest.” These twin provisions of the Constitution seek to promote transparency in policy-making and in the operations of the government, as well as provide the people sufficient information to exercise efectively other constitutional rights. These twin provisions are essential to the exercise of freedom of expression. If the government does not disclose its official acts, transactions and decisions to citizens, whatever citizens say, even if expressed without any restraint, will be speculative and amount to nothing. These twin provisions are also essential to hold public officials “at all times accountable to the people,” for unless citizens have the proper information, they cannot hold public officials accountable for anything. Armed with the right information, citizens can participate in public discussions leading to the formulation of government policies and their efective implementation. An informed citizenry is essential to the existence and proper functioning of any democracy.

KITY

The Fraternal Ateneo de Davao

TAU MU TAU MU TAU MU TAU MU TAU MU TAU MU

CONSTITUTIONAL LAW II Order of Saint Thomas More Atty. Philip John Pojas/Atty. Rovyne G. Jumao-as, RN University College of Law cannot be pried open by a co-equal branch of government. Hence, the SC ruled that the constitutional right to information includes official information on ongoing negotiations before a final contract. The information, however, must constitute definite propositions by the government and should not cover recognized exceptions like privileged information, military and diplomatic secrets and similar matters afecting national security and public order.

Executive Order No. 213 (Creation of a Philippine Coordinating Committee to study the feasibility of the Japan-Philippines Economic Partnership Agreement) to study and negotiate the proposed JPEPA, and to furnish the Committee with a copy of the latest draft of the JPEPA. Usec. Aquino did not heed the request, however. Congressman Aguja later requested for the same document, but Usec. Aquino replied that the Congressman shall be provided with a copy thereof “once the negotiations are completed and as soon as a thorough legal review of the proposed agreement has been conducted. Issue: WON the right to information includes an ongoing negotiation before a final agreement.

TAU MU TAU MU

Ruling: Public disclosure of the text of the JPEPA after its signing by the President, during the pendency of the present petition, has been largely rendered moot and academic. With the Senate deliberations on the JPEPA still pending, the agreement as it now stands cannot yet be considered as final and binding between the two States. To be covered by the right to information, the information sought must meet the threshold requirement that it be a matter of public concern. In determining whether or not a particular information is of public concern there is no rigid test which can be applied. ‘Public concern’ like ‘public interest’ is a term that eludes exact definition. Both terms embrace a broad spectrum of subjects which the public may want to know, either because these directly afect their lives, or simply because such matters naturally arouse the interest of an ordinary citizen. In the final analysis, it is for the courts to determine on a case by case basis whether the matter at issue is of interest or importance, as it relates to or afects the public. From the nature of the JPEPA as an international trade agreement, it is evident that the Philippine and Japanese ofers submitted during the negotiations towards its execution are matters of public concern. This, respondents do not dispute. They only claim that diplomatic negotiations are covered by the doctrine of executive privilege, thus constituting an exception to the right to information and the policy of full public disclosure. It is well-established in jurisprudence that neither the right to information nor the policy of full public disclosure is absolute, there being matters which, albeit of public concern or public interest, are recognized as privileged in nature. Whether a claim of executive privilege is valid depends on the ground invoked to justify it and the context in which it is made. In the present case, the ground for respondents’ claim AKBAYAN vs. AQUINO of privilege is that the categories of information that 558 SCRA 468 may be considered privileged includes matters of diplomatic character and under negotiation and Facts: Petitioners - non-government organizations, review. In this case, the privileged character of the Congresspersons, citizens and taxpayers – seek via diplomatic negotiations has been categorically invoked petition for mandamus and prohibition to obtain from and clearly explained by respondents particularly respondents the full text of the Japan-Philippines respondent DTI Senior Undersecretary. The ground Economic Partnership Agreement (JPEPA) including the relied upon by respondents is thus not simply that the Philippine and Japanese ofers submitted during the information sought involves a diplomatic matter, but negotiation process and all pertinent attachments and that it pertains to diplomatic negotiations then in annexes thereto. progress. Petitioners Congressmen. Tañada and Aguja It is clear that while the final text of the JPEPA filed House Resolution No. 551 calling for an inquiry may not be kept perpetually confidential – since there into the bilateral trade agreements then being should be “ample opportunity for discussion before a negotiated by the Philippine government, particularly treaty is approved” – the ofers exchanged by the the JPEPA. The Resolution became the basis of an parties during the negotiations continue to be inquiry subsequently conducted by the House Special privileged even after the JPEPA is published. It is Committee on Globalization (the House Committee) reasonable to conclude that the Japanese into the negotiations of the JPEPA. representatives submitted their ofers with the In the course of its inquiry, the House understanding that “historic confidentiality” would Committee requested Usec. Aquino, Chairman of the govern the same. Disclosing these ofers could impair Philippine Coordinating Committee created under the ability of the Philippines to deal not only with Japan ACADCOM 2010; Contributors: Gene Geocaniga, Jarissa Guiani, Darlene Magabilen TAU MU Page 89 of 179

TAU MU TAU MU

TAU MU

TAU MU

TAU MU

TAU MU

TAU MU

TAU MU

TAU MU

TAU MU

TAU MU

TAU MU TAU MU TAU MU

Facts: Petitioner Gonzales, in his capacity as a citizen and taxpayer, prays for an order compelling respondent Zamora to furnish with information for the names of executive officials holding multiple positions in the government, copied of their appointments and a list of the recipients of luxury vehicles seized by Bureau of Customs and turned over to Malacanang. Issue: WON the petitioner has the right to information.

TAU MU TAU MU TAU MU TAU MU TAU MU

Ruling: Section 7 of the Bill of Rights is a selfexecutory provision which can be invoked by any citizen before the courts. The right to information as a public right and “when a mandamus proceeding involves the assertion of a public right, the requirement of personal interest is satisfied by the mere fact that the petitioner is a citizen and therefore, part of the general ‘public’ which possesses the right.” However, Congress may provide for reasonable conditions upon the access to information. Such limitations were embodied in RA 6713, otherwise known as the “Code of Conduct and Ethical Standards for Public Officials and Employees.” This law provides that in the performance of their duties, all public officials and employees are obliged to respond to letters sent by the public within 15 working days from receipt thereof and to ensure the accessibility of all public documents for inspection by the public within reasonable working hours, subject to the reasonable claims of confidentiality. The incorporation of this right in the Constitution is a recognition of the fundamental role of the free exchange of information in a democracy. The information to which the public is entitled to those concerning “matters of public concern,” a term which embraces a broad spectrum of subjects which the public may want to know, either because these directly afect their lives, or simply because such matters naturally arouse the interest of an ordinary citizen. In the final analysis, it is for the courts to determine in a case to case basis whether the matter at issue is of interest or importance, as it relates to or afects the public. Respondent Zamora, in his official capacity as executive secretary, has a constitutional and statutory duty to answer petitioner’s letter dealing with matters which are unquestionably of public concern. With regard to petitioner’s request for copies of the appointment papers of certain officials, respondent Zamora is obliged to allow the inspection and copying of the same subject to the reasonable limitations required for the orderly conduct of official business.

The Fraternal Ateneo de Davao

TAU MU TAU MU

GONZALES vs. NARVASA, et. al 337 SCRA 733 (2000)

KITY

TAU MU TAU MU TAU MU TAU MU TAU MU TAU MU

CONSTITUTIONAL LAW II Order of Saint Thomas More Atty. Philip John Pojas/Atty. Rovyne G. Jumao-as, RN University College of Law but with other foreign governments in future negotiations. Diplomatic negotiations, therefore, are recognized as privileged in this jurisdiction, the JPEPA negotiations constituting no exception. It bears emphasis, however, that such privilege is only presumptive. Recognizing a type of information as privileged does not mean that it will be considered privileged in all instances. Only after a consideration of the context in which the claim is made may it be determined if there is a public interest that calls for the disclosure of the desired information, strong enough to overcome its traditionally privileged status. Hence, the SC held that petitioners’ demand to be furnished with a copy of the full text of the JPEPA has become moot and academic, it having been made accessible to the public since September 11, 2006. As for their demand for copies of the Philippine and Japanese ofers submitted during the JPEPA negotiations, the same must be denied, respondents’ claim of executive privilege being valid.

SENATE vs. ERMITA 488 SCRA 1 Facts: Senate President Drilon received from Executive Secretary Ermita a letter “respectfully requesting for the postponement of the hearing regarding the NorthRail project to which various officials of the Executive Department have been invited” in order to “aford said officials ample time and opportunity to study and prepare for the various issues so that they may better enlighten the Senate Committee on its investigation.” Senate President Drilon, however, wrote Executive Secretary Ermita that the Senators “are unable to accede to his request” as it “was sent belatedly” and “all preparations and arrangements as well as notices to all resource persons were completed the previous week.” Senate President Drilon likewise received a letter from the President of the North Luzon Railways Corporation Cortes, Jr. requesting that the hearing on the NorthRail project be postponed or cancelled until a copy of the report of the UP Law Center on the contract agreements relative to the project had been secured. In 2005, the President issued E.O. 464, “ENSURING OBSERVANCE OF THE PRINCIPLE OF SEPARATION OF POWERS, ADHERENCE TO THE RULE ON EXECUTIVE PRIVILEGE AND RESPECT FOR THE RIGHTS OF PUBLIC OFFICIALS APPEARING IN LEGISLATIVE INQUIRIES IN AID OF LEGISLATION UNDER THE CONSTITUTION, AND FOR OTHER PURPOSES.” Senate President Drilon received from Executive Secretary Ermita a copy of E.O. 464, and another letter informing him “that officials of the Executive Department invited to appear at the meeting regarding the NorthRail project will not be able to attend the same without the consent of the President, pursuant to E.O. 464” and that “said officials have not secured the required consent from the President.” On even date which was also the scheduled date of the hearing on the alleged wiretapping, Gen. Senga sent a letter to Senator Biazon, Chairperson of the Committee on National Defense and Security, informing him “that per instruction of President Arroyo, thru the Secretary of National Defense, no officer of the AFP is authorized to appear before any Senate or Congressional hearings without seeking a written approval from the President” and “that no approval has been granted by the President to any AFP officer to appear before the public hearing of the Senate Committee on National Defense and Security.Despite the communications received from Executive Secretary Ermita and Gen. Senga, the investigation scheduled by the Committee on National Defense and Security pushed through, with only Col. Balutan and Brig. Gen. Gudani among all the AFP officials invited attending. Issue: WON the withholding of the information violated petitioner’s right to information.

TAU MU

Ruling: E.O 464 is concerned only with the demands of Congress for the appearance of executive officials in the hearings conducted by it, and not with the demands of citizens for information pursuant to their right to information on matters of public concern. Petitioners are not amiss in claiming, however, that what is involved in the present controversy is not merely the legislative power of inquiry, but the right of the people to information. There are clear distinctions between the right of Congress to information which underlies the power of inquiry and the right of the people to information on matters of public concern. For one, the demand of a citizen for the production of documents pursuant to his right to information does not have the same obligatory force as a subpoena duces tecum issued by Congress. Neither does the right to information grant a citizen the ACADCOM 2010; Contributors: Gene Geocaniga, Jarissa Guiani, Darlene Magabilen TAU MU Page 90 of 179

TAU MU

TAU MU

TAU MU TAU MU TAU MU TAU MU TAU MU TAU MU

Facts: Petitioners mortgaged their property to the respondent GSIS to secure a housing loan. As petitioners failed to pay their loan when it fell due, GSIS foreclosed the mortgage. Petitioner wrote GSIS Board of Trustees to inform them of their desire to redeem the subject property and for advice of the procedure of redemption. GSIS respondent advised them to pay the total redemption price of Php154, 896 on or before the expiry date of redemption. Unable to find financing to repurchase the subject property, petitioner requested for re-mortgage through repurchase of the subject property. Respondent dela Cruz commenced negotiations with respondent GSIS for her purchase of the subject property. A deed of absolute sale over the subject property was executed between GSIS and respondent dela Cruz. Having learned about the sale of the subject property, petitioner wrote to the GSIS protesting the said sale and requesting its consideration and recall. Petitioners filed the instant case before the RTC of QC which was dismissed and affirmed by the CA.

The Fraternal Ateneo de Davao

TAU MU TAU MU

VDA. DE URBANO, et. al. vs. GSIS, et. al 367 SCRA 672 (2001)

KITY

TAU MU TAU MU TAU MU TAU MU TAU MU TAU MU

CONSTITUTIONAL LAW II Order of Saint Thomas More Atty. Philip John Pojas/Atty. Rovyne G. Jumao-as, RN University College of Law power to exact testimony from government officials. These powers belong only to Congress and not to an individual citizen. Thus, while Congress is composed of representatives elected by the people, it does not follow, except in a highly qualified sense, that in every exercise of its power of inquiry, the people are exercising their right to information. To the extent that investigations in aid of legislation are generally conducted in public, however, any executive issuance tending to unduly limit disclosures of information in such investigations necessarily deprives the people of information which, being presumed to be in aid of legislation, is presumed to be a matter of public concern. The citizens are thereby denied access to information which they can use in formulating their own opinions on the matter before Congress — opinions which they can then communicate to their representatives and other government officials through the various legal means allowed by their freedom of expression. Thus holds Valmonte v. Belmonte: It is in the interest of the State that the channels for free political discussion be maintained to the end that the government may perceive and be responsive to the people’s will. Yet, this open dialogue can be efective only to the extent that the citizenry is informed and thus able to formulate its will intelligently. Only when the participants in the discussion are aware of the issues and have access to information relating thereto can such bear fruit. The impairment of the right of the people to information as a consequence of E.O. 464 is, therefore just as direct as its violation of the legislature’s power of inquiry.

involving public interest.” The negotiation and subsequent sale of the subject property by the GSIS to dela Cruz was by no stretch of the imagination imbued with public interest as it was a purely private transaction. HILADO vs. REYES 496 SCRA 282 Facts: PR Benedicto, the surviving spouse of the deceased Roberto S. Benedicto, filed a petition for issuance of letters of administration, “Intestate Estate of Roberto S. Benedicto.” The case was raffled to Branch 21 presided by Public Respondent Judge Reyes. Private respondent was appointed Administratrix of the estate of Benedicto and letters of administration were thereafter issued in her favor. Petitioners, Hilado, Lacson, Tuvilla, Limjap, Lopez Sugar Corporation and First Farmers Holding Corporation had, during the lifetime of Benedicto, filed two complaints for damages or collection of sums of money against Roberto Benedicto et al. The Branch Clerk of Court of Branch 21 of the Manila RTC allowed petitioners through counsel Sedigo and Associates to regularly and periodically examine the records of the case and to secure certified true copies thereof. However, Atty. Paredes, an associate of petitioners’ counsel, was denied access to the last folder-record of the case which, according to the court’s clerical staf, could not be located and was probably inside the chambers of public respondent for safekeeping. Petitioners’ counsel thus requested public respondent to allow Atty. Paredes to personally check the records of the case. The Officer-In- Charge/Legal Researcher of Branch 21 advised petitioners’ counsel in writing that “per instruction of the Hon. Presiding Judge, only parties or those with authority from the parties are allowed to inquire or verify the status of the case pending in this Court,” and that they may be “allowed to go over the records of the case upon presentation of written authority from the administratrix.” Hence, petitioners filed petition for mandamus and prohibition to compel public respondent to allow them to access, examine, and obtain copies of any and all documents forming part of the records of the case and disqualify public respondent from further presiding Issue: WON petitioner’s right to information was violated.

TAU MU

Ruling: Insofar as the right to information relates to judicial records, an understanding of the term “judicial record” or “court record” is in order. The term “judicial record” or “court record” does not only refer to the orders, judgment or verdict of the courts. It comprises the official collection of all papers, exhibits and pleadings filed by the parties, all processes issued and returns made thereon, appearances, and word-for-word testimony which took place during the trial and which Issue: WON petitioners were deprived of their right to are in the possession, custody, or control of the information. judiciary or of the courts for purposes of rendering court decisions. It has also been described to include Ruling: The petitioner is devoid of merit. GSIS sold the any paper, letter, map, book, other document, tape, subject property to dela Cruz only after giving photograph, film, audio or video recording, court petitioners an almost one year opportunity to reporter’s notes, transcript, data compilation, or other repurchase the property and only after ascertaining materials, whether in physical or electronic form, made that the purchase price proposed by dela Cruz in or received pursuant to law or in connection with the payment of the subject property would benefit the transaction of any official business by the court, and GSIS. Nor can petitioners, on the strength of Valmonte includes all evidence it has received in a case. vs. Belmonte, impute bad faith on the part of GSIS In determining whether particular information when the latter did not disclose to petitioners that it is of public concern, there is no right test. In the final was negotiating with dela Cruz for the sale of the analysis, it is for the courts to determine on a case to subject property as soon as it started the negotiations. case basis whether the matter at issue is of interest or The constitutional right to information is limited to importance as it relates to or afect the public. It bears “maters of public concern,” and “to transaction emphasis that the interest of the public hinges on its ACADCOM 2010; Contributors: Gene Geocaniga, Jarissa Guiani, Darlene Magabilen TAU MU Page 91 of 179

TAU MU

TAU MU

TAU MU

TAU MU

KITY

The Fraternal Ateneo de Davao

TAU MU TAU MU TAU MU TAU MU TAU MU TAU MU TAU MU TAU MU

CONSTITUTIONAL LAW II Order of Saint Thomas More Atty. Philip John Pojas/Atty. Rovyne G. Jumao-as, RN University College of Law right to transparency in the administration of justice, to the end that it will serve to enhance the basic fairness of the judicial proceedings, safeguard the integrity of the fact-finding process, and foster an informed public discussion of governmental afairs. Decisions and opinions of a court are of course matters of public concern or interest for these are the authorized expositions and interpretations of the laws, binding upon all citizens, of which every citizen is charged with knowledge. Justice thus requires that all should have free access to the opinions of judges and justices, and it would be against sound public policy to prevent, suppress or keep the earliest knowledge of these from the public. Unlike court orders and decisions, however, pleadings and other documents filed by parties to a case need not be matters of public concern or interest. For they are filed for the purpose of establishing the basis upon which the court may issue an order or a judgment afecting their rights and interests. In thus determining which part or all of the records of a case may be accessed to, the purpose for which the parties filed them is to be considered. Information regarding the financial standing of a person at the time of his death and the manner by which his private estate may ultimately be settled is not a matter of general, public concern or one in which a citizen or the public has an interest by which its legal rights or liabilities maybe afected. Granting unrestricted public access and publicity to personal financial information may constitute an unwarranted invasion of privacy to which an individual may have an interest in limiting its disclosure or dissemination. If the information sought then is not a matter of public concern or interest, denial of access thereto does not violate a citizen’s constitutional right to information. Once particular information has been determined to be of public concern, the accessory right of access to official records, including judicial records, are open to the public. The accessory right to access public records may, however, be restricted on a showing of good cause. Thus, access to court records may be permitted at the discretion and subject to the supervisory and protective powers of the court, after considering the actual use or purpose for which the request for access is based and the obvious prejudice to any of the parties. In this case, the SC granted the petition for mandamus filed by Hilado. Public respondent Reyes was ordered to allow petitioners to access, examine and obtain copies of any and all documents-part of the records of the special proceedings.

seeking to disqualify the nominees of certain party-list organizations. Both petitioners appear not to have the names of the nominees sought to be disqualified since they still asked for a copy of the list of nominees. Meanwhile, reacting to the emerging public perception that the individuals behind the aforementioned 14 party-list groups do not actually represent the poor and marginalized sectors, petitioner Rosales addressed a letter to Director Dalaig of the Comelec’s Law Department requesting a list of that groups nominees. However, neither the Comelec Proper nor its Law Department responded to petitioner Rosales’ requests. Issue: WON there was a violation of the constitutional right to information when COMELEC refused to disclose the names of the party-lists.

TAU MU TAU MU

Ruling: Any citizen can challenge any attempt to obstruct the exercise of his right to information and may seek its enforcement by mandamus. And since every citizen by the simple fact of his citizenship possesses the right to be informed, objections on ground of locus standi are ordinarily unavailing. However, the right to information and its companion right of access to official records are not absolute. The people’s right to know is limited to “matters of public concern” and is further subject to such limitation as may be provided by law. Similarly, the policy of full disclosure is confined to transactions involving “public interest” and is subject to reasonable conditions prescribed by law. There is also the need of preserving a measure of confidentiality on some matters, such as military, trade, banking and diplomatic secrets or those afecting national security. In the case at bar, no national security or like concerns is involved in the disclosure of the names of the nominees of the party-list groups in question. Doubtless, the Comelec committed grave abuse of discretion in refusing the legitimate demands of the petitioners for a list of the nominees of the party-list groups subject of their respective petitions. Mandamus, therefore, lies. The last sentence of Section 7 of RA 7941 reading: “The names of the party-list nominees shall not be shown on the certified list” is certainly not a justifying card for the Comelec to deny the requested disclosure. The prohibition imposed on the Comelec under said Section 7 is limited in scope and duration, meaning, that it extends only to the certified list which the same provision requires to be posted in the polling places on election day. To stretch the coverage of the prohibition to the absolute is to read into the law something that is not intended. As it were, BANTAY REPUBLIC ACT 7941 vs. COMELEC there is absolutely nothing in RA 7941 that prohibits 523 SCRA 1 (2007) the Comelec from disclosing or even publishing Facts: The case filed before the SC are two through mediums other than the “Certified List” the consolidated petitions for certiorari and mandamus to names of the party-list nominees. nullify and set aside certain issuances of the COMELEC The Comelec obviously misread the limited respecting party list groups which have manifested non-disclosure aspect of the provision as an absolute their intention to participate in the party-list elections bar to public disclosure before the May 2007 elections. in May 2007. Both petitions seek to compel COMELEC The interpretation thus given by the Comelec virtually to disclose or publish the names of the nominees of the tacks an unconstitutional dimension on the last various party list groups. sentence of Section 7 of RA7941. The Comelec’s In January 2007, the COMELEC issued a reasoning that a party-list election is not an election of resolution prescribing rules and regulations to govern personalities is valid to a point. It cannot be taken, the filing of manifestation of intent to participate and however, to justify its assailed non-disclosure stance submission of names of nominees under the party-list which comes, as it were, with a weighty presumption of system of representation in connection with the May invalidity, impinging, as it does, on a fundamental right 2007 elections. Pursuant thereto, a number of to information. While the vote cast in a party-list organized groups filed the necessary manifestations. elections is a vote for a party, such vote, in the end, Among these – and ostensibly subsequently accredited would be a vote for its nominees, who, in appropriate by the Comelec to participate in the 2007 elections cases, would eventually sit in the House of are 14 party-list groups. Petitioners BA-RA 7941 and Representatives. UP-LR presented a longer, albeit an overlapping, list. Thus, the SC orderd the Comelec to Subsequent events saw BA-RA 7941 and UP-LR immediately disclose and release the names of the filing with the Comelec an Urgent Petition to Disqualify, nominees of the party-list groups, sectors or ACADCOM 2010; Contributors: Gene Geocaniga, Jarissa Guiani, Darlene Magabilen TAU MU Page 92 of 179

TAU MU

TAU MU

TAU MU

TAU MU

TAU MU

TAU MU

TAU MU

TAU MU

TAU MU

AQUINO vs. MORATO 203 SCRA 515 (1991) Facts: The petitioner, a member of the MTRCB, requested that she be allowed to examine the voting slips of other members. It is on the basis of these slips that the films are banned, classified or cut by the Board. Petitioner’s request was refused on the ground that the votes, which are purely personal, are private and confidential.

VALMONTE vs. BELMONTE 170 SCRA 256 (1989)

TAU MU TAU MU TAU MU TAU MU

Section 8. The right of the people, including those employed in the public and private sectors, to form unions, associations, or societies for purposes not contrary to law, shall not be abridged.

TAU MU

Ruling 2: The Court held that there is no law which grants GSIS’s confidentiality. Also, the right to confidentiality involves privacy which cannot be invoked by GSIS because matters about privacy are not available to an artificial person (such as corporations or government entities). The right of confidentiality pertains to borrowers. Privacy is granted only to natural beings.

TAU MU

Issue 2: Is the information excluded by law on the ground of confidentiality?

TAU MU

Ruling 1: Only information which are matters of public concern are covered by Section 7. The OCUrt held that the information sought is a matter of public concern. Since the GSIS is the custodian of public funds, it shall be the interest of the citizen which will determines as to how the money must be disturbed (NATURE OF THE FUNDS). The Court defines a MATTER OF PUBLIC CONCERN as any subject of which the public may want to know because it directly afects their lives or they arouse the interest of a citizen. And the Court also stated that the members of the Batasan are public figures, and this contributes to the idea that this is a matter of public concern (CHARACTER OF THE BORROWER).

TAU MU

Issue 1: Is the information sought a matter of public concern?

TAU MU TAU MU

Facts: Special civil action for mandamus with preliminary injunction. Valmonte wrote to Belmonte, GSIS General Manager, requesting the list of names of the opposition members of the Batasang Pambansa who were able to secure a clean loan on guaranty of Mrs. Imelda Marcos, certified true copies of the documents evidencing their loan or access to such documents. It was denied on the ground that a confidential relationship exists between GSIS and all those who borrow from it, whoever they may be.

The Fraternal Ateneo de Davao

TAU MU TAU MU

Ruling: The Court held that they are not private because the members of the Board occupy a public position. These are actually public records and are therefore not private or personal.

KITY

TAU MU TAU MU TAU MU TAU MU TAU MU TAU MU

CONSTITUTIONAL LAW II Order of Saint Thomas More Atty. Philip John Pojas/Atty. Rovyne G. Jumao-as, RN University College of Law organizations accredited to participate in the May 2007 party-list elections.

freedom of speech and expression, subject to the same limitation. Even without this constitutional provision, the right of association is part and parcel of the freedom of expression and assembly. It is also an aspect of liberty, which is covered by the freedom clause. It is also an aspect of contracts, which is also related to freedom of religion because normally people form associations to advance a specific belief. Limitations: 1. Imposed by law 2. Dominant police power of the State Rights included/ 2 aspects: 1. The positive aspect is the right to join or be a member of an association. 2. The negative aspect is the right not to be compelled to be a member of any association. Exception to Negative Aspect: When there is a closed shop agreement in the CBA. An employee can be compelled to be member of the union. Because a closed shop agreement is a valid exercise of the police power, and the state has an interest in promoting unionism to protect labor. So, in this case, a person can be compelled to join an association. Exception to the exception: In the case of Victoriano vs. Elizalde: a person cannot be compelled to be a member of a union, even under a legal closed shop agreement if his religion prohibits him to join associations because the freedom of religion is superior to the freedom of association. Note: But in Hernandez, mere membership in Huk Balahap is punishable. Tests used in limiting the right of association: 1. Test for valid exercise of police power - the right to association is limited when the state is invoking or exercising this police power. (Remember the tests for valid exercise of police power). This test is used when the purpose of association is to promote property rights. 2. Clear and Present Danger Test - used when the purpose of association is to promote advance basic human rights or political ideas. So if you're joining a political group, this test is used.  Under section 8, it says that even those employed in the public sector can join a strike. But there are limitations because the right of public employees are limited to the right to form associations. Actually, they have a right to form unions, but they do not have the constitutional right to strike. Private employees have the constitutional right to strike, but not the public employees.  PAFLU case- The purpose of the registration was for the acquisition of the legal personality. Here, nobody was being prevented from joining a union. The union exists. But without the registration, the union cannot enter into CBA, etc. Its just the activities that they can't enter into if they don't enter, but there is no violation of the right of association.  The right to form an association is superior to the freedom of assembly, because in association the people not only assemble but start to form an organization. And to form an association already involves a purpose, assembly is just assemble, get together

TAU MU

The right to form associations shall not be impaired without due process of law and is thus an aspect of the right of liberty. It is also an aspect of the freedom of contract. In addition, insofar as the associations may have for their object the advancement of beliefs and ideas, the freedom of association is an aspect of the ACADCOM 2010; Contributors: Gene Geocaniga, Jarissa Guiani, Darlene Magabilen TAU MU Page 93 of 179

 The right to form association DOES NOT INCLUDE the following: a. automatic acquisition of legal personality; b. possession of rights and privileges granted by law to legitimate labor organizations/associations; c. automatic registration of the association with any legal entity.

TAU MU TAU MU TAU MU TAU MU

Q: Can this right be exercised by the unemployed? Where is the seat of this right? A: YES. The right is recognized as belonging to the PEOPLE whether employed or not, and whether employed in government or the private sector.

TAU MU

Strictly speaking, this right is already comprehended in due process, particularly as it protects the person’s liberty. It was nevertheless deemed advisable to provide for this separate section by way of emphasis upon this specific right.  This right is also deemed embraced in the freedom of expression because the organization can be used as a vehicle for the expression of views that have a bearing on the public welfare. These views would be more efectively disseminated and enjoy a more respectful audience if articulated through and organization to w/c a person belongs than if he were to ventilate them as a mere individual.  The instinct to organize is a very basic human drive.  The provision guarantees the right to form association. IT does not include the right to compel others to form an association. BUT there may be situations w/c, by entering into a contract, one may also be agreeing to join an association [BEL-AIR VILLAGE ASSOCIATION vs. DIONISIO (174 SCRA 589)].

TAU MU TAU MU



The Fraternal Ateneo de Davao

TAU MU TAU MU

Q: Can a lawyer’s name be cancelled in the roll of attorney’s if he fails to pay IBP dues, or stated otherwise, can a lawyer be forced to join the IBP? A: Yes. (See: In Re: Edillion) Membership in IBP is not a violation of a person’s freedom of association – Bar association does not force lawyers to associate with anyone. It is the lawyers choice to attend or not to attend the meeting. The only compulsion is the payment of fees which is a valid exercise of police power.

KITY

TAU MU TAU MU TAU MU TAU MU TAU MU TAU MU

CONSTITUTIONAL LAW II Order of Saint Thomas More Atty. Philip John Pojas/Atty. Rovyne G. Jumao-as, RN University College of Law Q: Is membership in the communist party punishable as a criminal ofense? A: Mere membership and nothing more merely implies advocacy of abstract theory or principle, remember, the freedom to believe is absolute. It becomes a criminal ofense only if it is coupled with action or advocacy of action.

 To compel a lawyer to become a member of the

integrated bar is not violative of his freedom to associate [IN RE: EDILLON (127 SCRA 404)].  The right to form association does not necessarily include the right to be given legal personality --- must register!  REGISTRATION is merely a condition sine qua non for the acquisition of LEGAL PERSONALITY and the possession of the rights and privileges granted by law. CASES: RIGHT TO ASSOCIATE STA. CLARA HOMEOWNERS ASSOCIATION vs. SPOUSES GASTON 374 SCRA 399 (2002) Facts: Sps. Gaston purchased their lots at Sta. Clara Subdivision and at the same time purchase; there was no mention or requirement of membership in any homeowner’s association. Only member in good standing were issued stickers for their vehicles. Respondent’s son was prevented entry and demanded from him his driver’s license for identification. Also, the SCHA by-laws contain a provision that all real estate owners in Sta. Clara Subdivision automatically become members of the association. Spouses Gaston alleged that they had never joined the association. Issue: WON Sps. Gaston are SCHA member. Ruling: The constitutionally guaranteed freedom of association includes the freedom not to associate. The right to choose with whom one will associate oneself is the very foundation and essence of that partnership. It does not include the right to compel others to form or join one. Sps. Gaston cannot be compelled to become members of the SCHA by including them in the Article of Incorporation and by-laws without their express or implied consent. it may be to the mutual advantage of lot owners to band themselves together to promote their common welfare. But that is impossible only if the owner’s voluntarily agree, directly or indirectly, to become members of the association. Such membership may be acquired in various ways like deed of sale, torrens certificate or other forms of evidence of property ownership. Other that the Article of Incorporation and by-laws, there is no showing that Sps. Gaston have agreed to be SCHA members. PADCOM CONDOMINIUM CORPORATION vs. ORTIGAS CENTER ASSOCIATION 382 SCRA 222

TAU MU

Facts: PADCOM owns and manages the Padilla Office Condominium Building (PADCOM Building). The land on which the building stands was originally acquired from the Ortigas & Company, Limited Partnership (OCLP), by  “...for purposes not contrary to law” is a built-in Tierra Development Corporation (TDC) under a Deed of limitation of the right. All rights are subject to the Sale. Among the terms and conditions in the deed of inherent police power. sale was the requirement that the transferee and its  The right to associate includes the right NOT to successor-in-interest must become members of an associate. This right is superior to the right to assemble association for realty owners and long-term lessees in BUT must yield to the right to freedom of religion [INK the area later known as the Ortigas Center. case]. Subsequently, the said lot, together with improvements  The right does not include the right to strike. thereon, was conveyed by TDC in favor of PADCOM in a Q: Can civil servants unionize? Deed of Transfer. A: YES under Art. IX B Sec. 2 (5) “The right to selfIn 1982, respondent Ortigas was organized to organization shall not be denied to gov’t. employees” advance the interests and promote the general welfare --- whether performing proprietary or gov’t. function. of the real estate owners and long-term lessees of lots in the Ortigas Center. It sought the collection of  The closed-shop is a valid form of union security and membership dues from PADCOM. The corporate books a provisions therefor in a CBA is not considered a showed that PADCOM owed Ortigas which represents restriction of the right of association [VILLAR vs. membership dues, interests and penalty charges from INCIONG (121 SCRA 444)]. April 1983 to June 1993. The letters exchanged between the parties through the years showed repeated demands for payment, requests for ACADCOM 2010; Contributors: Gene Geocaniga, Jarissa Guiani, Darlene Magabilen TAU MU Page 94 of 179

TAU MU

TAU MU

TAU MU

TAU MU TAU MU TAU MU TAU MU TAU MU TAU MU TAU MU TAU MU

Issue: WON this is a restriction to Sec. 8 of the Bill of Rights.

TAU MU

Facts: There was a law passed requiring unions to be registered in order for them to acquire legal personality. Acquiring legal personality is important because if there is none, the union virtually cannot do anything. It cannot bargain with the management, be elected as bargaining representative and it cannot serve as agent to the employees. The efect is they exist as entities but they have no rights or privileges under the law. Their existence will have no relevance to the workers. This law was challenged as violative of the right to association.

TAU MU TAU MU

PAFLU vs. SECRETARY 27 SCRA 41 (1969)

The Fraternal Ateneo de Davao

TAU MU TAU MU

Issue: WON the automatic membership clause is a violation of PADCOM’S freedom of association. Ruling: It is undisputed that when the land in question was bought by PADCOM’s predecessor-in-interest, TDC, from OCLP, the sale bound TDC to comply with paragraph (G) of the covenants, conditions and restrictions of the Deed of Sale, which reads as follows: G. AUTOMATIC MEMBERSHIP WITH THE ASSOCIATION: “The owner of this lot, its successor-in-interest hereby binds himself to become a member of the ASSOCIATION which will be formed by and among purchasers, fully paid up Lot BUYERS, Building Owners and the COMPANY in respect to COMPANY OWNED LOTS. The OWNER of this lot shall abide by such rules and regulations that shall be laid down by the ASSOCIATION in the interest of security, maintenance, beautification and general welfare of the OFFICE BUILDING zone. The ASSOCIATION when organized shall also, among others, provide for and collect assessments which shall constitute a lien on the property, junior only to liens of the Government for taxes.” Evidently, it was agreed by the parties that dues shall be collected from an automatic member and such fees or assessments shall be a lien on the property. Hence, the SC ruled in the negative. PADCOM was never forced to join the association. It could have avoided such membership by not buying the land from TDC. Nobody forced it to buy the land when it bought the building with the annotation of the condition or lien on the Certificate of Title thereof and accepted the Deed. PADCOM voluntarily agreed to be bound by and respect the condition, and thus to join the Association.

KITY

TAU MU TAU MU TAU MU TAU MU TAU MU TAU MU

CONSTITUTIONAL LAW II Order of Saint Thomas More Atty. Philip John Pojas/Atty. Rovyne G. Jumao-as, RN University College of Law extensions of payment, and even a settlement scheme proposed by PADCOM In view of PADCOM’s failure and refusal to pay its arrears in monthly dues, including interests and penalties thereon, Ortigas filed a complaint for collection of sum of money. Ortigas averred that purchasers of lands within the Ortigas Center complex from OCLP are obligated under their contracts of sale to become members of the Association. This obligation was allegedly passed on to PADCOM when it bought the lot from TDC, its predecessor-in-interest. In its answer, PADCOM contended that it is a non-stock, nonprofit association, and for it to become a special member of the Association, it should first apply for and be accepted for membership by the latter’s Board of Directors. No automatic membership was apparently contemplated in the Association’s By-laws. PADCOM added that it could not be compelled to become a member without violating its right to freedom of association. And since it was not a member of the Association, it was not liable for membership dues, interests and penalties.

Ruling: The Court held that the right guaranteed by the Constitution is the right to form associations and the law does not violate this right. The employees can still form associations. However, in order to be able to do something, they must register first to be recognized. The Constitution does not automatically guarantee legal personalities to associations. The reason for the requirement of registration is a valid exercise of police power because the issue of unionism is impressed with public interest. This requirement of registration would protect labor and the public against fraud, abuses and impostors who pose as organizers. BEL AIR vs. DIONISIO 174 SCRA 589 (1989) Facts: Plaintif filed a complaint against the defendant for the collection of the association dues assessed on the lot owned by the defendant as member of the plaintif association. Without applying for membership in plaintif association, defendant in this case, like the other members, automatically became a member because he is the registered owner of a lot located inside the Bel Air Village. The inferior court rendered its decision in favor of the plantif. Issue: WON such membership collides with the constitutional guarantee of freedom of association. Ruling: There is no dispute that the title covering the subject parcel of land issued in the name of the petitioner contains an annotation to the efect that the lot owner becomes an automatic member of the respondent Bel Air Association and must abide by such rules and regulations laid down by the Association. When the petitioner voluntarily bought the subject parcel of land, it was understood that he took the same free of all encumbrances except notations at the back of the certificate of title, among them, that he automatically becomes a member of the respondent association. One of the obligations of a member of the respondent association is to pay certain amounts for the operation and activities of the association which is being collected by the Board of Governors. The dues collected are intended for garbage collection, salary of security guards, cleaning and maintenance of streets and street lights and establishments of parks. The amount to be paid by each lot owner is computed on the basis of the area per square meter of the lot owned by every member. The transaction between the defendants and the original seller (defendant’s immediate predecessor) is a sale and the conditions have been validly imposed by the said vendor and the same not being contrary to law, morals, good customs and public policy. The fact that it has been approved by the Land Registration Commission did not make it a governmental act subject to the constitutional restriction against infringement of the right of association. The Constitutional proscription that no person can be compelled to be a member of an association against his will applies only to government acts and not to private transactions like the one in question. If he does not desire to comply with the annotation or lien in question he can at any time exercise his inviolable freedom of disposing of the property and free himself from the burden of becoming a member of the plaintif association. OCCENA vs. COMELEC 127 SCRA 404 (1984) Facts: Occena challenged as violative of Section 8 of the Bill of Rights a law which prohibits a candidate of the brgy. election from representing himself as a member of the political party and also prohibited the

ACADCOM 2010; Contributors: Gene Geocaniga, Jarissa Guiani, Darlene Magabilen TAU MU Page 95 of 179

Ruling: The Court held that the provision is not absolute. It is subject to the police power of the State. The law is a valid exercise of such power. Note: Whether there is a permissible exercise of police power by the State, the Court did not really aver what should be the test to be applied in limiting this right. In this case, the Court referred to the Gonzales vs. COMELEC case and the implication that there is to apply the clear and present danger rule. The Court held that the right to organize is not violated by the law because the political parties can still organize, but only the activities are restricted.

TAU MU TAU MU TAU MU TAU MU TAU MU TAU MU TAU MU

Ruling: Petition is meritorious. The SC held that the closed-shop agreement was a valid form of union security, and such provision in a CBA is not a restriction of the right of freedom of association guaranteed by the Constitution. It was grave abuse of discretion for respondent court to order the reinstatement and back wages of Blanco who was found to be in an identical position as the 5 other employees whose separation from the service was upheld by the said Court. Blanco could be dismissed on the ground of willful disobedience to rules, orders and instructions of the employer which were reasonable and lawful, known to the employee and pertaining to duties which he discharged. If his name was not included in the list of 18 employees recommended for dismissal, it was because he had been dismissed three days before by the company. And if he had not been dismissed by the company, his dismissal would have been demanded by LEA considering that he was one of those investigated by LEA’s grievance committee which had approved the recommendation to dismiss them on the charge of being members of another union.

TAU MU

Issue: WON the CBA is a restriction of the right of freedom of association.

TAU MU

Facts: Employed in 1959 by LITEX, Blanco joined the then existing labor union, the Litex Employees Association (LEA). In 1960, LEA entered into a collective bargaining agreement with LITEX which was to expire in 1965. The CBA contained a closed shop provision. LEA’s constitution and by-laws also provided for the expulsion of members who are associated with other labor unions. In 1964, Blanco and several Litex members organized the Confederation of Industrial and Allied Labor Organization (CIAOLO). The Court of Industrial Relations (CIR) certified LEA as the sole bargaining representative of the rank and file employees of LITEX. LITEX dismissed 18 of its employees found to have violated the union constitution and by-laws and the CBA for having joined the CIALO. 6 of the dismissed employees and Blanco filed a case for illegal dismissal. The CIR dismissed the case as regards the 6 employees but ordered the reinstatement with back wages of Blanco. Failing to obtain reconsideration of the ruling, LITEX sought review.

TAU MU TAU MU

LIRAG vs. BLANCO and CIR 109 SCRA 87 (1981)

The Fraternal Ateneo de Davao

TAU MU TAU MU

Reason or policy arguments of the Court: It is necessary to insulate the barangay from partisan political activities. The barangay is a basis political structure in our society. Because of this important public interest, the Court considered resctricting the activities of political parties as valid.

KITY

TAU MU TAU MU TAU MU TAU MU TAU MU TAU MU

CONSTITUTIONAL LAW II Order of Saint Thomas More Atty. Philip John Pojas/Atty. Rovyne G. Jumao-as, RN University College of Law political parties from supporting a particular candidate in the elections.

Section 9. Private property shall not be taken for public use without just compensation. Definition of Eminent Domain It is the right of the State to acquire private property for public use upon payment of just compensation. This is based on the Regalian doctrine that all property belongs to the State. This power is inseparable in sovereignty, being essential to the existence of the State and inherent in government, even in its most primitive forms. No laws therefore are ever necessary to confer this right upon the government. It is the ultimate right of sovereign power to appropriate, not only the public, but [even] the private property of all citizens within the territorial sovereignty, to public purposes [Bernas, The 1987 Philippine Constitution: A Comprehensive Reviewer, p. 101]. Eminent domain is described as “the highest and most exact idea of property remaining in the government” that may be acquired for some public purpose through a method “in the nature of a compulsory sale to the State.” Being inherent, the power of eminent domain does not need to be specifically conferred on the government by the Constitution. As it happens, however, it is expressly provided in Article III, Section 9, that “private property shall not be taken for public use without just compensation.” This provision is not a grant but indeed a limitation of the power as its negative and restrictive language clearly suggests [Cruz, Constitutional Law 2007, p.62]. The procedure of taking is called expropriation. This provision, sec. 9, is included in the Bill of Rights is not a grant of powers, but a limitation of the power of eminent domain. Telecommunications vs. COMELEC 289 SCRA 337 (1998) Facts: GMA Network, Inc., is challenging Sec. 52 of BP Blg. 881, which provides: The Commission shall procure radio and television time to be known as “Comelec Time” which shall be allocated equally and impartially among candidates… For this purpose, the franchise of all radio broadcasting and television stations are hereby amended so as to provide radio or television time, free of charge, during the election period. Petitioner claims that this is a form of taking or property without compensation. Held: The argument has no merit. All broadcasting, whether by radio or television stations, is licensed by the government. Airwave frequencies have to be allocated as there are more individuals who want to broadcast than there are frequencies assign. A franchise is thus a privilege subject, among other things, to

ACADCOM 2010; Contributors: Gene Geocaniga, Jarissa Guiani, Darlene Magabilen TAU MU Page 96 of 179

TAU MU TAU MU TAU MU TAU MU

3. The airspace is not a property of the TV stations/networks. It is just a franchise given to

TAU MU

2. PPI vs. COMELEC -- there is unlawful taking here because there was not a need to expropriate. It wasn't shown that PPI was not willing to sell the newspaper space. 2nd, there was no just compensation. 3rd, the Comelec is not one of the entities empowered under the constitution to exercise the power of eminent domain. It was merely a resolution empowering itself which is very arbitrary on the part of COMELEC.

TAU MU

Note: 1. It is only necessary when the owner does not want or opposes the sale of his property. Thus, if a valid contract exists between the government and the owner, the government cannot exercise the power of eminent domain as a substitute to the enforcement of the contract.

TAU MU

1. When property is needed for public use, and the property owner does not want to sell his property. 2. When the owner wants/willing to sell, but there is failure to reach an agreement as to the price.

TAU MU

Situations where the Government is forced to expropriate or to exercise the power of eminent domain:

TAU MU

Who May Exercise the Power Congress, and by delegation, the President, administrative bodies, LGUs, and even private enterprises performing public services [Nachura, Outline Reviewer in Political Law 2006, p. 48].

TAU MU TAU MU

“The exercise of the right of eminent domain, whether directly by the State, or by its authorized agents, is necessarily in derogation of private rights, and the rule in that case is that the authority must be strictly construed.” [Cruz, Constitutional Law 2007, p.63]

The Fraternal Ateneo de Davao

TAU MU TAU MU

amendment by Congress in accordance with the Constitution if the common good so requires. In truth, radio and television broadcasting companies, which are given franchises, do not own the airwaves and frequencies though which they transmit broadcast signal and images. “Airtime” is not a finished product which become the property of the company, like oil produced form refining. Thus, no private property is taken. As the government spends public funds in granting broadcasting companies the privilege and in supervising them, it would be strange if they cannot be required to render public service by giving free airtime. Strict Construction against Expropriator The limiting function imposed by the Bill is in keeping with the philosophy of the Bill of Rights against the arbitrary exercise of governmental powers to the detriment of individual rights. Given this function, the provision of Article III, Section 9 should therefore be strictly interpreted against the expropriator and liberally in favor of the property owner.

KITY

TAU MU TAU MU TAU MU TAU MU TAU MU TAU MU

CONSTITUTIONAL LAW II Order of Saint Thomas More Atty. Philip John Pojas/Atty. Rovyne G. Jumao-as, RN University College of Law

them by the government, it is a privilege and not a right. And being so, the Congress can take it away anytime. In other words, there is no taking of private property. Under existing laws, the following may exercise the power of expropriation: f. The National Government i. Congress, through a law, and the law shall provide for the procedure of the expropriation ii. Executive, pursuant to legislation enacted by Congress iii. Local legislative bodies, like the Sanggunian, provided that it is granted by the local government code b. Local government units, pursuant to an ordinance enacted by their respective legislative bodies (under LGC) c. Public utilities, as may be delegated by law. Certain public corporations, like the Land Authority and the National Housing Authority; d. Quasi-public corporations like Philippine National Railways, PLDT and the MERALCO. Eminent Domain vs. Destruction from Necessity EMINENT DOMAIN DESTRUCTION FROM NECESSITY The right of eminent The right of necessity domain is a public arises under the laws of right; it arises from the society or society itself. laws of society and is It is the right of selfvested in the state or defense, of selfits grantee, acting preservation, whether under the right and applied to persons or to power of the state, or property. It is a private benefit of the state, or right vested in every those acting under it. individual, and with which the right of the state or state necessity has nothing to do. Requires payment of Does not require just compensation payment of just compensation Property must be for Cannot require the public use conversion of the property taken to public use Scope of the Power of Eminent Domain In the hands of Congress the scope of the power is, like the scope of legislative power itself, plenary. It is as broad as the scope of police power itself. It can thus reach every form of property which the State might need for public use. It can reach even private property already dedicated to public use or even property devoted to religious worship [Bernas, The 1987 Philippine Constitution: A Comprehensive Reviewer, p. 22]. Foundation of Right to Exercise the Power of Eminent Domain The foundation of the right to exercise eminent domain is genuine necessity and that necessity must be of public character. Government may not

ACADCOM 2010; Contributors: Gene Geocaniga, Jarissa Guiani, Darlene Magabilen TAU MU Page 97 of 179

capriciously or arbitrarily choose which private property should be expropriated [Nachura, Outline Reviewer in Political Law 2006, p. 49]. Private property may be expropriated for public use and upon payment of just compensation; condemnation of property is justified only if it is for the public good and there is genuine necessity therefor of a public character. Consequently, the courts have the power to inquire into the legality of the right of eminent domain and to determine whether or not there is a genuine necessity therefor [Cruz, Constitutional Law 2007, p.66].

TAU MU TAU MU TAU MU TAU MU TAU MU

Taking, in the constitutional sense, may include trespass without actual eviction of the owner, material impairment of the value of the property for which the property was intended. In People vs. Fajardo (104 Phil 44), a municipal ordinance prohibiting a building which would impair the view of the plaza from the highway was considered taking. The property owner was entitled to payment of just compensation [Nachura, Outline Reviewer in Political Law 2006, p. 50].

TAU MU

Two concepts of TAKING: 1. Physical taking or the transfer of title 2. Taking as prevention or impairment of ordinary use - This is any act which necessarily afects the useful or beneficial use of the property. (No actual transfer)

TAU MU

TAU MU

Requisites of Taking (Republic vs. Castellvi): 6. The expropriator must enter a private property; 7. The entry must be for more than a momentary period; 8. The entry must be under warrant or color of legal authority; 9. The property must be devoted to public use or otherwise informally appropriated or injuriously afected; 10. The utilization of the property for public use must be in such a way as to oust the owner and deprive him of the beneficial enjoyment of the property.

TAU MU TAU MU TAU MU

Note: 1. Compensable taking does not need to involve all the property interests which form part of the right of ownership. When one or more of the property rights are appropriated and applied to a

The Fraternal Ateneo de Davao

TAU MU TAU MU

Requisites for the Exercise of the Power of Eminent Domain f. TAKING Taking, as the term is commonly understood, imports a physical dispossession of the owner, as when he is ousted from his land or relieved of his watch or his car and is thus deprived of all beneficial use and enjoyment of his property. In law, however, the term has a broader connotation. Taking may include trespass without actual eviction of the owner, material impairment of the value of the property or prevention of the ordinary uses for which the property was intended [Cruz, Constitutional Law 2007, p.70].

KITY

TAU MU TAU MU TAU MU TAU MU TAU MU TAU MU

CONSTITUTIONAL LAW II Order of Saint Thomas More Atty. Philip John Pojas/Atty. Rovyne G. Jumao-as, RN University College of Law

public purpose, there is already a compensable taking, even if bare title still remains with the owner. 2. All kinds of property can be expropriated, private property, personal property, real property. But what cannot be expropriated? MONEY. So, if expropriations proceedings were instituted against a person's bank account, that is not allowed. When municipal property is taken by the State: There is compensable taking depending on the nature of the property. If it is patrimonial property of the municipality, that is, property acquired by the municipality with its private funds in its corporate or private capacity, compensation is required. However, if it is any other property such as public buildings or legua communal held by the municipality for the State in trust for the inhabitants, the State is free to dispose of it at will [Bernas, The 1987 Philippine Constitution: A Comprehensive Reviewer, p. 104]. Effect of Police Power and Eminent Domain on Private Property* EMINENT DOMAIN POLICE POWER Property is “taken” Property is “regulated” Transfer of ownership No transfer of ownership The term “taking” must The term “regulation” is be compensated. not compensable *[Bernas, The 1987 Philippine Constitution: A Comprehensive Reviewer, p. 104]

CASES: TAKING REPUBLIC vs. CASTELLVI 58 SCRA 336 (1974) Facts: In 1947, The PAF leased the property of Castellvi renewable year to year. The lease was renewed yearly until 1956, Castellvi refused to renew the contract with PAF any longer. In 1959, the government instituted an action to expropriate the property previously leased. Also, in the same year, the property was placed under the possession of the government by virtue of a court order. TAU MU Issues: 1. Castellvi and the government cannot agree on the price Government – P 0.20 per sq.meter Castellvi – P15.00 per sq.meter 2. They do not also agree on the date which will be the basis for the determination of the value of the land. Governement – value in 1947 Castellvi – value in 1959 Held: The court held that the value should be determined at the time of the taking. The taking here would mean 1959, where all the elements of taking were present. It cannot be 1947 since it did not satisfy the 2nd and 5th elements. DIDIPIO EARTH SAVERS’ vs. GUZON

ACADCOM 2010; Contributors: Gene Geocaniga, Jarissa Guiani, Darlene Magabilen TAU MU Page 98 of 179

485 SCRA 586 (2006) Facts: Sec. 76 of RA 7942 otherwise known as the Philippine Mining Act of 1995, provides: Entry into private lands and concession areas – Subject to prior notification, holders of mining rights shall not be prevented from entry into private lands and concession areas by surface owners, occupants, or concessionaires when conducting mining operations therein. Issue:Is the provision a form of taking which is subject to just compensation?

TAU MU TAU MU TAU MU TAU MU TAU MU TAU MU TAU MU

Held: Yes, there was taking. The ownership of land extends to the surface as well as to the subsoil under it. First, the argument that the landowners’ right extends to the subsoil insofar as necessary for their practical interests serves only to further weaken the NPC’s case. The theory would limit the right to the subsoil upon the economic utility which such area ofers to the surface owners.

TAU MU

Issue: Was there taking as to entitle the owners to compensation?

TAU MU

Facts: In 1978, NAPOCOR, without the landowners’ knowledge and prior consent, took possession of the sub-terrain area of their lands and constructed therein underground tunnels. When discovered by the owners in 1992, they asked for compensation. NAPOCOR denied arguing that: 3. The owners’ rights extend to the subsoil only insofar as necessary for their practical interests, and 4. The tunnels merely constitute and easement upon their property, which does not involve any loss or title or possession. Hence, they were not denied the beneficial use of their subject properties as to entitle them to just compensation by way of damages.

TAU MU TAU MU

NPC vs. IBRAHIM 526 SCRA 149 (2007)

The Fraternal Ateneo de Davao

TAU MU TAU MU

Held: Yes. The entry referred to in Sec. 76 is not just a simple right-of-way which is ordinarily allowed under the provisions of the Civil Code. Here, the holders of mining rights enter private lands for purposes of conducting mining activities such as exploration, extraction and processing of minerals. Mining right holders build mine infrastructure, dig mine shafts and connecting tunnels, prepare tailing ponds, storage areas and vehicle depots, install their machinery, equipment and server systems. On top of this, under Sec 75, easement rights are accorded to them where they may build warehouses, port facilities, electric transmission, railroads and other infrastructures necessary for mining operations. All these will definitely oust the owners or occupants of the afected areas the beneficial ownership of their lands. Without a doubt taking occurs once mining operations commence.

KITY

TAU MU TAU MU TAU MU TAU MU TAU MU TAU MU

CONSTITUTIONAL LAW II Order of Saint Thomas More Atty. Philip John Pojas/Atty. Rovyne G. Jumao-as, RN University College of Law

Presumably, the landowners’ right extends to such height or depth where it is possible for them to obtain some benefit or enjoyment, and it is extinguished beyond such limit as there would be no more interest protected by law. The owners could have dug upon their property motorized deep wells but were prevented from doing so by NPC precisely because of the construction and existence of the tunnels underneath the surface of their property. They, therefore, still had a legal interest in the sub-terrain portion insofar as they could have excavated the same for the construction of the deep well. Second, the manner in which the easement was created by NPC, however, violates the due process rights of the owners as it was, without notice and indemnity to them and did not go through proper expropriation proceedings. NPC could have, at any time, validly exercised the power of eminent domain to acquire the easement over the property as this power encompasses not only the taking or appropriation of title to and possession of the expropriated property but likewise covers even the imposition of a mere burden upon the owner of the condemned property. The underground tunnels impose limitations on the use of the property for an indefinite period and deprives them of its ordinary use. Based upon the foregoing, the owners are clearly entitled to the payment of just compensation. REPUBLIC vs. ANDAYA 524 SCRA 671 (2007) Facts: Ismael Andaya is the owner of two parcels of land in Butuan City, which were subject to a 60-meter wide perpetual easement for public highways, irrigation ditches, aqueducts, and other similar works of the government or public enterprise. Out of the 10,380 sq. m. property, 701 sq. m. was subject of the easement. During the expropriation proceedings, Andaya demanded that the consequential damages he must be paid should be passed on the remaining area of 9,679 sq. m. which is the entire property minus the easement. Andaya alleged that the easement would render his entire property unusable and unhabitable. Is he correct? Held: Yes. Taking, in the exercise of the power of eminent domain, occurs not only when the government actually deprives or dispossesses the property owner of his property or of its ordinary use, but also when there is a practical destruction or material impairment of the value of his property. Using this standard, there was undoubtedly a taking of the remaining area of Andaya’s property. True, no burden was imposed thereon and Andaya still retained title and possession of the property. But the nature and the efect of the floodwalls would deprive Andaya of the normal use of the remaining areas. It would prevent ingress and egress to the property and turn it into a catch basin for the floodwaters coming from the Agusan River. Hence, Andaya is

ACADCOM 2010; Contributors: Gene Geocaniga, Jarissa Guiani, Darlene Magabilen TAU MU Page 99 of 179

CARLOS SUPERDRUG vs. DSWD 526 SCRA 130 (2007) Facts: Carlos Superdrug questioned the constitutionality of the Expanded Senior Citizens Act of 2003, which grants to senior citizens 20% discount on their purchase of medicines. According to CS, the law is confiscatory because it compels drugstore owners to grant the discount and will result in a loss of profit and capital since it tailed to provide a scheme whereby drugstores will be justly compensated for the discount. Issue: Was there a valid taking?

TAU MU TAU MU TAU MU TAU MU TAU MU TAU MU TAU MU

As a requirement of eminent domain, “public use” is the general concept of meeting public need or public exigency. It is not confined to actual use by the public in its traditional sense. The idea that “public use” is strictly limited to clear cases of “use by the public” has been abandoned. The

The predominant precept is that upon abandonment of real property condemned for public purpose, the party who originally condemned the property recovers control of the land if the condemning party continues to use the property for public purpose; however, if the condemning authority ceases to use the property for a public purpose, property reverts to the owner in fee simple. The government’s taking of private property, and then transferring it to private persons under the guise of public use or purpose is the despotism found in the immense power of eminent domain. Moreover, the direct and unconstitutional state’s power to oblige a landowner to renounce his productive and invaluable possession to another citizen, who will use it predominantly for his own private gain, is ofensive to our laws [Bernas, The 1987 Philippine Constitution: A Comprehensive Reviewer, p. 104].

TAU MU

Note: 1. Public use, for purposes of expropriation, is synonymous with public welfare as the latter term is used in the concept of police power. 2. Examples of public use include land reform and socialized housing. 3. As a general rule, when private property is expropriated for public use and it is not used, it cannot be returned to the owner. But if there is an implication that the property has to be returned to the owner if the property is abandoned, then the property should be returned. But the original owner must return the compensation. (Heirs of Moreno case)

term “public use” has not been held to be synonymous with public interest, public benefit, public welfare, and public convenience [Nachura, Outline Reviewer in Political Law 2006, p. 51].

TAU MU

g. PUBLIC USE Two concepts of public use: 1. The traditional concept: those which are to be used by the public whether for a fee or for free. i.e. the roads, public plaza, etc. 2. The expanded concept: one which will satisfy public use even if the expropriated property is not made directly available to the public for as long as it results to an indirect advantage or benefit. The advantage doesn't have to be direct.

Ateneo de Davao

TAU MU TAU MU

Held: Yes. The Senior Citizens Act was enacted primarily to maximize the contribution of senior citizens to nation-building, and to grant benefits and privileges to them for their improvement and well-being as the State considers them an integral part of our society. The law is a legitimate exercise of police power which, similar to the power of eminent domain, has general welfare for its object. Police power is not capable of an exact definition but has been purposely veiled in general terms to underscore its comprehensiveness to meet all exigencies and provide enough room for an efficient and flexible response to conditions and circumstances, thus assuring the greatest benefits.

The Fraternal

TAU MU TAU MU

entitled to payment of just compensation, which must neither more nor less than the monetary equivalent of the land.

KITY

TAU MU TAU MU TAU MU TAU MU TAU MU TAU MU

CONSTITUTIONAL LAW II Order of Saint Thomas More Atty. Philip John Pojas/Atty. Rovyne G. Jumao-as, RN University College of Law

This original meaning of the phrase has now been broadened to cover uses which, while not directly available to the public, redound to their indirect advantage or benefit [Cruz, Constitutional Law 2007, p.76]. Private property condemned for public use sold to private user – effect (Heirs of Moreno vs. Mactan-Cebu, 2005):

CASES: PUBLIC USE MASIKIP vs. CITY OF PASIG 479 SCRA (2006) Facts: The City of Pasig, notified Masikip of its intention to expropriate a portion of her property to be used for the sports development and recreational activities. She replied that the intended expropriation of her property is unconstitutional, invalid, and oppressive, as the area of her lot is neither sufficient nor suitable to “provide land opportunities to deserving poor sectors of our community.” She contended that the City failed to establish a genuine necessity which justifies the condemnation of her property. While she does not dispute the intended public purpose, nonetheless, she insists that there must be a genuine necessity for the proposed use and purposes. Held: LGUs have no inherent power of eminent domain and may exercise it only when expressly authorized by statute. Section 19 of the LGC prescribes the delegation by Congress of the power of eminent domain to LGUs and lays down the parameters for its exercise. “SEC. 19. Eminent Domain. - A local government unit may, through its chief executive and acting pursuant to an ordinance, exercise the power of eminent domain for public use, or purpose, or welfare for the benefit of the poor and the landless, upon payment of just compensation,

ACADCOM 2010; Contributors: Gene Geocaniga, Jarissa Guiani, Darlene Magabilen TAU MU Page 100 of 179

TAU MU TAU MU TAU MU TAU MU TAU MU TAU MU TAU MU TAU MU

Held: The act of NHA of entering into a contract with a real estate developer for the construction of low cost housing on the expropriated lots cannot be taken to mean as a deviation from the stated public purpose of their taking. Jurisprudence has it that the expropriation of private land for slum clearance and urban development is for a public purpose even if the developed area is later sold to homeowners, commercial firms, service companies and other private concerns. Moreover, the Constitution itself allows the State to undertake, for the common good, and in cooperation with the private sector,

TAU MU

Facts: In 1977, the National Housing Authority (NHA) field complaints to expropriate sugarcane lands in Cavite. The purpose of the expropriation was the expansion of the Damariñas Resettlement Project to accommodate the squatters who were relocated from the Metropolitan Manila Area. The trial court rendered judgment ordering the expropriation, after payment of just compensation, of the subject lots which decision was affirmed by the SC in 1987. In 1992, the previous owners of the expropriated properties filed a case against NHA for their return on the ground that NHA deviated from the public purpose of the taking when it entered into a contract for the construction of low cost housing units to be sold to low-income beneficiaries.

TAU MU TAU MU

REYES vs. NHA 395 SCRA 495 (2003)

The Fraternal Ateneo de Davao

TAU MU TAU MU

pursuant to the provisions of the Constitution and pertinent laws: Provided, however, That the power of eminent domain may not be exercised unless a valid and definite offer has been previously made to the owner, and such offer was not accepted: Provided, further, That the local government unit may immediately take possession of the property upon the filing of the expropriation proceedings and upon making a deposit with the proper court of at least fifteen percent (15%) of the fair market value of the property based on the current tax declaration of the property to be expropriated: Provided, finally, That, the amount to be paid for the expropriated property shall be determined by the proper court, based on the fair market value at the time of the taking of the property.” According to Masikip, there is already an established sports development and recreational activity center. The City does not dispute this. Evidently, there is no “genuine necessity” to justify the expropriation. Where the taking by the State of private property is done for the benefit of a small community which seeks to have its own sports and recreational facility, notwithstanding that there is such a recreational facility only a short distance away, such taking cannot be considered to be for public use. Its expropriation is not valid. The right to take private property for public purposes necessarily originates from “the necessity” and the taking must be limited to such necessity.

KITY

TAU MU TAU MU TAU MU TAU MU TAU MU TAU MU

CONSTITUTIONAL LAW II Order of Saint Thomas More Atty. Philip John Pojas/Atty. Rovyne G. Jumao-as, RN University College of Law

a continuing program of urban land reform and housing which will make at afordable cost decent housing to homeless citizens. It follows that the low cost housing of the NHA on the expropriated lots is consistent with the public use requirement. ESTATE vs. PHILIPPINE EXPORT 349 SCRA 240 (2000) Facts: Philippine Export Processing Zone (PEZA) initiated before the RTC expropriation proceedings on 3 parcels of irrigated rice land. The power of eminent domain of PEZA is contained in its original charter, PD No. 66, which allows it to expropriate any property for export processing zones, low-cost housing, or for the construction of dams, terminal facilities, structures and approaches thereto. Considering that portions of the expropriated property would be leased to banks, is it still for a “public purpose?” Held: Yes. The term “public use” has acquired a more comprehensive coverage. To the literal import of the term signifying strict use by the public has been added the broader notion of indirect public benefit or advantage. It should be pointed out that PEZA was created to be a viable commercial, industrial and investment area. The expropriation of the lot for the purpose of being leased to banks and for the construction of a terminal has the purpose of making banking and transportation facilities easily accessible to persons working at industries located at PEZA. It therefore comes as a matter of necessity to bring life to the purpose of the law, as reaffirmed by RA No. 9716. Moreover, Congress can determine the necessity of expropriating private property. In such a case, it is a question exclusively for the legislature to decide. MANAPAT vs. CA and NHA 536 SCRA 32 (2007) Facts: The subject matter of this case were the parcels of land which form part of what was originally known as the Grace Park Subdivision and formerly owned by the Roman Catholic Archbishop of Manila (RCAM) and/or the Philippine Realty Corporation (PRC). In the 1960’s, RCAM allowed a number of individuals to occupy the Grace Park property on condition that they would vacate the premises should the former push through with the plan to construct a school in the area. The plan, however, did not materialize, thus, the occupants ofered to purchase the portions they occupied. Later, as they could not aford RCAM’s proposed price, the occupants petitioned the Government for the acquisition of the said property, its subdivision into home lots, and the resale of the subdivided lots to them at a low price. The Government, through the People’s Homesite and Housing Corporation (PHHC), negotiated for the acquisition of the property from RCAM/PRC. But because of the high asking price of RCAM and the budgetary constraints of the Government, the latter’s efort to purchase and/or to expropriate the property was

ACADCOM 2010; Contributors: Gene Geocaniga, Jarissa Guiani, Darlene Magabilen TAU MU Page 101 of 179

TAU MU TAU MU TAU MU TAU MU TAU MU TAU MU TAU MU TAU MU TAU MU

Ruling: Albeit the power of eminent domain partakes of a sovereign character, it is by no means absolute. Its exercise is subject to limitations, one of which is, precisely, Section 9, Article III of the Constitution. The SC has recognized the following requisites for the valid exercise of the power of eminent domain: (1) the property taken must be private property; (2) there must be genuine necessity to take the private property; (3) the taking must be for public use; (4) there must be payment of just compensation; and (5) the taking must comply with due process of law. It is incontrovertible that the parcels of land subject of these consolidated petitions are private property. Thus, the first requisite is satisfied. With respect to the second, the foundation of the right to exercise eminent domain is genuine necessity, and that necessity must be of a public character. As a rule, the determination of whether there is genuine necessity for the exercise is a justiciable question. However, when the power is exercised by the Legislature, the question of necessity is essentially a political question. In the case at bar, the authority to expropriate came from PD 1072. At that time, and as explicitly recognized under the 1973 Constitution, President Marcos had legislative powers. Perforce, the expropriation of the subject properties – identified with specificity in the P.D. --- was directed by legislation. The issue of necessity then assumed the nature of a political question. As to the third requisite of “public use,” the SC examined the purpose for which the expropriation was undertaken by NHA. As set forth in its petition, NHA justifies the taking of the subject property for the purpose of improving and upgrading the area by constructing roads and installing facilities thereon under the Government’s zonal improvement program and subdividing them into much smaller lots for distribution and sale at a low cost to qualified beneficiaries, mostly underprivileged long-time occupants of Grace Park. Around 510 families with approximately 5 members each will be

TAU MU TAU MU

Issue: WON NHA has a lawful right to take the lot of Manapat for supposed public use.

The Fraternal Ateneo de Davao

TAU MU TAU MU

discontinued. RCAM then decided to efect, on its own, the subdivision of the property and the sale of the individual subdivided lots to the public. The parties in these consolidated cases were among those who purchased individual subdivided lots of Grace Park directly from RCAM and/or PRC. A significant turn of events however happened when the President Marcos issued PD No. 1072, appropriating Php1.2M out of the President’s Special Operations Funds to cover the additional amount needed for the expropriation of Grace Park. The NHA, PHHC’s successor, then filed several expropriation proceedings over the already subdivided lots for the purpose of developing Grace Park under the Zonal Improvement Program (ZIP) and subdividing it into small lots for distribution and resale at a low cost to the residents of the area.

KITY

TAU MU TAU MU TAU MU TAU MU TAU MU TAU MU

CONSTITUTIONAL LAW II Order of Saint Thomas More Atty. Philip John Pojas/Atty. Rovyne G. Jumao-as, RN University College of Law

benefited by the project. The only remaining obstacle in the completion of this project is the lots subject of these consolidated petitions as the other lots in Grace Park have already been expropriated. The Zonal Improvement Program (ZIP), being implemented for government by NHA, draws breath from policy mandates found in the 1987 Constitution. It is an integral part of the government’s “socialized housing” program which the SC deemed compliant with the “public use” requirement, it being a program clearly devoted to a “public purpose.” It need only be added that the “public use” requisite for the valid exercise of the power of eminent domain is a flexible and evolving concept influenced by changing conditions. It may not be amiss to state that whatever is beneficially employed for the general welfare satisfies the requirement of public use. Still, Manapat insists that, being himself a beneficiary of the expropriation (because he has been a longtime resident of Grace Park), it would be incongruous for government to take his land away from him only to give it back to him. This contention sadly fails to comprehend the public purpose for the taking under the “socialized housing” program. The parcels of land subject of the expropriation are, precisely, being taken so that they can be subdivided into much smaller lots --- at an average of 66.5 square meters per lot --- for distribution to deserving dwellers in the area. Upon the completion of the project, Manapat, and those similarly situated as he, cannot assert any right to be awarded the very same lots they currently occupy, nor be entitled to the same area of the land they now have. Note: The power of eminent domain is an inherent and indispensable power of the State. Also called the power of expropriation, it is described as “the highest and most exact idea of property remaining in the government” that may be acquired for some public purpose through a method “in the nature of a compulsory sale to the State.” By virtue of its sovereign character, the exercise of the power prevails over the nonimpairment clause, and is clearly superior to the final and executory judgment rendered by a court in an ejectment case. Being inherent, the power need not be specifically conferred on the government by the Constitution. Section 9, Article III of the Constitution, which mandates that “private property shall not be taken for a public use without just compensation,” merely imposes a limit on the government’s exercise of the power and provides a measure of protection to the individual’s right to property. Just like its two companion fundamental powers of the State, the power of eminent domain is exercised by the Legislature. However, it may be delegated by Congress to the President, administrative bodies, local government units, and even to private enterprises performing public services. HEIRS OF MORENO vs. MACTAN CEBU 413 SCRA 502 (2003)

ACADCOM 2010; Contributors: Gene Geocaniga, Jarissa Guiani, Darlene Magabilen TAU MU Page 102 of 179

TAU MU TAU MU TAU MU TAU MU TAU MU TAU MU TAU MU TAU MU TAU MU TAU MU TAU MU

Held: In Fery, the SC declared that the government acquires only such rights in the expropriated parcels of land as may be allowed by the character of its title over the properties: “If x x x land is expropriated for a particular purpose, with the condition that when that purpose is ended or abandoned, the property shall return to its former owner, then or course, when the purpose is terminated or abandoned the former owner reacquires the property so expropriated. If x x x land is expropriated for a public street and the expropriation is granted upon condition that the city can only use it for a public street, it returns to the former owner, unless there is a statutory provision to the contrary x x x if, upon the contrary, however, the decree if expropriation gives to the entity a fee simple title, then, of course, the land becomes absolute property of the expropriator, whether it be a State, province, or municipality, and in that case the non-use does not have the effect of defeating the title acquired by the expropriation proceedings x x x” As for the public purpose of the expropriation proceeding, it cannot now be doubted. Although Mactan Airport is being constructed, it does not take away the actual usefulness and importance of the Lahug Airport: it is handling the air traffic both civilian and military. From it aircrafts fly to Mindanao and

The Fraternal Ateneo de Davao

TAU MU TAU MU

Facts: In 1949, the National Airport Corporation as the predecessor agency of respondent MactanCebu International Airport Authority (MCIAA) wanted to acquire Lot Nos. 916 and 920 among other parcels of land for the proposed expansion of Lahug Airport. To entice the landowners to cede their properties, the government assured them that they could repurchase their lands once Lahug Airport was closed or its operations transferred to Mactan Airport. TAU MU At the end of 1991, Lahug Airport ceased operations as the Mactan Airport was opened for incoming and outgoing flights. Lot Nos. 916 and 920 which had been expropriated for the extension of Lahug Airport were not utilized. Hence, petitioners filed a complaint for reconveyance and damages to compel the repurchase of Lot Nos. 916 and 920. Petitioners argue that Fery vs. Municipality of Cabanatuan does not apply to the case at bar since what was involved therein was the “right of reversion” and not the “right of repurchase” which they are invoking. They also diferentiate Mactan-Cebu International Airport Authority vs. CA from the instant case in that the landowners in the MCIAA case ofered inadmissible evidence to show their entitlement to a right of repurchase, while petitioners herein ofered evidence based on personal knowledge for which reason MCIAA did not object and thus waived whatever objection it might have had to the admissibility thereof. Finally, petitioners allege that their right to equal protection of the laws would be infringed if some landowners are given the right to repurchase their former properties even as they are denied the exercise of such prerogative.

KITY

TAU MU TAU MU TAU MU TAU MU TAU MU TAU MU

CONSTITUTIONAL LAW II Order of Saint Thomas More Atty. Philip John Pojas/Atty. Rovyne G. Jumao-as, RN University College of Law

Visayas and past through it or their flights to the North and Manila. Then, no evidence was adduced to show how soon is the Mactan Airport to be placed in operation and whether the Lahug Airport will be closed immediately thereafter. It is up to the other departments of the Government to determine said matters. The Court cannot substitute its judgment for those of the said departments or agencies. In the absence of such showing, the Court will presume that the Lahug Airport will continue to be in operation. The predicament of petitioners involves a constructive trust, one that is akin to the implied trust referred to in Art. 1454 of the NCC. “If an absolute conveyance of property is made in order to secure the performance of an obligation of the grantor toward the grantee, a trust by virtue of law is established. If the fulfillment of the obligation is offered by the grantor when it becomes due, he may demand the reconveyance of the property to him.” In the case at bar, petitioners conveyed Lot Nos. 916 and 920 to the government with the latter obliging itself to use the realties for the expansion of Lahug Airport; toiling to keep its bargain, the government can be compelled by petitioners to reconvey the parcels of land to them, otherwise, petitioners would be denied the use of their properties upon a state of afairs that was not conceived nor contemplated when the expropriation was authorized. Hence, respondent MCIAA as representative of the State is obliged to reconvey Lot Nos. 916 and 920 to petitioners who shall hold the same subject to existing liens thereon, ie leasehold right of DPWH. HEIRS OF MORENO vs. MACTAN-CEBU 466 SCRA 288 (2005) Facts: On MFR by Mactan International Airport Authority, it was argued that the decision of the Court which granted the owner’s right of repurchase efectively overturns the ruling in Fery, which requires that for an expropriation judgment to be conditional, it must clearly spell out said condition. Held: Nothing in the Fery case bespeaks that there should be an express condition in the dispositive portion of the decision before the condemned property can be returned to its former owner after the purpose for its taking has been abandoned or ended. The indisputable certainty in the present case is that there was a prior promise by the predecessor of Mactan that the expropriated properties may be recovered by the former owners since the airport is transferred to Mactan, Cebu. In fact, the witness for Mactan testified that 15 lots were already reconveyed to their previous owners. SUMULONG vs. GUERRERRO 154 SCRA 461 (1987)

ACADCOM 2010; Contributors: Gene Geocaniga, Jarissa Guiani, Darlene Magabilen TAU MU Page 103 of 179

Facts: The NHA wanted to use Sumulong’s property for socialized housing for the lower and middle class. The owner contended that socialized housing is not public use because not everyone can benefit from this, only the handful of people who to be given the houses.

TAU MU TAU MU TAU MU TAU MU TAU MU TAU MU TAU MU

Held: The power of eminent domain is inherent in every state and the provisions in the Constitution pertaining to such power only serve to limit its exercise in order to protect the individual against whose property the power is sought to be enforced. The State is not subject to any limitation other than those imposed by the Constitution which are: First, the taking must be for a public use; Second, the payment of just compensation must be made and;Third, due process must be observed in the taking. The due process clause cannot be rendered nugatory everytime a specific decree or law orders the expropriation of somebody’s property and provides its own peculiar manner of taking the same. Neither should the courts adopt a hands-of policy just because the public use has been ordained as existing by the decree of the just compensation has been fiexed and determined beforehand by a statute. Although due process does not always necessarily demand that a proceeding be had

TAU MU

Issue: W/N the government had the right to expropriate the said areas.

TAU MU

Facts: The President of the Philippines issued a Letter of Instruction instituting an nationwide slum improvement and resettlement program and to adopt slum improvement as a national housing policy. The Metro Manila Zonal Improvement Program included the properties known as the Tambunting Estate and the Sunog-Apog area in its priority list for a zonal improvement program. The President also designated the NHA to negotiate with the owners of the property for the acquisition of the same. This, however, did not materialize as the negotiations for the purchase of the property failed. The President issued Proc. No. 1810 declaring all sites identified by the Metro Manila local governments and approved by the Ministry of Human Settlements to be included in the ZIP upon proclamation of the President. The Tambunting Estate and the Sunog-Apog area were among the sites included.

TAU MU TAU MU

MANOTOK vs. CA 150 SCRA 87 (May 21, 1987)

The Fraternal Ateneo de Davao

TAU MU TAU MU

Held: The court held that the socialized housing is within the context of public use. Public use has acquired a more comprehensive meaning. That is whatever would result to indirect public benefit or welfare is also public use. It also ruled that it will benefit everyone in the sense that it will afect the safety, health and environment. Providing housing to these people will help in lessening the incidence of violence and problems concerning health. In the end, it will benefit everybody in a way. In short, socialize housing falls within the meaning of public use.

KITY

TAU MU TAU MU TAU MU TAU MU TAU MU TAU MU

CONSTITUTIONAL LAW II Order of Saint Thomas More Atty. Philip John Pojas/Atty. Rovyne G. Jumao-as, RN University College of Law

before a court of law, it still mandates some form of proceeding wherein notice and reasonable opportunity to be heard are given to the owner to protect his property rights. There are exceptional situations when, in the exercise of the power of eminent domain, the requirement of due process may not necessarily entail judicial process. But where it is alleged that in the taking of a person’s property, his right to due process of law has been violated, the courts will have to step in and probe into such an alleged violation. A necessity must exist for the taking of private property for the proposed uses and purposes but accepted the fact that modern decisions do not call for absolute necessity. It is enough if the condemnor can show a reasonable or practical necessity, which of course, varies with the time and peculiar circumstances of each case. In the instant petitions, there is no showing whatsoever as to why the properties involved were singled out for expropriation through decrees or what necessity impelled the particular choices or selections. The provision of P.D. 1669 which allows NHA, at its sole option, to put portions of the expropriated area to commercial use in order to defray the development costs of its housing projects cannot stand constitutional scrutiny. P.D. Nos. 1669 and 1670 is violative of the petitioners' right to due process of law and, therefore, they must fail the test of constitutionality. The decrees, do not by themselves, provide for any form of hearing or procedure by which the petitioners can question the propriety of the expropriation of their properties or the reasonableness of the just compensation. The fixing of the maximum amounts of compensation and the bases thereof which are the assessed values of the properties in 1978 deprive the petitioner of the opportunity to prove a higher value because, the actual or symbolic taking of such properties occurred only in 1980 when the questioned decrees were promulgated. The decision of the government to acquire a property through eminent domain should be made known to the property owner through a formal notice wherein a hearing or a judicial proceeding is contemplated. This shall be the time of reckoning the value of the property for the purpose of just compensation. The questioned decrees transgress the petitioners' right to just compensation. Having violated the due process and just compensation guarantees, P. D. Nos. 1669 and 1670 are unconstitutional and void. MANOSCA vs. CA 252 SCRA 412 (1996) Facts: Petitioners own a piece of land consisting of 492 square meters, which was later ascertained by the National Historical Institute to be the birth site of Felix Y. Manalo, the founder of the Iglesia Ni Cristo. Consequently, it was declared a national historical landmark, and the Republic instituted expropriation proceedings. Issue: Would the expropriation constitute an application of funds for the use, benefit or support of a religious entity?

ACADCOM 2010; Contributors: Gene Geocaniga, Jarissa Guiani, Darlene Magabilen TAU MU Page 104 of 179

TAU MU TAU MU TAU MU TAU MU TAU MU TAU MU TAU MU TAU MU

 Cross-reference with PubCorp.: Local legislative body can delegate its power to the Mayor. Under the Local Government Code the f. requisites must be present:

TAU MU

Note: 1. The medium of payment is money. It is always money. But we know that there is an exception to this, and that is under CARP, because the CARP is a revolutionary undertaking. (Association of Small Land Owners vs. Secretary) 2. The determination of the amount of just compensation is a judicial function. 3. Compensation is just if the owner receives a sum equivalent to the market value of his property. Market value is generally defined as the fair value of the property as between one who desires to purchase and one who desires to sell. 4. The point of reference use in determining fair value is the value at the time the property was taken. Thus, future potential use of the land is not considered in computing just compensation.

TAU MU TAU MU

h. JUST COMPENSATION Just compensation is described as a full and fair equivalent of the property taken from the private owner by the expropriator. This is intended to indemnify the owner fully for the loss he has sustained as a result of the expropriation. The measure of this compensation is not just the taker’s gain but the owner’s loss. The word “just” is used to intensify the meaning of the word “compensation,” to convey the idea that the equivalent to be rendered for the property taken shall be real, substantial, full, ample [Cruz, Constitutional Law 2007, p.76]. Expressed diferently, the compensation given to the owner is just if he receives for his property a sum equivalent to its “market value” [Bernas, The 1987 Philippine Constitution: A Comprehensive Reviewer, p. 104]. It is the just and complete equivalent of the loss which the owner of the thing appropriated has to sufer by reason of the expropriation. It is the market value to which can be added the consequential damages, if any and from which must be deducted consequential benefits, if any.

The Fraternal Ateneo de Davao

TAU MU TAU MU

Held: The attempt to give some religious perspective to the case deserves little consideration, for what should be significant is the principal objective of, not the casual consequences that might follow from, the exercise of power. The purpose in setting up a marker is essentially to recognize the distinctive contribution of the late Felix Manalo to the culture of the Philippines, rather than to commemorate his founding and leadership of the Iglesia Ni Cristo. The practical reality that the greater benefit may be derived by its members than by most others could well be true but such a peculiar advantage still remains to be merely incidental and secondary in nature. Indeed that only a few would actually benefit from the expropriation of the property does not necessarily diminish the essence and character of public use.

KITY

TAU MU TAU MU TAU MU TAU MU TAU MU TAU MU

CONSTITUTIONAL LAW II Order of Saint Thomas More Atty. Philip John Pojas/Atty. Rovyne G. Jumao-as, RN University College of Law

1. There must be a definite ofer which was previously made upon the owner; 2. An ordinance enacted by the council authorizing the Mayor to initiate the proceedings; 3. Exercised to expropriate the property for public use; 4. Just compensation  Section 9, Article III of the Constitution specifically mandates that “Private property shall not be taken for public use without just compensation.  In City of Manila vs. Estrada, just compensation means an equivalent for the value of land (property) taken. The use of the word “just” is to convey the idea that the equivalent to be rendered for the property taken shall be real, substantial, full, and ample. Thus, just compensation is defined as a fair and full equivalent for the loss sustained.  In Manila Railroad Co. v. Caligsahan, the SC held that “to be exactly just, the compensation should be estimated at the time of the taking.”  In Republic vs. Vda. De Castellvi, it was ruled that just compensation is determined as of the date of the taking of the property or the filing of the complaint, whichever came first.

CASES: JUST COMPENSATION SANTOS vs. LAND BANK 340 SCRA 59 (2000) Facts: An RTC ordered Land Bank to pay P49M to Santos for the taking of some 77 hectares of agricultural land. It was also ordered that the compensation shall be in the manner provided under the CARL, which allows compensation in cash and in bonds. Issue: Is the mode of payment under the CARL constitutional? Held: Yes. Traditionally, the medium of payment if just compensation is money and no other. However, in land reform, we do not deal with the traditional exercise of the power of eminent domain. This is a revolutionary kind of expropriation that does not afect only a specific property or a limited area. Accepting the theory that payment of just compensation need not always be made fully in money, we find that the proportion of cash payment is not unduly oppressive. No less important, the government financial instruments in making up the balance of the payment are negotiable at any time. -BASIS OF DETERMINATIONESLABAN vs. VDA DE ONORIO 360 SCRA 230 (2001)

ACADCOM 2010; Contributors: Gene Geocaniga, Jarissa Guiani, Darlene Magabilen TAU MU Page 105 of 179

KITY

Held: Sec. 4, of Rule 67 of the Rules of Court states that just compensation should be determined “as of the date of the taking of the property or the filing of the complaint, whichever comes first.” The price of the land at the time of the taking, not its value after the passage of time, represents the true value to be paid as just compensation. Thus, the just compensation to be paid respondent should be determined as of its taking by the NIA in 1981, not the filing of the complaint in 1990.

TAU MU TAU MU TAU MU TAU MU TAU MU TAU MU TAU MU TAU MU TAU MU TAU MU TAU MU

Facts: Ibrahim and his co-heirs claimed that they were owners of several parcels of land described in Survey Plan FP (VII-5) 2278 consisting of 70,000 square meters, divided into three (3) lots, i.e. Lots 1, 2, and 3 consisting of 31,894, 14,915, and 23,191 square meters each respectively. In 1978, NAPOCOR, through alleged stealth and without respondents’ knowledge and prior consent, took possession of the sub-terrain area of their lands and constructed therein underground tunnels. The existence of the tunnels was only discovered sometime in July 1992 by respondents and then later confirmed on November 13, 1992 by NAPOCOR itself through a memorandum issued by the latter’s Acting Assistant Project Manager. The tunnels were apparently being used by NAPOCOR in siphoning the water of Lake Lanao and in the operation of NAPOCOR’s projects. In 1992, respondent Maruhom, Ibrahim’s co-heirs, requested the Marawi City Water District for a permit to construct and/or install a motorized deep well in Lot 3 but his request was turned down because the construction of the deep well would cause danger to lives and property. Thereafter, respondents demanded that NAPOCOR pay damages and vacate the subterrain portion of their lands but the latter refused to vacate much less pay damages. Respondents further averred that the construction of the underground tunnels has endangered their lives and properties as Marawi City lies in an area of local volcanic and tectonic activity. Further, these illegally constructed tunnels caused them sleepless nights, serious anxiety and shock thereby entitling them to recover moral damages and that by way of example for the public good,

TAU MU TAU MU

Facts: Respondent owns a piece of land with an area of 39,512 square meters. On October 6, 1981, with the consent of respondent provided they would be paid for the area taken after the processing of documents by the Commission on Audit, the National Irrigation Administration constructed an irrigation canal. The canal afected 24,660 square meters. Since NIA failed to pay for the property, on Dec. 10,1990, respondent filed a complaint praying that NIA be ordered to pay compensation for the property used in the canal constructed. What year should be used as basis for determining just compensation?

NPC vs. IBRAHIM 526 SCRA 149 (2007)

The Fraternal Ateneo de Davao

TAU MU TAU MU TAU MU TAU MU TAU MU TAU MU

CONSTITUTIONAL LAW II Order of Saint Thomas More Atty. Philip John Pojas/Atty. Rovyne G. Jumao-as, RN University College of Law

NAPOCOR must be held liable for exemplary damages. Issue: WON the respondents are entitled to just compensation. Ruling: The Court has held that if the government takes property without expropriation and devotes the property to public use, after many years, the property owner may demand payment of just compensation in the event restoration of possession is neither convenient nor feasible. This is in accordance with the principle that persons shall not be deprived of their property except by competent authority and for public use and always upon payment of just compensation. Just compensation has been understood to be the just and complete equivalent of the loss and is ordinarily determined by referring to the value of the land and its character at the time it was taken by the expropriating authority. There is a “taking” in this sense when the owners are actually deprived or dispossessed of their property, where there is a practical destruction or a material impairment of the value of their property, or when they are deprived of the ordinary use thereof. There is a “taking” in this context when the expropriator enters private property not only for a momentary period but for more permanent duration, for the purpose of devoting the property to a public use in such a manner as to oust the owner and deprive him of all beneficial enjoyment thereof. Moreover, “taking” of the property for purposes of eminent domain entails that the entry into the property must be under warrant or color of legal authority. Under the factual backdrop of this case, the last element of taking mentioned is patently lacking. Hence, the taking was not made properly. The manner in which the easement was created by petitioner, however, violates the due process rights of respondents as it was without notice and indemnity to them and did not go through proper expropriation proceedings. Hence, respondents are entitled to just compensation. Petitioner could have validly exercised the power of eminent domain to acquire the easement over respondents’ property as this power encompasses not only the taking or appropriation of title to and possession of the expropriated property but likewise covers even the imposition of a mere burden upon the owner of the condemned property. In the case at bar, the SC made reference to the case Mangondato in computing the amount of compensation to be given to the respondents. In the Mangondato case, the general rule in determining “just compensation” in eminent domain is the value of the property as of the date of the filing of the complaint. The general rule, however, admits of an exception: where this Court fixed the value of the property as of the date it was taken and not the date of the commencement of the expropriation proceedings. Thus, to allow petitioner to use the date it constructed the tunnels as the date of valuation would be grossly unfair. First, it did not

ACADCOM 2010; Contributors: Gene Geocaniga, Jarissa Guiani, Darlene Magabilen TAU MU Page 106 of 179

enter the land under warrant or color of legal authority or with intent to expropriate the same. In fact, it did not bother to notify the owners and wrongly assumed it had the right to dig those tunnels under their property. Secondly, the “improvements” introduced by petitioner, namely, the tunnels, in no way contributed to an increase in the value of the land. The trial court, therefore, as affirmed by the CA, rightly computed the valuation of the property as of 1992, when respondents discovered the construction of the huge underground tunnels beneath their lands and petitioner confirmed the same and started negotiations for their purchase but no agreement could be reached.

TAU MU TAU MU TAU MU TAU MU TAU MU TAU MU TAU MU TAU MU TAU MU

Issue: WON the just compensation for petitioner’s property should be based on the BIR

TAU MU TAU MU

Facts: Prior to the transfer of the property to petitioner by an Antonio Development Corporation (SADC), Public Estates Authority (PEA) wrote SADC requesting permission to enter the latter’s property, then covered by TCT No. 439101, for the purpose of constructing thereon the southern abutment of the Zapote Bridge at the Coastal Road. PEA also proposed to SADC to start their negotiation for its acquisition of the latter’s property. SADC replied authorizing PEA to enter the property, subject to the condition that the latter should pay a monthly rental of Php10k. PEA then directed its contractor, the Philippine National Construction Corporation, to enter the property and begin the necessary engineering works on the Coastal Road. Thereafter, PEA requested SADC either to donate or sell the property to the government. SADC replied by ofering to sell the property to PEA. PEA informed SADC it has no plan to buy the whole lot, but only the 1,131 square meter portion above sea level. PEA then asked SADC to submit proofs of ownership and costs of the improvements which were demolished. Negotiations then ensued between the parties. However, for the past twenty (20) years, they failed to reach an agreement. In 2000, SADC asked PEA to pay compensation equivalent to the current zonal value plus interest of 10% per annum and a monthly rental of Php10k, also with the same interest. These sums, according to SADC, could be considered just compensation for the government’s use of the property since 1985 until September 2000 and thereafter. The following month, PEA inquired from the BIR District 53, the zonal value of the SADC property. It submitted to the BIR the appraisal reports prepared by two (2) independent licensed appraisers. On April 2001, petitioner acquired the property from SADC. The BIR sent a letter to PEA stating that the zonal value of the property is P2,900.00 per square meter, with the caveat that the said assessment is subject to review and approval by higher tax authorities. Thereafter, the BIR informed PEA that the current zonal value of the property is Php20k per square meter.

The Fraternal Ateneo de Davao

TAU MU TAU MU

TAN vs. REPUBLIC May 25, 2007

KITY

TAU MU TAU MU TAU MU TAU MU TAU MU TAU MU

CONSTITUTIONAL LAW II Order of Saint Thomas More Atty. Philip John Pojas/Atty. Rovyne G. Jumao-as, RN University College of Law

zonal valuation in 1985 when petitioner entered the subject property. Ruling: PEA’s entry into the property with the permission of SADC, its previous owner, was not for the purpose of expropriating the property. SADC allowed PEA to enter the land on condition that it should pay a monthly rental ofPhp10k. Thereafter, PEA requested SADC to donate or sell the land to the government. Thereafter, SADC ofered to sell the land to PEA for Php1,288,980.00, plus Php400,000.00 representing the value of the improvements destroyed by PEA when it entered the property. However, since 1985 up to the present, no agreement has been reached between PEA and SADC or herein petitioner who acquired the property from the latter. While PEA has been earning huge toll fees, it has refused to pay petitioner any compensation for the use of her property in violation of her right as an owner. The above circumstances clearly show that when PEA entered petitioner’s land in 1985, it was not for the purpose of expropriating it. After its entry, PEA wrote SADC requesting to donate or sell the land to the government. Indeed, there was no intention on the part of PEA to expropriate the subject property. Section 2, Rule 67 (on Expropriation) of the same Rules provides, among others, that upon the filing of the complaint or at any time thereafter and after due notice to the defendant, the plaintif shall have the right to take or enter upon the possession of the real property involved if he deposits with the authorized government depositary an amount equivalent to the assessed value of the property. There was no taking of the property in 1985 by PEA for purposes of expropriation. As shown by the records, PEA filed with the RTC its petition for expropriation only in September 2003. The trial court, therefore, was correct in ordering respondent, through PEA, upon the filing of its complaint for expropriation, to pay petitioner just compensation on the basis of the BIR zonal valuation of the subject property at Php20k per square meter. HEIRS of PIDACAN vs. AIR TRANSPORTATION OFFICE July 15, 2007 Facts: In 1935, spouses Mateo Pidacan and Romana Eigo acquired under the homestead provision of Act 2874 a parcel of land consisting of about 22 hectares. Patent No. 33883 and OCT No. 2204 were issued on the land, in the names of the Pidacan spouses. In 1948, the Civil Aeronautics Administration (now “ATO”) used a portion of the said property as an airport. Upon the death of the Pidacan spouses, the ATO constructed a perimeter fence and a new terminal building on the property. The ATO also lengthened, widened, and cemented the airport’s runway. The spouses’ heirs demanded from ATO the payment of the value of the property as well as rentals for the use of the occupied premises. However, they were told that payment could not be made because the property was still in their

ACADCOM 2010; Contributors: Gene Geocaniga, Jarissa Guiani, Darlene Magabilen TAU MU Page 107 of 179

right of eminent domain to take advantage of the ridiculously low value of the property at the time of taking that it arbitrarily chooses to the prejudice of petitioners. In this particular case, justice and fairness dictate that the appropriate reckoning point for the valuation of petitioners’ property is when the trial court made its order of expropriation in 2001. As for the fair value of the subject property, the SC believe that the amount arrived at by the commissioners appointed by the trial court, Php304.39 per square meter, constitutes just compensation to petitioners. TIONGSON vs. NHA 558 SCRA 56 (2008)

TAU MU

Facts: NHA took possession in 1978 of properties belonging to petitioners pursuant to P.D. No. 1669 (An Act Providing for the Expropriation of the Property known as the “Tambunting Estate”) and P.D. No. 1670 (An Act Providing for the Expropriation of the Property along the Estero de Sunog-Apog). The two presidential decrees were held to be unconstitutional in 1987 for being violative of right to due process. Petitioners argue that since P.D. No. 1669 pursuant to which NHA took possession of their properties in 1978 was declared unconstitutional, “necessarily, in thereafter resurrecting the filing of another complaint for expropriation of the same properties,” it would be unlawful to fix the reckoning period for purposes of computing the just compensation based on NHA’s previous unlawful taking of said properties in 1978.” They thus maintain that the trial court’s in 1997 holding that the determination of the just compensation of their properties should be reckoned from the date NHA filed the petition before the RTC in 1987 is in order.

TAU MU

Issue: WON the subject properties sought to be expropriated should be reckoned from the date of taking of the property or from the date of the filing of the complaint.

TAU MU

Ruling: The determination of just compensation should be reckoned from NHA’s taking of the properties in 1978. The parties admitted that NHA took possession of the subject properties as early as 1978. In NHA’s petition for expropriation filed before the RTC constitute judicial admission of NHA that it possessed the subject properties until this Court’s declaration promulgated on May 21, 1987, that P.D. No. 1669 pursuant to which NHA took possession of the properties of petitioners in 1978 was unconstitutional and, therefore, null and void. Hence, in the case at bar, just compensation of petitioners’ properties must be determined “as of the date of the filing of NHA’s complaint” on September 14, 1987.

TAU MU TAU MU TAU MU TAU MU TAU MU TAU MU TAU MU TAU MU

Issue: WON petitioners are entitled to just compensation. Ruling: Eminent domain or expropriation is the inherent right of the state to condemn private property to public use upon payment of just compensation. A number of circumstances must be present in the taking of property for purposes of eminent domain: (1) the expropriator must enter a private property; (2) the entrance into private property must be for more than a momentary period; (3) the entry into the property should be under warrant or color of legal authority; (4) the property must be devoted to a public use or otherwise informally appropriated or injuriously afected; and (5) the utilization of the property for public use must be in such a way as to oust the owner and deprive him of all beneficial enjoyment of the property. When private property is rendered uninhabitable by an entity with the power to exercise eminent domain, the taking is deemed complete. Taking occurs not only when the government actually deprives or dispossesses the property owner of his property or of its ordinary use, but also when there is a practical destruction or material impairment of the value of his property. In this case, it is undisputed that petitioners’ private property was converted into an airport by respondent ATO. As a consequence, petitioners were completely deprived of beneficial use and enjoyment of their property. Clearly, there was taking in the concept of expropriation as early as 1948 when the airport was constructed on petitioners’ private land. As a rule, the determination of just compensation in eminent domain cases is reckoned from the time of taking. In this case, however, application of the said rule would lead to grave injustice. Note that the ATO had been using petitioners’ property as airport since 1948 without having instituted the proper expropriation proceedings. To peg the value of the property at the time of taking in 1948, despite the exponential increase in its value considering the lapse of over half a century, would be iniquitous. Hence, the SC cannot allow the ATO to conveniently invoke the

The Fraternal Ateneo de Davao

TAU MU TAU MU

parents’ name. The heirs presented TCT No. T7160 and the death certificates of their parents to the ATO, but the latter still refused to pay them. The heirs claimed that they were entitled to payment of rentals plus the value of the property. The ATO countered that the heirs were not entitled to any payment, either of the value of the land or of the rentals because the property had been sold to its predecessor, the defunct Civil Aeronautics Administration for P0.70 per square meter. The ATO claimed that even if it failed to obtain title in its name, it had been declaring the property for taxation purposes. The heirs subsequently filed with the RTC a complaint against the ATO for payment of the value of the property as well as rentals for its use and occupation. The ATO, in turn, filed a complaint for expropriation, which was dismissed on the ground that it would be absurd for the ATO to expropriate a parcel of land it considered its own.

KITY

TAU MU TAU MU TAU MU TAU MU TAU MU TAU MU

CONSTITUTIONAL LAW II Order of Saint Thomas More Atty. Philip John Pojas/Atty. Rovyne G. Jumao-as, RN University College of Law

NEPUMUCENO vs. CITY of SURIGAO 560 SCRA 41 (2008) Facts: Petitioner alleged that the city government neither asked her permission to use the land nor instituted expropriation proceedings

ACADCOM 2010; Contributors: Gene Geocaniga, Jarissa Guiani, Darlene Magabilen TAU MU Page 108 of 179

TAU MU TAU MU TAU MU TAU MU TAU MU TAU MU

Facts: In 1969, the government instituted expropriation proceedings against tracts of land in Malolos, Bulacan, to be utilized for broadcast operation and use of radio transmitter facilities of the Voice of the Philippines. In 1979, the trial court rendered judgment condemning the

TAU MU

REPUBLIC vs. CA 383 SCRA 611 (2002)

TAU MU

-EFFECT OF NON-PAYMENT-

TAU MU

Ruling: Where actual taking is made without the benefit of expropriation proceedings and the owner seeks recovery of the possession of the property prior to the filing of expropriation proceedings, it is the value of the property at the time of taking that is controlling for purposes of compensation. As held in the case of Republic vs. Lara, the SC ruled that “The owner of private property should be compensated only for what he actually loses; it is not intended that his compensation shall extend beyond his loss or injury. And what he loses is only the actual value of his property at the time it is taken. This is the only way the compensation to be paid can be truly just; i.e., “just” not only to the individual whose property is taken, “but to the public, which is to pay for it. Thus, the value of petitioners’ property must be ascertained as of 1960 when it was actually taken. It is as of that time that the real measure of their loss may fairly be adjudged. The value, once fixed, shall earn interest at the legal rate until full payment is efected, conformably with other principles laid down by case law. Moreover, the damages prayed for by the petitioners should be denied there being no contractual obligations entered into between the parties.

TAU MU TAU MU

Issue: WON petitioners are entitled to just compensation.

The Fraternal Ateneo de Davao

TAU MU TAU MU

for its acquisition. She and her husband wrote respondent (then Surigao City Mayor) Salvador Sering a letter proposing an amicable settlement for the payment of the portion taken over by the city. They subsequently met with Mayor Sering to discuss their proposal but the mayor rebufed them in public and refused to pay them anything. Petitioners sought reconsideration of the mayor’s stand. But again, the city mayor turned this down in his reply. As a consequence, petitioners claimed that they sufered mental anguish, embarrassment, disappointment and emotional distress which entitled them to moral damages. Respondents admitted the existence of the road in question but alleged that it was constructed way back in the 1960s during the administration of former Mayor Pedro Espina. At that time, the lot was owned by the spouses Fernandez who signed a road right-of-way agreement in favor of the municipal government. However, a copy of the agreement could no longer be found because the records were completely destroyed and lost when the Office of the City Engineer was demolished by typhoon Nitang in 1994.

KITY

TAU MU TAU MU TAU MU TAU MU TAU MU TAU MU

CONSTITUTIONAL LAW II Order of Saint Thomas More Atty. Philip John Pojas/Atty. Rovyne G. Jumao-as, RN University College of Law

property and ordering the government to pay as just compensation P6.00 per square meter, which included the 76,589 square meter subject of the dispute. Eventually, portions of the land were used for the expansion of Bulacan State University. Despite the judgment however, and despite utilization for public use by the government, as of Sept. 16, 1999, the owners of the property were not compensated. May the owners demand the return of the property? If not, may they demand compensation based on the current value of the property? Held: By final and executory judgment in the expropriation proceedings, the previous owners are not entitled to recover possession of their expropriated land – which are still devoted to public use – but only to demand the fair market value. It is of no moment that the present property use difers from the original purpose of the expropriation. Surely, the government, as condemnor and owner of the property, is well within its rights to alter and decide the use, the only, limitation being that it is for public use. The compensation for the property should be computed at its market value at the time it was taken and appropriated by the State. However, the owners are entitled to an interest of 12% per annum computed from the date of the “taking” of the property, i.e. September 1969, until the due amount shall have been fully paid. REYES vs. NHA 395 SCRA 495 Ruling: The refusal of respondent NHA to pay just compensation, allegedly for failure of petitioners to pay capital gains tax and surrender the owners’ duplicate certificates of title, to be unfounded and unjustified. First, under the expropriation judgment the payment of just compensation is not subject to any condition. Second, it is a recognized rule that although the right to enter upon and appropriate the land to public use is completed prior to payment, title to the property expropriated shall pass from the owner to the expropriator only upon full payment of the just compensation. With respect to the amount of the just compensation still due and demandable from respondent NHA, the lower courts erred in not awarding interest computed from the time the property is actually taken to the time when compensation is actually paid or deposited in court. It is not disputed that respondent NHA took actual possession of the expropriated properties in 1977. Perforce, while petitioners are not entitled to the return of the expropriated property, they are entitled to be paid the balance of P1,218,574.35 with legal interest thereon at 12% per annum computed from the taking of the property in 1977 until the due amount shall have been fully paid. REPUBLIC vs. LIM 462 SCRA 265 (2005)

ACADCOM 2010; Contributors: Gene Geocaniga, Jarissa Guiani, Darlene Magabilen TAU MU Page 109 of 179

Facts: On April 5, 1948 an entry of judgment was made in the expropriation case filed by the Republic against the Denzons. The judgment allowed the Republic to expropriate their property after payment of just compensation in the amount of P4,062.10. After the lapse of more than 50 years, the Republic failed to pay the amount, though it took possession of the property. May the landowner be allowed to recover?

TAU MU TAU MU TAU MU TAU MU TAU MU TAU MU TAU MU

Held: In Panes v. Visayas, 264 SCRA 708 (1996), we ruled that the determination of the amount of provisional deposit for the issuance of the writ of possession is a judicial function. Consequently, a summary hearing must first be done by the judge to determine provisionally the value of the property, which full amount should be deposited with the national or provincial treasurer. However, under the 1987 Rules on Civil Procedure, it is provided that upon filing of the complain and after due notice to the defendant “the plaintif shall have the right to take or enter upon the possession of the real property involved if he deposits with the authorized government depositary an amount equivalent to the assessed value of the property for purposes of taxation to be held by such bank subject to the orders of the court.” There is therefore no need of a hearing, as the issuance of the writ of possession by the

TAU MU

Facts: Robern Development Corporation owns parcels of land in Davao City intended for a lowcost housing project. The National Power Corporation filed a complaint to expropriate them. Upon deposit of P6,121.20 with the Philippine National Bank representing the assessed value of the property, the Regional Trial Court issued a writ of possession in NPC’s favor as authorized by PD No. 42. Can the judge issue the writ without first conducting a hearing on the amount of just compensation?

TAU MU

ROBERN vs. QUITAIN 315 SCRA 150 (1999)

TAU MU TAU MU

-WRIT OF POSSESSION-

The Fraternal Ateneo de Davao

TAU MU TAU MU

Held: Yes. While we are aware of the doctrine that “non-payment of just compensation does not entitle the landowner to recover possession, the facts of the present case do not justify its application. It bears stressing that the Republic was ordered to pay just compensation twice, the first was in the expropriation proceedings and the second in, Valdehueza. Fifty-seven years have passed since then. We cannot but construe the Republic’s failure to pay just compensation as a deliberate refusal on its part. We thus rule that the special circumstances prevailing in this case entitle respondent to recover possession of the expropriated lot from the Republic. Unless this form of swift an efective relief is granted to him, the grave injustice committed against his predecessors-in-interest, though no fault or negligence on their part, will be perpetuated.

KITY

TAU MU TAU MU TAU MU TAU MU TAU MU TAU MU

CONSTITUTIONAL LAW II Order of Saint Thomas More Atty. Philip John Pojas/Atty. Rovyne G. Jumao-as, RN University College of Law

court becomes ministerial once the provisional compensation is deposited.

CITY of ILOILO vs. LEGASPI 444 SCRA 329 (2004) Facts: In 2001, the Sangguniang Panlungsod of the City of Iloilo enacted Regulation Ordinance granting authority to its City Mayor to institute expropriation proceedings on Lot No. 935, registered in the name of Manuela Yusay. The regulation ordinance was approved by then City Mayor Malabor. Mayor Malabor wrote the heirs of Yusay, through Mrs. Sylvia, Administratrix of the estate of Manuela Yusay, making a formal ofer to purchase their property known as Cadastral Lot No. 935 with an area of 85,320 square meters for P250 per square meter for the purpose of converting the same as an on-site relocation for the poor and landless residents of the city in line with the city’s housing development program. Mayor Malabor informed the administrators, Sylvia and Enrique, that their counter-proposal to the City’s proposal to purchase Lot No. 935 was not acceptable to the City Government, particularly to the City Council, which insisted that an expropriation case be filed. With their apparent refusal to sell the property, the City terminated further proceedings on the matter. Petitioner City of Iloilo, represented by Mayor Treñas, filed an Amended Complaint or Eminent Domain against private respondents Heirs of Yusay. Public respondent Judge Legaspi found the motion to be in order and meritorious, and the grounds of the opposition to be untenable; thus, he set the case for Preliminary Hearing on the Special and Affirmative Defenses. Petitioner filed a Motion for Issuance of Writ of Possession alleging that since it has deposited with the Court the amount of Php2,809,696.50 representing 15% of the fair market value of the property sought to be expropriated based on its current tax declaration, it may immediately take possession of the property in accordance with Section 19, Republic Act No. 7160. Issue: WON petitioner may immediately take possession of the property subject of expropriation by virtue of its power of eminent domain. Ruling: Petitioner has the irrefutable right to exercise its power of eminent domain. It being a local government unit, the basis for its exercise is granted under Section 19 of Rep. Act No. 7160. The requisites for authorizing immediate entry are as follows: (1) the filing of a complaint for expropriation sufficient in form and substance; and (2) the deposit of the amount equivalent to 15% of the fair market value of the property to be expropriated based on its current tax declaration. Upon compliance with these requirements, the issuance of a writ of possession becomes ministerial.

ACADCOM 2010; Contributors: Gene Geocaniga, Jarissa Guiani, Darlene Magabilen TAU MU Page 110 of 179

TAU MU TAU MU TAU MU TAU MU TAU MU TAU MU TAU MU TAU MU TAU MU

Facts: Petitioner, Capitol Steel, is a domestic corporation which owns 65 parcels of land located in the Municipality of Tagoloan, Province of Misamis Oriental. PHIVIDEC, is a governmentowned and controlled corporation organized and existing under PD 538, as amended, which is vested with governmental and proprietary functions including the power of eminent domain for the purpose of acquiring rights of way or any property for the establishment or expansion of the Phividec Industrial Areas. The properties of Capitol Steel were identified as the most ideal site for the Mindanao International Container Terminal Project (MICTP), a PHIVIDEC project which involves the phased production of an 800meter berth and the acquisition of port equipment to handle the volume of seaborne break-bulk and container traffic in Mindanao. Hence, PHIVIDEC filed an expropriation case. The trial court issued a writ of possession in favor of PHIVIDEC. Due, however, to the unauthorized engagement by PHIVIDEC of the legal services of a private lawyer, the expropriation case was dismissed. Capitol Steel requested the Technical Committee on Real Property Valuation (TCRPV) of the BIR for a revaluation of its properties. TCPRV issued a resolution fixing the “reasonable and realistic zonal valuation” of the properties at Php700 per square meter. PHIVIDEC informed Capitol Steel that it would file anew an expropriation case and that it had deposited Php116,563,500 in the name of Capitol Steel.PHIVIDEC further informed Capitol Steel that

TAU MU TAU MU

CAPITOL STEEL vs. PHIVIDEC 510 SCRA 590 (2006)

The Fraternal Ateneo de Davao

TAU MU TAU MU

In the case at bar, petitioner avers that the Amended Complaint it filed complies with both requisites, thus entitling it to a writ of possession as a matter of right and the issuance thereof becoming ministerial on the part of the lower court even without any hearing. On the other hand, private respondents allege that the Amended Complaint is not sufficient in form and substance since it failed to allege compliance with the mandatory requirements for the exercise of the power of eminent domain for purposes of socialized housing. The Court finds the Amended Complaint sufficient in form and substance, and the amount of Php2,809,696.50 deposited with the Regional Trial Court of Iloilo is equivalent to 15% of the fair market value of the property sought to be expropriated per current tax declaration.. For a writ of possession to issue, only two requirements are required: the sufficiency in form and substance of the complaint and the required provisional deposit. In fact, no hearing is required for the issuance of a writ of possession. The sufficiency in form and substance of the complaint for expropriation can be determined by the mere examination of the allegations of the complaint. In this case, the sufficiency of the Amended Complaint was further confirmed by public respondent when he set the case for pretrial and hearing.

KITY

TAU MU TAU MU TAU MU TAU MU TAU MU TAU MU

CONSTITUTIONAL LAW II Order of Saint Thomas More Atty. Philip John Pojas/Atty. Rovyne G. Jumao-as, RN University College of Law

the total amount deposited represents the zonal value of the properties, and may be withdrawn at any time. PHIVIDEC filed an Urgent Motion for the Issuance of a Writ of Possession, to which it attached a Certificate of Availability of Funds, and Certifications from the Landbank and the DBP that it deposited the total amount of Php116,563,500 required under RA 8974. Issue: WON the appellate court erred in ordering the RTC to issue a writ of possession in favor of respondent for the expropriation of the property. Ruling: After a writ of possession was issued in favor of respondent in the first expropriation case, respondent commenced the construction of infrastructure buildings and container port terminals. Possession of the properties has since remained with respondent, with the MICTP now complete and fully operational. When the second expropriation case was re-filed, R.A. 8974, which provides for substantive requirements before a writ of possession is issued, was already in force and in efect. Under R.A. 8974, the requirements for authorizing immediate entry in expropriation proceedings involving real property are: (1) the filing of a complaint for expropriation sufficient in form and substance; (2) due notice to the defendant; (3) payment of an amount equivalent to 100% of the value of the property based on the current relevant zonal valuation of the BIR including payment of the value of the improvements and/or structures if any, or if no such valuation is available and in cases of utmost urgency, the payment of the profered value of the property to be seized; and (4) presentation to the court of a certificate of availability of funds from the proper officials. Upon compliance with the requirements, a petitioner in an expropriation case, in this case the respondent, is entitled to a writ of possession as a matter of right and it becomes the ministerial duty of the trial court to forthwith issue the writ of possession. No hearing is required and the court neither exercises its discretion or judgment in determining the amount of the provisional value of the properties to be expropriated as the legislature has fixed the amount under Section 4 of R.A. 8974. The payment of the provisional value as a prerequisite to the issuance of a writ of possession difers from the payment of just compensation for the expropriated property. While the provisional value is based on the current relevant zonal valuation, just compensation is based on the prevailing fair market value of the property. There is no need for the determination with reasonable certainty of the final amount of just compensation before the writ of possession may be issued. -EXPROPRIATION BY LGU’SHEIRS OF SUGUITAN vs. CITY 328 SCRA 137 (2000) Facts: In 1994, the Sangguniang Panglungsod ng Mandaluyong City issued a resolution authorizing the mayor to institute expropriation proceedings

ACADCOM 2010; Contributors: Gene Geocaniga, Jarissa Guiani, Darlene Magabilen TAU MU Page 111 of 179

over the lot of Saguitan. Saguitan filed a motion to dismiss arguing that the Local Government Code requires an ordinance, not a mere resolution. The trial court, however, denied the motion reasoning that an ordinance would only be required to appropriate funds in payment of the expropriated property once the trial court has determined its value. Is the court correct?

TAU MU TAU MU TAU MU TAU MU TAU MU TAU MU

Facts: Respondent filed a complaint for expropriation against petitioners. Respondent needed petitioners' 16,256 sq. m. idle property at the junction of the North Expressway, MalhacanIba-Camalig main road artery and the MacArthur Highway. It planned to use it to establish a common public terminal for all types of public

TAU MU

FRANCIA Jr. vs. MUN. OF MEYCAUAYAN, BULACAN 549 SCRA 424 (2008)

TAU MU

Held: The applicable law as to the point of reckoning for the determination of just compensation is the Local Government Code. The 1997 Rules on Civil Procedure cannot prevail over RA No. 7160, which is a substantive law. However, since the parties voluntarily agreed to be bound by the report of the commission and approved by the trial court, said valuation of the property has the force of law between the parties and should be complied with in good faith.

TAU MU

Facts: On Sept. 17, 1993, the City of Cebu filed expropriation proceedings against respondents who owned parcels of land which the former wanted to use for the construction of a public road. On Sept. 21, 1994, the court issued a writ of possession in favor of the City. Sec. 19 of the Local Government Code of 1991 provides that just compensation shall be determined as of the time of actual taking. However, Sec. 4, Rule 67 of the 1997 Rules of Procedure provides that just compensation shall be determined at the time of the filing of the complaint for expropriation. Which shall prevail?

TAU MU TAU MU

CITY OF CEBU vs. DEDAMO 387 SCRA 754 May 7, 2002

The Fraternal Ateneo de Davao

TAU MU TAU MU

Held: No. Under the Local Government Code, the exercise of eminent domain must comply with the following requisites: 1) an ordinance enacted by the council authorizing the chief executive; 2) the power is exercised for public use, purpose or welfare, or for the benefit of the poor and the landless; 3) payment of just compensation; and 4) a definite ofer was previously made which was not accepted by the property owner. An examination of the law shows that an ordinance is necessary to authorize the filing of the complaint with the proper court since, beginning at that point, the power of eminent domain stats to be exercised. The determination of the award of just compensation is but the last stage which cannot be arrived at without an initial finding by the court that plaintif has a right to take property.

KITY

TAU MU TAU MU TAU MU TAU MU TAU MU TAU MU

CONSTITUTIONAL LAW II Order of Saint Thomas More Atty. Philip John Pojas/Atty. Rovyne G. Jumao-as, RN University College of Law

utility vehicles with a weighing scale for heavy trucks. Petitioners denied that the property sought to be expropriated was raw land. It was in fact developed and there were plans for further development. For this reason, respondent’s ofer price of Php2,333,500 was too low. After trial, the RTC ruled that the expropriation was for a public purpose. The construction of a common terminal for all public utility conveyances (serving as a two-way loading and unloading point for commuters and goods) would improve the flow of vehicular traffic during rush hours. Moreover, the property was the best site for the proposed terminal because of its accessibility. Issue: WON prior determination of the existence of a public purpose was not necessary for the issuance of a writ of possession in expropriation cases. Ruling: Section 19 of Republic Act 7160 provides that: “Section 19. Eminent Domain. ― A local government unit may, through its chief executive and acting pursuant to an ordinance, exercise the power of eminent domain for public use, or purpose, or welfare for the benefit of the poor and the landless, upon payment of just compensation, pursuant to the provisions of the Constitution and pertinent laws; Provided, however, That the power of eminent domain may not be exercised unless a valid and definite ofer has been previously made to the owner, and that such ofer was not accepted; Provided, further, That the local government unit may immediately take possession of the property upon the filing of the expropriation proceedings and upon making a deposit with the proper court of at least 15% of the fair market value of the property based on the current tax declaration of the property to be expropriated; Provided, finally, That, the amount to be paid for the expropriated property shall be determined by the proper court, based on the fair market value at the time of the taking of the property.” Before a local government unit may enter into the possession of the property sought to be expropriated, it must (1) file a complaint for expropriation sufficient in form and substance in the proper court and (2) deposit with the said court at least 15% of the property's fair market value based on its current tax declaration. The law does not make the determination of a public purpose a condition precedent to the issuance of a writ of possession. i. NECESSITY Questions of necessity or wisdom are essentially political when decided by the national legislature and are usually not subject to judicial review [Cruz, Constitutional Law 2007, p.65]. But when exercised by a delegate, the determination of whether there is genuine necessity for the exercise is a justiciable question [Nachura, Outline Reviewer in Political Law 2006, p. 49]. The issue of the necessity of the expropriation is a matter properly addressed to the RTC in the

ACADCOM 2010; Contributors: Gene Geocaniga, Jarissa Guiani, Darlene Magabilen TAU MU Page 112 of 179

course of the expropriation proceedings. If the property owner objects to the necessity of the takeover, he should say so in his Answer to the Complaint. The RTC has the power to inquire into the legality of the exercise of the right of eminent domain and to determine whether there is a genuine necessity for it. [Nachura, Outline Reviewer in Political Law 2006, p. 49].

TAU MU TAU MU TAU MU TAU MU TAU MU TAU MU

1. The amount of just compensation 2. The choice of the property (Note: that private property should be the last resort) 3. To determine the necessity of the taking 4. To determine the "public use" character of the taking. However, if the expropriation is pursuant to a specific law passed by Congress,

TAU MU

Matters under Judicial review of the exercise of the power of eminent domain (FYI: Res Judicate does not apply)

TAU MU

PROCEDURE: 1. Expropriating authority files a complaint for expropriation against the landowner; 2. Expropriating authority must deposit in an amount equivalent to the assessed value of the property for the purposes of taxation in the bank account of the land owner. (This is what you call the provisional compensation, the basis is the assessed value for purposes of taxation.) 3. After due notice to the land owner that the amount has been deposited, the government can now take possession because the court will issue a writ of possession after the provisional compensation has been deposited.

TAU MU

Property already devoted to public use is still subject to expropriation, provided this is done directly by the national legislature or under a specific grant of authority to the delegate. A mere general authority may not suffice. In such a case, the courts will have authority to inquire into the necessity of the expropriation and, finding none, refuse to allow it [Cruz, Constitutional Law 2007, p.68].

TAU MU TAU MU

Even services may be subject to eminent domain [Nachura, Outline Reviewer in Political Law 2006, p. 50].

The Fraternal Ateneo de Davao

TAU MU TAU MU

j. PRIVATE PROPERTY Anything that can come under the dominion of man is subject to expropriation. This will include real and personal, tangible and intangible properties [Cruz, Constitutional Law 2007, p.67]. All private property capable of ownership may be expropriated, except money and choses in action [Nachura, Outline Reviewer in Political Law 2006, p. 50]. Expropriation of money would be a futile act because of the requirement for the payment of just compensation, usually also in money. A chose in action is “a personal right not reduced into possession but recoverable by a suit at law, a right to receive, demand or recover a debt, demand or damages on a cause of action ex contractu or for a tort or omission of duty.” It is essentially conjectural both as to its validity and its value [Cruz, Constitutional Law 2007, p.68].

KITY

TAU MU TAU MU TAU MU TAU MU TAU MU TAU MU

CONSTITUTIONAL LAW II Order of Saint Thomas More Atty. Philip John Pojas/Atty. Rovyne G. Jumao-as, RN University College of Law

the courts cannot question the public use character of the taking. 5. Authority of the expropriating authorities; 6. Whether the formalities have been complied with. Note: If the power is being exercised by an agency possessing power delegated by Congress, meaning the LGU's, President, Public utilities, all issues are subject to judicial review. Including the necessity and the purpose of public taking. 

Issues which are NOT subject to judicial review: 1. When the power is exercised by Congress and it involves the issue of necessity. It is the Congress will decide if the taking is necessary. 2. The public use character of the purpose of the taking property. If the Congres will decide that this is the purpose and this is for public use, the court cannot look into that.

CASE: JUDICIAL REVIEW DE KNECHT vs. BAUTISTA 100 SCRA 660 (1980) Facts: The plan to extend EDSA to Roxas Boulevard to be ultimately linked to the Cavite Coastal Road Project, originally called for the expropriation of properties along Cuneta Avenue in Pasay City. Later on, however, the Ministry of Public Highways decided to make the proposed extension pass through Fernando Rein and Del Pan Streets. Because of the protests of residents of the latter, the Commission on Human Settlements recommended the reversion to the original plan, but the Ministry argued the new route withh save the government P2 million. The government filed expropriation proceedings against the owners of Fernando Rein and Del Pan streets, among whom was petitioner. Held: The choice of Fernando Rein and Del Pan streets is arbitrary and should not receive judicial aprpoval. The Human Settlements Commission concluded that the cost factor is so minimal that it can be disregarded in making a choice between the two lines. The factor of functionality strongly militates against the choice of Fernando Rein and Del Pan streets, while the factor of social and economic impact bears grievously on the residents of Cuneta Avenue. While the issue would seem to boil down to a choice between people, on one hand, and progress and development, on the other, it is to be remembered that progress and development are carried out for the benefit of the people. Point of reference for valuating a piece of property: General rule: The value must be that as of the time of the filing of the complaint for expropriation.

ACADCOM 2010; Contributors: Gene Geocaniga, Jarissa Guiani, Darlene Magabilen TAU MU Page 113 of 179

Exception: When the filing of the case comes later than the time of taking and meanwhile the value of the property has increased because of the use to which the expropriator has put it, the value is that of the time of the earlier taking. BUT if the value increased independently of what the expropriator did, then the value is that of the latter filing of the case.

TAU MU TAU MU TAU MU TAU MU TAU MU TAU MU TAU MU

Consequential Damages/Consequential Benefits Consequential damages consist of injuries directly caused on the residue of the private property taken by the reason of expropriation. Where, for example, the expropriator takes only a part of a parcel of land, leaving the remainder with an odd shape or area as to be virtually unusable, the owner can claim consequential damages. On the other hand, if the remainder is as a result of the expropriation placed in a better location, such as fronting a street where it used to be an interior lot, the owner will enjoy consequential benefits which should be deducted from the consequential damages. Consequential benefits, like consequential damages, must be direct and particular and not merely shared with the rest of the properties in the area, as where there is a general appreciation of lad values because of the public use to which the condemned properties are devoted [Cruz, Constitutional Law 2007, p.79].

TAU MU

Title to the property does not pass until after payment, except in agrarian reform [Nachura, Outline Reviewer in Political Law 2006, p. 57].

TAU MU

When a parcel of land is taken by eminent domain, the owner of the fee is not necessarily the only person who is entitled to compensation. Every person having interest at law or in equity in the land taken is entitled to share in the award. If a person claiming an interest in the land sought to be condemned is not made a party, he is given the right to intervene and lay claim to the compensation [Bernas, The 1987 Philippine Constitution: A Comprehensive Reviewer, p. 108].

TAU MU TAU MU

Owner (Concept) According to Knecht vs. CA, the term “owner” as applied in eminent domain cases refers to all those who have lawful interest in the property to be condemned, including a mortgagee, a lessee and a vendee in possession under an executory contract [Cruz, Constitutional Law 2007, p.78].

The Fraternal Ateneo de Davao

TAU MU TAU MU

Market Value (Definition) “Market value” has been described in a variety of ways. It is the “price fixed by the buyer and seller in the open market in the usual and ordinary course of legal trade and competition; the price and value of the article established or shown by sale, public or private, in the ordinary way of business; the fair value of property as between one who desires to purchase and one who desires to sell; the current price; the general or ordinary price for which property may be sold in that locality” [Bernas, The 1987 Philippine Constitution: A Comprehensive Reviewer, p. 104].

KITY

TAU MU TAU MU TAU MU TAU MU TAU MU TAU MU

CONSTITUTIONAL LAW II Order of Saint Thomas More Atty. Philip John Pojas/Atty. Rovyne G. Jumao-as, RN University College of Law

In such a case, the owner is not restricted to payment of the market value of the portion actually taken. In addition to the market value of the portion taken, he is also entitled to payment of consequential damages, if any to the remaining part of the property. At the same time, from the total compensation must be deducted the value of consequential benefits, if any, provided consequential benefits shall not exceed consequential damages [Nachura, Outline Reviewer in Political Law 2006, p. 51]. If the consequential benefits exceed the consequential damages, these items should be disregarded altogether as the basic value of the property should be paid in every case [Cruz, Constitutional Law 2007, p.79]. Reasonable Period Just compensation includes not only the correct determination of the amount to be paid to owner of the land but also the payment of the land within a reasonable period of time from its taking [Bernas, The 1987 Philippine Constitution: A Comprehensive Reviewer, p. 105]. Without prompt payment, compensation cannot be considered “just,” for the property owner is made to sufer the consequence of being immediately deprived of his land while being made to wait for a decade or more before actually receiving the amount necessary to cope with his loss. To allow the taking of the landowner’s properties and in the meantime leave them empty-handed by withholding payment of just compensation while the government speculates on whether or not it will pursue expropriation, or worse, for government to subsequently decide to abandon the property and return it to the landowner when it has already been rendered useless by force majeure, it is undoubtedly an oppressive exercise of eminent domain that must never be sanctioned [Nachura, Outline Reviewer in Political Law 2006, p. 54]. Property Assessment The property taken should be assessed as of the time of the taking, which usually coincides with the commencement of the expropriation proceedings. Where entry precedes the filing of the complaint for expropriation, the assessment should be made as of the time of the entry [Cruz, Constitutional Law 2007, p.83]. The general rule is that the value must be that as of the time of the filing of the complaint for expropriation (Section 4, Rule 67, Rules of Court). Moreover, the filing of the case generally coincides with the taking. When, however, the filing of the case comes later than the time of taking and meanwhile the value of the property has increased because of the use to which the expropriator has put it, the value is that of the time of the earlier taking. Otherwise, the owner would gain undeserved profit. But if the value increased independently of what the expropriator did, then the value is that of the later filing of the case [Bernas, The 1987 Philippine Constitution: A Comprehensive Reviewer, p. 109].

ACADCOM 2010; Contributors: Gene Geocaniga, Jarissa Guiani, Darlene Magabilen TAU MU Page 114 of 179

When eminent domain is exercised by a local government unit, the “amount to be paid for the expropriated property shall be determined by the proper court, based on the fair market value at the time of the taking of the property (Sec. 19, RA7160) precisely because the Rules of Court cannot prevail over RA7160, a substantive law [Nachura, Outline Reviewer in Political Law 2006, p. 56].

TAU MU TAU MU TAU MU TAU MU TAU MU TAU MU

Right of Owner in case of Non-payment of Just Compensation As a rule, non-payment of just compensation in an expropriation proceeding does not entitle the private landowner to recover possession of the expropriated lots, but only to demand payment of the fair market value of the property.

TAU MU

In some expropriation cases, the Court imposed an interest of 12% per annum on the just compensation due the landowner. It must be stressed, however, that in these cases, the imposition of interest was in the nature of damages for delay in payment which, in efect, makes the on the part of the government one of forbearance. It follows that the interest in the form of damages cannot be applied where there was prompt and valid payment of just compensation. Conversely, where there was delay in tendering a valid payment of just compensation, imposition of interest is in order [Nachura, Outline Reviewer in Political Law 2006, p. 56].

TAU MU

When there is delay in the payment of just compensation, the owner is entitled to payment of interest, if claimed; otherwise, interest is deemed waived. In NAPOCOR vs. Angas, the Supreme Court held that the interest due the property owner is at the rate of 6% per annum, prescribed in Article 2209 of the Civil Code, and not 12% per annum under Central Bank Circular No. 416, because the latter applies to loans or forbearance of money, goods or credits, or judgments involving such loans or forbearance of money goods or credits. The kind of interest involved here is by way of damages, hence Art. 2209 of the Civil Code applies.

TAU MU TAU MU

Form of Compensation As explicitly provided by Sec. 16(e), RA 6657, the deposit of compensation must be in “cash” or in “Land Bank bonds,” not in any other form, and certainly not in a “trust account.” Entitlement of Owner to Interest

The Fraternal Ateneo de Davao

TAU MU TAU MU

Determination of Just Compensation The principal criterion in determining just compensation is the character of the land at the time of the taking. The tax declaration is only one of the factors to be used in determining the market value of the property for purposes of arriving at the amount to be paid by way of just compensation [Nachura, Outline Reviewer in Political Law 2006, p. 55]

KITY

TAU MU TAU MU TAU MU TAU MU TAU MU TAU MU

CONSTITUTIONAL LAW II Order of Saint Thomas More Atty. Philip John Pojas/Atty. Rovyne G. Jumao-as, RN University College of Law

However, in Republic vs. Vicente Lim, the Supreme Court said that the facts of the case do not justify the application of the rule. It was held that where the government failed to pay just compensation within five (5) years from the finality of the judgment in the expropriation proceedings, the owners concerned shall have the right to recover possession of their property [Nachura, Outline Reviewer in Political Law 2006, p.57]. Writ of Possession The issuance of the writ of possession becomes ministerial upon: 1. Filing of a complaint for expropriation sufficient in form and substance; 2. Upon deposit made by the government of the amount equivalent to fifteen percent (15%) of the fair market value of the property sought to be expropriated per current tax declaration [Nachura, Outline Reviewer in Political Law 2006, p. 58]. Plaintif’s right to dismiss the complaint in eminent domain In expropriation cases, there is no such thing as the plaintif’s “matter-of-right” to dismiss the complaint, precisely because the landowner may have already sufered damages at the start of the taking. The plaintif’s right to dismiss the complaint has always been subject to Court approval and to certain conditions [Nachura, Outline Reviewer in Political Law 2006, p. 58]. Right to repurchase or re-acquire the property The property owner’s right to repurchase the property depends upon the character of the title acquired by the expropriator, e.g. if land is expropriated for a particular purpose with the condition that when that purpose is ended or abandoned, the property shall revert to the former owner, then the former owner can reacquire the property [Nachura, Outline Reviewer in Political Law 2006, p. 58]. Temporary takeover of private business or utility Temporary takeover by the government extends only to the operation of the business and not to the ownership thereof. As such, the government is not required to compensate the private entityowner of the said business as there is not transfer of ownership, whether permanent or temporary. The private entity-owner afected cannot, likewise, claim just compensation for the use of said business and its properties, as the temporary takeover by the government is in the exercise of the police power and not the power of eminent domain [Nachura, Outline Reviewer in Political Law 2006, p. 59].

TAU MU

RA7279 (Urban Development and Housing Act of 1992) Under RA7279, lands for socialized housing are to be acquired in the following order: 1. Government lands 2. Alienable lands of the public domain

ACADCOM 2010; Contributors: Gene Geocaniga, Jarissa Guiani, Darlene Magabilen TAU MU Page 115 of 179

3. Unregistered, abandoned, or idle lands 4. Lands within the declared Areas for Priority Development, Zonal Improvement Program sites, Slum Improvement and Resettlement sites which have not yet been acquired 5. BLISS site which have not yet been acquired 6. Privately owned lands

TAU MU TAU MU TAU MU TAU MU TAU MU TAU MU TAU MU

While the principle of res judicata does not denigrate the right of the State to exercise eminent domain, it does not apply to specific issues decided in a previous case. For example, a final judgment dismissing an expropriation suit on the ground that there was not prior ofer precluded another suit raising the same issue; it cannot, however, bar the State or its agent from thereafter complying with this requirement, as prescribed by law, and subsequently exercising its power of eminent domain over the same

TAU MU

Res Judicata The principle of res judicata does not bar the right of the State or its agent to expropriate private property. “The very nature of the eminent domain, as an inherent power of the State, dictates that the right to exercise the power be absolute and unfettered by a prior judgment or res judicata. The scope of eminent domain is plenary and, like police power, can ‘reach every form of property which the State might need for public use.’”

TAU MU

Instance where exercise of the power of eminent domain is not subject to judicial review When land is expropriated for subdivision and resale for social justice purposes directly by the legislature and not through an inferior agency of the state, the necessity and public purpose of the taking are not subject to judicial review. Article XIII, Section 4, constitutes a textual commitment of discretion on the subject to the legislature. However, if the power is exercised by an agency possessing power delegated by the legislature or when the issue is the adequacy of the compensation, judicial review is still proper.

TAU MU TAU MU

The exercise of the power of eminent domain is subject to judicial review. The following aspects of the exercise of the power have been subject to judicial scrutiny: 1. The adequacy of the compensation 2. The necessity of the taking 3. The “public use” character of the purpose of the taking

The Fraternal Ateneo de Davao

TAU MU TAU MU

The mode of expropriation is subject to two conditions, namely: 1. It shall be resorted to only when the other modes of acquisition have been exhausted 2. Parcels owned by small property owners are exempt from such acquisition [Nachura, Outline Reviewer in Political Law 2006, p. 60] Judicial Review [Bernas, The 1987 Philippine Constitution: A Comprehensive Reviewer, p.112]

KITY

TAU MU TAU MU TAU MU TAU MU TAU MU TAU MU

CONSTITUTIONAL LAW II Order of Saint Thomas More Atty. Philip John Pojas/Atty. Rovyne G. Jumao-as, RN University College of Law

property. (Municipality of Paranaque vs. VM Realty Corporation) Expropriation as substitute for the enforcement of a valid contract Expropriation lies only when it is made necessary by the opposition of the owner to the sale or by the lack of any agreement as to price. Where there is a valid and subsisting contract, between the owners of the property and the expropriating authority, there is no reason for the expropriation. The State may not enter into a contract which in efect binds it not to exercise the power of eminent domain. Like police power and the power of taxation, the power of eminent domain is inalienable. “There can be no right to restrain by contract the power of eminent domain, it must also of necessity to follow that any contract by which it was sought to accomplish that result would be inefficacious for want of power.” Exercise of Eminent Domain by Local Government Unit By express legislative authority granted by Congress in Section 19, RA7160, local government units may expropriate private property for public use, or purpose, or welfare, for the benefit of the poor and the landless. Thus, in Moday vs. CA, the Supreme Court held that the Sangguniang Panlalawigan of Agusan del Sur was without authority to disapprove Bunawan Municipal Reso 43-89 because clearly, the Municipality of Bunawan has the authority to exercise the power of eminent domain and its Sangguniang Bayan the capacity to promulgate the assailed resolution [Nachura, Outline Reviewer in Political Law 2006, p. 52]. Ordinance (Requisite for Valid Expropriation) In Municipality of Paranaque vs. VM Realty Corporation, the Supreme Court declared that there was lack of compliance with Sec. 19, RA7160, where the Municipal Mayor filed a complaint for eminent domain over two parcels of land on the strength of a resolution passed by the Sangguniang Bayan, because what is required by law is an ordinance. Essential requisites for LGU to validly exercise the power of eminent domain (Section 19, RA7160): e. An ordinance is enacted by the local legislative council authorizing the local chief executive, in behalf of the LGU, to exercise the power of eminent domain or pursue expropriation proceedings over a particular private property; f. The power of eminent domain is exercised for public use, purpose or welfare, or for the benefit of the poor and the landless; g. There is payment of just compensation as required under Sec. 9, Article III of the Constitution, and other pertinent laws; h. A valid and definite ofer has been previously made to the owner of the property sought to be expropriated, but said ofer was not accepted.

ACADCOM 2010; Contributors: Gene Geocaniga, Jarissa Guiani, Darlene Magabilen TAU MU Page 116 of 179

Limitations on power of eminent domain of LGU: (Estate of JBL Reyes vs. City of Manila) Since local governments possess only delegated power of eminent domain, it is subject to limitations specified by law on the delegated power. Thus there are mandatory limits with respect to: 3. The order of priority in acquiring land for socialized housing; and 4. The resort to expropriation proceedings as a means to acquiring it.

TAU MU TAU MU TAU MU TAU MU TAU MU TAU MU TAU MU

LIMITATIONS TO THE NON-IMPAIRMENT CLAUSE 1. If the law is a valid exercise of police power it will prevail over the contract.  Into each contract are read the provisions of existing law, and always a reservation of the police power as long as the agreement deals w/ a matter afecting the public welfare.  The legislature cannot bargain away the police power through the medium of contract. Neither may the private parties fetter the legislative authority by contracting on matters that are essentially within the power of the lawmaking body to regulate.

TAU MU

Q: Is the protection provided in this provision absolute? A: NO. There are instances when contracts valid @ the time of their conclusion may become invalid, or some of their provisions may be rendered inoperative or illegal, by virtue of supervening legislation.

TAU MU

NON-IMPAIRMENT CLAUSE  The PURPOSE of this clause is to safeguard the integrity of valid contractual agreements against the warranted interference by the state.  Contracts should be respected by the legislature and not tampered w/ by subsequent laws that will 1. change the intention of the parties or 2. will modify the parties’ rights and obligations. The will of the obligor and the obligee must be observed.  This right is a limitation to Congress’ power to enact laws.

TAU MU TAU MU

Section 10. No law impairing the obligation of contracts shall be passed.

The Fraternal Ateneo de Davao

TAU MU TAU MU

Private lands rank last in the order of priority for purposes of socialized housing. In the same vein, expropriation proceedings may not be resorted to only after the other modes of acquisition are exhausted. Compliance with these conditions are mandatory because these are the only safeguards of often time helpless owners of private property against what may be tyrannical violation of due process when their property is forcibly taken from them allegedly for public use.

KITY

TAU MU TAU MU TAU MU TAU MU TAU MU TAU MU

CONSTITUTIONAL LAW II Order of Saint Thomas More Atty. Philip John Pojas/Atty. Rovyne G. Jumao-as, RN University College of Law

2. The freedom of contract w/c is constitutionally protected is freedom to enter into LAWFUL contracts.  Contracts that contravene public policy are not lawful. 3. The inherent powers of taxation and imminent domain may also limit the nonimpairment clause.  A lawful tax on a new subject or an increased tax on an old one does not interfere w/ a contract or impair its obligation w/i the meaning of the constitution. Even though such taxation may afect particular contracts, as it may increase the debt of one person and lessen the security of another, or may impose additional burdens upon one class and release the burdens of another, still the tax must be paid unless prohibited by the constitution, nor can it be said that it impairs the obligation of any contract in its true legal sense.  Where a law grants a tax exemption in exchange for a valuable consideration, such exemption is considered a contract and cannot be repealed because of this section [CASSANOVA vs. HORD].  All other tax exemptions are not contractual and so may be revokes @ will by the legislature. 4. Freedom of Speech. Expression and of the Press (Sec. 4) 5. Right to Assemble and Petition (Sec. 4). 6. Freedom of Religion (Sec. 5). CONTRACT  any lawful agreement on property or property rights, whether real or personal, tangible or intangible  the agreement may be executed or executory  the PARTIES may be private persons only, natural or artificial, or private persons and the gov’t. or its agencies.  It does not cover licenses NOT COVERED BY SEC. 10 1. licenses – because they only involve grants of privileges that are essentially revocable 2. marriage contract – because marriage is regarded as more than a an agreement between the spouses as it is a social institution subject @ all times to regulation by the legislature and to change of the original conditions 3. public office EXCEPTION: where the salary has already been earned, it will be deemed vested property right that cannot be withdrawn or reduced by retroactive legislation LAW  includes statutes enacted by the national legislature, executive orders & administrative regulations promulgated under a valid power, and municipal ordinances passes by local legislative bodies

ACADCOM 2010; Contributors: Gene Geocaniga, Jarissa Guiani, Darlene Magabilen TAU MU Page 117 of 179

 it does NOT include judicial decisions or adjudication made by administrative bodies in their QJ functions  TO IMPAIR it (law) must retroact so as to afect existing contracts concluded before its enactment  NO IMPAIRMENT if it (law) is made to operate prospectively only --- to cover contracts entered into after its enactment

TAU MU TAU MU TAU MU TAU MU TAU MU

Coverage: Any lawful contract agreement; any contract which involves property, i.e. intellectual property, personal, movable, etc.; franchises or

TAU MU

2. It can be invoked against a law, an executive act, an ordinance, but not against executive officials performing judicial actions

TAU MU

Note: 1. A mere change in PROCEDURAL REMEDIES which does not change the substance of the contract, and which still leaves an efficacious remedy for enforcement does NOT impair the obligation of contracts.

TAU MU

 BUT in the case of REMEDIES, there will be impairment only when all of them are withdrawn, w/ the result that either of the parties will be unable to enforce his rights under the original agreement.  There is NO IMPAIRMENT as long as substantial and efficacious remedy remains. And this rule holds true even if the remedy retained is the most difficult to employ and it is the easier ones that are withdrawn [MANILA TRADING CO. vs. REYES (62 Phil. 461)].

TAU MU

Q: When is there impairment? A: 1. When the law changes the terms of the contract; 2. When the law imposes new obligations or conditions; 3. When the law dispenses w/ those agreed upon; or 4. When the law withdraws the remedies for the enforcement of the rights of the parties.

TAU MU TAU MU

IMPAIRMENT  is anything that diminishes the efficacy of the contract  the degree of diminution is immaterial as long as the rights of either party are changed to his prejudice there is impairment of the obligation of the contract.  To impair, the law must retroact so as to afect existing contracts concluded before its enactment. There will be no impairment if the law is made to operate prospectively only, to cover contracts entered into after its enactment.

The Fraternal Ateneo de Davao

TAU MU TAU MU

OBLIGATION  is the vinculum juris --- the tie that binds the parties to each other  it refers to the law/duty that binds the parties to perform their undertaking/agreement according to its terms and intent

KITY

TAU MU TAU MU TAU MU TAU MU TAU MU TAU MU

CONSTITUTIONAL LAW II Order of Saint Thomas More Atty. Philip John Pojas/Atty. Rovyne G. Jumao-as, RN University College of Law

charters granted to a private person; timber licenses. What can be impaired: Licenses issued by the Government, licenses for exploiting mining, marbles; marriage contracts; public office The following are superior to the non impairment clause: 1. Police Power; (reclassification of property) 2. Eminent domain; (lessor for lease but expropriated) 3. Taxation power 4. Freedom of religion *The right to non-impairment of contracts cannot be invoked against judicial acts.

CASES: NON-IMPAIRMENT OF CONTRACTS UNITED BF HOMEOWNERS vs. CITY MAYOR 515 SCRA 1 (2007) Facts: BF Homes Parañaque Subdivision is the largest subdivision in the country. In 1997, the Municipal Council of Parañaque enacted a Municipal Ordinance entitled, "An Ordinance Prescribing the Comprehensive Land Use Plan & Zoning of the Municipality of Parañaque Pursuant to the Local Government Code of 1991 and Other Pertinent Laws." Sections 11.5 and 11.6 of the Ordinance seeks to reclassify El Grande and Aguirre Avenues in BF Homes Parañaque from residential to commercial areas. Petitioners filed a petition for prohibition with an application for TRO and preliminary injunction. They questioned the constitutionality of certain sections of the municipal ordinance and alleged that the reclassification of certain portions of BF Homes Parañaque from residential to commercial zone is unconstitutional because it amounts to impairment of the contracts between the developer of BF Homes Parañaque and the lot buyers. Petitioners cited the annotation on the lot buyers’ titles which provides that "the property shall be used for residential purposes only and for no other purpose." On the other hand, public respondents alleged that the passage of Municipal Ordinance is a valid exercise of police power by the Municipal Council of Parañaque and that such ordinance can nullify or supersede the contractual obligations entered into by the petitioners and the developer. Meanwhile, EL ACTO claimed that its members are lot owners, residents, and operators of commercial establishments along El Grande and Aguirre Avenues in BF Homes Parañaque, who will be afected if Municipal Ordinance is declared unconstitutional. EL ACTO asserted that Municipal Ordinance is a valid exercise of police power and that petitioners are guilty of estoppel since petitioners endorsed the opening of many of these commercial establishments in BF Homes Parañaque. EL ACTO further alleged that the instant petition should have been initially filed

ACADCOM 2010; Contributors: Gene Geocaniga, Jarissa Guiani, Darlene Magabilen TAU MU Page 118 of 179

with the Regional Trial Court in accordance with the principle of hierarchy of courts. Issue: WON the ordinance impaired the contractual obligation annotated in homeowners’ titles.

TAU MU TAU MU TAU MU TAU MU TAU MU TAU MU TAU MU TAU MU TAU MU

Facts: In June 1987, Harrison Motors Corporation sold 2 Isuzu Elf trucks to private respondent Rachel Navarro. Petitioner, a known importer, assembler and manufacturer, assembled the 2 trucks using imported component parts. Prior to the sale, Claros represented to Navarro that all the BIR taxes and customs duties for the parts used on the 2 trucks had been paid for. In September 1987, BIR and the LTO entered into a MOA which provided that prior to registration in the LTO of any assembled or re-assembled motor vehicle which used imported parts, a Certificate of Payment should first be obtained from the BIR to prove payment of all taxes required under existing laws. The Bureau of Customs issued Customs Memorandum Order No. 44-87 promulgating rules, regulations and procedure for the voluntary payment of duties and taxes on

TAU MU TAU MU

HARRISON vs. NAVARRO 331 SCRA 202 (2000)

The Fraternal Ateneo de Davao

TAU MU TAU MU

Ruling: The Court has upheld in several cases the superiority of police power over the nonimpairment clause. The constitutional guaranty of non-impairment of contracts is limited by the exercise of the police power of the State, in the interest of public health, safety, morals and general welfare. Contractual restrictions on the use of property could not prevail over the reasonable exercise of police power through zoning regulations. While non-impairment of contracts is constitutionally guaranteed, the rule is not absolute, since it has to be reconciled with the legitimate exercise of police power, i.e., "the power to prescribe regulations to promote the health, morals, peace, education, good order or safety and general welfare of the people." Invariably described as "the most essential, insistent, and illimitable of powers" and "in a sense, the greatest and most powerful attribute of government," the exercise of the power may be judicially inquired into and corrected only if it is capricious, whimsical, unjust or unreasonable, there having been a denial of due process or a violation of any other applicable constitutional guarantee. The State guarantees sanctity of contract and is said to be the "law between the contracting parties," but while it is so, it cannot contravene "law, morals, good customs, public order, or public policy." Above all, it cannot be raised as a deterrent to police power, designed precisely to promote health, safety, peace, and enhance the common good, at the expense of contractual rights, whenever necessary. Hence, the ordinance, as held by the SC, is a legitimate exercise of police power and the reclassification of El Grande and Aguirre Avenues in BF Homes Parañaque is not arbitrary or unreasonable.

KITY

TAU MU TAU MU TAU MU TAU MU TAU MU TAU MU

CONSTITUTIONAL LAW II Order of Saint Thomas More Atty. Philip John Pojas/Atty. Rovyne G. Jumao-as, RN University College of Law

imported motor vehicles assembled by nonassemblers.Pursuant to the MOA between the BIR and the LTO, the BIR issued Revenue Memorandum Order No. 44-87 which provided the procedure governing the processing and issuance of the Certificate of Payment of internal revenue taxes for purposes of registering motor vehicles. In 1988, the BIR, BOC and LTO entered into a tripartite MOA which provided that prior to the registration in the LTO of any locally assembled motor vehicle using imported component parts, a Certificate of Payment should first be obtained from the BIR and the BOC to prove that all existing taxes and customs duties have been paid. On that same year, government agents seized and detained the 2 Elf trucks of respondent after discovering that there were still unpaid BIR taxes and customs duties thereon. The BIR and the BOC ordered Navarro to pay the proper assessments or her trucks would be impounded. She went to Claros to ask for the receipts evidencing payment of BIR taxes and customs duties; however, Claros refused to comply. She then demanded from Claros that he pay the assessed taxes and warned him that he would have to reimburse her should she be forced to pay for the assessments herself. Her demands were again ignored.But wanting to secure the immediate release of the trucks to comply with her business commitments, Navarro paid the assessed BIR taxes and customs duties. Consequently, she returned to petitioner's office to ask for reimbursement, but petitioner again refused. Issue: WON the payment of the taxes specified in the administrative regulations violated the constitutional mandate of non-impairment clause. Ruling: The SC ruled that the Memorandum Orders and the MOA did not impose any additional BIR taxes or customs duties. What Sec. 10, Art. III, of the Constitution prohibits is the passage of a law which enlarges, abridges or in any manner changes the intention of the contracting parties. The Memorandum Orders and the 2 Memoranda of Agreement do not impose any additional taxes which would unduly impair the contract of sale between petitioner and private respondent. Instead, these administrative regulations were passed to enforce payment of existing BIR taxes and customs duties at the time of importation. Administrative rulings and regulations are generally prospective in nature. An inspection of the 2 Memoranda of Agreement however demonstrates that their intent is to enforce payment of taxes on assemblers/manufacturers who import component parts without paying the correct assessments. Thus, although Navarro is the one required by the administrative regulations to secure the Certificate of Payment for the purpose of registration, petitioner as the importer and the assembler/manufacturer of the two 2 Elf trucks is still the one liable for payment of revenue taxes and customs duties. Petitioner's obligation to pay does not arise from the administrative regulations

ACADCOM 2010; Contributors: Gene Geocaniga, Jarissa Guiani, Darlene Magabilen TAU MU Page 119 of 179

TAU MU TAU MU TAU MU TAU MU TAU MU TAU MU TAU MU TAU MU TAU MU

Facts: The instant case involves a rich tract of mineral land known as the "Diwalwal Gold Rush Area." On March 10, 1988, Marcopper Mining Corporation was granted Exploration Permit No. 133 which included the hotly-contested Diwalwal area. Marcopper's acquisition of mining rights over Diwalwal under its EP No. 133 was subsequently challenged where Marcopper's claim was sustained over that of another mining firm, Apex Mining Corporation. In 1991, Republic Act 7076 or the People's Small-Scale Mining Act was enacted. The law established a People's Small-Scale Mining Program to be implemented by the Secretary of the DENR and created the Provincial Mining Regulatory Board under the DENR Secretary's direct supervision and control. The statute also authorized the PMRB to declare and set aside small-scale mining areas subject to review by the DENR Secretary 5 and award mining contracts to small-scale miners under certain conditions. In 1991, DENR Sec. Factoran issued Department Administrative Order No. 66, declaring 729 hectares of the Diwalwal area as non-forest land open to small-scale mining. Subsequently, a petition for the cancellation of EP No. 133 and the admission of a Mineral Production Sharing Arrangement (MPSA) proposal over Diwalwal was filed before the DENR Regional Executive Director. While the RED Mines case was pending, Marcopper assigned its EP No. 133 to petitioner Southeast Mindanao Gold Mining Corporation (SEM), which in turn applied for an integrated MPSA over the land covered by the permit. The Mines and Geosciences Bureau accepted and registered the integrated MPSA application of petitioner. In the meantime, Republic Act 7942, the Philippine Mining Act, was enacted. Provincial Mining Regulatory Board of Davao passed Resolution No. 26 authorizing the issuance of ore transport permits to small-scale miners operating in the Diwalwal mines. Thus, petitioner filed a complaint for damages against the DENR Secretary and PMRB-Davao. SEM alleged that the illegal issuance of the OTPs

TAU MU TAU MU

SMGMC vs. BALITE PORTAL 380 SCRA 145 (2002)

The Fraternal Ateneo de Davao

TAU MU TAU MU

but from the tax laws existing at the time of importation. Hence, even if private respondent already owned the 2 trucks when the Memorandum Orders and Memoranda of Agreement took efect, the fact remains that petitioner was still the one duty-bound to pay for the BIR taxes and customs duties. As between petitioner, who is the importer-assembler/manufacturer, and Navarro, who is merely the buyer, it is petitioner which has the obligation to pay taxes to the BIR and the BOC. Petitioner would be unjustly enriched if Navarro should be denied reimbursement. It would inequitably amass profits from selling assembled trucks even if it did not pay the taxes due on its imported spare parts. Imposing the tax burden on private respondent would only encourage the proliferation of smugglers who scheme to evade taxes by passing on their tax obligations to their unsuspecting buyers.

KITY

TAU MU TAU MU TAU MU TAU MU TAU MU TAU MU

CONSTITUTIONAL LAW II Order of Saint Thomas More Atty. Philip John Pojas/Atty. Rovyne G. Jumao-as, RN University College of Law

allowed the extraction and hauling of P60,000.00 worth of gold ore per truckload from SEM's mining claim. Meanwhile, the DENR Secretary issued Memorandum Order No. 97-03. With this, petitioner filed a special civil action for certiorari, prohibition and mandamus against PMRB-Davao, the DENR Secretary and Balite Communal Portal Mining Cooperative (BCPMC), which represented all the OTP grantees. It prayed for the nullification of the Memorandum Order No. 97-03 on the ground that the "direct state utilization" espoused therein would efectively impair its vested rights under EP No. 133; that the DENR Secretary unduly usurped and interfered with the jurisdiction of the RPA which had dismissed all adverse claims against SEM in the Consolidated Mines cases; and that the memorandum order arbitrarily imposed the unwarranted condition that certain studies be conducted before mining and environmental laws are enforced by the DENR. Issue: WON the non-impairment clause mandated in the Constitution was violated. Ruling: Under no circumstances may petitioner's rights under EP No. 133 be regarded as total and absolute. EP No. 133 merely evidences a privilege granted by the State, which may be amended, modified or rescinded when the national interest so requires. This is necessarily so since the exploration, development and utilization of the country's natural mineral resources are matters impressed with great public interest. Like timber permits, mining exploration permits do not vest in the grantee any permanent or irrevocable right within the purview of the non-impairment of contract and due process clauses of the Constitution, since the State, under its allencompassing police power, may alter, modify or amend the same, in accordance with the demands of the general welfare. Thus, the State may pursue the constitutional policy of full control and supervision of the exploration, development and utilization of the country's natural mineral resources, by either directly undertaking the same or by entering into agreements with qualified entities. The DENR Secretary acted within his authority when he ordered a study of the first option, which may be undertaken consistently in accordance with the constitutional policy enunciated above. Obviously, the State may not be precluded from considering a direct takeover of the mines, if it is the only plausible remedy in sight to the gnawing complexities generated by the gold rush. The State need be guided only by the demands of public interest in settling for this option, as well as its material and logistic feasibility. PICOP RESOURCES vs. BASE METALS 510 SCRA 400 (2006) Facts: In 1987, the Central Mindanao Mining and Development Corporation entered into a Mines Operating Agreement with Banahaw Mining and

ACADCOM 2010; Contributors: Gene Geocaniga, Jarissa Guiani, Darlene Magabilen TAU MU Page 120 of 179

TAU MU TAU MU TAU MU TAU MU TAU MU TAU MU TAU MU TAU MU TAU MU

Ruling: An examination of the Presidential Warranty at once reveals that it simply reassures PICOP of the government's commitment to uphold the terms and conditions of its timber license and guarantees PICOP's peaceful and adequate possession and enjoyment of the areas which are the basic sources of raw materials for its wood processing complex. The warranty covers only the right to cut, collect, and remove timber in its concession area, and does not extend to the utilization of other resources, such as mineral resources, occurring within the concession. The Presidential Warranty cannot be considered a contract distinct from PTLA No. 47 and IFMA No. 35. The SC agreed with the OSG's position that it is merely a collateral undertaking

TAU MU TAU MU

Issue: WON the approval of the application and the issuance of the MPSA of Base Metals will violate the constitutional mandate against impairment of obligation in a contract.

The Fraternal Ateneo de Davao

TAU MU TAU MU

Development Corporation whereby the latter agreed to act as Mine Operator for the exploration, development, and eventual commercial operation of CMMCI's 18 mining claims. Pursuant to the terms of the Agreement, Banahaw Mining filed applications for Mining Lease Contracts over the mining claims with the Bureau of Mines. Banahaw Mining was issued a Mines Temporary Permit authorizing it to extract and dispose of precious minerals found within its mining claims. Upon its expiration, the temporary permit was subsequently renewed thrice by the Bureau of Mines. Since a portion of Banahaw Mining's mining claims was located in petitioner PICOP's logging concession, Banahaw Mining and petitioner PICOP entered into a Memorandum of Agreement, whereby, in mutual recognition of each other's right to the area concerned, petitioner PICOP allowed Banahaw Mining an access/right of way to its mining claims. In 1991, Banahaw Mining converted its mining claims to applications for Mineral Production Sharing Agreements. While the MPSA were pending, Banahaw Mining decided to sell/assign its rights and interests over 37 mining claims in favor of private respondent Base Metals Mineral Resources Corporation. The transfer included mining claims held by Banahaw Mining in its own right as claim owner, as well as those covered by its mining operating agreement with CMMCI. Upon being informed of the development, CMMCI, as claim owner, immediately approved the assignment made by Banahaw Mining in favor of private respondent Base Metals, thereby recognizing private respondent Base Metals as the new operator of its claims. Pivate respondent Base Metals amended Banahaw Mining's pending MPSA applications with the Bureau of Mines to substitute itself as applicant and to submit additional documents in support of the application. Area clearances were submitted, as required. Thereafter, Base Metals’ amended MPSA applications were published. Hence, petitioner PICOP filed with the Mines Geo-Sciences Bureau an Adverse Claim and/or Opposition to private respondent Base Metals' application.

KITY

TAU MU TAU MU TAU MU TAU MU TAU MU TAU MU

CONSTITUTIONAL LAW II Order of Saint Thomas More Atty. Philip John Pojas/Atty. Rovyne G. Jumao-as, RN University College of Law

which cannot amplify PICOP's rights under its timber license. A timber license is not a contract within the purview of the non-impairment clause is edifying. A timber license is not a contract within the purview of the due process clause; it is only a license or a privilege, which can be validly withdrawn whenever dictated by public interest or public welfare. A license is merely a permit or privilege to do what otherwise would be unlawful, and is not a contract between the authority, federal, state, or municipal, granting it and the person to whom it is granted; neither is it a property or a property right, nor does it create a vested right; nor is it taxation. Thus, he granting of license does not create irrevocable rights, neither is it property or property rights. Timber licenses, permits and license agreements are the principal instruments by which the State regulates the utilization and disposition of forest resources to the end that public welfare is promoted. And it can hardly be gainsaid that they merely evidence a privilege granted by the State to qualified entities, and do not vest in the latter a permanent or irrevocable right to the particular concession area and the forest products therein. They may be validly amended, modified, replaced or rescinded by the Chief Executive when national interests so require. Thus, they are not deemed contracts within the purview of the due process of law clause. Since timber licenses are not contracts, the non-impairment clause, cannot be invoked. REPUBLIC vs. RMDC 426 SCRA 517 (2004) Facts: The four petitioners, after having been granted permission to prospect for marble deposits in the mountains of Biak-na-Bato, San Miguel, Bulacan, succeeded in discovering marble deposits of high quality and in commercial quantities in Mount Mabio which forms part of the Biak-na-Bato mountain range. Having succeeded in discovering said marble deposits, and as a result of their tedious eforts and substantial expenses, the petitioners applied with the Bureau of Mines, now Mines and Geosciences Bureau, for the issuance of the corresponding license to exploit said marble deposits. Thereafter, License No. 33 was issued by the Bureau of Mines in favor of the herein petitioners. Shortly after Respondent Maceda was appointed Minister of the DENR, petitioners’ License No. 33 was cancelled by him, by virtue of Proclamation No. 84, through his letter to ROSEMOOR MINING AND DEVELOPMENT CORPORATION for the reasons stated therein. Issue: WON Proclamation No. 84 which confirmed the cancellation of the license of the petitioners violated the non-impairment clause. Ruling: Proclamation No. 84 cannot be stigmatized as a violation of the non-impairment clause. As pointed out earlier, respondents’ license is not a contract to which the protection accorded by the non-impairment clause may extend. Even if the license were, it is settled that

ACADCOM 2010; Contributors: Gene Geocaniga, Jarissa Guiani, Darlene Magabilen TAU MU Page 121 of 179

provisions of existing laws and a reservation of police power are deemed read into it, because it concerns a subject impressed with public welfare. As it is, the non-impairment clause must yield to the police power of the state. Thus, a mining license that contravenes a mandatory provision of the law under which it is granted is void. Being a mere privilege, a license does not vest absolute rights in the holder. Thus, without ofending the due process and the non-impairment clauses of the Constitution, it can be revoked by the State in the public interest. BELTRAN vs. SECRETARY 476 SCRA 168 (2005)

TAU MU TAU MU TAU MU TAU MU TAU MU TAU MU TAU MU TAU MU TAU MU TAU MU

Sale of Human Blood, and the Establishment and Operation of Blood Banks and Blood Processing Laboratories.” The law, which was enacted in 1956, allowed the establishment and operation by licensed physicians of blood banks and blood processing laboratories. Petitioners assail the constitutionality of the questioned legal provisions, namely, Section 7 of RA 7719, which provides: “Section 7. Phaseout of Commercial Blood Banks - All commercial blood banks shall be phased-out over a period of two (2) years after the efectivity of this Act, extendable to a maximum period of two (2) years by the Secretary.” and Section 23 of Administrative Order No. 9, which provides that: “Section 23. Process of Phasing Out. -- The Department shall efect the phasing-out of all commercial blood banks over a period of two (2) years, extendible for a maximum period of two (2) years after the efectivity of R.A. 7719. The decision to extend shall be based on the result of a careful study and review of the blood supply and demand and public safety.”

The Fraternal Ateneo de Davao

TAU MU TAU MU

Facts: RA 7719 or the National Blood Services Act of 1994 seeks to provide an adequate supply of safe blood by promoting voluntary blood donation and by regulating blood banks in the country. It was approved by then President Ramos on May 15, 1994 and it law took efect on August 23, 1994. Blood banking and blood transfusion services in the country have been arranged in 4 categories: blood centers run by the PNRC, government-run blood services, private hospital blood banks, and commercial blood services. Years prior to the passage of the RA 7719, petitioners have already been operating commercial blood banks under RA 1517, entitled “An Act Regulating the Collection, Processing and PAUPER INDIGENT  need not be  persons who have no persons so poor that property or sources of they must be income sufficient for supported at public their support aside from expense their own labor though  those protected self-supporting when include lowly-paid able to work and in employees, domes- employment tic servants and laborers

KITY

TAU MU TAU MU TAU MU TAU MU TAU MU TAU MU

CONSTITUTIONAL LAW II Order of Saint Thomas More Atty. Philip John Pojas/Atty. Rovyne G. Jumao-as, RN University College of Law

Issue: WON Section 7 of R.A. 7719 and its implementing rules and regulations violated the non-impairment clause. Ruling: The National Blood Services Act is a valid exercise of the State’s police power. Therefore, the Legislature, under the circumstances, adopted a course of action that is both necessary and reasonable for the common good. Police power is the State authority to enact legislation that may interfere with personal liberty or property in order to promote the general welfare. The State, in order to promote the general welfare, may interfere with personal liberty, with property, and with business and occupations. Thus, persons may be subjected to certain kinds of restraints and burdens in order to secure the general welfare of the State and to this fundamental aim of government, the rights of the individual may be subordinated. hat the nonimpairment clause of the Constitution must yield to the loftier purposes targeted by the government. The right granted by this provision must submit to the demands and necessities of the State’s power of regulation. While the Court understands the grave implications of Section 7 of the law in question, the concern of the Government in this case, however, is not necessarily to maintain profits of business firms. In the ordinary sequence of events, it is profits that sufer as a result of government regulation. Furthermore, the freedom to contract is not absolute; all contracts and all rights are subject to the police power of the State and not only may regulations which afect them be established by the State, but all such regulations must be subject to change from time to time, as the general well-being of the community may require, or as the circumstances may change, or as experience may demonstrate the necessity. As found by the SC, individual rights to contract and to property have to give way to police power exercised for public welfare. BPI vs. SEC 541 SCRA 294 (2007)

TAU MU

Facts: BPI, through its predecessor-in- interest, Far East Bank and Trust Company, extended credit accommodations to the ASB Group secured by a real estate mortgage over 2 properties. The ASB Group filed a petition for rehabilitation and suspension of payments before the SEC. Thereafter, the interim receiver submitted its Proposed Rehabilitation Plan for the ASB Group. The Rehabilitation Plan provides, among others, a dacion en pago by the ASB Group to BPI of one of the properties mortgaged to the latter at the ASB Group against the total amount of the ASB Group’s exposure to the bank. In turn, ASB Group would require the release of the other property mortgaged to BPI, to be thereafter placed in the asset pool. The dacion would constitute full payment of the entire obligation due to BPI because the balance was then to be considered waived, as per the Rehabilitation Plan.BPI opposed the Rehabilitation

ACADCOM 2010; Contributors: Gene Geocaniga, Jarissa Guiani, Darlene Magabilen TAU MU Page 122 of 179

Plan and moved for the dismissal of the ASB Group’s petition for rehabilitation. However, the SEC hearing panel issued an order approving ASB Group’s proposed rehabilitation plan and appointed Mr. Fortunato Cruz as rehabilitation receiver. BPI filed a petition for review before the SEC en banc, imputing grave abuse of discretion on the part of the hearing panel. It argued that the Order constituted an arbitrary violation of BPI’s freedom and right to contract since the Rehabilitation Plan compelled BPI to enter into a dacion en pago agreement with the ASB Group. The SEC en banc denied the petition.

TAU MU TAU MU TAU MU TAU MU TAU MU TAU MU

Ruling: SEC’s approval of the Rehabilitation Plan did not impair BPI’s right to contract. As correctly contended by private respondents, the nonimpairment clause is a limit on the exercise of legislative power and not of judicial or quasijudicial power. The SEC, through the hearing panel that heard the petition for approval of the Rehabilitation Plan, was acting as a quasi-judicial body and thus, its order approving the plan cannot constitute an impairment of the right and the freedom to contract. Besides, the mere fact that the Rehabilitation Plan proposes a dacion en pago approach does not render it defective on the ground of impairment of the right to contract. Dacion en pago is a special mode of payment where the debtor ofers another thing to the creditor who accepts it as equivalent of payment of an outstanding debt. The undertaking really partakes in a sense of the nature of sale, that is, the creditor is really buying the thing or property of the debtor, the payment for which is to be charged against the debtor’s debt. As such, the essential elements of a contract of sale, namely; consent, object certain, and cause or consideration must be present. Being a form of contract, the dacion en pago agreement cannot be perfected without the consent of the parties involved. Section 11. Free access to the courts and quasi-judicial bodies and adequate legal assistance shall not be denied to any person by reason of poverty.

TAU MU

The courts of justice should be as available to the pauper as to the affluent in the protection of their respective rights --- Rule 3 Sec. 22 Rules of Court.

TAU MU



MARTINEZ vs. PEOPLE 332 SCRA 694 (2000)

TAU MU TAU MU

Facts: Martinez was accused of homicide. While his petition for certiorari was pending before the Court of Appeals, he filed a motion with said court to litigate as pauper litigant. The court denied it because under Sec. 16, Rule 41, of the Revised Rules of Court, appellate courts are not allowed to entertain such petitions

TAU MU

CASE: FREE ACCESS TO COURTS

The Fraternal Ateneo de Davao

TAU MU TAU MU

Issue: WON there was a violation of the nonimpairment clause.

KITY

TAU MU TAU MU TAU MU TAU MU TAU MU TAU MU

CONSTITUTIONAL LAW II Order of Saint Thomas More Atty. Philip John Pojas/Atty. Rovyne G. Jumao-as, RN University College of Law

Held: The 1997 Rules of Civil Procedure has removed the prohibition on appellate courts from entertaining petitions to litigate as paper. While accused filed his petition in 1994, the new policy shall be made applicable to pending actions. This interpretation is more in keeping with Art. III. Sec. 11 of the Constitution which decrees that “free access to the courts and quasi-judicial bodies and adequate legal assistance shall not be denied to any person by reason of poverty.” ACAR vs. ROSAL Facts: This is a civil case which was filed by Sacada workers against the Sugar Central. Complainants prayed that they be authorized to sue as pauper litigants (to be exempted to pay certain fees). The court denied their motion because they were workers, and not paupers. Ruling: The court held that there was a denial of access to courts by reason of poverty. An indigent was defined as persons who have no property or source of income sufficient for their own labor, though selfsupporting when able to work and in employment. Section 12. (1) Any person under investigation for the commission of an offense shall have the right to be informed of his right to remain silent and to have competent and independent counsel preferably of his own choice. If the person cannot afford the services of counsel, he must be provided with one. These rights cannot be waived except in writing and in the presence of counsel. (2) No torture, force, violence, threat, intimidation, or any other means which vitiate the free will shall be used against him. Secret detention places, solitary, incommunicado, or other similar forms of detention are prohibited. (3) Any confession or admission obtained in violation of this or Section 17 hereof shall be inadmissible in evidence against him. (4) The law shall provide for penal and civil sanctions for violations of this section as well as compensation to the rehabilitation of victims of torture or similar practices, and their families. SECTION 12 (1): CUSTODIAL INVESTIGATION  Time when the investigation is no longer a general inquiry into an unsolved crime BUT has begun to focus on a particular suspect who has been taken into police custody and subjected by

ACADCOM 2010; Contributors: Gene Geocaniga, Jarissa Guiani, Darlene Magabilen TAU MU Page 123 of 179

TAU MU TAU MU

confessions made prior to its efectivity. The same is true with the Morales decision which is a judge-made rule. Prior to April 26, 1983, the guidelines requiring that waiver of the right to counsel by an accused-can be properly made only with the assistance of counsel, had yet to be promulgated by the Court. The principle of prospectivity of statutes shall apply to judicial decisions, which although in themselves are not laws, are nevertheless evidences of what the law means. RIGHTS OF PERSON UNDER CUSTODIAL INVESTIGATION: (1) Right to remain silent  Only an accused has the absolute right to remain silent.  A person who is not an accused may assume the stance of silence only when asked an incriminating question.  A person under custodial investigation has the right to refuse to answer any question. Moreover, his silence may NOT be used against him [PEOPLE vs. FRAGO,232 SCRA 653]. (2) Right to have competent and independent counsel, preferably of his own choice  The counsel made available to the person under custodial investigation must be competent and independent and preferably to the detainee’s own choice. Q: What is the privilege given to that person under investigation? A: If a person cannot aford a counsel, he must be provided with one. So, that is a right and a privilege. It is diferent from the right to counsel because the right to counsel, everybody has the right to counsel, but the person who cannot aford counsel has the privilege and is given one. (3) Right to be provided with the services of counsel if he cannot afford the services of one. (4) Right to be informed of these rights.

TAU MU TAU MU TAU MU TAU MU TAU MU

Ruling: Yes. Sec. 12, Art III of the 1987 Constitution cannot be applied to extra-judicial

TAU MU

Facts: Accused are charged with Estafa Through Falsification of Public Documents. On February 12, 1982, an agent of the National Bureau of Investigation took their statements after they orally waived their right to counsel without the assistance of a lawyer. Under the 1973 Constitution, there was no requirement that the waiver must be in writing and made with the assistance of counsel. However, on April 26, 1983, the Supreme Court rendered the decision in Morales v. Enrile requiring that the waiver be made with the assistance of counsel to be valid. Considering that trial took place under the 1987 Constitution which bars such evidence, are the statements admissible?

TAU MU

Santos v. Sandiganbayan 347 SCRA 386 (2000)

TAU MU TAU MU

Essence of protection: What is being eschewed is the evil of “extorting” a confession from the mouth of the person being interrogated. As defined, “extortion” is an act or practice of taking or obtaining anything from a person by illegal use of fear, whether by force, threats or any undue exercise of power. In the context of obtaining an admission, “extorting” means “compelling or coercing a confession or information by any means serving to overcome his power of resistance, or making the confession or admission involuntary.” [PEOPLE vs. TIN LANG UY, 475 SCRA 248]

The Fraternal Ateneo de Davao

TAU MU TAU MU

the police to a process of interrogations that lends itself to eliciting incriminating statements  Any questioning initiated by the law enforcers after a person has been taken into custody or otherwise deprived of his freedom  It exists only in “custodial interrogations,” or “in-custody interrogation of accused persons.” By custodial interrogation is meant “questioning initiated by law enforcement officers after a person has been taken into custody or otherwise deprived of his freedom of action in any significant way.”  It refers to a “custodial” investigation where a suspect has already been taken into police custody and the investigating officers begin to ask questions to elicit information and confessions or admissions from the suspect.  Involves any questioning initiated by law enforcement authorities after a person is taken into custody or otherwise deprived of his freedom of action in any significant manner. And, the rule begins to operate at once as soon as the investigation ceases to be a general inquiry into an unsolved crime and direction is then aimed upon a particular suspect who has been taken into custody and to whom the police would then direct interrogatory question which tend to elicit incriminating statements. It refers to the critical pre-trial stage when the investigation ceases to be a general inquiry into an unsolved crime but has begun to focus on a particular person as a suspect.

KITY

TAU MU TAU MU TAU MU TAU MU TAU MU TAU MU

CONSTITUTIONAL LAW II Order of Saint Thomas More Atty. Philip John Pojas/Atty. Rovyne G. Jumao-as, RN University College of Law

CRIMINAL PROCESS [MIRANDA Case]: 1. Investigation prior to the filing of the charges; 2. Preliminary Examination and Investigation after charges are filed; and 3. Trial Period. Q: When do the rights under Sec. 12 (1) end? A: The rights under this section end when the investigation prior to the filing of the charges also end. GENERAL RULE: The rights can be waived. BUT the waiver must be done in writing and in the presence of counsel. The provisions say that these rights can be waived, if it is in writing and in the presence of counsel. Q: What cannot be waived from the abovementioned rights?

ACADCOM 2010; Contributors: Gene Geocaniga, Jarissa Guiani, Darlene Magabilen TAU MU Page 124 of 179

A: The right to be informed cannot be waived, it is an absolute right. PURPOSES OF THE RIGHTS IN SEC. 12: (1) To prohibit incommunicado investigation in a police dominated atmosphere resulting in selfincriminating statements without the Miranda Warning. (2) To prevent coerced confessions given in a police dominated atmosphere. NOTE: In Galman case, there was a law compelling the accused to testify under the pain of contempt while in Ayson casem there is none.

TAU MU TAU MU TAU MU TAU MU TAU MU TAU MU TAU MU TAU MU TAU MU

WHEN THE RIGHT ATTACHES OR WHEN CAN A PERSON INVOKE THESE RIGHTS: 1. Such person must be under investigation;  The right attaches at the start of the investigation when the investigating officers start to ask and question immediate information. Now, the question has to be specific, meaning you are being singled out, targeted as a perpetrator. If only asked a general question, that is not considered under investigation because it says here that you must be interrogated or questioned about matters relating to the crime, so it must be a specific kind of investigation.  When a suspect is being invited to the police station and asked questions about the crime, even if he is not arrested. But if all the requisites are present, then the right attaches. [PEOPLE vs. TAN]  When the questioning is not a general inquiry for any possible lead to the perpetration of the crime under investigation. [PEOPLE vs. BRAVO, 318 SCRA 812]  The accused was arrested and confession was made while walking along the highway with the policemen. The admission is inadmissible. [PEOPLE vs. BARIQUIT, 341 SCRA 600]

TAU MU TAU MU

REQUISITES WHEN RIGHT ATTACHES: 1. He must be under the custody of law enforcers or is in jail or deprived of freedom of action in a significant way. 2. He must be under investigation subjected to questioning or interrogation by a law enforcer. 3. Such questioning must be on relation to a crime.

The Fraternal Ateneo de Davao

TAU MU TAU MU

GENERAL RULE: In administrative investigations, the rights under Sec. 12 do not attach. EXCEPTION: Galman case because there was a law or EO compelling the accused to testify WHEN RIGHTS ARE AVAILABLE: (1) AFTER a person has been taken into custody or (2) When a person is otherwise deprived of his freedom of action in any significant way. (3) When the investigation is being conducted by the government (police, DOJ, NBI) with respect to a criminal ofense. (4) Signing of arrest reports and booking sheets.

KITY

TAU MU TAU MU TAU MU TAU MU TAU MU TAU MU

CONSTITUTIONAL LAW II Order of Saint Thomas More Atty. Philip John Pojas/Atty. Rovyne G. Jumao-as, RN University College of Law

2. The person being investigated must be: a. In the custody of a law enforcer, or b. Must be in jail, or c. Deprived of freedom of action in significant way.

a

 He was arrested, he was on board a police vehicle already, he was not in jail. But he was already deprived of his freedom of action in a significant way because he was already in the vehicle of the police and it was the police who questioned him. [PEOPLE vs. BOLANOS] 3. The investigation must be made by a law enforcer. [Note: ALL elements must concur.] CASES: RIGHT APPLIES PEOPLE vs. MALNGAN 503 SCRA 204 (2006) Facts: Malngan was accused of burning her employer’s house. She was apprehended by the Brgy. Chairman and the tanods and brought her to the Barangay Hall. Upon inspection, a disposable lighter was found inside Malngan’s bag. Thereafter, she confessed to Bernardo in the presence of multitudes of angry residents that she set her employer’s house on fire because she had not been paid her salary for about a year and that she wanted to go home to her province but her employer told her to just ride a broomstick in going home. Malngan was then turned over to arson investigators headed by SFO4 Danilo Talusan, who brought her to the San Lazaro Fire Station in Sta. Cruz, Manila where she was further investigated and then detained. When interviewed by a reporter of ABS-CBN Network, Malngan while under detention was heard by SFO4 Talusan as having admitted the crime and even narrated the manner how she accomplished it. He was able to hear the same confession, this time at his home, while watching the television program “True Crime” hosted by Gus Abelgas also of ABS-CBN Network. Hence, she was charged with the crime of arson with multiple homicide. Issue: WON the uncounselled extrajudicial confession of Malngan to Bernardo and to the media is inadmissible in evidence as being violative of Article 3, Sec. 12(1) of the Constitution. Ruling: The provision under Sec. 12 Article 3 of the Constitution applies to the stage of custodial investigation – when the investigation is no longer a general inquiry into an unsolved crime but starts to focus on a particular person as a suspect. Said constitutional guarantee has also been extended to situations in which an individual has not been formally arrested but has merely been “invited” for questioning. To be admissible in evidence against an accused, the extrajudicial confessions made must satisfy the

ACADCOM 2010; Contributors: Gene Geocaniga, Jarissa Guiani, Darlene Magabilen TAU MU Page 125 of 179

TAU MU TAU MU TAU MU TAU MU TAU MU TAU MU

Facts: Mojello was found guilty beyond reasonable doubt for the crime of rape with homicide. He admitted the perpetrated crime and

TAU MU

PEOPLE vs. MOJELLO 425 SCRA 11 (2004)

TAU MU

Ruling: No. The settled rule is that an uncouselled extra-judicial confession without a valid waiver of the right to counsel—that is, in writing and in the presence of counsel—is inadmissible in evidence. The belated presence of a lawyer the following say even if prior to the actual signing of the uncounselled confession does not cure the defect for the investigator was already able to extract incriminatory statements from the suspects. We have previously held that admissions obtained during custodial interrogations without the benefit of counsel although later reduced to writing and in the presence of counsel are still flawed under the Constitution.

TAU MU

Facts: Since accused had no counsel and there was no lawyer in Kaputian, Island Garden City of Samal, the police investigator took their confessions but did not require them to sign. The following day, he escorted the two accused to Davao City, along with their affidavits, before the Public Attorney’s Office. The lawyer, after advising them of their constitutional rights and explaining the contents of their statements, and after they affirmed the veracity and voluntariness of their execution, asked them to affix their signatures. Are the confessions admissible?

TAU MU TAU MU

PEOPLE vs. QUIDATO JR. 297 SCRA 1 (1998)

The Fraternal Ateneo de Davao

TAU MU TAU MU

following requirements: (1) it must be voluntary; (2) it must be made with the assistance of competent and independent counsel; (3) it must be express; and (4) it must be in writing. The inadmissibility of Malngan’s confession to Bernardo and the lighter does not automatically lead to her acquittal. It should well be recalled that the constitutional safeguards during custodial investigations do not apply to those not elicited through questioning by the police or their agents but given in an ordinary manner whereby the accused verbally admits to having committed the ofense as what happened in the case at bar when Malngan admitted to one of the witnesses to having started the fire in the Separas’ house. The testimony of the said witness recounting said admission is admissible in evidence against her and is not covered by the constitutional guarantee. Article III of the Constitution, or the Bill of Rights, solely governs the relationship between the individual on one hand and the State (and its agents) on the other; it does not concern itself with the relation between a private individual and another private individual – as both Malngan and prosecution witness undoubtedly are. There was also no evidence on record to show that said witness was acting under police authority.

KITY

TAU MU TAU MU TAU MU TAU MU TAU MU TAU MU

CONSTITUTIONAL LAW II Order of Saint Thomas More Atty. Philip John Pojas/Atty. Rovyne G. Jumao-as, RN University College of Law

assisted by counsel during custodial investigation. Issue: WON the extrajudicial confession executed is admissible in evidence. Ruling: The extra-judicial confession of the appellant is valid and admissible in evidence. The phrase “preferably of his own choice” does not convey the message that the choice of a lawyer by a person under investigation is exclusive to preclude other equally competent and independent attorneys from handling the case, otherwise, the temporariness of custodial investigation will be solely in the hands of the accused. The suspect has the final choice as he may reject the counsel. A lawyer is deemed engaged when the accused does not raise any objection during the investigation. The first custodial investigation violated the Miranda doctrine since no counsel was present and employees’ waiver of right of counsel. The Dec. 23, 1996 custodial investigation was upheld for having complied with Art. 3, Section 12. Even though improper interrogation methods were used at the outset, there is still a possibility of obtaining a legally valid confession later on by properly interrogating the subject under diferent conditions and circumstances than those which prevailed originally. People vs. Arondain 366 SCRA 98 (2001) Facts: After the robbing and killing of the taxi driver, accused was arrested by the police not far from the crime scene. The gun used in the killing was confiscated from him. When asked by the arresting officers why he shot the victim, accused answered that the latter resisted his demand for money. He added that after the shooting the driver, he panicked and immediately scampered away, leaving the money bill scattered on the floor of the taxi cab. Is his statement admissible in evidence? Ruling: No. The statement made by the accused admitting the crime cannot be admitted even as part of res gestae. It must be stressed that said statement was obtained in violation of his constitutional rights. Said confession was giver after he was arrested and without the assistance of counsel. He was not even informed of his right to remain silent or to counsel. From the time he was arrested and deprived of his freedom, all the questions propounded on him by the police for the purpose of eliciting, admissions, confessions or any information came within the ambit of custodial investigation. Failing to observe the constitutional mandate the confession cannot be admitted as evidence against him. PEOPLE vs. TAN 286 SCRA 207 (1998) Facts: In connection with the crime highway robbery with murder, the police invited accused, then a suspect, to the police station. In the course of the conversation inside the station, and without being warned of his right to counsel, but

ACADCOM 2010; Contributors: Gene Geocaniga, Jarissa Guiani, Darlene Magabilen TAU MU Page 126 of 179

without being subjected to any pressure, accused allegedly admitted authorship of the crime. Is his oral confession admissible?

PEOPLE VS. BRAVO 318 SCRA 812 (1999)

TAU MU TAU MU TAU MU TAU MU TAU MU TAU MU

Ruling: No. The interrogation by the police falls under the term “custodial investigation.” It is of no moment that the questioning was done along

TAU MU

Facts: Accused were suspects in a crime of robbery with homicide. Upon their arrest, the police immediately commenced investigation by asking them questions regarding the commission of the crime even while they were still walking along the highway, on their way to the police station. Without any lawyer to assist them, the 2 confessed to the crime. Are the confessions admissible?

TAU MU

PEOPLE VS. BARINQUIT 341 SCRA 600 (2000)

TAU MU

Ruling: No. Accused was under arrest for the rape and killing and any statements made by him pertaining to possible complicity in the crime without prior warning of his rights is inadmissible in evidence. The policeman’s apparent attempt to circumvent the rule by insisting that the admission was made during an “informal talk” prior to custodial investigation proper is not tenable. Appellant was not invited to the police station as part of a general inquiry for any possible lead to the perpetrators of the crime under investigation. Courts are not allowed to distinguish between preliminary questioning and custodial investigation proper when applying the exclusionary rule.

TAU MU TAU MU

Facts: On Jan. 15, 1994, the decomposing body of a child was found. After interviewing a witness who saw accused with the child on the night of Jan. 12, the police located accused at his work place. He was informed that he was a suspect and asked to come to the precinct for questioning. Accused was not informed of his constitutional rights, and while he was being informally interviewed by a policeman before being turned over to the assigned investigator, accused admitted that he was with the girl and he carried her on his shoulder but he was so drunk that night that he did not remember what he did to her. Is the statement admissible?

The Fraternal Ateneo de Davao

TAU MU TAU MU

Ruling: No. Under the Constitution and RA No. 7438, a confession to be admissible must satisfy the following requirements: (1) it must be voluntary; (2) it must be made with the assistance of competent and independent counsel; (3) it must be express; and (4) it must be in writing. Further, under RA No. 7438 it is provided that “custodial investigation shall include the practice of issuing an invitation” to a person who is investigated in connection with an ofense he is suspected to have committed, without prejudice to the liability of the inviting officer for any violation of law.”

KITY

TAU MU TAU MU TAU MU TAU MU TAU MU TAU MU

CONSTITUTIONAL LAW II Order of Saint Thomas More Atty. Philip John Pojas/Atty. Rovyne G. Jumao-as, RN University College of Law

the highway while accused were being led by the police to the station. To put it diferently, the place of interrogation is not at all a reliable barometer to determine the existence or absence of custodial investigation. It is significant that at the time the police already presumed accused as the perpetrators of the crime and singled them out as the authors thereof. The police should have properly apprised them of their constitutional rights, without which such uncounselled admissions are inadmissible in evidence. GUTANG vs. PEOPLE 335 SCRA 479 (2000) Facts: After conducting a search by virtue of a warrant in the residence of accused and the confiscation of marijuana, “shabu” and paraphernalia for the use of “shabu”, the police prepared a receipt. Accused, without the assistance of counsel, signed the receipt. The Philippine National Police Crime Laboratory subsequently conducted an examination of the specimens and it rendered a report that they were positive to test for the presence of prohibited and/or regulated drugs. Are the Receipts of Property Seized and Chemistry and Physical Science Reports admissible in evidence? Ruling: The Receipt of Property Seized describing the property taken from accused is inadmissible in evidence. It was signed by him without the assistance of a lawyer. It is tantamount to an uncounselled extra-judicial confession. However, the Physical Science and Chemistry Reports are admissible inasmuch as the examined materials were legally seized on the strength of a valid warrant. Thus, the tests conducted thereon were legally done. In other words, even without the Receipt of Property Seized, the guilt of the accused for the crimes charged was proven beyond reasonable doubt. WHEN RIGHTS ARE NOT AVAILABLE: (1) During a police line-up. Exception: Once there is a move among the investigators to elicit admissions or confessions from the suspect, or once you are targeted as a suspect. (2) During administrative investigations. (3) Confessions made by an accused at the time he voluntarily surrendered to the police or outside the context of a formal investigation. (4) Statements made to a private person. WHEN THE RIGHT DOES NOT ATTACH: (1) General inquiries (2) General rule for police line-up (3) when person investigating is not a police officer or not a law enforcer (4) Administrative investigations because sec. 12 only covers criminal investigations (5) Fact finding investigations even if akin to a criminal investigation, still look at the person investigating: not a police officer or law enforcer, but a fact finding body (so it is still not allowed)

ACADCOM 2010; Contributors: Gene Geocaniga, Jarissa Guiani, Darlene Magabilen TAU MU Page 127 of 179

(6) A barangay captain is not considered a police officer or law enforcer. [Note: The evidence is admissible perhaps because one or all of the requisites are missing.] REQUISITES OF A VALID WAIVER: (1)Waiver should be made in WRITING. (2)Waiver should be made in the PRESENCE OF COUNSEL.

TAU MU TAU MU TAU MU TAU MU TAU MU TAU MU TAU MU TAU MU

[Miranda Warning by CJ Warren] “He must be warned prior to any questioning that he has the right to remain silent, that anything he

TAU MU

It is now incumbent upon the prosecution to prove during trial that prior to questioning the confessant was warned of his constitutional rights.  The rights under this section are available only to a person facing custodial investigation. These rights do not apply to a spontaneous statement, NOT elicited through questioning by the authorities, BUT given in an ordinary manner whereby the accused orally admits having committed the crime [PEOPLE vs. BALOLOY (4/12/2002)].  The rights under this section can be lost by neglect as when the defense fails to object to the admissibility of the evidence immediately as required by Rule 132 Sec. 36 --- deemed WAIVER. 

TAU MU TAU MU

MIRANDSA RIGHTS/WARNING: 1. The person must be informed in the outset in clear and unequivocal terms that he has the right to remain silent; 2. Thereafter he must be told that anything he says can and will be used against him in court; 3. He must be clearly informed that he has the right to consult with a lawyer and to have the lawyer with him during the interrogation; 4. He should be warned that not only has he the right to consult with a lawyer but also that if he is indigent, a lawyer will be appointed to represent him; 5. Even if the person consents to answer questions without assistance of counsel, the moment he asks for a lawyer at any point in the investigation, the interrogation must cease until an attorney is present; 6. If the foregoing protections and warnings are NOT demonstrated to have been observed by the prosecution, no evidence obtained as a result of the interrogation can be used against the accused.

The Fraternal Ateneo de Davao

TAU MU TAU MU

[PEOPLE vs. GOMEZ]: The guy was in HK, he was in jail in HK. The NBI or some government agency sent some people to investigate, but these are law enforcers. Now, there was an admission. Can this be used against him? The SC said that no, he can invoke his rights under sec. 12 even if the investigation is being done in a foreign country for as long as he is under investigation. For as long as he is in custody of a law enforcer, (he was in jail in HK), and for as long as the person investigating is a law enforcer, he can invoke his right because at that time that he was investigated in HK, there was no lawyer present.

KITY

TAU MU TAU MU TAU MU TAU MU TAU MU TAU MU

CONSTITUTIONAL LAW II Order of Saint Thomas More Atty. Philip John Pojas/Atty. Rovyne G. Jumao-as, RN University College of Law

says can be used against him in a court of law, that he has the right to the presence of an attorney, and that if he cannot aford an attorney one will be appointed for him prior to any questioning if he so desires. Opportunity to exercise those rights must be aforded to him throughout the interrogation. After such warnings have been given, such opportunity aforded him, the individual may knowingly and intelligently waive these rights and agree to answer or make a statement. But unless and until such warnings and waivers are demonstrated by the prosecution at the trial, no evidence obtained as a result of interrogation can be used against him.” The objective is to prohibit “incommunicado interrogation of individuals in a police-dominated atmosphere, resulting in selfincriminating statement without full warnings of constitutional rights.”  The employee may, of course, refuse to submit any statement at the investigation; that is his privilege. But if he should opt to do so, in his defense to the accusation against him, it would be absurd to reject his statements, whether at the administrative investigation, or at a subsequent criminal action brought against him, because he had not been accorded, prior to his making and presenting them, his “Miranda rights” (to silence and to counsel and to be informed thereof, etc) which, to repeat, are relevant in custodial investigations. [PEOPLE vs. AYSON] Note: An interrogation of the accused by the police, if any there had been would already have been ended at the time of the filing of the criminal case in court (or the public prosecutor’s office). Hence, with respect to a defendant in a criminal case already pending in court (or the public prosecutor’s office), there is no occasion to speak of his right while under ‘custodial interrogation’ laid down by the second and subsequent sentences of Section 20, Article IV of the 1973 Constitution [now Section 12, Article III of the 1987 Constitution], for the obvious reason that he is no longer under ‘custodial interrogation.’ RIGHTS OF THE ACCUSED WHETHER IN COURT OR UNDERGOING PRELIMINARY INVESTIGATION BEFORE PUBLIC PROSECUTOR: 1) The right to refuse to be made witnesses; 2) The right not to have any prejudice whatsoever imputed to them by such refusal; 3) The right to testify on their own behalf, subject to cross-examination by the prosecution; and 4) While testifying, the right to refuse to answer a specific question that tends to incriminate them for some crime other than that for which they are being prosecuted. [LADIANA vs. PEOPLE, 393 SCRA 419] CASES: RIGHT DOES NOT APPLY PEOPLE vs. BALOLOY 381 SCRA 31 (2002)

ACADCOM 2010; Contributors: Gene Geocaniga, Jarissa Guiani, Darlene Magabilen TAU MU Page 128 of 179

CONSTITUTIONAL LAW II Order of Saint Thomas More Atty. Philip John Pojas/Atty. Rovyne G. Jumao-as, RN University College of Law

KITY

Ateneo de Davao

TAU MU TAU MU TAU MU TAU MU TAU MU TAU MU TAU MU TAU MU TAU MU TAU MU TAU MU

Ruling: In the case at bar, there is merit in Baloloy’s claim that his constitutional rights during custodial investigation were violated by Judge Dicon when the latter propounded to him incriminating questions without informing him of his constitutional rights. It is settled that at the

TAU MU TAU MU

of

TAU MU TAU MU TAU MU TAU MU TAU MU TAU MU

Facts: A dead body of an 11-year-old girl Genelyn Camacho was found. The one who caused its discovery was accused-appellant Baloloy himself, who claimed that he had caught sight of it while he was catching frogs in a nearby creek. At Genelyn’s wake, Brgy. Captain Ceniza asked Baloloy why his rope was found where Genelyn’s body was discovered. He answered, ““I have to claim this as my rope because I can commit sin to God if I will not claim this as mine because this is mine.” Baloloy told Ceniza that his intention was only to frighten the girl, not to molest and kill her. When Genelyn ran away, he chased her. As to how he raped her, Baloloy told Ceniza that he first inserted his fingers into GENELYN’s vagina and then raped her. Thereafter, he threw her body into the ravine. After such confession, Ceniza examined his body and found a wound on his right shoulder, as well as abrasions and scratches on other parts of his body. Upon further inquiry, Baloloy told her that the wound on his shoulder was caused by the bite of Genelyn. Ceniza then turned over Baloloy to a policeman for his own protection, as the crowd became unruly when she announced to them that Baloloy was the culprit. He was forthwith brought to the police headquarters. Ceniza informed him the police officer that Baloloy was the suspect in the killing of Genelyn, and she turned over to him a black rope which belonged to Baloloy. He wanted to interrogate Baloloy, but Ceniza cautioned him not to proceed with his inquiry because the people around were getting unruly and might hurt Baloloy. He immediately brought Baloloy to the police station, and on that same day, he took the affidavits of the witnesses. Judge. Dicon said that when he arrived in his office, several people, including Ceniza, were already in his courtroom. He learned that they came to swear to their affidavits before him. After reading the affidavit of Ceniza, he asked Ceniza whether her statements were true. Ceniza answered in the affirmative and pointed to Baloloy as the culprit. Judge Dicon turned to him and asked him whether the charge against him was true. He replied: “I was demonized”. While Judge Dicon realized that he should not have asked him as to the truthfulness of the allegations against him, he felt justified in doing so because the latter was not under custodial investigation. Judge Dicon thus proceeded to ask Baloloy whether he had a daughter as old as the victim and whether he was aware of what he had done to the girl Again, Baloloy responded that he was demonized, and he spontaneously narrated that after he struck Genelyn’s head with a stone he dropped her body into the precipice. Issue: WON the extrajudicial confession Baloloy to Ceniza is admissible in evidence.

The Fraternal

moment the accused voluntarily surrenders to, or is arrested by, the police officers, the custodial investigation is deemed to have started. So, he could not thenceforth be asked about his complicity in the ofense without the assistance of counsel. Judge Dicon’s claim that no complaint has yet been filed and that neither was he conducting a preliminary investigation deserves scant consideration. The fact remains that at that time JUANITO was already under the custody of the police authorities, who had already taken the statement of the witnesses who were then before Judge Dicon for the administration of their oaths on their statements. Moreover, contrary to what the police officer claims that Baloloy was not arrested but was rather brought to the police headquarters for his protection, the records reveal that he was in fact arrested. If indeed Baloloy’s safety was the primordial concern of the police authorities, the need to detain and deprive him of his freedom of action would not have been necessary. Arrest is the taking of a person into custody in order that he may be bound to answer for the commission of an ofense, and it is made by an actual restraint of the person to be arrested, or by his submission to the person making the arrest. At any rate, while it is true that his extrajudicial confession before Judge Dicon was made without the advice and assistance of counsel and hence inadmissible in evidence, it could however be treated as a verbal admission of the accused, which could be established through the testimonies of the persons who heard it or who conducted the investigation of the accused. PEOPLE vs. ENDINO 352 SCRA 307 (2001) Facts: For a murder that took place in Palawan, policemen arrested accused in Antipolo. On their way to the airport, they stopped at ABS-CBN television station where reporters interviewed accused. Video footages were taken that showed accused admitting his guilt. The interview was shown over ABS-CBN evening news program TV Patrol? Is the video footage admissible in evidence? Ruling: Yes. The confession does not form part of custodial investigation as it was not given to police officers but to media men. However, because of inherent danger in the use of the television as a medium for admitting one’s guilt, and the recurrence of the phenomenon in several cases, it is prudent that trial courts are reminded that extreme caution must be taken in further admitting similar confessions. For all probability, the police, with the connivance of unscrupulous media practitioners, may attempt to legitimize coerced confessions and place them beyond the exclusionary rule by having an accused admit an ofense on television. Courts should never presume that all media confessions described as voluntary have been freely given. This type of confession always remains suspect and therefore should be thoroughly scrutinized.

ACADCOM 2010; Contributors: Gene Geocaniga, Jarissa Guiani, Darlene Magabilen TAU MU Page 129 of 179

PEOPLE vs. TABOGA 376 SCRA 505 (2002) Facts: Accused was charged with Robbery with Homicide, as well as Arson, for robbing and killing an old woman and burning her house. After he was arrested, he was interviewed by a radio reporter inside the investigation room of the police station were policemen were present. The radio announcer informed him at the outset that he was a reporter who will be interviewing him to get side of the incident. During the interview, he admitted authorship of the crimes. Is his confession admissible?

TAU MU TAU MU TAU MU TAU MU TAU MU TAU MU

Ruling: The SC ruled that although the testimony of SFO4 Talusan is a hearsay because he was not present when Gus Abelgas interviewed Malngan, it may nevertheless be admitted in evidence as

TAU MU

Issue: WON the testimony of SF04 Talusan is inadmissible for being a hearsay.

TAU MU

Malngan was then turned over to arson investigators headed by SFO4 Danilo Talusan, who brought her to the San Lazaro Fire Station in Sta. Cruz, Manila where she was further investigated and then detained. When interviewed by a reporter of ABS-CBN Network, Malngan while under detention was heard by SFO4 Talusan as having admitted the crime and even narrated the manner how she accomplished it. He was able to hear the same confession, this time at his home, while watching the television program “True Crime” hosted by Gus Abelgas also of ABS-CBN Network. Hence, she was charged with the crime of arson with multiple homicide.

TAU MU

Facts: Malngan was accused of burning her employer’s house. She was apprehended by the Brgy. Chairman and the tanods and brought her to the Barangay Hall. Upon inspection, a disposable lighter was found inside Malngan’s bag. Thereafter, she confessed to Bernardo in the presence of multitudes of angry residents that she set her employer’s house on fire because she had not been paid her salary for about a year and that she wanted to go home to her province but her employer told her to just ride a broomstick in going home.

TAU MU TAU MU

PEOPLE vs. MALNGAN 503 SCRA 204 (2006)

The Fraternal Ateneo de Davao

TAU MU TAU MU

Ruling: Yes. The confession made by the accused did not form part of custodial investigation. It was not given to police officers but to a media man in an apparent attempt to elicit sympathy. There is nothing in the record to show that the radio announcer colluded with the police authorities to elicit inculpatory evidence against accused. Nether is there anything on record which even remotely suggests that the radio announcer was instructed by the police to extract information from him on the details of the crimes. Thus, assistance of counsel is not required.

KITY

TAU MU TAU MU TAU MU TAU MU TAU MU TAU MU

CONSTITUTIONAL LAW II Order of Saint Thomas More Atty. Philip John Pojas/Atty. Rovyne G. Jumao-as, RN University College of Law

an independently relevant statement to establish not the truth but the tenor of the statement or the fact that the statement was made. Under the doctrine of independently relevant statements, regardless of their truth or falsity, the fact that such statements have been made is relevant. The hearsay rule does not apply, and the statements are admissible as evidence. Evidence as to the making of such statement is not secondary but primary, for the statement itself may constitute a fact in issue or be circumstantially relevant as to the existence of such a fact.” BPI vs. CASA MONTESSORI, YABUT 430 SCRA 261 (2004) Facts: Casa opened an account with BPI. BPI discovered that 9 checks were encashed by Sonny Santos, a fictitious name used by Yabut, external auditor of Casa. Casa filed a complaint for collection with damages against BPI. Issue: WON the voluntary admission of Yabut violated his constitutional rights on custodial investigation. Ruling: He was not under custodial investigation. His affidavit was executed in private and before private individuals. To fall within the ambit of Sec. 12, there must be an arrest or a deprivation of freedom. It does not apply to spontaneous statements made in a voluntary manner whereby an individual orally admits to authorship of a crime. What the constitution proscribes is the compulsory or coercive disclosure of incriminating facts. The Bill of Rights does not concern itself with the relation between a private individual and another individual. It governs the relationship between the individual and the State. ASTUDILLO and ORELLANA vs. PEOPLE 509 SCRA 509 (2006) Facts: Petitioners were hired by Western Marketing Corporation, a chain of appliance stores, as salespersons. It was found out later on that the daily cash collection report did not reflect any remittance of payments from the transactions covered by certain invoices. It was also discovered that Flormarie was the one who filled it up and received the payment reflected therein and that the goods covered thereby were missing. Concluding that the transactions under the said invoices were made but no payment was remitted to Western, the accountant reported the matter to the branch manager. Benitez and Orellana pleaded with them not to report the matter to the management. Flormarie made a similar plea as she admitted to stealing the missing booklet of invoices, she explaining that her father was sick and had to undergo medical operation, and ofering to pay for the goods covered thereby. In a subsequent meeting with Lily, Orellana admitted having brought home some appliances while Benitez gave a handwritten statement asking for apology and explaining why there was a “short-over”. In a still subsequent meeting with the representative of

ACADCOM 2010; Contributors: Gene Geocaniga, Jarissa Guiani, Darlene Magabilen TAU MU Page 130 of 179

Western, Orellana made a written statement in the former’s presence stating that she will not repeat what she did when she brought items outside the establishment. Also in a meeting with the representative, Astudillo, who was earlier implicated by Flormarie’s husband in his telephone conversation with the branch manager, wrote in her written statement her apology and her regrets for what she did. Flormarie subsequently appeared before a notary public to execute a similar statement. She later executed a statement before SPO1 Gregorio.

The Fraternal Ateneo de Davao

TAU MU TAU MU TAU MU TAU MU

In an inventory of stocks conducted at the branch office of Western, several other appliances were found missing as were unauthorized deductions from the cash collections. The total missing merchandise was valued at P797,984.00 as reflected in the inventory report. And discrepancies between the actual sales per cash sales invoice and the cash remittance to the company in the sum of P34,376.00 were also discovered, prompting Western to initiate the criminal complaints for Qualified Theft.

KITY

TAU MU TAU MU TAU MU TAU MU TAU MU TAU MU

CONSTITUTIONAL LAW II Order of Saint Thomas More Atty. Philip John Pojas/Atty. Rovyne G. Jumao-as, RN University College of Law

TAU MU

Ruling: In the case at bar, petitioners were not under custodial investigation to call for the presence of counsel of their own choice, hence, their written incriminatory statements are admissible in evidence. The extra-judicial confession before the police of Flormarie in which she incriminated petitioners bears a diferent complexion, however, as it was made under custodial investigation. When she gave the statement, the investigation was no longer a general inquiry into an unsolved crime but had begun to focus on a particular suspect. The records show that the accountant had priorly reported the thievery to the same police authorities and identified Flormarie and Benitez as initial suspects. It is always incumbent upon the prosecution to prove at the trial that prior to incustody questioning, the confessant was informed of his constitutional rights. The presumption of regularity of official acts does not prevail over the constitutional presumption of innocence. Hence, in the absence of proof that the arresting officers complied with these constitutional safeguards, extrajudicial statements, whether inculpatory or exculpatory, made during custodial investigation are inadmissible and cannot be considered in the adjudication of a case. In other words, confessions and admissions in violation of Section 12 (1), Article III of the Constitution are inadmissible in evidence against the declarant and more so against third persons. This is so even if such statements are gospel truth and voluntarily given. There is nothing on record, however, buttressing petitioners’ claim other than their self-serving assertion. The presumption that no

TAU MU TAU MU

Issue: WON the employees’ extra-judicial admissions taken before an employer in the course of an administrative inquiry are admissible in a criminal case filed against them.

person of normal mind would deliberately and knowingly confess to a crime unless prompted by truth and conscience such that it is presumed to be voluntary until the contrary is proved thus stands. Only Astudillo was acquitted. SEBASTIAN V. GARCHITORENA 343 SCRA 463 (2000) Facts: Petitioners were postal employees charged with Malversation of Public Funds. During a fact-finding investigation conducted relative to missing postage stamps they executed sworn statements without the assistance of counsel and without being warned of their rights. Their written statements were ofered by the prosecution in evidence during the trial in the criminal case. Is there a violation of the right to counsel? Ruling: No. The rights under Sec. 12 apply only to custodial investigation or questioning initiated by law enforcement officers after a person has been taken into custody or otherwise deprived of his freedom of action in any significant way. The fact finding relative to the missing stamps conducted by the Chief Postal Service Office is not a custodial investigation but a mere administrative investigation. While an administrative investigation may be akin to a criminal proceeding, irrespective of the nature of the charges and of the respondent’s capacity to represent himself, no duty rests on the investigator to provide the person being investigated with counsel. REMOLINA VS. CSC 362 SCRA 304 (2001)

TAU MU TAU MU

Facts: For having secured for his wife a fake Report of Rating with a passing mark in the teacher’s board examination, petitioner was dismissed from government service for dishonesty. Among the evidence against him was his written statement made during the investigation by the Civil Service Commission admitting his guilt. No counsel assisted him nor was he advised of rights under the Constitution when he signed the statement. Is it admissible?

TAU MU TAU MU TAU MU TAU MU

Ruling: Yes. While investigations conducted by an administrative body may at times be akin to a criminal proceedings, the fact remains that under existing laws, a party in an administrative inquiry may not be assisted by counsel, irrespective of the nature of the charges or the respondent’s capacity to represent himself. Under the Civil Service Act, a respondent has the option of engaging the services of counsel or not. The right to counsel is not always imperative in an administrative investigation because such inquiries are conducted merely to determine whether there are facts that merit disciplinary measure against erring pubic officials. In this case, respondents was not accused of any crime in the investigation conducted by the CSC field office. It was conducted for the purpose of

ACADCOM 2010; Contributors: Gene Geocaniga, Jarissa Guiani, Darlene Magabilen TAU MU Page 131 of 179

CONSTITUTIONAL LAW II Order of Saint Thomas More Atty. Philip John Pojas/Atty. Rovyne G. Jumao-as, RN University College of Law

he

should

The Fraternal Ateneo de Davao

be

PEOPLE vs. TING LAN UY 475 SCRA 248 (2005)

TAU MU TAU MU TAU MU TAU MU TAU MU TAU MU TAU MU TAU MU TAU MU

Ruling: The rights enumerated by the constitutional provision invoked by accusedappellant are not available before government investigators enter the picture. Thus, as held in one case, the admissions made during the course of an administrative investigation by Philippine Airlines do not come within the purview of Section 12. The protective mantle of the constitutional provision also does not extend to admissions or confessions made to a private individual, or to a verbal admission made to a radio announcer who was not part of the investigation, or even to a mayor approached as a personal confidante and not in his official capacity. Neither does the constitutional provision on custodial investigation extends to a spontaneous statement, not elicited through questioning by the authorities, but given in an ordinary manner whereby the accused orally admits having committed the crime, nor to a person undergoing an audit examination because an audit examiner is not a law enforcement officer. Thus, the flaw in appellant’s argument in this regard becomes immediately apparent vis-à-

TAU MU TAU MU

Issue: WON a sworn statement which was taken without the benefit of counsel violated Section 12.

TAU MU TAU MU

Facts: The prosecution theorizes that accused diverted the funds covered by the two PNB Manager’s checks by falsifying a commercial document called an “Application for Cashier’s Check” by inserting an account number of a private individual after the name of the payee, UCPB. It claims that NPC did not authorize the insertion considering that the Payment Instruction issued by NPC instructing PNB to prepare a Manager’s check to be charged to NPC’s savings account did not contain any account number. Through the insertion, the accused allegedly succeeded in diverting the funds from the UCPB in favor of several persons. In his defense, appellant asserts that there was no evidence that he committed any of the acts alleged in the information, particularly the intercalation on the ACC; that he deposited the checks subsequently issued or that he received the proceeds thereof; or that he conspired with any of his co-accused. He claims that his conviction was based on the alleged sworn statement and the transcript of stenographic notes of a supposed interview with appellant by the NPC personnel and the report of the NBI. Appellant maintains that he signed the sworn statement while confined at the Philippine Heart Center and upon assurance that it would not be used against him. He was not assisted by counsel nor was he apprised of his constitutional rights when he executed the affidavit.

TAU MU TAU MU TAU MU TAU MU TAU MU TAU MU

ascertaining whether administratively charged.

KITY

vis the foregoing legal yardsticks, considering that his statement was taken during the administrative investigation of NPC’s audit team and before he was taken into custody. As such, the inquest was still a general inquiry into an unsolved ofense at the time and there was, as yet, no specific suspect. The fact that an NBI investigation was being contemporaneously conducted at the time the sworn statement was taken will not extricate appellant from his predicament. The essence of the constitutional safeguard is protection from coercion. The interview where the sworn statement is based was conducted by NPC personnel for the NPC’s administrative investigation. Any investigation conducted by the NBI is a proceeding separate, distinct and independent from the NPC inquiry and should not be confused or lumped together with the latter. PEOPLE vs. SALONGA 359 SCRA 310 (2001) Facts: Accused was an Acting Assistant Cashier of Metrobank. During a spot audit conducted by the Department of Internal Afairs of the bank, he admitted having issued a cashier’s check without any legitimate transaction and that he benefited P8,500.00 from the amount of the check. His admissions were reduced into writing and ofered in evidence by the prosecution. When he made the admissions, however, there was no lawyer assisting him. Is it admissible? Ruling: Yes. The constitutional right to counsel may be invoked only by a person under custodial investigation. Custodial investigation is the stage where the police investigation is no longer a general inquiry into an unsolved crime but has begun to focus on a particular suspect taken into custody by the police who carry out an interrogation to elicit incriminating statements. In this case, the bank auditor who questioned accused was not a police officer but a private person. He was not under custodial investigation so that the legal formalities required by the fundamental law do not apply. PEOPLE vs. BALOLOY supra. The constitutional provision on custodial investigation does not apply to a spontaneous statement, not elicited through questioning by the authorities but given in an ordinary manner whereby the suspect orally admits having committed the crime. Neither can it apply to admissions or confessions made by a suspect in the commission of a crime before he is placed under investigation. What the Constitution bars is the compulsory disclosure of incriminating facts or confessions. The rights under Section 12 of the Constitution are guaranteed to preclude the slightest use of coercion by the state as would lead the accused to admit something false, not to prevent him from freely and voluntarily telling the truth.

ACADCOM 2010; Contributors: Gene Geocaniga, Jarissa Guiani, Darlene Magabilen TAU MU Page 132 of 179

PEOPLE vs. PAVILLARE 329 SCRA 684 (2000) Facts: Accused was suspect in a kidnapping of an Indian national. After his arrest for another ofense, he was identified in a police line-up by his victim at the police station as one of the kidnappers. Considering that accused was not assisted by counsel, is he identification admissible?

TAU MU TAU MU TAU MU TAU MU TAU MU TAU MU TAU MU

Ruling: The legal formalities required by the fundamental law of the land apply only to extrajudicial confessions or admissions obtained during custodial investigations. Indeed, the rights enumerated in the constitutional provision “exist only in custodial interrogations, or in-custody interrogation of accused persons.

TAU MU

Issue (1): WON absence of the counsel when petitioner executed the affidavit violated his right under Section 12.

TAU MU

Facts: Caridad declared that she is the wife of Barangay Captain Francisco, the victim in the case at bar. Caridad testified that Francisco was shot and killed by accused Ladiana, who happens to be also a distant relative of the decedent Prior to the conduct of the examination-in-chief on Cortez, a retired assistant prosecutor, the defense counsel made an admission as to the authorship, authenticity, and voluntariness of the execution of the counter-affidavit of Ladiana, which was subscribed and sworn to before Cortez. In said counter-affidavit, accused Ladiana allegedly admitted to making the fatal shots on Francisco. However, Ladiana allegedly did so in self-defense as Francisco was then purportedly attacking Ladiana and had, in fact, already inflicted a stab wound on the arm of accused Ladiana. However, Cortez emphasized that he was not the one who conducted the preliminary investigation of the complaint which led to the filing of the subject case. Additionally, Cortez testified that he would not be able to anymore recognize the face of the affiant in the said counter-affidavit, but maintained that there was a person who appeared and identified himself as Ladiana before he affixed his signature on the counter-affidavit.

TAU MU TAU MU

LADIANA vs. PEOPLE 393 SCRA 419 (2002)

The Fraternal Ateneo de Davao

TAU MU TAU MU

Ruling: Yes. The right to counsel under Sec. 12 (1) of the Bill of Rights does not extend to a person in a police line-up because that stage of criminal law enforcement process is not yet part of custodial investigation. Custodial investigation commences when a person is taken into custody and is singled out as a suspect in the commission of the crime under investigation and the police officers begin to ask questions on the suspect’s participation therein which tend to elicit an admission. A police line-up has been held to the outside the mantle of protection of the right to counsel because it involves a general inquiry into an unsolved crime.

KITY

TAU MU TAU MU TAU MU TAU MU TAU MU TAU MU

CONSTITUTIONAL LAW II Order of Saint Thomas More Atty. Philip John Pojas/Atty. Rovyne G. Jumao-as, RN University College of Law

Contrary to what Ladiana claims that the questioned statements were made during the preliminary investigation, not during the custodial investigation, hence, the right to competent and independent counsel also applies during preliminary investigations is untenable. A preliminary investigation is an inquiry or a proceeding to determine whether there is sufficient ground to engender a well-founded belief that a crime has been committed, and that the respondent is probably guilty thereof and should be held for trial. A person undergoing preliminary investigation before the public prosecutor cannot be considered as being under custodial investigation. In fact, the Court has unequivocally declared that a defendant on trial or under preliminary investigation is not under custodial interrogation. There is no question that even in the absence of counsel; the admissions made by petitioner in his Counter-Affidavit are not violative of his constitutional rights. It is clear from the undisputed facts that it was not exacted by the police while he was under custody or interrogation. Hence, the constitutional rights of a person under custodial investigation as embodied in Article III, Section 12 of the 1987 Constitution, are not at issue in this case. Issue (2): WON the counter-affidavit filed by petitioner during preliminary investigation constitutes as an extra-judicial admission. Ruling: Petitioner’s counter-affidavit is not an extrajudicial admission but only an admission. In a confession, there is an acknowledgment of guilt; in an admission, there is merely a statement of fact not directly involving an acknowledgment of guilt or of the criminal intent to commit the offense with which one is charged. RIGHT TO COUNSEL  The right to independent counsel, choice.

have competent preferably of his

and own

REQUISITES FOR COUNSEL DURING CUSTODIAL INVESTIGATION: a. The counsel must be preferably of the own choice of the person being investigated. b. The counsel must be competent and independent; and according to the SC c. The counsel must be vigilant and efective. WHEN IS THE PERSON WHO IS BEING INVESTIGATED NOT DEPRIVED OF HIS RIGHT TO COUNSEL: (1) When he personally has his own lawyer; (2) When the person expressly agrees to the lawyer chosen or given to him by the investigator; (3) When the person gives his implied consent to the lawyer chosen by the police investigator – [PEOPLE Vs. PORIO and TIZON]

ACADCOM 2010; Contributors: Gene Geocaniga, Jarissa Guiani, Darlene Magabilen TAU MU Page 133 of 179

Because if you do not object, you impliedly consented to the lawyer given to you. EXCEPTION: if the person had no opportunity to object to the lawyer assigned to him because the environment is intimidating. It cannot be considered as implied consent.

TAU MU TAU MU TAU MU TAU MU TAU MU TAU MU TAU MU

(b) Any public officer or employee, or anyone acting under his order or his place, who arrests, detains or investigates any person for the commission of an ofense shall inform the latter, in a language known to and understood by him,

TAU MU

Section 2. Rights of Persons Arrested, Detained or Under Custodial Investigation; Duties of Public Officers. - (a) Any person arrested detained or under custodial investigation shall at all times be assisted by counsel.

TAU MU

REPUBLIC ACT No. 7438 -AN ACT DEFINING CERTAIN RIGHTS OF PERSON ARRESTED, DETAINED OR UNDER CUSTODIAL INVESTIGATION AS WELL AS THE DUTIES OF THE ARRESTING, DETAINING AND INVESTIGATING OFFICERS, AND PROVIDING PENALTIES FOR VIOLATIONS THEREOF

TAU MU TAU MU

[PEOPLE vs. TOMAQUIN]: There was a violation of the right to counsel when the accused confessed to the barangay captain and there was no lawyer. Although the barangay captain was a lawyer, he was not considered independent. WAIVER OF THE RIGHT TO COUNSEL DURING INVESTIGATION History of the waiver. 1. 1935 Constitution- no such thing. 2. Under the 1973 Constitution protected the right to remain silent and the right to counsel. However, the right to remain silent can be waived just by talking. But the right to counsel can be waived, there were no formalities. The only requirement was that it was done voluntarily and intelligently. 3. 1983: Enrile case: By jurisprudence, the SC held that the right to counsel can be waived but only with the assistance of counsel. There's no form, as long as the counsel is present. You need to have a lawyer to waive your right to a lawyer during investigation 4. 1987: The right of counsel can be waived but only with the assistance of counsel and the waiver must be in writing.

The Fraternal Ateneo de Davao

TAU MU TAU MU

WHEN IS A PERSON BEING INVESIGATED BEING DEPRIVED OF HIS RIGHT TO COUNSEL: (1) If he was not asked if he has a counsel of choice, (2) And whether he could hire such counsel, and if he could not, (3) Whether he would agree to the one that would be provided for him. [PEOPLE vs. AGUSTIN] (4) When the lawyer is not vigilant and efective; (5) When the lawyer is not independent [PEOPLE vs. CULALA]

KITY

TAU MU TAU MU TAU MU TAU MU TAU MU TAU MU

CONSTITUTIONAL LAW II Order of Saint Thomas More Atty. Philip John Pojas/Atty. Rovyne G. Jumao-as, RN University College of Law

of his rights to remain silent and to have competent and independent counsel, preferably of his own choice, who shall at all times be allowed to confer privately with the person arrested, detained or under custodial investigation. If such person cannot aford the services of his own counsel, he must be provided with a competent and independent counsel by the investigating officer.lawphi1Ÿ (c) The custodial investigation report shall be reduced to writing by the investigating officer, provided that before such report is signed, or thumbmarked if the person arrested or detained does not know how to read and write, it shall be read and adequately explained to him by his counsel or by the assisting counsel provided by the investigating officer in the language or dialect known to such arrested or detained person, otherwise, such investigation report shall be null and void and of no efect whatsoever. (d) Any extrajudicial confession made by a person arrested, detained or under custodial investigation shall be in writing and signed by such person in the presence of his counsel or in the latter's absence, upon a valid waiver, and in the presence of any of the parents, elder brothers and sisters, his spouse, the municipal mayor, the municipal judge, district school supervisor, or priest or minister of the gospel as chosen by him; otherwise, such extrajudicial confession shall be inadmissible as evidence in any proceeding. (e) Any waiver by a person arrested or detained under the provisions of Article 125 of the Revised Penal Code, or under custodial investigation, shall be in writing and signed by such person in the presence of his counsel; otherwise the waiver shall be null and void and of no effect. (f) Any person arrested or detained or under custodial investigation shall be allowed visits by or conferences with any member of his immediate family, or any medical doctor or priest or religious minister chosen by him or by any member of his immediate family or by his counsel, or by any national non-governmental organization duly accredited by the Commission on Human Rights of by any international nongovernmental organization duly accredited by the Office of the President. The person's "immediate family" shall include his or her spouse, fiancé, parent or child, brother or sister, grandparent or grandchild, uncle or aunt, nephew or niece, and guardian or ward. As used in this Act, "custodial investigation" shall include the practice of issuing an "invitation" to a person who is investigated in connection with an ofense he is suspected to have committed, without prejudice to the liability of the "inviting" officer for any violation of law.

ACADCOM 2010; Contributors: Gene Geocaniga, Jarissa Guiani, Darlene Magabilen TAU MU Page 134 of 179

In the absence of any lawyer, no custodial investigation shall be conducted and the suspected person can only be detained by the investigating officer in accordance with the provisions of Article 125 of the Revised Penal Code.

TAU MU TAU MU TAU MU TAU MU TAU MU TAU MU TAU MU

Facts: The appellants were charged with 4 counts of rape with homicide. During trial, the police officers invariably testified that they informed the appellants of their rights and of the consequences of their acts before their statements were taken down. Atty. Guinalon purportedly acted as counsel for all the suspects at the time their statements were taken down. Known to the four as a leader in the community, Atty. Guinalon was requested to assist in the execution of their extra-judicial statements. The lawyer asked the suspects why they requested him in particular. They replied that they knew him and that he, in turn, knew all of them. Atty. Guinalon conferred with the suspects, who expressly signified their intention to put into writing what happened that fateful night. He explained to them that by making a confession, they would be admitting to the commission of a grave crime, which carried with it a severe penalty. After Atty. Guinalon apprised them of their constitutional rights, the four proceeded to execute their respective statements. Atty. Guinalon was in front of the suspects when they gave their statements and was present during the entire investigation. Assistant Prosecutor Cardinal subscribed the extra-judicial statements of the four accused. He explained to the suspect the consequences of his action, making sure that the latter understood the contents of his statement. The Assistant City Prosecutor told Tizon, Jr. that the same could be used against him and that he could be severely punished for his crime. Cardinal also subscribed the extra-judicial confessions of the other appellants. Prior to the signing of these statements, he asked the suspects if the police

Issue (1): WON the rights of the accused under Section 12 was violated by reason of the extrajudicial declarations extracted from them.

TAU MU

PEOPLE vs. TIZON JR. 385 SCRA 364

threatened them or forced them to sign the statements. They answered that they were not. To avert any compulsion, the prosecutor even asked the police officers to leave his cubicle before asking the suspects any question. He also examined their hands and bodies for any injuries and asked them whether they were promised any reward. He found no signs of injury on the suspects, who categorically declared that they were not threatened and that no reward was promised them. Assistant City Prosecutor Cardinal then instructed them to examine every page of the documents and to sign the statements in the presence of their counsel

TAU MU

CASES: COUNSEL OF CHOICE

Ateneo de Davao

TAU MU TAU MU

RA 7438 has extended the guaranty to situations in which an individual has not been formally arrested but has been merely “invited” for questioning.  RA 7438 which expanded the provision in the 1987 Constitution by providing that: "If the accused waives his right to counsel, the waiver must be waived (1) with the assistance of counsel, and (2) the waiver must be in writing and signed in the presence of parents, brothers or sisters, spouse, municipal mayor, municipal judge, district or school supervisor, etc. 

The Fraternal

TAU MU TAU MU

Section 3. Assisting Counsel. - Assisting counsel is any lawyer, except those directly afected by the case, those charged with conducting preliminary investigation or those charged with the prosecution of crimes.

KITY

TAU MU TAU MU TAU MU TAU MU TAU MU TAU MU

CONSTITUTIONAL LAW II Order of Saint Thomas More Atty. Philip John Pojas/Atty. Rovyne G. Jumao-as, RN University College of Law

Ruling: The right to be informed of one's constitutional rights during custodial investigation refers to an efective communication between the investigating officer and the suspected individual, with the purpose of making the latter understand these rights. Understanding would mean that information transmitted was efectively received and comprehended. Hence, the Constitution does not merely require the investigating officers to "inform" the person under investigation; rather, it requires that the latter be “informed.” Appellants' extra-judicial statements were subscribed to before an assistant prosecutor who testified that he asked the appellants if they were forced by the police to sign their statements. They declared that everything was of their own free will. The prosecutor examined appellants’ bodies to determine if they were harmed. The prosecutor even told the police to get out of the office to preclude any intimidation. Moved by remorse, appellants said they were giving their statements freely and voluntarily. Issue (2): WON the appellants were not accorded a competent and independent counsel of their own choice as provided for under Section 12. Ruling: Appellants deny that they sought the services of Atty. Guinalon as their counsel during the taking of their extra-judicial statements. If Atty. Guinalon was not really appellants’ chosen counsel, they could have requested for another lawyer or voiced their objection. Appellants never did but instead voluntarily executed their extra-judicial statements. While the initial choice of the lawyer in cases where a person under custodial investigation cannot aford the services of a lawyer is naturally lodged in the police investigators, the accused really has the final choice as he may reject the counsel chosen for him and ask for another one. A lawyer provided by the investigators is deemed engaged by the accused where he never raised any objection against the lawyer’s appointment during the course of the investigation and the accused thereafter subscribes to the veracity of his statement before the swearing officer.

ACADCOM 2010; Contributors: Gene Geocaniga, Jarissa Guiani, Darlene Magabilen TAU MU Page 135 of 179

TAU MU TAU MU TAU MU TAU MU

Ruling: No. A Municipal Attorney cannot be an independent counsel as required by the Constitution. As a legal officer of the municipality, he provides legal assistance and support to the mayor of the municipality in carrying out the delivery of basic services to the people, including the maintenance of peace and order. It is therefore seriously doubted whether he can efectively undertake the defense of the accused without running into conflict of interest. He is no better that a fiscal or prosecutor who cannot represent the accused during custodial investigations.

TAU MU

Facts: Accused was a suspect in a robbery with homicide case. After his arrest, he executed an extra-judicial confession admitting the commission of the crime with the assistance of the Municipal Attorney of Valenzuela, Metro Manila. The lawyer testified that he apprised accused of his constitutional rights. Is the confession admissible?

TAU MU

PEOPLE VS. CULALA 316 SCRA 582 (1999)

TAU MU

Ruling: The accused has the final choice of the lawyer as he may reject the counsel chosen for him and ask for another one. However, a lawyer provided by the investigator is deemed engaged by the accused where, as in this case, he never raised any objection against the officer’s appointment during the course of investigation and the accused thereafter subscribed to the veracity of his statement before the swearing officer.

TAU MU

Facts: Accused was convicted of murder. Among the evidence presented during the trial was his extra-judicial confession wherein he admitted the crime. The document was signed in the presence of Atty. Teofilo Tumulak and subscribed before the municipal mayor. He now alleges that the lawyer was not his choice.

TAU MU TAU MU

PEOPLE VS. GONZALES 311 SCRA 551 (1999)

The Fraternal Ateneo de Davao

TAU MU TAU MU

Atty. Guinalon is a competent and independent counsel, having been a public prosecutor, a member of the Practicing Lawyer Association of Negros Occidental, and of the Integrated Bar of the Philippines and a civil leader in the community, and the way he steadfastly and painstakingly assisted the four accused during the entire proceedings in the custodial investigation to safeguard their constitutional rights and his attendance before Asst. City Prosecutor Manuel Cardinal when the four accused subscribed to their corresponding extrajudicial statements. He even signed these statements to attest to his presence and the regularity of the proceedings. Thus, the appellants were accorded a competent and independent counsel in the person of Atty. Guinalon.

KITY

TAU MU TAU MU TAU MU TAU MU TAU MU TAU MU

CONSTITUTIONAL LAW II Order of Saint Thomas More Atty. Philip John Pojas/Atty. Rovyne G. Jumao-as, RN University College of Law

PEOPLE vs. VELARDE 384 SCRA 646 (2002) Facts: Accused declared that he has been detained for more than one year already because he was told that he was the one who committed a crime against his cousin. According to him, he was arrested while selling balot by four Barangay Officials. When they asked him where he brought the victim, he told them he ‘don’t know’. He did not insist answering them ‘because I don’t know what they were asking about the child’. He just went with them because if he will not go with them ‘di nila lulubayan and pamilya ko’. He was brought to the Barangay Hall. He was kicked and mauled by the father and brothers of Brenda, the victim. After hurting him inside the Barangay Hall he was made to sign by one of the Barangay Officials. He signed without reading what he signed because he cannot read very well. During trial, the accused was candid enough to admit that the signature is his signature; that Atty. Domingo is known to him because he was then the Mayor of Malolos; that he hired or engaged the services of Atty. Domingo; that he was also candid enough to testify that ‘wala akong alam diyan.’ His educational attainment was up to Grade four (4) only. He claims that he does not know the police investigator who typed the ‘Sinumpaang Salaysay’.” Issue (1): WON the extrajudicial confession taken during the investigation with the presence of Atty. Domingo is inadmissible in evidence. Ruling: Based on the facts of the case, upon the advice of the Mayor, Atty. Domingo, Velarde’s written extrajudicial confession was taken. During the investigation, appellant was assisted by the mayor as counsel. In this case, Atty. Domingo cannot be considered as an independent counsel. He was the mayor of Malolos at the time. As such, he exercised “operational supervision and control over the PNP unit in that municipality. His powers included the utilization of the elements thereof for the maintenance of peace and order, the prevention of crimes, the arrest of criminal ofenders and the bringing of ofenders to justice. Hence, the extrajudicial confession taken from Velarde cannot be used as evidence. Issue (2): WON appellant was assisted by a competent and independent counsel during the custodial investigation.

TAU MU

Ruling: The right to counsel is a fundamental right and contemplates not just the mere presence of a lawyer beside the accused. The competent and independent lawyer so engaged should be present “at all stages of the interview, counseling or advising caution reasonably at every turn of the investigation, and stopping the interrogation once in a while either to give advice to the accused that he may either continue, choose to remain silent or terminate the interview. The desired role of counsel in the

ACADCOM 2010; Contributors: Gene Geocaniga, Jarissa Guiani, Darlene Magabilen TAU MU Page 136 of 179

TAU MU TAU MU TAU MU TAU MU TAU MU TAU MU TAU MU TAU MU

Ruling: No. In providing that during the taking of the extra-judicial confession the parents of the accused, older brother and sisters, his spouse, the municipal mayor, municipal judge, district school supervisor, priest or minister of the gospel may be present, RA No. 7438 does not suppose that they appear as substitute for counsel. It applies only when counsel of the accused is absent and a valid waiver had been executed. Hence, in this case, in the absence of such valid waiver, the priest, mayor or relatives of the accused could not stand in lieu of counsel. The apparent consent of the accused in continuing with the investigation was of no moment as a

TAU MU

Facts: Accused were suspects in a rape with homicide case. They voluntarily went to the police station to confess. Since there was no lawyer to assist them, the police requested the presence of the parish priest and the municipal mayor as well as the relatives of the accused to obviate the possibility of coercion. Are their confessions admissible in evidence?

TAU MU TAU MU

PEOPLE vs. ORDONO 334 SCRA 673 (2000)

The Fraternal Ateneo de Davao

TAU MU TAU MU

process of custodial investigation is rendered meaningless if the lawyer merely gives perfunctory advice as opposed to a meaningful advocacy of the rights of the person undergoing questioning. If the advice given is so cursory as to be useless, voluntariness is impaired. As mayor of Malolos, his duties were inconsistent with those of his responsibilities to appellant, who was already incarcerated and tagged as the main suspect in the rape-slay case. Serving as counsel of appellant placed him in direct conflict with his duty of “operational supervision and control” over the police. “What the Constitution requires in Article III Section 12 (1) is the presence of competent and independent counsel, one who will efectively undertake his client’s defense without any intervening conflict of interest.” Evidently Atty. Domingo, being the mayor of the place where the investigation was taken, could not act as counsel, independent or otherwise, of appellant. During the investigation, Atty. Domingo failed to act as the independent and competent counsel envisioned by the Constitution. He failed to give any meaningful advice to protect the rights of appellant. The former did not even bother to inform the latter of the consequences of an extrajudicial confession. It is significant to point out that, during the cross-examination and perhaps in total confusion, the investigator even went so far as to state that Atty. Domingo had not acted as appellant’s lawyer. If this were so, then appellant had absolutely no counsel when his extra-judicial confession was taken. Thus, it is clear that, in palpable violation of the Constitution, appellant was not assisted by a competent and independent counsel during the custodial investigation and the taking of his extra-judicial confession. Hence, the Court is duty-bound to disregard it.

KITY

TAU MU TAU MU TAU MU TAU MU TAU MU TAU MU

CONSTITUTIONAL LAW II Order of Saint Thomas More Atty. Philip John Pojas/Atty. Rovyne G. Jumao-as, RN University College of Law

waiver to be efective must be made in writing and in the presence of counsel. PEOPLE vs. TOMAQUIN 435 SCRA 23 (2004) Facts: Appellant was convicted for the crime of murder. Issue: WON the extra-judicial confession executed by appellant is admissible in evidence against him. Ruling: It is not legally sociable to consider a brgy. captain an independent counsel of appellant. What the Constitution requires is the presence of an independent and competent counsel, one who will efectively undertake his client’s defense without conflict of interest. The appellant chose Atty. Parawan who did not stop him from complaining about the latter’s failure to safeguard his rights. Atty. Parawan failed to meet the exacting standards of an independent and competent counsel as required by Constitution. The extra-judicial confession executed by appellant, even if gospel truth, is deemed an uncounselled confession and therefore, inadmissible in evidence. PEOPLE vs. LABTAN 320 SCRA 140 (1999) Facts: Accused was charged with highway robbery and robbery with homicide. After trial, he was convicted mainly based on his extra-judicial confession. While he agreed to be assisted by a lawyer given to him by the investigator, said lawyer has been engaged by the police 3 times and would sometimes be paid by the police for his services. He was also the one who notarized the confession. Is the confession admissible? Ruling: No. The Constitution requires that the counsel assisting the suspect must be competent and independent. The independence of the lawyer in this case is suspect since he was regularly engaged by the police as counsel for suspects who cannot aford the services of a lawyer and even received money from the police as payment of his services. His act of notarizing the statement also seriously compromised his independence. By doing so, he vouched for the regularity of the circumstances surrounding the taking of the sworn statement by the police. He cannot serve as counsel of the accused and the police at the same time. There was a serious conflict of interest on his part. PEOPLE vs. OBERO 332 SCRA 190 (2000) Facts: Accused was a suspect in a robbery with homicide case. He signified his intention to confess but had no lawyer to assist him. Coincidentally, the Station Commander of the WPD Headquarters, UN Avenue, Manila, who was a lawyer, happened to be at Station 7 of the WPD,

ACADCOM 2010; Contributors: Gene Geocaniga, Jarissa Guiani, Darlene Magabilen TAU MU Page 137 of 179

so that the investigator requested him to assist accused. After the lawyer apprised him of his constitutional rights, accused confessed to the commission of the crime. Is the confession admissible?

TAU MU TAU MU TAU MU TAU MU TAU MU TAU MU

RIGHT TO BE INFORMED  This presupposes a transmission of meaningful information, not just a ceremonial recitation of an abstract principle.  Presupposes a transmission of meaningful information, not just ceremonial recitation of an abstract principle. The police must explain the meaning of the provision in a language the accused fairly understands. This will depend on factors like, the educational background and the intelligence of the accused.

TAU MU

Ruling: No. Even as the person under custodial investigation enjoys the right to counsel from its inception, so does he enjoy such right until its termination – indeed, “in every phase of the investigation.” An efective and vigilant counsel necessarily and logically requires that the lawyer be present and able to advise and assist his client from the time the confessant answers the first question asked by the investigating officer until the signing of the extra-judicial confession. The lawyer in this case can hardly be the counsel that the framers contemplated as “competent”. Neither can he be described as “vigilant” and “efective” which jurisprudence require.

TAU MU

Facts: Accused was convicted of robbery with homicide. At the start of the custodial investigation he was assisted by a lawyer who warned him of his rights and asked him if he is willing to answer the questions of the police investigator. During the investigation, the lawyer “would come and go” within the police station. Midway along, after all “the material points” have been asked, the lawyer left for an important engagement with the consent of accused. The lawyer also instructed the police to bring the accused and the document to him after the written confession had been prepared for “further examination.” Is the confession admissible?

TAU MU TAU MU

PEOPLE vs. MORIAL 363 SCRA 96 (20001)

The Fraternal Ateneo de Davao

TAU MU TAU MU

Held: No. Accused was assisted by a lawyer, who, though presumably competent, cannot be considered as “independent counsel” as contemplated by law for the reason that he was station commander of the WPD. The lawyer was part of the police force who could not be expected to have efectively and scrupulously assist accused in the investigation, his claim to the contrary notwithstanding. To allow him to assist in confessions would render illusory the protection given to the suspect during custodial investigation.

KITY

TAU MU TAU MU TAU MU TAU MU TAU MU TAU MU

CONSTITUTIONAL LAW II Order of Saint Thomas More Atty. Philip John Pojas/Atty. Rovyne G. Jumao-as, RN University College of Law

Q: Is the legal officer of a municipality qualified to assist the accused? A: He is not an independent counsel because as legal officer, it is his function to assist the mayor including the reinforcement of peace and order. Obviously, there is conflict of interest. CASES: TO BE INFORMED PEOPLE vs. CANELA Facts: Prior to the investigation, the police let the accused read the rights. When the case reached the SC, the accused raised the issue on whether he was sufficiently informed in accordance with the constitutional requirement. Ruling: The court held that asking the accused just to read his rights will not be sufficient under the idea of “meaningful transmission of information”. The accused should be made sure to have understood his rights. PEOPLE vs. NICANDRO Facts: Police investigator was presented by the fiscal. The fiscal asked about the investigation. The policeman answered “the first thing I did was to inform the accused of his rights. And the next I questioned her about the marijuana”. Issue: What is the meaning of “meaningful transmission of information”? Ruling: The court held that there was no compliance with the requirement of the Constitution. If this statement only appeared during the direct examination, it is obvious that the police did not inform her as to what these specific rights are: This is a general statement. The policeman, must explain to her in practical terms which she can understand. PEOPLE vs. KADIWA Facts: The accused, upon being informed of his rights was only made to answer one word , “OPO”. Ruling: The court ruled there was no sufficient compliance of the right to be informed. The function was kilometric while the answer was monosyllabic, which does not show that the accused properly understood his rights. (NOTE: There is no formula but merely depends on the education, literacy, etc. of the accused) PEOPLE vs. BANDULA Facts: Two people were arrested and they were immediately investigated without counsel. Next day, some of the accused executed extra judicial confessions assisted by counsel. Two weeks later, the other one also made a confession.

TAU MU

Issue: Whether the confession of accused be considered as admissible.

ACADCOM 2010; Contributors: Gene Geocaniga, Jarissa Guiani, Darlene Magabilen TAU MU Page 138 of 179

the

two

Ruling: The court rules the confession as not admissible. Confession made without counsel although later reduced to writing with the assistance of counsel is not admissible. Late arrival of counsel will not cure the defect. ADMISSIBILITY OF CONFESSIONS 1987 Constitution

JUDICIAL

RA 7438 (April 1992) 1. Must be in writing under the situation wherein the right to counsel attaches. 2. Must be with counsel

TAU MU TAU MU TAU MU TAU MU

A mayor cannot be considered the independent lawyer referred to by the Constitution. [PEOPLE vs. TALIMAN, 342 SCRA 534]

TAU MU

A municipal attorney could not be an independent counsel as required by the Constitution. We reasoned that as legal officer of the municipality, he provides legal assistance and support to the mayor and the municipality in carrying out the delivery of basic services to the people, including the maintenance of peace and order. It is therefore seriously doubted whether he can efectively undertake the defense of the accused without running into conflict of interests. [PEOPLE vs. BANDULA, ]

TAU MU

The extrajudicial confession of the accusedappellant was inadmissible as he was ‘assisted’ by the incumbent municipal attorney. [PEOPLE vs. CULALA]

TAU MU TAU MU

3. With counsel in writing -- admissible with counsel, not in writing -not admissible. (When is it admissible? When the right does not attach)

The Fraternal Ateneo de Davao

TAU MU TAU MU

1. Need not be in writing, but must be done in the presence of counsel 2. if extra judicial confession made w/o counsel - inadmissible, because it has to be in the presence of counsel 3. If made in the presence of counsel, but not in writingvalid, admissible (because need not be in writing. If the police will testify that the confession was made in the presence of counsel, then that testimony can be admitted in evidence)

EXTRA

KITY

TAU MU TAU MU TAU MU TAU MU TAU MU TAU MU

CONSTITUTIONAL LAW II Order of Saint Thomas More Atty. Philip John Pojas/Atty. Rovyne G. Jumao-as, RN University College of Law

WAIVER of the rights under Sec. 12 (1):  The law says that “these rights cannot be waived except in writing and in the presence of counsel”.

TAU MU

Q: What rights may be waived? A: Only the right to remain silent and the right to counsel may be waived. The right to be informed of the said rights cannot be waived.

TAU MU

Q: How can the waiver be made?

A: The waiver must be: 1. In writing and 2. Done in the presence of counsel [Sec. 12 (1)] 3. The waiver must be in language w/c clearly manifests the desire to waive the right [PEOPLE vs. GALIT, 135 SCRA 465]. In localities where there are no lawyers, the state must bring the individual to a place where there is one or bring counsel to the place where the person is held. 

WAIVER It is defined as the abandonment or relinquishment of a right. What can the accused waive? - right to remain silent - right to counsel NOTE: The right to be informed cannot be waived APPLICATION OF THE RIGHTS IN SECTION 12: NOTE: Enactment of 1935 Constitution – January 17, 1973 Moncupa ruling was promulgated – April 26, 1983 Enactment of the 1987 Constitution – February 2, 1987 Enactment of RA 7438 – Prior to January 17, 1973 - There was no right to counsel and the right to remain silent under the 1935 Constitution. (Miranda vs State of Arizona was only decided on 1956) MAGTOTO vs. MANGERA Facts: The accused made an extra judicial confession prior to Jan. 17, 1973. He had not been informed and had not availed of his right to the assistance of counsel. He was tried after the efectivity of the 1935 Constitution. RULING: The court held the 1935 Constitution had no retroactive efect. And the confession he made was held admissible against him.

TAU MU

After January 17, 1973 - The right to remain silent and the right to counsel is now protected and that those rights may be waived, and there is no formality. It can be waived in any manner, but must be voluntary and intelligently made even without a lawyer to assist the accused. (No specific form of waiver). From April 26, 1983 MONCUPA VS ENRILE The right to counsel may only be waived with the assistance of counsel. And it must also

ACADCOM 2010; Contributors: Gene Geocaniga, Jarissa Guiani, Darlene Magabilen TAU MU Page 139 of 179

CONSTITUTIONAL LAW II Order of Saint Thomas More Atty. Philip John Pojas/Atty. Rovyne G. Jumao-as, RN University College of Law

KITY

Ateneo de Davao

PEOPLE vs. DACUYCUY Facts: Confession was obtained prior to April 26, 1983 and accused waived his right without the assistance of a counsel. The issue is whether there is a retroactive efect in this rule. Ruling: The court held that this rule should be applied even to confessions obtained prior to April 26, 1983. This was given a retroactive efect.

TAU MU TAU MU TAU MU TAU MU TAU MU

Under RA 7438, Sec. 2(1) - There is the requirement of a form in order for the waiver to be valid. - After the lawyer signs the waiver, he can already leave and need not be present during the time when the confession is made. - However, under RA 7438, after a valid waiver, signed by the lawyer, it is now required that the confession is signed by the accused in the presence of other additional people although they need not sign anymore. And these are the either of the following: - parents - brothers - sisters - spouse - mayor - priest

TAU MU

NOTE: Under the 1987 Constitution, there is no requirement that the confession should be in writing. The requirement is only that the waiver should be in writing.

TAU MU

a

TAU MU

According to Fr. Bernas, if asked in the BAR, just cite the two sets of cases.

TAU MU

Prior to the Moncupa rule, it is unfair to expect the police to follow the said rule since the decision has not been promulgated. So why give efect to it? IMPORTANT: There are two sets of decisions in this matter that do not reach a consensus one agrees that there is retroactive efect and the other agrees that it has none.

TAU MU TAU MU

There was retroactive efect to the rule in Moncupa. However, in People vs Magalona and People vs Ponce, the Court stated that the rule that waiver must be done with the assistance of counsel has no retroactive efect. The rule on Moncupa should only be used as a guideline after the decision on that case.

TAU MU TAU MU

PEOPLE vs. PECARDAL

TAU MU TAU MU TAU MU TAU MU TAU MU TAU MU

be made in writing. Otherwise, the confession is not admissible in court.

From February 2, 1987 - The waiver must be done: - In the presence of counsel - It must be in writing

The Fraternal

- etc. - In addition to the requirements of the waiver to be with assistance of counsel and in writing, the confession itself must be signed in the presence of the other people (exclusively enumerated in the law, but their signature is not required. - If there is no waiver, there is no requirement that the other people should be present, only his counsel. OTHER CHANGES BROUGHT ABOUT BY RA7438 Under RA 7438, it requires that the Custodial Investigation report same with the confession must be in writing, otherwise, such report shall be void. Oral confessions are now also void. The list of visitors are now expanded – Section 2(f) A penalty is provided for failure to inform and to provide competent counsel to the accused. (8 to 10 years of imprisonment). It also defined as to who can assist the accused as counsel – Section 3 It also interpreted the meaning of whether the police or investigator can change the lawyer for the accused – Section 8 Custodial investigation now includes “invitations”. SUMMARY OF RIGHTS GRANTED BY SECTION 12(1): 1. The right to remain silent: meaning he has the right to refuse to answer any question and his silence cannot be used against him. 2. The right to competent and independent counsel preferably of his own choice: to preclude the slightest coercion that would be used against the accused to confess something 3. The right to be informed of these rights: it is not enough that the police officer will just repeat to the person under investigation the constitutional provision, he must also explain the efects in practical terms. It contemplates an efective communication that results in the understanding of the rights contemplated. 4. (Right added by 7438): Visitorial rights: The right to be visited by family, friends, lawyers, doctors, etc. The person being investigated can be visited. SECTION 12 (2)  This provision contains the right against coerced confessions. SECTION 12 (3)  The exclusionary rule. The law says “Any confession or admission obtained in violation of this or Sec. 17 hereof shall be inadmissible in evidence against him”. Confessions made in violation of Sec. 12 are inadmissible in evidence. How do you understand "made in violation of Sec. 12"? a. If the confession is done without the lawyer; (uncounseled confession)

ACADCOM 2010; Contributors: Gene Geocaniga, Jarissa Guiani, Darlene Magabilen TAU MU Page 140 of 179

b. When the waiver is not valid (when without presence of lawyer, and not in writing) c. Confession is gathered through force, intimidation, etc. (sec. 12 paragraph 2) d. No meaningful transmission of right. e. Oral confession (in violation of RA 1348), because confessions have to be in writing. OTHER CONFESSIONS WHICH ARE EXCLUDED: (even with counsel or if such was true)

TAU MU TAU MU

According to the Court, there is no presumption that the police have given the Miranda warning to the accused. During the trial,

TAU MU

PEOPLE vs. TOLENTINO

TAU MU

Miranda

TAU MU

1. No presumptions that the Warning has been given. (General Rule)

TAU MU

PRESUMPTIONS Presumptions are Assumptions of Fact which are made by law in order to dispense with evidence.

TAU MU

EXCEPTIONS: (A) To prove that there is a conspiracy. (B) To be used as evidence against police abuse. (C) To impeach the accused person’s credibility.

TAU MU

SCOPE OF INADMISSIBILITY: (1) They are inadmissible against the confessant. (2) They are inadmissible for the purpose of any proceeding: (criminal, civil and administrative proceedings). However, if a confession is made in an administrative proceeding, then that is admissible. (3) When the confession was made and where the confession was made are diferent. You have to look into both. (4) The confession is inadmissible against third persons.

TAU MU

1. Principle of Humanity – the law abhors all forms of violence inflicted upon another. 2. Also, these confessions are generally unreliable.

TAU MU TAU MU

REASONS WHY CONFESSIONS OBTAINED THROUGH DURESS OR TORTURE ARE INADMISSIBLE:

The Fraternal Ateneo de Davao

TAU MU TAU MU

1. Where the accused is promised that he would be released if he signs. (People vs Eglipa) 2. When the accused is promised a better treatment if he signs. (People vs Albano, 145 S 155) 3. All oral confessions are inadmissible. It should always be in writing to be admissible as evidence. 4. Confessions obtained after a valid waiver, but not signed in the presence of person enumerated in RA 7438 5. All other means which vitiate the person’s will.

KITY

TAU MU TAU MU TAU MU TAU MU TAU MU TAU MU

CONSTITUTIONAL LAW II Order of Saint Thomas More Atty. Philip John Pojas/Atty. Rovyne G. Jumao-as, RN University College of Law

the prosecution must prove that the police have warned the accused of his rights under Section 12 in the absence of proof. It is presumed that there was no warning given and the evidence is inadmissible. There is no presumption of regularity of official acts under the Miranda Rule. 2.

No presumption on the validity of a waiver.

PEOPLE vs. JARA Whenever there is a waiver of constitutional rights, then presumption is always against the waiver. The presumption is that there was no waiver given. If the prosecution claims there was a waiver. They must prove with strong and convincing evidence to the satisfaction of the Court. Also, there is no presumption of the regularity of official acts in this case. What must the prosecution prove? The presumption is that there was no valid waiver except if the prosecution can prove: That the accused willingly and voluntarily made his confession That the accused was not interested in having a lawyer That the waiver complied with all the requirements to make a valid one There was a valid Miranda Warning Mere presentation of the text of the waiver is not sufficient to prove the above- mentioned. 3. There now the presumption Voluntariness of the Confession.

of

PEOPLE vs. ENANORIA An opposite presumption. Atty. Jocon was present to assist the accused in giving his statement. There is now a presumption on voluntariness. There is now a presumption of the regularity of official acts. This means that there is a presumption that the police regularly perform and comply with proper procedures, and that they do not have to torture the accused to get the confession. It is presumed that the confession was voluntarily made. The police do not have to prove that they did not torture the accused in order to get the confession. The law does not presume that the police are torturers. The burden is now on the accused to prove that he was tortured and the confession was involuntarily made. NOTE: There was strong evidence in this case that the accused gave his testimony uncoerced and freely. PEOPLE VS BALLISTEROS An obiter dictum. One accused signed without the assistance of a lawyer. Later, during the trial, he was made a State witness against his co-accused. He repeated his confession in open court. His confession was considered admissible as evidence.

ACADCOM 2010; Contributors: Gene Geocaniga, Jarissa Guiani, Darlene Magabilen TAU MU Page 141 of 179

The rule stands that any confession obtained with violation of the Constitution shall be held inadmissible as evidence in Court. This is based on the 1987 Constitution. His confession shall be inadmissible in Court only if this is used against him, but this does not include confessions to be used as evidence against third persons. (A dangerous ruling is made. This should not be considered as a new string.)

CASES: EXCLUSIONARY RULE

TAU MU TAU MU

Ruling: The alleged extra-judicial confession of Janson cannot be admitted in evidence. The manner which it was obtained violated accused constitutional right to counsel. Even if the confession contains a grain of truth, it was made without the assistance of counsel, it becomes inadmissible in evidence, regardless of the absence of coercion or even if it had been voluntarily given. Assistance of counsel is

TAU MU

Issue: WON the alleged extra-judicial confession of Janson is inadmissible because it was obtained in violation of the constitutional rights of the accused.

TAU MU

Facts: The RTC declared the appellants guilty for the crime of robbery with rape. Janson’s statement was signed by him without force. No lawyer was present during the custodial investigation. The affidavit was signed by Atty. Zerudo after the investigation was conducted.

TAU MU

PEOPLE vs. JANSON 400 SCRA 584 (2003)

TAU MU

Q: Are they admissible against the persons who obtained the confession/admission? A: YES. They may be used as evidence against the person violating the constitutional prohibition.

TAU MU

It is the person alone who made the confession/admission who can ask for its exclusion [PEOPLE vs. BALISTEROS, 237 SCRA 499]. 

TAU MU

ADMISSION  is the act, declaration or omission of a party as to a relevant fact [Rule 130 Sec. 26]

TAU MU

CONFESSION  the declaration of an accused acknowledging his guilt of the ofense charged, or of any ofense necessarily included therein [Rule 130 Sec. 33]

TAU MU TAU MU

Q: Against whom does the confession/admission be inadmissible as evidence? A: Against the person making the confession or admission.

The Fraternal Ateneo de Davao

TAU MU TAU MU

PRESUMPTIONS: 1. There is no presumption that Miranda warning has been given; 2. No presumption that the waiver is valid; 3. Presumption that the confession is voluntarily made;

KITY

TAU MU TAU MU TAU MU TAU MU TAU MU TAU MU

CONSTITUTIONAL LAW II Order of Saint Thomas More Atty. Philip John Pojas/Atty. Rovyne G. Jumao-as, RN University College of Law

required even when he waives the right to counsel. A confession, to be admissible, must satisfy the following requirement: (1) it must be voluntary, (2) it must be made with the assistance of an independent and competent counsel, (3) it must be express and (4) it must be in writing. The purpose is to curb the uncivilized practice of extracting confession by coercion no matter how slight, as would lead the accused to admit something false. Any waiver of the right of counsel without the assistance of counsel has no evidentiary rule. In this case, it cannot be said that the waiver of the right to counsel was made knowingly and intelligently. Admissions obtained during custodial investigation without the benefit of counsel, although later reduced in writing and signed in the presence of counsel, are still flawed under the Constitution. If the lawyer’s role is reduced to being that of a mere witness to the signing of a priority prepared document albeit indicating thereon compliance with the accused constitutional rights, the constitutional standard is not met. MARCELO vs. SANDIGANBAYAN 302 SCRA 102 (1999) Facts: Accused was an emergency laborer of Makati Central Post Office. He was caught stealing 622 letters some of them consisting US dollar bills. After his arrest, he was asked to affix his signatures on the envelopes of the letters in the presence of National Bureau of Investigation (NBI) agents but without the assistance of counsel. Are the letters admissible in evidence? Ruling: Accused signed the envelopes following his arrest. Hence, he was at that time under custodial investigation. Under the Constitution, at this stage he had the right to counsel. Since the signatures are actually evidence of admission obtained from accused under custodial investigation, they should be excluded. However, the letters are themselves not inadmissible. The letters were validly seized from accused as an incident to a valid arrest. A ruling that his admission that the letters in question were those seized from him does not extend to the letter themselves. SECTION 12 (4)  The way the law is worded, it seems that it will need implementing legislation.

SEC. 13 BAIL – All persons, except those charged with offenses punishable by Reclusion Perpetua when evidence of guilt is strong, shall, before conviction, be bailable by sufficient sureties, or be released on recognizance as my be provided by law. The right to bail shall not be impaired even when the privilege of the writ of habeas corpus is suspended. Excessive bail shall not be required.

ACADCOM 2010; Contributors: Gene Geocaniga, Jarissa Guiani, Darlene Magabilen TAU MU Page 142 of 179

BAIL  is a mode short of confinement w/c would, w/ reasonable certainty, insure the attendance of the accused [DE LA CAMARA vs. ENAGE, 41 SCRA 1]  It is the security given for the provisional release of a person accused of a crime while the case is still pending or trial is still going on.

TAU MU TAU MU TAU MU TAU MU TAU MU TAU MU

Sec. 5. Bail, when discretionary. – Upon conviction by the Regional Trial Court of an ofense not punishable by death, reclusion perpetua, or life imprisonment, admission to bail is discretionary. The application for bail may be filed and acted upon by the trial court despite the filing of a notice of appeal, provided it has not transmitted the original record to the appellate court. However, if the decision of the trial court conviction the accused changed the nature of the ofense from non-bailable to bailable, the application for bail can only be filed with and

TAU MU

Sec. 4. Bail, a matter of right; exception. – All persons in custody shall be admitted to bail as a matter of right, with sufficient sureties, or released on recognizance as prescribed by law or this Rule (a) before or after conviction by the Metropolitan Trial Court, Municipal Trial Court, Municipal Trial Court in Cities, or Municipal Circuit Trial Court, and (b) before conviction by the Regional Trial court of an ofense not punishable by death, reclusion perpetua, or life imprisonment.

TAU MU

Sec. 3. No release or transfer except on court order or bail. – No person under detention by legal process shall be released or transferred except upon order of the court or when he is admitted to bail.

TAU MU TAU MU

Sec. 2. Conditions of the bail; requirements. – All kinds of bail are subject to the following conditions: (a) The undertaking shall be efective upon approval, and unless cancelled, shall remain in force at all stages of the case until promulgation of the judgment of the Regional Trial Court, irrespective of whether the case was originally filed in or appealed to it; (b) The accused shall appear before the proper court whenever required by the court of these Rules; (c) The failure of the accused to appear at the trial without justification and despite due notice shall be deemed a waiver of his right to be present thereat. In such case, the trial may proceed in absentia; and (d) The bondsman shall surrender the accused to the court for execution of the final judgment. The original papers shall state the full name and address of the accused, the amount of the undertaking and the conditions required by this section. Photographs (passport size) taken within the last six (6) months showing the face, left and right profiles of the accused must be attached to the bail.

The Fraternal Ateneo de Davao

TAU MU TAU MU

RULE 114 – BAIL Section 1. Bail defined. – Bail is the security given for the release of a person in custody of the law, furnished by him or a bondsman, to guarantee his appearance before any court as required under the conditions hereinafter specified. Bail may be given in the form of corporate surety, property bond, cash deposit or recognizance.

KITY

TAU MU TAU MU TAU MU TAU MU TAU MU TAU MU

CONSTITUTIONAL LAW II Order of Saint Thomas More Atty. Philip John Pojas/Atty. Rovyne G. Jumao-as, RN University College of Law

If the penalty imposed by the trial court is imprisonment exceeding six (6) years, the accused shall be denied bail, or his bail shall be cancelled upon a showing by the prosecution, with notice to the accuse, of the following or other similar circumstances: (a) That he is a recidivist, quasi-recidivist, or habitual delinquent, or has committed the crime aggravated by the circumstance of reiteration; (b) That he has previously escaped from legal confinement, evaded sentence, or violated the conditions of his bail without valid justification; (c) That he committed the ofense while under probation, parole, or conditional pardon; (d) That the circumstances of his case indicate the probability of flight if released on bail; or (e) That there is undue risk that he may commit another crime during the pendency of the appeal. The appellate court may, motu proprio or on motion of any party, review the resolution of the Regional Trial Court after notice to the adverse party in either case. Sec. 6. Capital offense defined. – A capital ofense is an ofense which, under the law existing at the time of its commission and of the application for admission to bail, may be punished with death. Sec. 7. Capital offense or an offense punishable by reclusion perpetua or life imprisonment, not bailable. – No person charged with a capital ofense, or an ofense punishable by reclusion perpetua or life imprisonment, shall be admitted to bail when evidence of guilt is strong, regardless of the state of the criminal prosecution. Sec. 8. Burden of proof in bail application. – At the hearing of an application for bail filed by a person who is in custody for the commission of an ofense punishable by death, reclusion perpetua, or life imprisonment, the prosecution has the burden of showing that evidence of guilt is strong. The evidence presented during the bail hearing shall be considered automatically reproduced at the trial but, upon motion of either party, the court may recall any witness for additional examination unless the latter is dead, outside the Philippines, or otherwise unable to testify. Sec. 9. Amount of bail; guidelines. – The judge who issued the warrant or granted the application shall fix a reasonable amount of bail considering primarily, but not limited to, the following factors: (a) Financial liability of the accused to give bail; (b) Nature and circumstance of the ofense; (c) Penalty for the ofense charged; (d) Character and reputation of the accused; (e) Age and health of the accused; (f) Weight of the evidence against the accused; (g) Probability of the accused appearing at the trial; (h) Forfeiture of other bail; (i) The fact that the accused was a fugitive from justice when arrested; and (j) Pendency of other cases where the accused is on bail. Excessive bail shall not be required.

TAU MU

Sec. 10. Corporate surety. – Any domestic or foreign corporation, licensed as a surety in accordance with law and currently authorized to act as such, may ACADCOM 2010; Contributors: Gene Geocaniga, Jarissa Guiani, Darlene Magabilen provide bail by a bond subscribed jointly by the TAU MU Page 143 of 179 accused and an officer of the corporation duly authorized by its board of directors.

tax declaration in the office of the provincial, city and municipal assessor concerned. Within the same period, the accused shall submit to the court his compliance and his failure to do so shall be sufficient cause for the cancellation of the property bond and his re-arrest and detention.

TAU MU TAU MU TAU MU TAU MU TAU MU TAU MU

When a person has been in custody for a period equal to or more than the possible maximum imprisonment prescribed for the ofense charged, he shall be released immediately, without prejudice to the continuation of the trial or the proceedings on appeal. If the maximum penalty to which the accused may be sentenced is destierro, he shall be released after thirty (30) days of preventive imprisonment.

TAU MU

Sec. 16. Bail, when not required; reduced bail or recognizance. – No bail shall be required when the law or these Rules so provide.

TAU MU

Sec. 15. Recognizance. – Whenever allowed by law or these Rules, the court may release a person in custody on his own recognizance or that of a responsible person.

TAU MU

Sec. 14. Deposit of cash as bail. – The accused or any person acting in his behalf may deposit in cash with the nearest collector of internal revenue or provincial, city, or municipal treasurer the amount of bail fixed by the court, or recommended by the prosecutor who investigated or filed the case. Upon submission of a proper certificate of deposit and a written undertaking showing compliance with the requirements of section 2 of this Rule, the accused shall be discharged from custody. The money deposited shall be considered as bail and applied to the payment of fine and costs while the excess, if any, shall be returned to the accused or to whoever made the deposit.

TAU MU TAU MU

Sec. 13. Justification of sureties. – Every surety shall justify by affidavit taken before the judge that he possesses the qualification prescribed in the preceding section. He shall describe the property given as security, stating the nature of his title, its encumbrances, the number and amount of other bails entered into by him and still undischarged, and his other liabilities. The court may examine the sureties upon oath concerning their sufficiency in such manner as it may deem proper. No bail shall be approved unless the surety is qualified.

The Fraternal Ateneo de Davao

TAU MU TAU MU

Sec. 12. Qualifications of sureties in property bond. – The qualifications of sureties in a property bond shall be as follows: (a) Each must be a resident owner of real estate within the Philippines; (b) Where there is only one surety, his real estate must be worth at least the amount of undertaking; (c) If there are two or more sureties, each may justify in an amount less than that expressed in the undertaking but the aggregate of the justified sums must be equivalent to the whole amount of the bail demanded. In all cases, every surety must be worth the amount specified in his own undertaking over and above all just debts, obligations and properties exempt from execution.

KITY

TAU MU TAU MU TAU MU TAU MU TAU MU TAU MU

CONSTITUTIONAL LAW II Order of Saint Thomas More Atty. Philip John Pojas/Atty. Rovyne G. Jumao-as, RN University College of Law

trial judge in the province, city or municipality. If the accused is arrested in a province, city, or municipality other than where the case is pending, bail may also be filed with any regional trial court of said place, of if no judge thereof is available, with any metropolitan trial judge, municipal trial judge, or municipal circuit trial judge therein. (b) Where the grant of bail is a matter of discretion, or the accused seeks to be released on recognizance, the application may only be filed in the court where the case is pending, whether on preliminary investigation, trial, or appeal. Any person in custody who is not yet charged in court may apply for bail with any court in the province, city, or municipality where he is held. Sec. 18. Notice of application to prosecutor. – In the application for bail under section 8 of this Rule, the court must give reasonable notice of the hearing to the prosecutor or require him to submit his recommendation. Sec. 19. Release on bail. – The accused must be discharged upon approval of the bail by the judge with whom it was filed in accordance with section 17 of this Rule. When bail is filed with a court other than where the case is pending, the judge who accepted the bail shall forward it, together with the order of release and other supporting papers, to the court where the case is pending, which may, for good reason, require a diferent one to be filed. Sec. 20. Increase or reduction of bail. – After the accused is admitted to bail, the court may, upon good cause, either increase or reduce its amount. When increased, the accused may be committed to custody if he does not give bail in the increased amount within a reasonable period. An accused held to answer a criminal charge, who is released without bail upon filing of the complaint or information, may, at any subsequent stage of the proceedings and whenever a strong showing of guilt appears to the court, be required to give bail in the amount fixed, or in lieu thereof, committed to custody. Sec. 21. Forfeiture of bail. – When the presence of the accused is required by the court or these Rules, his bondsmen shall be notified to produce him before the court on a given date and time. If the accused fails to appear in person as required, his bail shall be declared forfeited and the bondsmen given thirty (30) days within which to produce their principal and to show why no judgment should be rendered against them for the amount of their bail. Within the said period, the bondsmen must: (a) produce the body of their principal or give the reason for his non-production; and (b) explain why the accused did not appear before the court when first required to do so. Failing in these two requisites, a judgment shall be rendered against the bondsmen, jointly and severally, for the amount of the bail. The court shall not reduce or otherwise mitigate the liability of the bondsmen, unless the accused has been surrendered or is acquitted.

Sec. 22. Cancellation of bail. – Upon application of A person in custody for a period equal to or more the bondsmen, with due notice to the prosecutor, ACADCOM 2010; Contributors: Geocaniga, Darlene Magabilen than the minimum of the principalGene penalty prescribed Jarissa Guiani, the bail may be cancelled upon surrender of the TAU MU Page 144 of 179 for the ofense charged, without application of the accused or proof of his death. Indeterminate Sentence Law or any modifying circumstance, shall be released on a reduced bail or The bail shall be deemed automatically cancelled on his own recognizance, at the discretion of the upon acquittal of the accused, dismissal of the

KITY

undertaking, cause him to be arrested by a police officer or any other person of suitable age and discretion.

TAU MU TAU MU

An accused released on bail may be re-arrested without the necessity of a warrant if he attempts to depart from the Philippines without permission of the court where the case is pending.

TAU MU TAU MU

Sec. 24. No bail after final judgment; exception. – No bail shall be allowed after a judgment of conviction has become final. If before such finality, the accused applies for probation, he may be allowed temporary liberty under his bail. When no bail was filed or the accused is incapable of filing one, the court may allow his release on recognizance to the custody of a responsible member of the community. In no case shall bail be allowed after the accused has commenced to serve sentence.

1. that the accused shall appear before the proper court whenever required by the court or by the Rules 2. absence without justification is a waiver of his right to be present in the hearing WHO ARE ENTITLED TO BAIL: 1) All persons ACTUALLY DETAINED 2) shall, BEFORE CONVICTION 3) Be entitled to bail. WHO ARE NOT ENTITLED TO BAIL: 1) Persons charged with ofenses PUNISHABLE by RECLUSION PERPETUA or DEATH, when evidence of guilt is strong 2) Persons CONVICTED by the trial court. Bail is only discretionary pending appeal. 3) Persons who are members of the AFP facing a court martial. OTHER RIGHTS IN RELATION TO BAIL: 1) The right to bail shall NOT be impaired even when the privilege of the writ of habeas corpus is suspended. 2) Excessive bail shall not be required.  As soon as one is placed under the custody of law, the accused that was confined in the hospital was allowed to post bail. The reason is he expressly submitted to physical and legal control of the court over his person by filing of application for bail, furnishing the court of his whereabouts and unequivocally recognizing the jurisdiction of the court. [PADERANGA vs. CATheory of Constructive Custody] WHEN IS BAIL A MATTER OF RIGHT: 1. Before conviction in the MTC 2. All appealed cases from the MTC to the RTC and to the CA or the SC, but only when original court is MTC 3. Original cases decided by the RTC when the imposable penalty is less than RP 4. Original cases decided by the RTC when the penalty is RP or above but the evidence of guilt is not strong.

TAU MU

IMPLICIT LIMITATIONS ON THE RIGHT TO BAIL: 1. The person claiming the right must be in actual detention or custody of the law. 2. The constitutional right is available only in criminal cases, not, e.g. in deportation proceedings.

TAU MU

Note: 1. Right to bail is not available in the military. 2. Apart from bail, a person may attain provisional liberty through recognizance.

TAU MU

TAU MU

DUTIES IMPOSED ON THE JUDGE WHEN AN APPLICATION FOR BAIL IS FILED:

TAU MU

CONDITIONS OF BAIL

TAU MU

Sec. 26. Bail not a bar to objections on illegal arrest, lack of or irregular preliminary investigation. – An application for or admission to bail shall not bar the accused from challenging the validity of his arrest or the legality of the warrant issued therefore, or from

TAU MU

A monthly report of such visitation shall be submitted by the executive judges to the Court Administrator which shall state the total number of detainees, the names of those held for more than thirty (30) days, the duration of detention, the crime charged, the status of the case, the cause for detention, and other pertinent information.

TAU MU

In cities and municipalities to be specified by the Supreme Court, the municipal trial judges or municipal circuit trial judges shall conduct monthly personal inspections of the municipal jails in their respective municipalities and submit a report to the executive judge of the Regional Trial Court having jurisdiction therein.

TAU MU

Sec. 25. Court supervision of detainees. – The court shall exercise supervision over all persons in custody for the purpose of eliminating unnecessary detention. The executive judges of the Regional Trial Courts shall conduct monthly personal inspections of provincial, city, and municipal jails and the prisoners within their respective jurisdictions. They shall ascertain the number of detainees, inquire on their proper accommodation and health and examine the condition of the jail facilities. They shall order the segregation of sexes and of minors from adults, ensure the observance of the right of detainees to confer privately with counsel, and strive to eliminate conditions inimical to the detainees.

PURPOSE OF BAIL To recognize the presumption of innocence accorded to every accused upon whom should not be inflicted incarceration at the outset since after trial he would be entitled to acquittal, unless his guilt be established beyond reasonable doubt.

The Fraternal Ateneo de Davao

TAU MU TAU MU TAU MU TAU MU TAU MU TAU MU

CONSTITUTIONAL LAW II Order of Saint Thomas More Atty. Philip John Pojas/Atty. Rovyne G. Jumao-as, RN University College of Law

1. notify the prosecutor of the hearing of the application for bail and require him to submit his recommendation 2. conduct a hearing (mandatory!) of the application regardless of whether the prosecution

ACADCOM 2010; Contributors: Gene Geocaniga, Jarissa Guiani, Darlene Magabilen TAU MU Page 145 of 179

presents evidence or not that the guilt of the accused is strong; 3. decide whether or not the evidence of accused’s guilt is strong based on the summary of prosecution’s evidence; 4. if accused’s guilt is NOT strong, discharge him upon approval of the bailbond.

TAU MU TAU MU TAU MU TAU MU TAU MU TAU MU

BAIL SHOULD BE DENIED in Rule 114 

TAU MU

before conviction by the RTC on an 

TAU MU

Bail is a MATTER Bail is a MATTER OF OF RIGHT DISCRETION  all persons  persons charged w/ charged w/ an ofenses punishable by ofenses NOT RP when evidence of punishable by RP guilt is strong [Sec. 13] [Sec. 13]  upon conviction by  before or after the RTC of an ofense conviction by the NOT punishable by MTC, MTCC, MeTC or death, RP, or life MCTC [Rule 114 Sec. imprisonment [Rule 114 4] Sec. 5]

TAU MU

BAIL AS A MATTER OF DISCRETION: 1. Cases decided by the RTC, appealed to the CA/SC and the circumstances of AO 12-94 is not present.

TAU MU TAU MU

WHEN SHOULD BAIL BE DENIED: 1. After final judgment by any court 2. Before conviction for an ofense punishable by death or reclusion perpetua where the evidence of guilt is strong. 3. After conviction for a crime punishable by reclusion perpetua or death while the case is on appeal [PEOPLE vs. VALERIANO] 4. After conviction for an ofense with the penalty exceeding 6 years but not more than 20 years, if: a. accused is a recidivist, quasirecidivist, habitual delinquent or has committed a crime aggravated by reiteracion b. accused is found to have previously escaped from legal confinement c. accused committed the ofense while on probation, parole or conditional pardon d. circumstances of accused or his case indicate the probability of flight e. there is undue risk that during the pendency of the appeal, accused may commit another crime. (SC Administrative Circular No. 12-94) This includes punishment of life imprisonment.

The Fraternal Ateneo de Davao

TAU MU TAU MU

The court order granting/refusing bail should contain a summary of the prosecution’s evidence followed by the conclusion on whether or not the evidence of guilt is strong.  The court cannot merely rely on the affidavits and recitals of its contents, if timely objected to, because they are only hearsay --- insufficient to establish quantum of proof needed w/c is strong evidence. 

KITY

TAU MU TAU MU TAU MU TAU MU TAU MU TAU MU

CONSTITUTIONAL LAW II Order of Saint Thomas More Atty. Philip John Pojas/Atty. Rovyne G. Jumao-as, RN University College of Law

ofense NOT punishable by death, RP, or life imprisonment [Rule 114 Sec. 4]

Sec. 5 par. 3

WHERE SHOULD BAIL BE FILED (SEC 17) General rule: in the court where the case pending Exceptions: 0 (1) Where the judge of the court where the case is pending is absent or unavailable, bail may be filed to any RTC or MTC of the province, city or municipality where the case pending. 1 (2) Where the accused is arrested in a province, city, or municipality other than where the case is pending, bail may be filed in any RTC or MTC of that place 2 (3) Any person in custody who is not yet charged, in the RTC or MTC of the province, city or municipality where he is held.  Mitigating circumstances are not yet considered to determine the penalty because in the end, it will only result in a full blown trial. Although mitigating circumstances in general are not considered yet. The mitigating circumstance of MINORITY may be considered in granting bail.  AC # 12-94 is constitutional because it only limits the right to bail after conviction while the case is on appeal. The right to bail is only guaranteed by the Constitution before conviction, not after conviction while the case is on appeal. The Constitution does not distinguish whether before final decision or before appeal. WHEN BAIL NOT REQUIRED 1. When a person has been in custody for a period equal to or more than the maximum imposable penalty for the ofense charged. 2. After 30 days of preventive imprisonment if the maximum penalty for the ofense is destierro. BAIL IN MILITARY COURTS Bail has traditionally not been recognized and is not available in the military as an exception to the general rule embodied in the Bill of Rights. The unique structure of the military should be enough reason to exempt military men from the constitutional coverage of the right to bail. Military are allowed the fiduciary use of firearms which could very well result in the overthrow of duly constituted authorities. POLICY ARGUMENTS (NOT LEGAL ARGUMENTS) WHICH COULD BE CONSIDERED IMPORTANT: 1. Military officers are diferent from ordinary criminals because they are entrusted with trust and confidence of the people. 2. It would also be dangerous if there are armed coup plotters walking in the streets because they are allowed to go out on bail. NOTE: These are only policy arguments and decisions should not be based solely on these.

ACADCOM 2010; Contributors: Gene Geocaniga, Jarissa Guiani, Darlene Magabilen TAU MU Page 146 of 179

COMMENDADOR vs. DE VILLA The accused was charged with violation of the Articles of War before a Court Martial. ISSUE: Whether or not a person charged before a court martial has a right to bail. RULING: The court decided that traditionally, the right to bail has not been recognized in the military courts.

PEOPLE vs. DONATO June 5, 1991 BAIL RIGHT

MTC (6 YRS AND BELOW) Matter of right all the time 

NOT ALLOW ED/ DENIED

N/A because it is always a matter of right.

DISCRE TION

N/a

RTC (6 YRS 1 DAY -)  if the accused appeals, all appealed cases from MTC, for as long as hindi pa final judgment, bail is a matter of right;  if penalty imposed is less than RP  pena lty is RP or higher, BUT the evidence of guilt is strong if the penalty is RP or higher and the evidence of guilt is strong N/A

TAU MU TAU MU TAU MU TAU MU

Efect of conviction: imprisoned.

TAU MU

4. Recognizance- the accused is entrusted to the custody of a prominent person who is likely to guarantee his appearance in court. Only for minor ofenses.

TAU MU

Efect of conviction or acquittal: the title is returned to the accused. This is limited to real property only.

TAU MU

3. Property bond- if the accused has no cash and no surety, title of real property is deposited in court. It does not need to be in the name of the accused, he can borrow. Upon tax assessment, if the value of the property is double the value of the recommended bail, the court accepts it and the accused is free to go. The accused may also use the title of another person provided he is authorized to do so.

WAIVER OF RIGHT TO BAIL

TAU MU

2. Surety Bond - a bonding company guarantees the appearance in court. Bonding company guarantee the appearance of the accused in Court, provided he pays a certain percentage of the recommended bail to the bonding company (15-20%). It should be renewed annually. The accused pays a premium every year and he cannot recover that what he has paid. (same as in an insurance premium)

Efect of acquittal: he goes free.

TAU MU

Efect of conviction or acquittal: the bond is returned. Exception: if the crime involves civil liability, the cash may go to the aggrieved party

Ateneo de Davao

TAU MU TAU MU

FORMS OF BAIL 1. Cash Bond- when the fiscal recommends a bail, and the judge approves. All the accused has to do is to deposit the cash bond to the clerk of court, and then he is free.

The Fraternal

TAU MU TAU MU

FACTORS CONSIDERED IN SETTING THE AMOUNT OF BAIL: 1) Ability to post bail 2) Nature of the ofense 3) Penalty imposed by law 4) Character and reputation of the accused 5) Health of the accused 6) Strength of the evidence 7) Probability of appearing at the trial 8) Forfeiture of previous bail bonds 9) Whether accused was a fugitive from justice when arrested 10) If accused is under bond in other cases

KITY

TAU MU TAU MU TAU MU TAU MU TAU MU TAU MU

CONSTITUTIONAL LAW II Order of Saint Thomas More Atty. Philip John Pojas/Atty. Rovyne G. Jumao-as, RN University College of Law

ACADCOM 2010; Contributors: Gene Geocaniga, Jarissa Guiani, Darlene Magabilen TAU MU Page 147 of 179

CA/SC  Wh en the case was originally decided by the MTC, appealed to the RTC and appealed to the higher courts.

Only applicable here. Because it only involves cases decided by the RTC and the penalty is Prision Mayor -- RT [If any of the instances in AO 1294 exists then bail should be denied If none of the instances exist, then bail is a matter of discretion.]

Salas was charged with rebellion with a penalty of prision mayor. (Bail here is a matter of right). The fiscal opposed the fixing of the bail bond saying that Salas is dangerous, and at the same time, citing many circumstances. RULING: The court held that if the bail is a matter of right, there is no more need of a hearing to determine whether he can enjoy the right or not, even if he is a very notorious criminal. The right is absolute. Even the fear of possibility that the accused will evade sentence is not a valid ground for denial of bail. A hearing may only be needed to fix the amount of bail and not to determine whether the accused can enjoy the right or not.

TAU MU TAU MU TAU MU TAU MU TAU MU TAU MU

Q: If a person is out on bail and he commits a crime, then he applies for bail again, can bail be granted? A: Depends. If the crime is punished by 6 years and below, kahit pa 100 times nya gawin yung crime, pwede syang magbail kasi bail is a matter of right sa MTC diba? Pero matamaan sya sa amount of bail.  The right to bail, a corollary to the right to be presumed innocent, is like the privilege of habeas

TAU MU

WHEN IS THE ACCUSED EXEMPT FROM BAIL (RA6036): 1. If ofense is punishable is 6 months or less (he is released on recognizance) 2. When the crime is covered by summary rule because there is no arrest, only notices. No arrest, no bail. 3. Rule 112 Sec. 9 (B)- if judge is satisfied that there is no necessity for placing the accused on custody, then he may issue summons instead of a warrant of arrest.

TAU MU

Note: Accused failed to raise the issue of not hearing his application the soonest possible. He only raised it during appeal. [PEOPLE vs. PANES]

TAU MU

Rights covered in the second type of rights: 1. Right to due process 2. Right against torture 3. right to free access to courts

TAU MU TAU MU

Is this considered a valid waiver of his right to bail? The court distinguished two rights: 1. Rights which are purely personal to the accused. This right can be waived. 2. Rights where the accused as well as the government are interested in. This right cannot be waived because of public policy. According to the Constitution, the right to bail is purely personal, thus, it can be waived. Hence, the waiver of Salas is valid and binding.

The Fraternal Ateneo de Davao

TAU MU TAU MU

There is also a second issue. The Solicitor General and the lawyers for Salas entered into an agreement whereby the companion of Salas would be released while Salas remains in jail. Immediately after, he filed the petition for bail. The government contended that he cannot do so because has waived his right to bail when he entered into the agreement.

KITY

TAU MU TAU MU TAU MU TAU MU TAU MU TAU MU

CONSTITUTIONAL LAW II Order of Saint Thomas More Atty. Philip John Pojas/Atty. Rovyne G. Jumao-as, RN University College of Law

corpus, another means of obtaining liberty albeit only provisional. Q: May an alien invoke the Constitutional right to bail during the pendency of deportation proceedings? A: Yes. According to the Lao Gi case, the court held that due to the harsh consequences of deportation proceedings to the life and liberty of a person, the rules on criminal procedure including the right to bail, should be applied to deportation proceedings. NOTE: Section 13 also covers proceedings not criminal in nature. STRONG EVIDENCE PROOF EVIDENT or EVIDENT PROOF  clear, strong evidence w/c leads a well-guarded dispassionate judgment to the conclusion that: a. the ofense has been committed as charged, b. accused is the guilty agent, and c. he will probably be punished capitally if the law is administered

PRESUMPTION GREAT  exists when the circumstances testified to are such that the inference of guilt naturally to be drawn therefrom is strong, clear and convincing to an unbiased judgment and excludes all reasonable probability of any other conclusion

The person claiming the right under this provision must be under actual detention or custody of the law [MENDOZA vs. CFI, 51 SCRA 369].  One is under the custody of the law either when he has been arrested or has surrendered himself to the jurisdiction of the court.  The right may NOT be impaired even when the privilege for habeas corpus is suspended. 

GENERAL RULE: The constitutional right to bail is available only in criminal proceedings. Q: How do you determine whether the evidence of guilt is strong? A: There has to be a summary hearing to determine if the evidence guilt is strong. The prosecution must be given the opportunity to present evidence, and only then can the judge grant or deny bail if the evidence of guilt is strong or not. And if the accused does not apply for bail, the judge has no right to outright fix the amount of bail. LIFE IMPRISONMENT  a penalty in special laws  does not carry accessory penalties  is indefinite in duration

ACADCOM 2010; Contributors: Gene Geocaniga, Jarissa Guiani, Darlene Magabilen TAU MU Page 148 of 179

RECLUSION PERPETUA  is imposed by the RPC  carries w/ it accessory penalties  is for a duration of 30 years, after w/c the prisoner is eligible for

RECOGNIZANCE  is an obligation of record entered into before a court guaranteeing the appearance of the accused for trial  it is in the nature of a contract between the surety and the state [PEOPLE vs. ABNER (87 Phil. 569)]  the details on how this can be obtained or when it is applicable is left to legislation RIGHT AGAINST EXCESSIVE BAIL  The right to bail can be rendered useless by a bail bond set at an exorbitant amount.

CASES: RIGHT TO BAIL PEOPLE vs. MANES 303 SCRA 231 (1999)

TAU MU TAU MU TAU MU

Facts: Fitzgerald, an Australian Citizen, actuated by lust and by the use of a “laced” drug, induced a 13 year old minor to engage in prostitution and

TAU MU

PEOPLE vs. FITZGERALD 505 SCRA 573 (2006)

TAU MU

Held: No. Trial proceeded without the accused calling the attention of the trial court to his unresolved petition. It was only appeal that he raised the issue. Thus, for failure to raise the issue at the earliest opportune time, accused is deemed to have waived the right to bail. In addition, the issue has been rendered academic by the conviction of accused. When an accused is charged with an ofense punishable b reclusion perpetua of life imprisonment or death, and the evidence of guilt is strong, bail must be denied.

TAU MU

Facts: Accused as charged with murder. He applied for bail but the judge set the case for trial without acting on his applications. He was convicted and on appeal he contended that the judge committed a serious error of law when he tried the case without resolving the application for bail. Is the contention of accused correct?

TAU MU

NOTE: However, it is very hard to determine what is excessive and what is not. It depends on many factors.

Issue: WON the CA erred when it granted Fitzgerald’s application for bail.

TAU MU

RULING: The court held that the amount of bail is excessive and it violates the Constitution. It would have been more honest if the judge denied bail rather than to grant bail on an amount beyond the person’s reach. It is like not granting bail at all.

had carnal knowledge of her. He was then charged for violating certain provisions of RA 7160. He was found guilty by the trial court. Hence, he applied for bail which was denied. He filed for a motion for new trial before the CA on the ground that new and material evidence not previously available had surfaced. The CA granted the motion. And subsequently, CA granted Fitzgerald’s application for bail. Hence, petitioner opposed and argued that CA erred in granting Fitzgerald’s Motion for Bail despite the fact that the latter was charged with a crime punishable by reclusion perpetua and the evidence of his guilt is strong.

TAU MU

The mayor of Magsaysay was implicated in the killing of 14 people and injuring 12 others in Agusan. The judge ordered his bail at P1,195,200.00.

Ateneo de Davao

TAU MU TAU MU

DELA CAMARA vs. ENAGE

The Fraternal

TAU MU TAU MU



pardon bail is a matter of discretion --- should be denied if evidence of guilt is strong!

KITY

TAU MU TAU MU TAU MU TAU MU TAU MU TAU MU

CONSTITUTIONAL LAW II Order of Saint Thomas More Atty. Philip John Pojas/Atty. Rovyne G. Jumao-as, RN University College of Law

Ruling: The right to bail emenates from of the right to be presumed innocent. It is accorded to a person in the custody of the law who may, by reason of the presumption of innocence he enjoys, be allowed provisional liberty upon filing of a security to guarantee his appearance before any court, as required under specified conditions. In this case, bail was not a matter of right for Fitzgerald but a mere privilege subject to the discretion of the CA to be exercised in accordance with the stringent requirements of Sec. 5, Rule 114. However, it is bad enough that the CA granted bail on grounds other than those stated in the Motion filed by Fitzgerald; it is worse that it granted bail on the mere claim of his illness. Bail is not a sick pass for an ailing or aged detainee or prisoner needing medical care outside the prison facility. A mere claim of illness is not a ground for bail. It may be that the trend now is for courts to permit bail for prisoners who are seriously sick. There may also be an existing proposition for the “selective decarceration of older prisoners” based on findings that recidivism rates decrease as age increases. But, in this particular case, the CA made no specific finding that respondent sufers from an ailment of such gravity that his continued confinement during trial will permanently impair his health or put his life in danger. It merely declared respondent not in the best of health even when the only evidence on record as to the latter’s state of health is an unverified medical certificate. Hence, the CA erred when it granted Fitzgerald’s application for bail. PEOPLE vs. SANDIGANBAYAN and ESTRADA 529 SCRA 764 (2007) Facts: Jinggoy Estrada was charged, together with ERAP and several others, for the crime of plunder, defined and penalized under RA No. 7080. When he was arrested, he filed a motion alleging that: (1) no probable cause exists to put him on trial and hold him liable for plunder, it appearing that he was only allegedly involved in illegal gambling and not in a “series or combination of overt or criminal acts” as required in RA 7080; and (2) he is entitled to bail as a matter of right. By reason of the denial of his motion, he interposed a petition for certiorari before the SC claiming that the respondent

ACADCOM 2010; Contributors: Gene Geocaniga, Jarissa Guiani, Darlene Magabilen TAU MU Page 149 of 179

Issue: WON the grant of bail was proper considering the fact that Jinggoy is equally guilty and liable as ERAP by his indispensable cooperation and/or direct participation in the commission of the crime of plunder.

TAU MU TAU MU TAU MU TAU MU TAU MU TAU MU TAU MU

Ruling: No. Extradition proceedings are not criminal in nature. By using the term “before conviction”, it is apparent that the right to bail in Sec. 13, Art III of the Constitution does not apply to extradition proceedings because extradition courts do not render judgments of conviction or acquittal. Moreover, the right to bail flows from

Facts: The Republic of the Philippines and the then British Crown Colony of Hong Kong signed an “Agreement for the Surrender of Accused and Convicted Persons. Two years after the agreement was signed, Hong Kong reverted back to the People’s Republic of China and became the Hong Kong Special Administrative Region. Muñoz was charged before the Hong Kong Court with 3 counts of the ofense of “accepting an advantage as agent.” He also faces 7 counts of the ofense of conspiracy to defraud, penalized by the common law of Hong Kong. Thereafter, warrants of arrest were issued against him. If convicted, he faces a jail term of 7 to 14 years for each charge. The DOJ received from the Hong Kong DOJ a request for the provisional arrest of Muñoz. The DOJ then forwarded the request to the NBI which, in turn, filed an application for the provisional arrest of private respondent. Hence, an order of arrest was issued against Muñoz. Petitioner filed a petition for the extradition of Muñoz. For his part, Muñoz filed, in the same case, a petition for bail which was opposed by petitioner. The application for bail was denied holding that there is no Philippine law granting bail in extradition cases and that private respondent is a high “flight risk.” However, when the case was raffled to PR Judge Olalia since the previous judge inhibited himself, he granted Muñoz’s application for bail.

TAU MU

Facts: The Government of the United States of America, through the Department of Justice, filed with the Regional Trial Court a petition for the extradition of Mark Jimenez who was facing various criminal charges in Florida. Petitioners prayed for his immediate arrest, and the judge set it for hearing. Thereafter, the judge issued an order for his arrest and fixed bail for his temporary liberty at P1 million. In extradition proceedings, is the prospective extraditee entitled to post bail while the extradition proceedings are pending?

the presumption of innocence in favor of accused who is entitled to acquittal unless his guilt is proved beyond reasonable doubt. It follows, therefore, that the provision on bail will not apply to extradition he is sought to be extradited are bailable in the United States is no argument. Extradition proceedings are separate and distinct from the trial for the ofenses for which he is charged. He should apply for bail before the US court, not before the extradition court.

TAU MU

GOVERNMENT OF USA vs. PURGANAN September 24, 2002

Ateneo de Davao

TAU MU TAU MU

Ruling: The SC ruled that the imputation of grave abuse of discretion to Sandiganbayn was untenable. To begin with, Section 13 of Article III (Bill of Rights) of the Constitution mandates: Section 13. All persons, except those charged with ofenses punishable by reclusion perpetua when evidence of guilt is strong, shall, before conviction, be bailable by sufficient sureties, or be released on recognizance as may be provided by law. Even if the capital ofense charged is bailable owing to the weakness of the evidence of guilt, the right to bail may justifiably still be denied if the probability of escape is great. In this case, Jinggoy does not, as determined by Sandiganbayan, seem to be a flight risk. The likelihood of escape on the part Jinggoy is now almost nil, given his election on May 2004, as Senator of the Republic of the Philippines. The Court takes stock of the fact that those who usually jump bail are shadowy characters mindless of their reputation in the eyes of the people for as long as they can flee from the retribution of justice. On the other hand, those with a reputation and a respectable name to protect and preserve are very unlikely to jump bail. The Court, to be sure, cannot accept any suggestion that someone who has a popular mandate to serve as Senator is harboring any plan to give up his Senate seat in exchange for becoming a fugitive from justice.

The Fraternal

TAU MU TAU MU

Sandiganbayan committed grave abuse of discretion in (a) sustaining the charge against him for alleged ofenses and with alleged conspirators with whom he is not even connected, and (b) in not fixing bail for him. But later on, Sandiganbayan granted his application for bail. Hence, this petition.

KITY

TAU MU TAU MU TAU MU TAU MU TAU MU TAU MU

CONSTITUTIONAL LAW II Order of Saint Thomas More Atty. Philip John Pojas/Atty. Rovyne G. Jumao-as, RN University College of Law

GOVERNMENT OF HONG KONG vs. OLALIA AND MUNOZ 521 SCRA 470 (2007)

Issue: WON a prospective extraditee may be granted bail. Ruling: Philippine jurisprudence has not limited the exercise of the right to bail to criminal proceedings only. This Court has admitted to bail persons who are not involved in criminal proceedings. In fact, bail has been allowed in this jurisdiction to persons in detention during the pendency of administrative proceedings, taking into cognizance the obligation of the Philippines under international conventions to uphold human rights. The right of a prospective extraditee to apply for bail in this jurisdiction must be viewed in the light of the various treaty obligations of the Philippines concerning respect for the promotion and protection of human rights. Under these treaties, the presumption lies in favor of human liberty. Thus, the Philippines should see to it that the right to liberty of every individual is not impaired.

ACADCOM 2010; Contributors: Gene Geocaniga, Jarissa Guiani, Darlene Magabilen TAU MU Page 150 of 179

TAU MU TAU MU TAU MU TAU MU TAU MU TAU MU TAU MU

Ruling: In extradition cases, prior notice and hearing should be aforded the extraditee even when a possible extradition is still being evaluated. The Court qualified and declared that prospective extraditees are entitled to notice and hearing only when the case is filed in court and not during the process of evaluation. The SC ruled that the benefits of continued temporary liberty on bail should not be revoked and their grant of bail should not be cancelled, without the co-petitioner, Imelda Rodriguez in this case, being given notice and without her being heard why her temporary liberty should not be discontinued. Imelda’s copetitioner was already facing his charges against him, hence, the issue posted is already moot and academic. Thus, it emphasized that bail may be granted to a possible extraditee only upon a clear

TAU MU

Issue (1): WON a prospective extraditee is entitled to notice and hearing before the cancellation of his or her bail.

TAU MU

Facts: This case stemmed from a petition for extradition which was filed by the Government of the United States of America through the DOJ against the petitioners. After their arrest, petitioners applied for bail which the trial court granted. The bail was set for one million pesos for each. Petitioners then posted cash bonds. The US government moved for reconsideration of the grant of bail, but the motion was denied by the trial court. The SC directed the trial court to resolve the matter of bail which shall be subject to whatever ruling that it may have in the similar case of Mark Jimenez. In compliance with the SC’s directive, the trial court, without prior notice and hearing, cancelled the cash bond of the petitioners and ordered the issuance of a warrant of arrest. Thus, petitioners filed a motion for reconsideration of the cancellation of their bail.

TAU MU TAU MU

RODRIGUEZ vs. PRESIDING JUDGE and US GOVERNMENT 483 SCRA 290 (2006)

The Fraternal Ateneo de Davao

TAU MU TAU MU

Muñoz was arrested on September 23, 1999, and remained incarcerated until December 20, 2001, when the trial court ordered his admission to bail. In other words, he had been detained for over two (2) years without having been convicted of any crime. By any standard, such an extended period of detention is a serious deprivation of his fundamental right to liberty. In fact, it was this prolonged deprivation of liberty which prompted the extradition court to grant him bail. While our extradition law does not provide for the grant of bail to an extraditee, however, there is no provision prohibiting him or her from filing a motion for bail, a right to due process under the Constitution. Thus, in this case, there is no showing that Muñoz presented evidence to show that he is not a flight risk. Consequently, this case should be remanded to the trial court to determine whether private respondent may be granted bail on the basis of “clear and convincing evidence.”

KITY

TAU MU TAU MU TAU MU TAU MU TAU MU TAU MU

CONSTITUTIONAL LAW II Order of Saint Thomas More Atty. Philip John Pojas/Atty. Rovyne G. Jumao-as, RN University College of Law

and convincing showing (1) that he will not be a flight risk or a danger to the community, and (2) that there exist special, humanitarian and compelling circumstances. Therefore, the SC held that Imelda’s bail should be restored. Issue (2): WON an order to issue warrant of arrest against petitioners and to cancel the bail of extraditees was a grave abuse of discretion of the trial court. Ruling: The cancellation of co-petitioner’s bail, without prior notice and hearing, could be considered a violation of co-petitioner’s right to due process tantamount to grave abuse of discretion. Considering that Imelda has not been shown to be a flight risk nor a danger to the community, she is entitled to notice and hearing before her bail could be cancelled. Thus, absent prior notice and hearing, the bail’s cancellation was in violation of Imelda’s right to due process. YAP, JR. V. CA 358 SCRA 564 (2001) Facts: Accused was convicted of misappropriating P5.5M and was sentenced to a jail term of 8 years to 20 years. While his appeal was pending before the Court of Appeals, he applied for bail which was granted. However, the Court of Appeals fixed it at P5.5M, with a restriction that he could not change residence without the approval of the court and a directive to the Commissioner of Immigration and Deportation to issue a hold departure order against him. Was his right against excessive bail violated? Ruling: Yes. The amount of P5.5M is unreasonable, excessive and constitutes an efective denial of petitioners’ right to bail. The purpose for bail is to guarantee the presence of accused whenever required by the court. The amount should be high enough to assure his presence, but no higher than is reasonably calculated to fulfill this purpose. To fix bail at an amount equivalent to the civil liability of which petitioner is charged is to permit the impression that the amount paid as bail is an exaction of the civil liability that accused is charged of, thus the court cannot allow because bail is not intended as a punishment, nor a satisfaction of civil liability which should necessarily await the judgment of the appellate court. OBOSA vs. CA 266 SCRA 281 (1997) Facts: Accused was charged with two counts of murder, a capital ofense, but after trial he was conceived only of two counts of homicide, a noncapital ofense. On appeal, is he entitled to bail as a matter of right? Ruling: Petitioner applied for bail after the efectivity of Administrative Circular No. 12-94. Under the Circular, if the court imposed a penalty of imprisonment exceeding six years but not

ACADCOM 2010; Contributors: Gene Geocaniga, Jarissa Guiani, Darlene Magabilen TAU MU Page 151 of 179

more that twenty ears, then bail is a matter of discretion, except when any of the circumstances mentioned in paragraph 3, Sec. 5 of the Circular is present, in which case bail has to be denied. Since petitioner was convicted of two counts of homicide, his circumstances found in the Circular justify the denial of bail, except that a retroactive application of the Circular is barred as it would be unfavorable to the petitioner.

TAU MU TAU MU TAU MU TAU MU TAU MU TAU MU TAU MU TAU MU

Q: Why does the accused has more rights? A: (1) Because it is better to free to guilty people than to convict 1 innocent person; (2) In a criminal trial, it is one person against the people of the state with all its machinations; and (3) It should not be proof by preponderance of evidence since it would result in the conviction of more innocent people.

TAU MU

RIGHTS OF A PERSON CHARGED WITH A CRIMINAL OFFENSE: 1. Right to due process of law – waivable by making a plea of guilty 2. Right to be presumed innocent – waivable by making a plea of guilty 3. Right to be heard by himself and counsel – waivable 4. Right to be informed of the nature and cause of the accusation against him – not waivable 5. Right to have a speedy trial – waivable 6. Right to an impartial trial – not waivable 7. Right to public trial – waivable 8. Right to meet the witness face to face – waivable 9. Right to have compulsory process to secure the attendance of witnesses and the production of evidence in his behalf – waivable by making an advance waiver.

TAU MU TAU MU

Section 14. (1) No person shall be held to answer for a criminal offense without due process of law. (2) In all criminal prosecutions, the accused shall be presumed innocent until the contrary is proved, and shall enjoy the right to be heard by himself and counsel, to be informed of the nature and cause of the accusation against him, to have a speedy, impartial, and public trial, to meet the witnesses face to face, and to have compulsory process to secure the attendance of witnesses and the production of evidence in his behalf. However, after arraignment, trial may proceed notwithstanding the absence of the accused: Provided, that he has been duly notified and his failure to appear is unjustifiable.

The Fraternal Ateneo de Davao

TAU MU TAU MU

Be that as it may, the rules on bail prior to the efectivity of Administrative Circular No. 12-94 does not favor him either. In accordance with previous decisions, when he appealed he opened the whole case for review so that the possibility of conviction upon the original charge of murder is ever present. Likewise, since the prosecution has previously demonstrated that evidence of guilt is strong, such determination subsists on appeal, despite conviction for a lesser ofense, since such determination is for the purpose of resolving whether to grant or deny bail and does not have any bearing on whether petitioner will ultimately be acquitted or convicted of the charge.

KITY

TAU MU TAU MU TAU MU TAU MU TAU MU TAU MU

CONSTITUTIONAL LAW II Order of Saint Thomas More Atty. Philip John Pojas/Atty. Rovyne G. Jumao-as, RN University College of Law

NOTE: The rights of the accused apply only to PROCEDURAL JUDICIAL PROCESS IN CRIMINAL PROCEEDINGS as compared to Section 1 of Article III of the Constitution that applies to all types of proceedings. 1. DUE PROCESS a. The right to due process is the biggest right of all. According to Justice Cruz, the right to due process mentioned here is only a procedural due process, the procedure laid down by law in trying an accused who is being charged of a crime. b. This means that the accused can only be convicted by a tribunal which is required to comply with the stringent requirements of the rules of criminal procedure. 

Criminal due process requires that the accused be tried by an impartial and competent court in accordance with the procedure prescribed by law and with proper observance of all the rights accorded him under the Constitution and the applicable statutes. Accordingly, to illustrate, denial from him of the right to preliminary investigation, as required by law, will constitute a denial of due process.  It should be noted that the right to a preliminary investigation is not among the rights granted to the accused in the Bill of Rights. It is purely statutory. Even so, denial of this right, in the absence of a valid waiver, will violate due process.  Obviously, the basic ingredient of criminal due process is a trial conducted in accordance with the rudiments of fair play. Hence, the accused has a right to complain if the judge has a personal or pecuniary interest in the outcome of the case, as where he is allowed to share in the fines he may impose or where he is covered by the disqualification enumerated in the rules of court.  Due process is also denied where a person is impleaded for violation of a law, administrative regulation or municipal ordinance not previously published as he would not know what acts he must do or avoid to prevent prosecution. Where appeal is permitted by the Constitution or by statute, denial thereof will also militate against due process. ELEMENTS OF CRIMINAL DUE PROCESS [BANCO ESPANOLA vs. PALANCA, 37 PHIL 921]: 1) There must be an impartial court or tribunal clothed with judicial power to hear and determine the matter before it. 2) Jurisdiction must be lawfully acquired over the person of the accused and over the property which is the subject matter of the proceeding. 3) Accused was given an opportunity to be heard 4) Judgment must be rendered upon lawful hearing. NOTE: All elements must be present. The absence of one means there is a violation of criminal due process. PROCEDURAL GUARANTY IN CRIMINAL PROCEEDINGS [NUNEZ vs. SANDIGANBAYAN 11 SCRA 542]: 1. The accused must be informed why is he being charged and what are the charges against him; 2. He should have the full opportunity to rebut it; 3 The conviction should rest on evidence that is not tainted with falsity 4. The sentence imposed against him should be in accordance with a valid law; 5. The court that rendered judgment is one of competent jurisdiction.

ACADCOM 2010; Contributors: Gene Geocaniga, Jarissa Guiani, Darlene Magabilen TAU MU Page 152 of 179

TAU MU TAU MU TAU MU TAU MU

Issue: WON the appellant was deprived of his right to due process.

TAU MU

Facts: Macarang was sentenced to the penalty of death for qualified rape. The trial court considered appellant to have waived his right to present his evidence without any showing that the latter was fully aware of the consequence of such waiver.

TAU MU

Ruling: The right of the accused to due process was violated. No trial was conducted based on the procedure in the Rules of Court and accused was not given his full day in court. It cannot be argued that accused waived his right to confront and cross-examine the witness because the existence of the waiver must be shown to have been done knowingly and with sufficient awareness of the consequence. The case should be remanded for further proceedings. PEOPLE vs. MACARANG 424 SCRA 18 (2004)

TAU MU

Facts: After accused was arraigned, the prosecution presented the rape victim who identified her affidavit of desistance and reaffirmed that she had no further interest in prosecuting accused. The judge then asked clarificatory questions to determine the truth and volutariness of both her affidavit-complaint and affidavit of desistance. Counsel for the accused did not anymore cross-examine the witness. The prosecutor then moved to dismiss the case as she could no longer prove the guilt of the accused. About two months later, the court convicted accused of rape and sentenced him to reclusion perpetua.

defense evidence, he would have deemed his right to present it, did not satisfy appellant’s constitutional right to due process. The trial court granted Motion of Appellant’s counsel to withdraw his appearance. Appellant, therefore, had no more counsel. It is obvious that the appellant was deprived of his right to due process. PAGASIAN vs. AZURA Facts: In a criminal case for theft entitled People vs Dumo, Barangay Captain Pagasian was a witness. It appeared that because of a report, and accompanied by police, he seized the carabao from the house of the accused which allegedly is the subject matter of the theft. After trial, the Judge acquitted Dumo but convicted Pagasian for clear violation of the fundamental law of the land and against human rights. He was sentenced to two days of jail term and a fine of P200.00. Ruling: The court ruled that the judge violated due process. The barangay captain was not informed of the charges against him and in fact, he had no idea that he was on trial. He did not even present evidences in his own behalf. OLAGUER vs. MILITARY COMMISSION

TAU MU

ALONTE vs. SAVELLANO JR. 287 SCRA 245 (1998)

TAU MU TAU MU

CASES ON DUE PROCESS:

The Fraternal Ateneo de Davao

TAU MU TAU MU

REQUIREMENTS FOR DUE PROCEES CLAUSE IF THE ACCUSED PLEADS GUILTY TO A CAPITAL OFFENSE [PEOPLE vs. STA. TERESA] (although no more capital punishment): 1. The trial court must conduct a searching inquiry into the voluntariness of the plea and the full comprehension of what he confessed thereof. 2. The court must order the prosecution to present evidence to prove the guilt of the accused beyond reasonable doubt and the precise degree of his liability 3. The accused must be asked if he would like to present evidence on his behalf, and allow him to do so, if he desires.

KITY

TAU MU TAU MU TAU MU TAU MU TAU MU TAU MU

CONSTITUTIONAL LAW II Order of Saint Thomas More Atty. Philip John Pojas/Atty. Rovyne G. Jumao-as, RN University College of Law  [OLAGUER vs. MILITARY COMMISSION] The court held that Due Process in Section 14 means trial by judicial process not by executive or military process. Military tribunal is not part of judiciary, but of executive to discipline the army. Therefore, there is a violation of due process.  Even during martial law, all cases should be tried under civilian courts. AS LONG AS THE CIVILIAN COURTS ARE OPEN and FUNCTIONING, decisions by military tribunals on civilians are null and void.  Pending cases in Tribunals should be transferred to civilian courts when national emergency already ceases to exist. Military Tribunals have jurisdiction only as long as national emergency exists. Double jeopardy will not attach.

TAU MU

Ruling: In criminal cases where the impossible penalty may be death, the presiding judge is called upon to see to it that the accused is made aware of the consequences of not heeding the warrant given by the trial court. A simple forewarning to the appellant that the next time that he would not be ready with his ACADCOM 2010; Contributors: Gene Geocaniga, Jarissa TAU MU Page 153 of 179

Facts: Olaguer was a civilian tried and sentenced to die by the military tribunal during Martial Law. After Marcos was deposed, Olaguer went to SC challenging the validity of his conviction saying that his conviction was a violation of his right to due process. Ruling: The court held that due process in Sec. 14 means trial by judicial process not by executive of military process. Military tribunal is not part of judiciary but of the executive branch for discipline of army. Therefore there is a violation of due process. Even during martial law, all cases should be tried under civilian courts. As long as civilian courts are open and functioning, the decisions of the military tribunal on civilians are null and void. Pending cases involving civilians in military tribunals should be transferred to civilian courts when national emergency already ceased to exist. Military tribunals have jurisdiction only as long as national emergency exists. Double jeopardy will not lie here. 2. PRESUMPTION OF INNOCENCE In all criminal prosecution, the accused shall be presumed innocent until the contrary is proved. DEMURRER OF EVIDENCE It is motion to dismiss based on insufficiency of evidence. The government has not overcome the presumption of innocence since it is the duty of the prosecution to present evidence against the accused. The evidences presented will not be based on the strength or weakness of the accused but of the PROSECUTION.  The Constitution does not prohibit the legislature

from providing that proof of certain facts leads to a prima facie presumption of guilt, provided that the facts proved have a reasonable connection to the ultimate fact presumed.  Presumption of guilt should not be conclusive. Q: When is there a REVERSE TRIAL? A: (1) When the accused puts up an affirmative defense (like self-defense) and; (2) When a law is made by the Congress that there is presumption of guilt Guiani, Darlene Magabilen

Q: Why does it not violate the Constitution? A: (1) There is a logical connection between the fact presumed and the fact proved; and (2) The presumption is rebuttable.

TAU MU TAU MU TAU MU TAU MU TAU MU

Facts: Petitioners challenge the provision of PD No. 704, the Fisheries Decree, which provides that the discovery of explosives or obnoxious substance in any fishing boat shall constitute a presumption that the owner or operator were fishing with the use of

TAU MU

HIZON vs. CA supra

Ruling: It is conceded that the legislature has the power to provide that proof of certain facts can constitute a prima facie evidence of guilt provided there is a rational connection between the fact proved and the fact presumed. To avoid any constitutional infirmity, the inference of one form proof of the other must not be arbitrary and unreasonable. P.D. 704 creates a presumption of guilt based on facts proved and hence is not constitutionally impermissible. However, the statutory presumption can only be prima facie. It cannot, under the guise of regulating the presentation of evidence, operate to preclude accused from presenting his defense to rebut the fact presumed

THE RIGH TO PRESENT EVIDENCE BEHALF INCLUDES: a) The right to testify in one’s favor; b) The right to call witnesses. THE RIGHT TO BE FOLLOWING RIGHTS: 

TAU MU

CASE ON PRESUMPTION OF INNOCENCE:

3. RIGHT TO BE HEARD BY HIMSELF AND COUNSEL Simply stated, this is the right to present evidence on one's behalf, which includes the right to testify in one's favor and the right to call witnesses. It includes the right to defend yourself, the right to confrontation, and the right to call witnesses.

TAU MU

 [DIZON-PAMINTUAN vs. PEOPLE, 234 SCRA 63]: There is no constitutional infirmity to the reversed presumptions.  [HIZON vs. CA]: Laws providing presumptions in criminal cases that a crime being or has been committed is not a violation of the presumption of innocence right provided that they are based on facts and these facts must be part of the crime when committed.  To avoid any constitutional infirmity, the inference of one from proof of the other must not be arbitrary and unreasonable.  The legislature has the power to provide that proof of certain facts can constitute prima facie evidence of guilt of the accused and shift the burden of proof to the accused provided that there is a rational connection between the facts proved and the ultimate fact presumed. (Natural inference)  The statutory presumption is merely prima facie and cannot preclude the accused from presenting his defense to rebut it and at no instance can the accused be denied the right to rebut the presumption.

explosives or poisonous substance, as a violation of the constitutional presumption of innocence.

TAU MU

Q: Are these presumptions constitutional? A: Yes. Clearly, the fact presumed is but a natural inference from the fact proved so that it cannot be said that there is no rational connection between the two. Furter, the statute establishes only a prima facie presumption thus giving the accused an opportunity to rebut it.

Ateneo de Davao

TAU MU TAU MU

GENERAL RULE: The accused shall be presumed innocence until the contrary is proved. EXCEPTIONS: 1. Article 217 of RPC, the failure of the public officer to produce money in his charge is prima facie evidence of malversation. So the burden of proof is shifted to the defense to prove otherwise (that he is not guilty). 2. Violation of the Anti-fencing Law 3. Possession of poisonous substances if you are on board a fishing vessel. (Reason: There is a presumption of involvement in illegal fishing)

The Fraternal

TAU MU TAU MU

Q: Do laws create by Congress to create a certain presumption of guilt violate the presumption of innocence? A: NO, because clearly, the fact presumed is but a natural inference from the fact proved so that it cannot be said that there is no rational connection between the 2. Furthermore, the statute establishes only a prima facie presumption, thus giving the accused an opportunity to rebut it.

KITY

TAU MU TAU MU TAU MU TAU MU TAU MU TAU MU

CONSTITUTIONAL LAW II Order of Saint Thomas More Atty. Philip John Pojas/Atty. Rovyne G. Jumao-as, RN University College of Law Note: In case of a reverse trial, the burden of evidence shifts to the accused to present the existence of factual evidence on whatever exculpatory defense he presents.

HEARD

IN

YOUR

INCLUDES

THE

Right to be present at the trial

A. The right to be present covers the period from ARRAIGNMENT to PROMULGATION of sentence. B. After arraignment, trial may proceed notwithstanding absence of accused, provided 2 requisites are met. Note, that trial in absentia is allowed only if the accused has been validly arraigned. (i) Accused has been duly notified; and (ii) His failure to appear is unjustifiable. C. The accused may waive the right to be present at the trial by not showing up. However, the court can still compel the attendance of the accused if necessary for identification purposes. EXCEPTION: If the accused, after arraignment, has stipulated that he is indeed the person charged with the ofense and named in the information, and that any time a witness refers to a name by which he is known, the witness is to be understood as referring to him. D. While the accused is entitled to be present during promulgation of judgement, the absence of his counsel during such promulgation does not afect its validity. 

Right to counsel

A. Right to counsel means the right to EFFECTIVE REPRESENTATION. B. If the accused appears at arraignment without counsel, the judge must: (i) Inform the accused that he has a right to a counsel before arraignment (ii) Ask the accused if he desires the aid of counsel (iii) If the accused desires counsel, but cannot aford one, a counsel de oficio must be appointed (iv) If the accused desires to obtain his own counsel, the court must give him a reasonable time to get one. BASIC ELEMENTS OF THE RIGHT TO COUNSEL:

ACADCOM 2010; Contributors: Gene Geocaniga, Jarissa Guiani, Darlene Magabilen TAU MU Page 154 of 179

TAU MU TAU MU TAU MU TAU MU TAU MU TAU MU TAU MU

DOCTRINAL CASES:  [PEOPLE vs. JOSE 37 SCRA 450]. BUT under Sec. 12 (1) the right to counsel also exists before arraignment.  [DELGADO vs. CA]: Deprivation of right to counsel because the accused was represented by a fake lawyer;  Waiver of this right to counsel need not be done in the presence of counsel because under our procedure, the accused can defend himself;  [PEOPLE vs. NADERA]: If the evidence presented by the prosecution is strong, it is the duty of the defense to try to rebut it in anyway he can. Here, the defense lawyer believed the victim. That cannot be done, the defense lawyer must try his best to rebut the evidence. He must not be swayed by the testimony of the victim because it is his duty to defend his client, and if he does not do so then there is a deprivation of the right to counsel.  [CHIONGBIAN CASE]: The Court said that the counsel of choice thing applies only to cases when the accused is under custodial investigation. There are other rights that cannot be violated, like the right to speedy trial. So the accused cannot delay the case just because he wants to get his own lawyer  [PEOPLE vs. TULIN]: Accused was first represented by a non lawyer, who did a good job. When he found out, he got a real lawyer, who adopted the nonlawyer's strategy. Accused was convicted. Hence, there was no deprivation because he should have said to his

TAU MU

when the accused has no counsel of choice and desires to employ the services of one is MANDATORY ONLY AT THE TIME OF ARRAIGMENT. The counsel need not be one who is the choice of the accused.  The right to counsel may be waived.  The rights enumerated in Sec. 14 (2) are rights in all criminal prosecutions which cover the period from arraignment to rendition of judgment. The right to counsel exists only during that period.

TAU MU

 The duty of the court to appoint counsel de officio

TAU MU TAU MU

Note: The duty of the court is not ended with such appointment of a counsel de oficio, as it should also see to it that the counsel does his duty by the defendant.

The Fraternal Ateneo de Davao

TAU MU TAU MU

NOTE: The right to counsel during trial can be waived. The accused may represent himself in any litigation. PRE ARRAIGNMENT DUTIES OF THE JUDGE [Rule 116 Sec.1]: a. to inform the accused that he has the right to have his own counsel before being arraigned; b. after informing the accused he has to ask the latter whether he desires the aid of counsel; c. if the accused so desires to procure the services of counsel, the court must grant him reasonable time to do so; d. if ha so desires to have counsel but is unable to employ one, the court must assign counsel de officio.

KITY

TAU MU TAU MU TAU MU TAU MU TAU MU TAU MU

CONSTITUTIONAL LAW II Order of Saint Thomas More Atty. Philip John Pojas/Atty. Rovyne G. Jumao-as, RN University College of Law a. The court is duty bound to inform the accused that he has the right to an attorney before he is arraigned; b. The court must ask him if he desires the service of counsel; c. If he does, and is unable to get one, the court must assign a counsel de officio; d. Or, if the accused wishes to procure private counsel, the court must give him time to obtain one [PEOPLE vs. HOLGADO (85 Phil. 753)]; e. Where the duly authorized members of the bar are not available, the court may appoint a person resident of the province and of good repute for probity and ability [Rule 116 Sec. 1];

counsel that he does not want to adopt the strategy of the non-lawyer. There was waiver on the part of the accused. Right to an impartial judge Right of confrontation and crossexamination  Right to compulsory process to secure the attendance of witnesses  

CASES ON COUNSEL:

RIGHT

TO

BE

HEARD

AND

TO

PEOPLE vs. SANTOCILDES 321 SCRA 310 (1999) Facts: Accused was charged and convicted of the crime of rape and sentenced to reclusion perpetua. During trial, he was represented by a certain Gualberto C. Ompong, who turned out to be a non-lawyer. On appeal, he argues that his right to counsel was violated so that he should be acquitted of the crime charged. Ruling: The right of accused to counsel was violated, no matter that the person who represented him had the ability of a seasoned lawyer and handled the case in a professional and skillful manner. This is so because an accused person is entitled to be represented by a member of the bar in a criminal case filed against him. Unless he is represented by a lawyer, there is a great danger that any defense presented will be inadequate considering the legal skills needed in court proceedings. The judgment is set aside, and the case remanded for new trial. PEOPLE vs. TULIN 364 SCRA 10 (2001) Facts: Accused was tried for piracy. They were represented by Tomas Posadas who presented and examined 7 witnesses before the court discovered that he was a non-lawyer. Their new lawyer, however, manifested that accused were adopting the evidence adduced when they were represented by a non-lawyer. Convicted by the trial court, they now claim that their right to counsel was violated. Ruling: The law entitles the accused to be present and to defend himself in a person and by counsel at every stage of the proceedings. However, it is also provided that rights may be waived, unless the waiver is contrary to law, public order, public policy, etc. The Rules also states that “upon motion, the accused may be allowed to defend himself in person when it sufficiently appears to the court that he can properly protect his rights without the assistance of counsel. By analogy, it is amply shown that the rights of accused were sufficiently and properly protected by Tomas Posadas. He knew the technical rules of procedure. Hence, there was a valid waiver of the right to sufficient representation during the trial, considering that it was unequivocally, knowingly, and intelligently made with the full assistance of their new bona fide lawyer. PEOPLE vs. BERMAS 306 SCRA 135 (1999) Facts: Accused, an indigent, was charged with rape, convicted and sentenced to death. His first lawyer withdrew after the direct examination of the victim and cross-examination was done by another de oficio lawyer who only prepare for 10 minutes. The third de oficio lawyer also wanted to withdrew but was prevailed upon by the court and he presented the accused as witness, but later also ceased to appear for unknown reasons.

ACADCOM 2010; Contributors: Gene Geocaniga, Jarissa Guiani, Darlene Magabilen TAU MU Page 155 of 179

TAU MU TAU MU TAU MU TAU MU TAU MU

Ruling: There is no denial of the right to counsel de officio was appointed during the absence of the accused’s counsel de parte, pursuant to the court’s desire to finish the case as easy as practicable under the continuous trial system. The appointment of counsel de officio under such circumstances is not proscribed by the Constitution. The “preference in the choice of counsel” pertains more aptly and specifically to a person under investigation. Neither is there a violation of appellant’s right to counsel just because the trial court did not grant their request for suspension of the hearing pending their search for new counsel. The right of the accused to select his own counsel must be exercised on a reasonable time and in a reasonable manner. The court may deny accused’s application is not made for purpose of delay. That the trial court limited the length of time to cross examine a witness cannot be labelled as a violation of a constitutional right. Due process of law is not denied by the reclusion of irrelevant, immaterial or incompetent evidence, or testimony of an incompetent witness.

TAU MU

Issue: Whether there is a violation of appellant’s right to due process.

TAU MU

Facts: Appellants were found guilty beyond reasonable doubt of the crimes of kidnapping and serious illegal detention.

TAU MU

Ruling: Accused has the right to be heard by himself and counsel. He has also the right to present evidence. Accordingly, denial of due process can be successfully invoked where no valid waiver of rights has been made. In this case, we find that under the circumstances, the accused was denied due process when the successive non-appearance of his counsel was construed as a waiver of his right to present evidence. Since he imposable penalty may be death, the trial court should have been more circumspect in outrightly denying accused his opportunity to present his side, particularly since he himself was present during the four hearings. Clearly, such presence is a strong indication that accused was interested in defending himself. PEOPLE vs. LARRANAGA 421 SCRA 530 (2004)

TAU MU TAU MU

Facts: Accused was convicted of raping his own daughter and sentenced to death. In appears that after the prosecution rested, the case was set for reception of defense evidence. However, in all four settings counsel for the accused failed to appear despite notice. This was treated by the trial court as a waiver by the accused of his right to present evidence, and it considered the case submitted for decision.

The Fraternal Ateneo de Davao

TAU MU TAU MU

PEOPLE vs. DIAZ 311 SCRA 585 (1999)

KITY

TAU MU TAU MU TAU MU TAU MU TAU MU TAU MU

CONSTITUTIONAL LAW II Order of Saint Thomas More Atty. Philip John Pojas/Atty. Rovyne G. Jumao-as, RN University College of Law Ruling: Accused was not properly and efectively accorded the right to counsel. The right to counsel proceeds from the fundamental principle of due process which basically means that a person must be heard before being condemned. The right to counsel must be more than just the presence of a lawyer in the court room or the mere propounding of standard questions and objections. The right to counsel means that the accused is amply accorded legal assistance extended by a counsel who commits himself to the cause of the defense and acts accordingly. The right assumes an active involvement by a lawyer in the proceedings, his bearing constantly in mind of the basic rights of the accused, his being well-versed on the case and his knowing the fundamental procedures, essential laws and existing jurisprudence.

PEOPLE vs. CHIONGBIAN 301 SCRA 614 (1999) Facts: Due to several postponements sought by a lawyer of accused, the judge appointed a counsel de oficio for him during trial. Accused objected to the services of counsel de oficio since he can aford to hire a counsel de parte of his own choice. Was there a violation of the right of the accused to counsel of his own choice? Ruling: No. An examination of related provisions in the Constitution concerning the right to counsel will show that the “preference in the choice of counsel” pertains more aptly and specifically to a person under custodial investigation rather than one who is an accused in a criminal prosecution. But even if we were to extend the application of the concept of “preference in the choice of counsel” to an accused in a criminal prosecution, such preferential discretion cannot partake of a discretion so absolute and arbitrary as would make the choice of counsel refer exclusively to the predilection of accused. Otherwise, the pace of criminal prosecutions would be entirely dictated by the accused. PEOPLE vs. RIVERA 362 SCRA 153 (2001) Facts: Accused was charged with and convicted of incestuous rape. During order trial date his counsel failed to appear so that the judge assigned him a counsel de officio. The lawyer moved for a postponement of the cross-examination of the physician-witness, but the judge denied the motion because the witness was a very busy person. Thus, the lawyer decided not to cross-examine. The accused now argues that the assignment by a counsel de officio in the absence of his own lawyer violated the right to counsel? Ruling: While the Constitution recognized the right of the accused to be represented b counsel of his own choice, his option to secure the services of a private counsel is not absolute. For considering the State’s and the ofended party’s right to speedy and adequate justice, the court may restrict the option of the accused to retain a private counsel if he insists on a counsel he cannot aford, or if the chosen counsel is not a member of the bar, or the latter declines to represent accused. Courts are not required to wait indefinitely the pleasure and convenience of the accused as they are also mandated to promote speedy and orderly administration of justice. PEOPLE vs. LIWANAG 363 SCRA 62 (2001) Facts: Accused was convicted of highway robbery with multiple rape. During trial he was assisted b counsel de oficio, a PAO lawyer. In the middle of the trial, he retained the services of counsel de parte. After he was convicted by the trial court, another lawyer filed the notice of appeal but failed to file the appellant’s brief. Before the Supreme Court he was represented by another counsel de officio. He now claims that his right to counsel was violated because his counsel made insufficient cross-examination of the prosecution witnesses, and failed to impeach the testimony of complainant through the use of contradictory evidence.

TAU MU

Ruling: The “right to be assisted by counsel” does not presuppose “the right to an intelligent counsel”. The requirement is not for counsel to be intelligent, but to be efective. While fairness is the object of Art. III, Sec. 14 (2) of the Constitution, the assistance aforded by the counsel to an accused need only be in accordance ACADCOM 2010; Contributors: Gene Geocaniga, Jarissa Guiani, Darlene Magabilen TAU MU Page 156 of 179

TAU MU TAU MU TAU MU

PURPOSES OF THE RIGHT: 1) To furnish the accused with a description of the charge against him as will enable him to make his defenses 2) To avail himself of his conviction or acquittal against a further prosecution for the same cause 3) To inform the court of the facts alleged.

TAU MU

4. RIGHT TO BE INFORMED OF THE NATURE AND CAUSE OF ACCUSATION AGAINST HIM In all criminal prosecutions, the accused shall have the right to be informed of the nature and cause of the accusation against him.

TAU MU

Ruling: Counsel’s decision not to cross-examine the witness is a glaring example of his manifest lack of enthusiasm for his client’s clause. It may be that defense counsel personally found the testimony to be believable. Nevertheless, he had the bounded duty to scrutinize private complainant’s testimony to ensure that the accused’s constitutional rights to confront and examine the witnesses against him was not rendered for naught.

TAU MU

Facts: After the rape victim testified, the following dialogue occurred: COURT: Any cross? ATTY. BROTONEL: If Your Honor please, we are not conducting any cross-examination, because this representation, from eh demeanor of the witness, I am convinced that she is telling the truth.

 [PEOPLE vs. GRADA]: It was not the accused name listed in the information, but that of his brother. Deprivation? No, because he knew of the nature and the cause of the accusation against him. It was just a wrong name; it does not vitiate the case. Besides, he did not raise it upon arraignment, so he is deemed to have waived the right.  Higher crime charged but convicted of lower ofense, there is no violation of this right; lower ofense charged but convicted of higher ofense, there is a violation of the right to be informed because lower crime does not involve the higher crime;  VOID FOR VAGUENESS RULE: The accused is denied of his right to be informed of the charge against him and to due process as well, when the statute itself is stated in such indefinite language that it is not possible for men of ordinary intelligence to determine what acts or omissions are punished. In such a case, the law is deemed void.  The accused must be informed of the facts that are imputed to him (cardinal requisite). Thus, the information referred to must describe the act with sufficient particularity.  The right to assail the sufficiency of the information or the admission of evidence may be waived by the accused if he fails to object to its sufficiency during trial and the deficiency is cured by competent evidence presented therein [PEOPLE vs. PALARCA May 29, 2002)].  The information need not allege the precise time of commission of the ofense, unless the time of an essential element of the crime charged.

TAU MU

PEOPLE vs. NADERA 324 SCRA 490 (2000)

2. You can be convicted in possession if acquitted in using prohibited drugs. 3. The word “minor” is not sufficient, age must be mentioned and if uncle did the act of rape, it must be stated that it is within the 4th civil degree.

TAU MU

Ruling: Accused acted as his own counsel. To allege now that his right to be assisted by counsel was violated is to bend the truth too far. The constitutional right of the accused to counsel is not violated where he was represented by a prominent and competent member of the Bar, namely himself, even if there were others available. He is now estopped from claiming that the trial court violated his right to be represented by counsel of his own choice.

Ateneo de Davao

TAU MU TAU MU

Facts: Accused was charged with murder. Being a practicing lawyer, he insisted on representing himself. Despite proddings by the court and an ofer of the possibility of assistance from the Public Attorney’s Office, he handled his own defense and was convicted. On appeal, he claims that his right to counsel was violated.

The Fraternal

TAU MU TAU MU

PEOPLE vs. SESBRENO 314 SCRA 87 (1999)

KITY

TAU MU TAU MU TAU MU TAU MU TAU MU TAU MU

CONSTITUTIONAL LAW II Order of Saint Thomas More Atty. Philip John Pojas/Atty. Rovyne G. Jumao-as, RN University College of Law with the provisions of Rules of Court and the Code of Professional Responsibility. In the Philippine setting, a counsel assisting an accused is presumed to be providing all the necessary legal defense which are reasonable under the circumstances in accordance with said norms. The proper measure of attorney performance remains simply reasonableness under prevailing norms. Coupled with the presumption that counsel’s performance was reasonable under the circumstances, as long as the trial was fair in that the accused was accorded due process by means of an efective assistance of counsel, then the constitutional requirement that an accused shall have the right to be heard by himself and counsel is satisfied.



If the information fails to allege the material elements of the ofense, the accused cannot be convicted thereof even if the prosecution is able to present evidence during the trial with respect to such elements.  The real nature of the crime charged is determined from the recital of facts in the information. It is not determined based on the caption or preamble thereof or from the specification of the provision of law allegedly violated.

RULE 110 – PROSECUTION OF OFFENSES Sec. 6. Sufficiency of complaint or information. – A complaint or information is sufficient if it states the name of the accused; the designation of the ofense given by the statute; the acts or omissions complained of as constituting the ofense; the name of the ofended party; the approximate date of the commission of the ofense; and the place where the ofense was committed. xxx xxx xxx Sec. 8. Designation of the offense. – The complaint or information shall state the designation of the ofense given by the statute, aver the acts or omissions constituting the ofense, and specify its qualifying and aggravating circumstances. If there is no designation of the ofense, reference shall be made to the section or subsection of the statute punishing it.

 What is essential is that the accused be informed of the facts alleged against him, not the characterization of the crime which is a conclusion of law. 5. RIGHT TO SPEEDY, IMPARTIAL AND PUBLIC TRIAL

TAU MU

ADEQUACY OF INFORMATION 1. If the information is more serious to less IMPARTIAL TRIAL serious – it can be done. ACADCOM 2010; Contributors: Gene Geocaniga, Jarissa Guiani, Darlene Magabilen TAU MU Page 157 of 179

TAU MU TAU MU TAU MU TAU MU TAU MU

 The right to a speedy disposition of a case, like the right to speedy trial, is deemed violated only when the proceeding is attended by vexatious, capricious, and oppressive delays; or when unjustified postponements of the trial are asked for and secured, or when without cause or justifiable motive a long period of time is

TAU MU

REMEDY OF THE ACCUSED IF HIS RIGHT TO SPEEDY TRIAL HAS BEEN VIOLATED: 1. He can move for the dismissal of the case. 2. If he is detained, he can file a petition for the issuance of writ of habeas corpus.

TAU MU

EFFECT OF DISMISSAL BASED ON THE GROUND OF VIOLATION OF THE ACCUSED’S RIGHT TO SPEEDY TRIAL: If the dismissal is valid, it amounts to an acquittal and can be used as basis to claim double jeopardy. This would be the efect even if the dismissal was made with the consent of the accused

TAU MU

FACTORS USED IN DETERMINING WHETHER THE RIGHT TO A SPEEDY TRIAL HAS BEEN VIOLATED: 1) Time expired from the filing of the information 2) Length of delay involved or extent of delay (actual lapse of time) 3) Reasons for the delay 4) Assertion or non-assertion of the right by the accused or the invocation of the accused of this right. Take Note: This right can be waived and when there is no objection against any postponement, then there is implied waiver). 5) Prejudice caused to the defendant.

TAU MU

 Right to speedy trial

TAU MU TAU MU

SPEEDY TRIAL It is one free from vexatious, capricious and oppressive delays” and is in inconveniences before sentence is pronounced upon him. This would also be consonant with Section 16 of the Bill of Rights providing that “all persons shall have the right to a speedy disposition of their cases before all judicial, quasi-judicial, or administrative bodies.” Although the trial is an indispensable and, indeed, the most important part of the proceedings against the accused, it has been held that the right to be present thereat is a personal right and therefore may be validly waived.

The Fraternal Ateneo de Davao

TAU MU TAU MU

PUBLICITY OF THE TRIAL It is necessary to prevent abuses that may be committed by the court to the prejudice of the defendant. The people have the right to attend the proceedings not only because of their interest therein but also so they can see whether or not the constitutional safeguards for the benefit of the accused are being observed. The accused is also entitled to the company of his relatives and friends to give him the moral support he needs during his ordeal. This rule is not absolute, however, for it is competent for the court to bar the public in certain cases, like rape trials, where the purpose of the spectators in attending the proceedings might be only to pander to their morbid curiosity, more since their presence is likely to inhibit testimony and embarrass some of the parties.

KITY

TAU MU TAU MU TAU MU TAU MU TAU MU TAU MU

CONSTITUTIONAL LAW II Order of Saint Thomas More Atty. Philip John Pojas/Atty. Rovyne G. Jumao-as, RN University College of Law It has been previously remarked that this requirement will call for no less than “the cold neutrality of an IMPARTIAL judge,” to insure that justice is done to the defendant. In another case, it was held as part of the rule that the judge must not only be impartial but must also appear to be impartial.

allowed to elapse without the party having his case tried. Equally applicable is the balancing test used to determine whether a defendant has been denied his right to a speedy trial, or a speedy disposition of a case for that matter, in which the conduct of both the prosecution and the defendant are weighed, and such factors as length of the delay, reason for the delay, the defendant’s assertion or non-assertion of his right, and prejudice to the defendant resulting from the delay, are considered. [SOLAR TEAM ENTERTAINMENT vs. HOW, 338 SCRA 511]  Right to impartial trial Definition of impartial trial The accused is entitled to the full “cold neutrality of an impartial judge”. It is an element of due process. The right to an impartial judge must not only be in reality, but also in appearance. The judge must not only be impartial but must look impartial; otherwise, the judge must inhibit himself from trying the case.  [TABUENA vs. SANDIGANBAYAN]: Judges can only clarify matters, and not impeach the accused. The judge cannot ask questions to find out whether the accused is guilty or not. The judge should not act as the prosecutor of the accused.  The right to have an impartial Judge. Impartiality must not only be in reality, but also even in appearance.  The judge should not be bias but objective; ruling based on information.  The right to an impartial trial is not a bar to a judge’s intervention in cross-examination.  A litigant is entitled to the cold neutrality of an impartial judge.  The judge should file a motion for inhibition if he is impartial. TWO PROCEEDINGS AGAINST A BIASED JUDGE: a) Disqualification; b) Inhibition. PUBLIC TRIAL The attendance at the trial is open to all irrespective of their relationship to the accused. However, if the evidence to be adduced is ofensive to decency or public morals, the public may be excluded.  The right of the accused to a public trial is not violated if the hearings are conducted on Saturdays, either with the consent of the accused or if failed to object thereto. PURPOSE OF PUBLIC TRIAL 1. To prevent possible abuses of judicial power that could be committed against him. 2. To prevent the courts from being instruments of persecution. NOTE: The right belongs to the accused and therefore, it can be WAIVED.  But waiver is not to be inferred from mere failure to urge the trial of the case. Waiver or abandonment may be presumed only when the postponement of the trial has been sought and obtained by the accused himself or by his attorney [KALAW vs. APOSTOL (64 Phil. 859)].  The presumption is always against the waiver of constitutionally protected rights.  The REMEDY of the accused for VIOLATION of his right to SPEEDY TRIAL is dismissal of the case and, if he is under detention, release by habeas corpus. Moreover, dismissal for violation of the right to speedy trial is equivalent to acquittal and is therefore a bar to a subsequent prosecution for the same ofense.

ACADCOM 2010; Contributors: Gene Geocaniga, Jarissa Guiani, Darlene Magabilen TAU MU Page 158 of 179

TAU MU TAU MU TAU MU

PEOPLE vs. SANCHEZ 302 SCRA 21 (1999)

TAU MU

CASES ON RIGHT TO SPEEDY, IMPARTIAL AND PUBLIC TRIAL

TAU MU

only because of their interest therein but also so they can see whether or not the constitutional safeguards for the benefit of the accused are being observed.  This rule is not absolute, however, for it is competent for the court to bar the public in certain cases, like rape trials, where the purpose of the spectators in attending the proceedings might be only to pander to their morbid curiosity, more since their presence is likely to inhibit testimony and embarrass some of the parties.

TAU MU

 People have a right to attend the proceedings not

TAU MU

TRIAL WITH LIVE MEDIA COVERAGE (ESTRADA CASE):  Balancing of rights present in the case: a) Accused  Fair Trial b) Press  Freedom of the Press c) Public  Right to information d) Court  Proper Administration of Justice  There would be no live media coverage.  The Estrada case covers immunity by suit, thus it is not yet a trial.  EXCEPTION: 1 Camera – for the purpose of history but not simultaneous with the trial.

TAU MU

 [CRUZ vs. SILVA]: Where the Fiscal chose the courtroom, installed microphones and invited the press, allowing them to ask questions, in a preliminary investigation involving a sensational killing. The court held the Fiscal in contempt. This is a case of trial by publicity.

TAU MU

TRIAL BY PUBLICITY This is a violation of the right to an impartial trial as too much publicity pressures the judge. However, it is not that easy to use this ground as a violation of this right. The test to be passed is the test of actual prejudice. To warrant a finding of prejudicial trial by publicity, there must be allegation and proof that the judges have been actually influenced, and not simply that they might be influenced. So, there is a diference between the judges MAY BE influenced, that is not enough, there must be proof that the judges were actually influenced.

TAU MU TAU MU

A PUBLIC TRIAL WHICH IS NOT HELD IN COURT IS VALID IF: 1) Public was not excluded. 2) Accused was not prejudiced. 3) Accused did not object during trial

The Fraternal Ateneo de Davao

TAU MU TAU MU

Q: What is the reason why trial has to be public? A: So that the public may see that the accused is fairly dealt with, and to give the judge a line to attend to his responsibility.

KITY

TAU MU TAU MU TAU MU TAU MU TAU MU TAU MU

CONSTITUTIONAL LAW II Order of Saint Thomas More Atty. Philip John Pojas/Atty. Rovyne G. Jumao-as, RN University College of Law  The general public may be excluded from trial when the evidence to be presented in the proceeding is “ofensive to public decency and morals” [Rule 119 Sec. 13]. BUT even in such instances the accused should be allowed to have his friends, relatives and counsel present [IN RE: OLIVER 333 US 257].  The purpose of the RIGHT TO A PUBLIC TRIAL is to safeguard against any attempt to employ our courts as instruments of persecution. IT is an efective restraint against possible abuse of judicial power because it the trial will be subject to contemporaneous review in the form of public opinion.

Facts: Mayor Sanchez was convicted of seven courts of rape with homicide. Considering the position of accused, the trial was accompanied by widespread media coverage. On appeal, Sanchez claims that his right to a fair trial was violated due to the intense publicity. Ruling: The right of an accused to a fair trial is not incompatible with a free press. Pervasive publicity is not per se prejudicial to the right of an accused to fair trial. It does not by itself prove that the publicity so permeated the mind of the trial judge and impaired his impartiality. At best appellant can only conjure possibility of prejudice on the part of the trial judge due to the barrage of publicity. But the test is not the possibility of prejudice but actual prejudice. To warrant a finding of prejudicial publicity, there must be allegation and proof that judges have been unduly influenced, not simply that they might be. Appellant must discharge this burden. In this case, there is no proof that the judge acquired a fixed opinion, or actual bias as a consequence of extensive media coverage. ESTRADA vs. DESIERTO 356 SCRA 109 (2001) Facts: Petitioner seeks a reconsideration of the decision of the Supreme Court declaring that having resigned from the presidency, he may be prosecuted for Plunder. Among others, he contends that his right to an impartial trial has been afected by the prejudicial pre-trial publicity of the proceedings before the Ombudsman. He also points to the alleged hate campaign launched by some newspapers so that the prosecution and the judiciary can no longer assure him of a sporting chance. He urges the Court to apply the rule on res ipsa loquitor. Has petitioner’s right to fair trial been violated? Ruling: No. The mere fact that the proceedings was given a day to day coverage does not prove that the publicity so permeated the mind of the tribunal and impaired his impartiality. To warrant a finding of prejudicial publicity, there must be allegation and proof that the judges have been unduly influenced, not simply that they might be. In this case, petitioner has failed to adduce any proof of actual prejudice developed by the members of the Panel of Investigators of the Ombudsman. We cannot replace this test of actual prejudice with the rule of res ipsa loquitur. The latter rule assumes that an injury has been sufered and then shifts the burden to the panel of investigators to prove that the impartiality of its members has not been afected by said publicity. Such a rule will overturn our case law that pervasive publicity is not per se prejudicial to the right of an accused to fair trial. For this reason, we continue to hold that it is not enough for petitioner to conjure possibility of prejudice but must prove actual prejudice on the part of his investigators for the Court to sustain his plea. RE: REQUEST RADIO… 360 SCRA 248 (2001) Facts: The Kapisanan ng mga BroadKaster ng Pilipinas, an association representing duly franchised and authorized television and radio networks, requested the Supreme Court to allow live media coverage of the trial of former President Estrada. The request was anchored on the need to assure the public of full transparency in the proceedings. In efect, the request seeks reconsideration of the 1991 resolution of the Court which barred live media coverage of all court proceedings.

ACADCOM 2010; Contributors: Gene Geocaniga, Jarissa Guiani, Darlene Magabilen TAU MU Page 159 of 179

TAU MU TAU MU TAU MU TAU MU

Facts: The direct examination of the witness presented by the accused for his defense consisted of 16 questions, while the prosecutor propounded 6 crossexamination questions. The court asked 27 questions and added 10 more later. When accused himself testified, the prosecutor asked 14 cross-examination questions, while the court asked 67 questions which were in the nature of cross-examination. Was there a violation of the right to due process? Ruling: The accused basic constitutional right to due process was violated and this appears to be a more compelling reason for their acquittal. While a trial judge has a right to question witnesses, his examination should be limited to asking clarificatory questions. The impression cannot be avoided that the Sandiganbayan had taken the cudgels for the prosecution. The “cold neutrality of an impartial judge” requirement of due process was denied the accused

TAU MU

TABUENA vs. SANDIGANBAYAN 268 SCRA 332 (1997)

TAU MU

Ruling: The motion is denied. However, because of the significance of the trial and the importance of preserving the records, there should be an audio visual recording of the proceedings. The recordings will not be for live or real time broadcast but for documentary purposes. Only later will they be available for public showing, after the Sandiganbayan shall have promulgated its decision. The master film shall be deposited in the National Museum and the Records Management ad Archives Office for historical preservation and exhibition pursuant to law. The audio-visual recording shall be made under the supervision and control of the Sandiganbayan.

TAU MU TAU MU

Facts: The Secretary of Justice seeks a reconsideration of the resolution denying permission to televise and broadcast live the trial of President Estrada before the Sandiganbayan. Among others, he argues that if there is a clash between the rights of the people to public information and the freedom of the press, on the one hand, and the right of the accused to fair trial, it should be resolved in favor of the right of the people, because the people, as repository of democracy are entitled to information; and that live media coverage is a safeguard against attempts by any party to use the courts as instruments for the pursuit of selfish interest

The Fraternal Ateneo de Davao

TAU MU TAU MU

RE: REQUEST FOR LIVE… 365 SCRA 62 (2001)

KITY

TAU MU TAU MU TAU MU TAU MU TAU MU TAU MU

CONSTITUTIONAL LAW II Order of Saint Thomas More Atty. Philip John Pojas/Atty. Rovyne G. Jumao-as, RN University College of Law Ruling: The issue involves the weighing out of constitutional guarantees of freedom of the press and the right to public information, of the one hand, and the fundamental rights of the accused, on the other hand, along with the constitutional power of a court to control its proceedings in ensuring a fair and impartial trial. When these rights race against each other, the right of the accused must be preferred. With the possibility of losing his life or liberty, it must be ensured that accused receives a verdict decreed by an imprejudiced judge. Television coverage, however, can impair the testimony in criminal trials, can the afect the performance of the judge, and can destroy the case of the accused in the eyes of the public. Accordingly, to protect the parties right to due process, to prevent the distraction of the participants in the proceedings and in the last analysis, to avoid miscarriage of justice, the request is denied.

Facts: Accused was charged with violation of the Dangerous Drugs Act. Arraigned on August 8, 1995, trial never commenced despite the lapse of 1 year and 3 months due to 11 postponements, 9 of which were secured by the prosecution. The grounds for postponements ranged from the absence of witnesses for the prosecution, absence of the prosecutor and reraffling of the case to another branch. Must the case be dismissed on the ground of speedy trial? Ruling: No. The reasons for the prosecution’s postponements were reasonable and were not intended merely to delay the proceedings of the case. It would be unjust to pounce on the absence of the witnesses as a basis for dismissing the case when there was a valid excuse for their absence, that is, there was no proof that they were duly served with subpoena. The other reasons for postponements were due to circumstances beyond the control of the prosecution. The right of the accused to speedy trial should not be utilized to deprive the State of a reasonable opportunity of fairly prosecuting criminals. DE ZUZUARREGUI vs. ROSETE 382 SCRA 1 (2002) Facts: A complaint was filed against Judge Rosete for gross ignorance of the law for dismissing with prejudiced criminal case for falsification of a private document filed by a complainant. Issue: Whether there was an abuse of discretion on the part of the respondent. Ruling: The requirement for postponement was not for any flimsy excuse but complainant was in the US for an operation. The right to a speedy trial is a relative one. Courts should do more than a mathematical computation of the number of postponements. What ofends the right to speedy trial are unjustified postponements which prolong trial for an unreasonable length of time. LUMANLAW vs. PERALTA 482 SCRA 396 (2006) Facts: Lumanlaw was apprehended for illegal possession of a dangerous drug. A year had passed since the filing of the Information, yet Lumanlaw remained uninformed of the charges against him, while continuing to be in detention and despair all throughout that period of limbo. Owing to this insuferable state of afairs, petitioner’s counsel manifested his intention to file a motion to dismiss on account of the violation of his client’s right to a speedy trial. Issue: WON there was a violation of Lumanlaw’s constitutional right to speedy trial.

TAU MU

Ruling: There were fourteen postponements in this case. Given the length and the unreasonableness of the majority of the delays, a violation of the right of petitioner to speedy trial becomes manifest. Almost two years elapsed from the filing of the Information against him until the filing of this Petition; incredibly, he has not been arraigned. An arraignment takes, at most, ten minutes of the court’s business and does not normally entail legal gymnastics. It consists simply of reading to the accused the charges leveled against them, ensuring their understanding of those charges, and obtaining their plea to the charges. A prudent and TAI LIM vs. CA resolute judge can conduct an arraignment as soon as 317 SCRA 521 (1999) the accused are presented before the court. In fact, by fixing a period of only thirty days from the filing of the information to the conduct of an ACADCOM 2010; Contributors: Gene Geocaniga, Jarissa Guiani, Darlene Magabilen TAU MU Page 160 of 179

TAU MU

TAU MU

SUMBANG vs. GENERAL COURT MARTIAL 337 SCRA 227 (2000)

TAU MU TAU MU TAU MU TAU MU TAU MU

FAILURE OF THE ACCUSED TO CROSS-EXAMINE A WITNESS: If the failure of the accused to cross-examine a witness is due to his own fault or was not due to the fault of the prosecution, the testimony of the witness should be excluded.

TAU MU

 Testimony not subjected to cross-examination must be excluded from consideration. HOWEVER if crossexamine actually commenced, but for lack of material time, it was not completed and the witness died before it could be resumed, so much of the testimony already covered by cross-examination is admissible [PEOPLE vs. SEBERIS 99 SCRA 92].  An exception to the requisite of confrontation is a dying declaration w/c are “declarations made in extremity, when the party is at the point of death, and when every hope of this world is gone; when every motive to falsehood is silenced and the mind is induced by the most powerful considerations to speak the truth. A situation so solemn and so awful as to be considered by the law as creating an obligation equal to that which is imposed by a positive oath in a court of justice” [US vs. GIL (1909)].

TAU MU

PURPOSES: a. to aford the accused an opportunity to test the testimony of the witness by cross-examination and b. to allow the judge to observe the deportment of the witness [GO vs. COLLECTOR OF CUSTOMS (1934)].

TAU MU

6. RIGHT TO MEET WITNESS FACE TO FACE (CONFRONTATION)

TAU MU TAU MU

Ruling: The determination of whether an accused has been denied the right to speedy trial must have to depend on the surrounding circumstances of each case. Although it is unfortunate that it took about 8 years form 1991 before the trial resumed in 1999, the delay does not amount to violation of the right considering that it could not be attributable to the prosecution. The delay was due to the changes in the composition of the court martial. Notably, from the time petitioner’s motion to dismiss was file in 1991, he did not take action to assert his right. Thus, the supervening delay seems to have been without his objection hence impliedly with his acquiescence. The right to speedy trial may be waived.

The Fraternal Ateneo de Davao

TAU MU TAU MU

Facts: Petitioner was a member of the Philippine Constabulary accused before a court martial of killing a civilian on May 29, 1988. In view of the efectivity of RA No. 6975 otherwise known as the Philippine National Police Law, the composition of the court martial was changed so that the case remained pending for years. On Sept. 29, 1999, petitioner moved for the dismissal of the case on the ground of violation of the right to speedy trial.

KITY

TAU MU TAU MU TAU MU TAU MU TAU MU TAU MU

CONSTITUTIONAL LAW II Order of Saint Thomas More Atty. Philip John Pojas/Atty. Rovyne G. Jumao-as, RN University College of Law arraignment, RA 8493 recognizes that this fundamental right should and can be done with minimal delay. The protracted delay became all the more oppressive and vexatious when viewed from the perspective that the liberty of the accused was being curtailed for the entire duration. Hence, in the light of the numerous and unreasonable delays in the arraignment of petitioner, the sought for dismissal of the Information filed against him is in order.

WHEN IS THE RIGHT TO CROSS-EXAMINE DEMANDABLE: It is demandable only during trials. Thus, it cannot be availed of during preliminary investigations. PRINCIPAL EXCEPTIONS TO THE RIGHT OF CONFRONTATION: 1. The admissibility of “dying declarations” 2. Trial in absentia under Section 14(2) 3. With respect to child testimony Q: What are not admissible as evidence in court? A: (1) Extra judicial statements of an accused implicating another when not repeated in open court; and (2) Affidavits of Witnesses not produced in Court to be cross-examined. 7. RIGHT TO HAVE COMPULSORY PROCESS TO SECURE THE ATTENDANCE OF WITNESSES AND HE PRODUCTION OF EVIDENCE IN HIS BEHALF This right to compulsory process must be invoked during the trial. Failure to do so constitutes a waiver that cannot be rectified or undone on appeal. 

A person accused can obtain a subpoena from the court in order to compel the attendance of witnesses in his behalf.  A refusal of subpoena will result to detention until such time that compliance is made. EXCEPTION: If the person resides more than 100 kilometers from the place of trial, he is not bound by a subpoena. This rule applies only to Civil cases, not to Criminal cases. .  This right includes the right to testify in one’s favor [PEOPLE vs. SANTIAGO (46 Phil. 734)] and the right to be given time to call witnesses [US vs. LAO CHUECO (37 Phil. 53)].  This right may NOT be invoked on appeal if the accused made no efort during trial to avail of himself of it [US vs. GARCIA (1908)]. To establish the right to continuance by reason of the absence of witnesses, the accused must show that: a. the witness is really material; b. he is guilty of no neglect of previously obtaining attendance of said witness; c. the witness will be available at the time desired; and d. no similar evidence could be obtained. [PEOPLE vs. SANDAL (54 Phil. 883)] TRIAL IN ABSENCIA  The right of the accused to be present during trial after arraignment may be waived because this provision allows trial to continue even w/o the presence of the accused.  The accused’s present during trial may be totally waived except when, after arraignment, his presence is needed for purposes of identification. The reason is to forestall the accused from using the defense that he was never identified as the person charge in the information and therefor entitled to acquittal.  For an accused to be excused from attending trial, he must unqualifiedly admit that every time a witness mentions a name by w/c he is known, the witness is to be understood as referring to him [VILLAVICENCIO vs. LUKBAN (39 Phil. 778)].  The purpose of this manner of trial is to prevent unnecessary delays of the trial.

TAU MU

REQUISITES of TRIAL IN ABSENCIA: 1. Accused has been arraigned; ACADCOM 2010; Contributors: Gene Geocaniga, Jarissa Guiani, Darlene Magabilen TAU MU Page 161 of 179

GENERAL RULE: The right to attend trial is waivable. EXCEPTION: 1. Arraignment 2. During the identification stage (witness testifies to the identity of the accused) 3. Promulgation of judgment Q: When can an accused be compelled by the court to attend trial? A: (1) Arraignment; (2) During the identification stage (witness testifies to identify the accused); and (3) Promulgation of judgment.

KITY

TAU MU TAU MU

Note: The accused may not invoke such right on appeal if he made no efort during the trial to avail himself of it. CASE ON TRIAL IN ABSENTIA:

TAU MU TAU MU TAU MU

Ruling: No. Once jurisdiction is acquired, it is never lost.

TAU MU

Issue 1: Whether the court loses jurisdiction over a person who escapes.

TAU MU

Facts: There were several accused and after he was arraigned accused escaped. The trial proceeded in his absence and the judge convicted his co-accused. However, insofar as the one who escaped, the judge withheld his decision that he can enjoy his constitutional right to confrontation if he is arrested. (Some think that this is wrong. Accused should be judged basing on the evidences presented during the trial in his absence.)

TAU MU

JIMENEZ vs. NAZARENO

TAU MU

Ruling: No. Under Sec. 14 (2) of the Bill of Rights, the following are the requisites of a valid trial in absentia: 1) accused had already been arraigned; 2) he has been duly notified of the trial, and 3) his failure to appear is unjustifiable. In this case accused had not been duly notified of the trial because notice of hearing was sent to his former address despite the fact he notified the court of his change of address.

TAU MU

Facts: Accused was charged with estafa and was out on bail. While trial was going on, accused changed his address notifying the court through his counsel as well as the bonding company. When accused failed to appear during a hearing because notice was sent to his old address, the judge issued a warrant for his arrest, appointed a counsel de officio for him, ordered a trial in absentia, and convicted him on the theory that he waived his right to present evidence. Was there a valid trial in absentia?

TAU MU TAU MU

TO ESTABLISH THE RIGHT TO CONTINUANCE BY REASON OF THE ABSENCE OF WITNESS, THE ACCUSED MUST SHOW: 1) That the witness is really material; 2) That he is guilty of not neglect previously obtaining attendance of said witness; 3) That the witness will be available at the time desired. 4) That no similar evidence could be obtained from other witnesses.

PARADA vs. VENERACION 269 SCRA 371 (1997)

The Fraternal Ateneo de Davao

TAU MU TAU MU TAU MU TAU MU TAU MU TAU MU

CONSTITUTIONAL LAW II Order of Saint Thomas More Atty. Philip John Pojas/Atty. Rovyne G. Jumao-as, RN University College of Law 2. Notice of the trial was duly served to him and properly returned; 3. His failure to appear is unjustified or he waives his right to be present.

Issue 2: Whether the accused who escaped retains his right to confrontation to present evidence to cross examination and to be heard. Ruling: These rights are no longer retained once the accused escapes. The provision on trial in absentia will be useless. Escape tantamounts tp a waiver to your right to present evidence, confrontation, and etc. Sec. 15: The privilege of the writ of habeas corpus shall not be suspended except in cases of invasion or rebellion when public safety requires it. WRIT OF HABEAS CORPUS  It is a prerogative writ of liberty employed to test the validity of a person’s detention  It is directed to the person detaining another, commanding him to produce the body of the prisoner @ a designated time and place, w/ the day and cause of his caption and detention, to do, to submit to, and receive whatever the court or judge awarding the writ shall consider in his behalf  The OBJECT of this writ is the liberty of those who may be in prison w/o sufficient cause  An order issued by a court directed to a person (illegally) detaining another, commanding him to produce the body of the prisoner at a designated time and place, and to explain the cause of his detention. (Cross reference: Special Proceedings)  The Habeas Corpus is not filed against a person. It is a special proceeding and not a civil action. If it were civil, then there is the plaintif and defendant. Here, there is only the petitioner. You petition the court to issue a writ directing someone to return the person illegally detained. 

If a person is restrained of his liberty, he or someone acting on his behalf may file a petition for habeas corpus to secure his release. This action shall take precedence in the calendar of the court and must be acted upon immediately.  Mere delay in the resolution of the issue will by itself constitute an invalid derogation of a person’s right to unlawful restraint. Rule 102, Rules of Court: Sec. 1. To what habeas corpus extends. - Except as otherwise expressly provided by law, the writ of habeas corpus shall extend to all cases of illegal confinement or detention by which any person is deprived of his liberty, or by which the rightful custody of any person is withheld from the person entitled thereto.  The following grounds are recognized by the SC for the issuance of the writ of habeas corpus: 4. deprivation of any fundamental or constitutional right; 5. lack of jurisdiction of the court to impose the sentence; 6. imposition of excessive penalties; Q: When is this remedy available? A: The writ may be resorted to where a person’s liberty is subjected to physical restraint. Q: Is this right/remedy limited to cases of physical restrain? A: NO. Even moral restraint is a ground for the issuance of this writ [CUANCA vs. SALAZAR].

TAU MU

INSTANCES WHEN THE WRIT CAN BE AVAILED OF: 1. If a person has finished serving his sentence, his incarceration has become unlawful.

ACADCOM 2010; Contributors: Gene Geocaniga, Jarissa Guiani, Darlene Magabilen TAU MU Page 162 of 179

TAU MU TAU MU TAU MU TAU MU TAU MU TAU MU

INSTANCES WHEN THE JUDGE WILL NOT ISSUE THE WRIT: 1. If the person is in the custody of an officer and under process issued by a court or judge or by virtue of a judgment or order of a court of record, and that the court or judge had jurisdiction to issue the process, render the judgment, or make the order. Meaning legal detention. No way will the court issue the writ; 2. When a person has been judicially charged with or convicted or a crime in the Philippines. Again, legal detention. You cannot use the writ if the ofense is bailable and you cannot post bail. You cannot use the writ to get out of jail; 3. If the jurisdiction appears after the writ is allowed, the person shall not be discharged by reason of any informality or defect in the process, judgment or order; 4. A person sufering imprisonment under lawful judgment; 5. In the case of Panillo vs. Salonga (June 24, 1994), if the issue of detention raised in the petition for a writ of habeas corpus is necessarily related with another pending case; 6. If the petitioner has the remedy of appeal; 7. When the writ is suspended and the person is being detained for the crimes covered by the suspension.

TAU MU

Note: Non-production of the person detained is sufficient ground to hold the ofender in contempt of court. The ofender must exert all eforts to produce the body, mere writing of letters is not enough.

TAU MU TAU MU

THREE WAYS OF COMPLYING WITH THE WRIT OF HABEAS CORPUS: 1. Produce the body; 2. If the person is sick and cannot be safely brought to court, affidavit; 3. Proving, by means of affidavit, that the person detained has waived his right to be present (the person who is being detained will produce the affidavit)

The Fraternal Ateneo de Davao

TAU MU TAU MU

PURPOSE: Best remedy for personal freedom because by the Writ of HC, the object is the speedy release by judicial decree of persons who are illegally detained or deprived of their liberty. This is a weapon against arbitrary use of State power.

KITY

TAU MU TAU MU TAU MU TAU MU TAU MU TAU MU

CONSTITUTIONAL LAW II Order of Saint Thomas More Atty. Philip John Pojas/Atty. Rovyne G. Jumao-as, RN University College of Law The authorities no longer have any right to keep him in prison. If he is not released, then his relatives can file for Habeas Corpus. 2. When patients are detained in hospitals for failure to pay the bill. If the corpse is not released, will the writ apply? No, because the Bill of Rights can only be availed of by living citizens of the Philippines, or aliens. 3. Custody of Children 4. When a person is denied of one or more of his constitutional rights. 5. When restraint is involuntary and unnecessary 6. When the deprivation of freedom which was originally valid becomes arbitrary later on 7. When the court has no jurisdiction to impose the sentence 8. Imposition of excessive penalties

Q: Can the judge inquire as to the reason of the detention of the detainee? A: No, because the detainee is already covered. And the judge can no longer order the release of the detainee. What happens? Is the writ really suspended? It is only the privilege of the writ that is suspended. So, even if there is rebellion, invasion, anybody can still petition for habeas corpus. And just because there is rebellion, the judge cannot say " I will not issue a writ.". So if there is an application, and it is shown that there is illegal detention, the judge will issue the writ. Unless the decision is legal. Now, when the judge issues the writ, what happens. It is addressed to the person detaining another. As a detaining officer, you have to prove, you have to show the judge that he (the person detained) is covered by the suspension, and once he is covered by the detention, the judge cannot do anything anymore. But he (the judge) has to issue the writ, even if the privilege is suspended.  Art. 7, Sec. 18: The suspension of the privilege of the writ shall apply only to persons judicially charged with rebellion or ofenses inherent in, or directly connected with invasion. LIMITATIONS ON THE POWER OF THE PRESIDENT TO SUSPEND THE WRIT: 1. Suspension is limited to 60 days only; 2. Congress may extend the period, upon initiative of the President, if the invasion or rebellion still exists after the 60 day period, with the condition that public safety requires it. 3. Congress by a majority vote can revoke the suspension, and this suspension cannot be set aside by the president. 4. Any citizen can file a petition in the SC challenging the decision and the court can look into the legality of the decision. (The issue now becomes a political question, and the Court must promulgate its decision within 30 days) 5. If the person is detained, these authorities can only detain him for 3 days. (if he is arrested without a warrant) He has to be charged with an ofense. 6. The suspension of the privilege of the writ shall apply only to persons judicially charges with rebellion or ofenses inherent in, or directly connected with invasion. A prisoner may avail of this remedy to secure his release if he was convicted by a court w/o jurisdiction or his sentence had become invalid [ALCANTARA vs. DIR. OF PRISONS 75 Phil. 749].  The right is also available to a person sentenced to a longer penalty than that subsequently meted out to another person convicted of the same ofense [GUMABON vs. DIR. OF PRISONS 37 SCRA 420].  Habeas Corpus may also be resorted to in case of a wrongful denial of bail [ZAFRA vs. CITY WARDEN].  HOWEVER where a decision is only tainted w/ errors of law, a petition for habeas corpus will not lie. If the error does not go to the jurisdiction of the court, the proper remedy would not be the prerogative writ but an ordinary appeal [CELESTE vs. PEOPLE 31 SCRA 391].  BUT if the error alleged denies the right to a speedy trial, the same is considered jurisdictional and so may be corrected on habeas corpus [CONDE vs. RIVERA 45 Phil. 650]. 

TAU MU

Note: Even if the President suspends the privilege of the Writ, anybody can still file a petition of Habeas Corpus. The judge can still issue the Writ. But once the writ reaches the detaining officer and it is proven that the detainee is covered by the suspension that is the time that the writ shall not be efective. The detaining officer merely has to prove that the person detained is covered. ACADCOM 2010; Contributors: Gene Geocaniga, Jarissa Guiani, Darlene Magabilen TAU MU Page 163 of 179

TAU MU

TAU MU TAU MU TAU MU

Facts: Petitioner was convicted of 3 counts of falsification of public documents, which convictions were affirmed by the Court of Appeals and the Supreme Court. Claiming that the proceedings were attended by violations of her constitutional rights, consisting of the failure of the judge to appreciate certain evidence in favor, as well as the presentation by the prosecution of a witness who might have been perjured, petitioner prays for the issuance of a writ of habeas corpus. Is the remedy proper?

TAU MU

IN RE: GARCIA 339 SCRA 292 (2000)

TAU MU

Ruling: Petitioners are correct that the extra-ordinary writ of habeas corpus is the appropriate remedy to inquire into questions of violations of constitutional rights and that this Court has jurisdiction to entertain the review. However, in this case there was no violation of the constitutional rights of the accused since they did not make any confession or admission during the identification in a police line-up. Thus, there was no need for them to be assisted by counsel.

TAU MU

Facts: Accused was sentenced to death by the Regional Trial Court for rape with homicide which decision had been affirmed by the Supreme Court. They later filed a petition for habeas corpus on the ground that the trial court was ousted of jurisdiction to try their case since the pre-trial identification was made without the assistance of counsel. The quoted Justice Teehankee in Olagner who stated that: “Once a deprivation of a constitutional right is shown to exist, the court that rendered the judgment is deemed ousted of its jurisdiction and habeas corpus is the appropriate remedy to assail the legality of detention.”

TAU MU TAU MU

ANDAL vs. PEOPLE 307 SCRA 650 (1999)

The Fraternal Ateneo de Davao

TAU MU TAU MU

CASES: HABEAS CORPUS

KITY

TAU MU TAU MU TAU MU TAU MU TAU MU TAU MU

CONSTITUTIONAL LAW II Order of Saint Thomas More Atty. Philip John Pojas/Atty. Rovyne G. Jumao-as, RN University College of Law  The writ itself is not suspended --- thus it may be issued --- but only the privilege --- meaning it cannot be given efect. NOTE: See also ARTICLE VII Sec. 18.  The SC has the power to annul the suspension of the privilege if the same is not based on the grounds enumerated in the constitution: 1.invasion or rebellion 2.when the public safety so requires.  While the President may suspend the privilege of the writ or impose martial law for 60 days, Congress may, on his initiative, extend such measures with the concurrence of only a majority of its members voting jointly --- same vote needed to revoke presidential acts. The extension may be indefinite, depending on the discretion of the legislature; which may be dictated by the President.  The SC has the power to revoke extension if it is without factual basis.

TAU MU

Ruling: The high prerogative writ of habeas corpus was devised as a speedy and efectual remedy in cases of unlawful restraint. Its object is to inquire into the legality of ones’ detention, and if found illegal, to order the release of he detainee. However, it will not issue where the person in whose behalf the writ is sought is in custody under process issued by a court with jurisdiction to issue the process or order. In this case, petitioner is not entitled to the relief via a petition for habeas corpus since she has been convicted by final judgment of the crime of falsification of public document. While it has been held that once a deprivation of a constitutional right is shown to exist, the court that rendered judgment is deemed ousted of ACADCOM 2010; Contributors: Gene Geocaniga, Jarissa TAU MU Page 164 of 179

jurisdiction and habeas corpus is the proper remedy, petitioner has failed to persuade the Court that the proceedings before the trial court were attended by violations of her constitutional rights. FERIA vs. CA 325 SCRA 525 (2002) Facts: Petitioner has been in jail since May 1981 and convicted of Robbery with Homicide. In 1993, he sought to be transferred to Muntinlupa but the transfer could not be efected because the documents relating to his conviction were not submitted. Further inquiry revealed that the entries records appear to have been destroyed in a fire. Does the destruction of the records render the conviction void as to warrant the release of the convict by virtue of a writ of habeas corpus? Ruling: No. The absence of a copy of a valid judgment does not mean that there is no legal basis to detain petitioner. Based on the hearings conducted, there is sufficient evidence to establish the fact of his conviction. Petitioner himself admitted in one of his motions that he was actually convicted. The mere loss or destruction of the records of the case does not invalidate the judgment of the commitment, or authorize the prisoner’s release. His remedy is not a petition for habeas corpus but a proceeding for the reconstitution of judicial records. CALVAN vs. CA, SALES 341 SCRA 806 (2000) Facts: MFR of a decision of the SC. Sales shot former Mayor Benemerito. After the incident, he placed himself under the custody of the municipal police. A criminal complaint for murder was charged against him. Judge Calvan concluded the preliminary investigation without allowing him to submit counteraffidavit and present his witnesses. Sales filed a petition for habeas corpus. CA dismissed the petition initially but granted it thereafter. Issue: Whether the petition for writ of habeas corpus should be granted. Ruling: A judge has the constructive custody over Sales for having issued the order and warrant for his arrest. In habeas corpus, the concern is not merely ordering or holding the petitioner in custody, but whether such error is sufficient to render void the judgment. The illegal order and warrant of arrest issued by Judge subsists and ofered no speedy, adequate remedy or appeal in the ordinary court of law. The writ of habeas corpus can be invoked by the attendance of a special circumstance that requires immediate action. Respondent cannot resort to the remedy of a motion to quash, the case no longer being with the judge. IN RE: AQUINO vs. ESPERON August 31, 2007

TAU MU TAU MU

Facts: The facts leading to the arrest of Major Aquino, show that, along with several military men allegedly met to plot a breach of the Camp Defense Plan of Camp General Emilio Aguinaldo and to take over Camp Aquinaldo, as well as the Headquarters of the Philippine Army. In the wake of the group’s alleged withdrawal of support from the Armed Forces of the Philippines chain of command and the current administration of PGMA, Major Aquino was ordered arrested and confined at the Intelligence Service Group, upon the order Lt. Gen. Esperon. Hence, petitioner filed a petition for habeas corpus with the CA praying that the AFP Chief of Staf and the Commanding General of the Philippine Army, Guiani, Darlene Magabilen

TAU MU TAU MU TAU MU TAU MU

WRIT OF HABEAS DATA  Section 6 of the Rule on the Writ of Habeas Data requires the following material allegations of ultimate facts in a petition for the issuance of a writ of habeas data: (a) The personal circumstances of the petitioner and the respondent; (b) The manner the right to privacy is violated or threatened and how it afects the right to life, liberty or security of the aggrieved party;

TAU MU

RULING: The court held a release that renders a petition for habeas corpus moot and academic is one that is unconditional. In this case, the restrictions efectively denied him of his constitutional rights. Restrictions are declared null and void.

TAU MU

Facts: Moncupa was arrested for being a member of the NDF. His lawyer filed a petition for habeas corpus. While the petition was pending, the military released him. After his release, the military attached many conditions (an example is he cannot travel without permission). After the military argued that the petition was moot and academic since he was already released.

TAU MU

MONCUPA vs. ENRILE

TAPUZ vs. DEL ROSARIO 554 SCRA 768

TAU MU

Ruling: The court held that any restraint which will preclude freedom of action is sufficient reason to issue the writ. The forcible taking of the women by the city officials who handed those to other parties and deposited them in a distant region deprived them of locomotion as efectively as if they had been imprisoned. Without money or belongings, they were prevented from exercising their right of going where they want. So restraint continues until they are brought back to Manila, unless they do not want to return. NOTE: The petition for habeas corpus can be filed by anybody in behalf of the detainee.

(c) The actions and recourses taken by the petitioner to secure the data or information; (d) The location of the files, registers or databases, the government office, and the person in charge, in possession or in control of the data or information, if known; (e) The reliefs prayed for, which may include the updating, rectification, suppression or destruction of the database or information or files kept by the respondent. In case of threats, the relief may include a prayer for an order enjoining the act complained of; and (f) Such other relevant reliefs as are just and equitable."

TAU MU

Facts: Mayor Lukban of Manila herded 120 prostitutes into a beer and brought from Manila to Davao. The relatives of some went and filed a petition for habeas corpus. Mayor’s defense was that habeas corpus was not the proper remedy because there was no restraint of liberty.

Ateneo de Davao

TAU MU TAU MU

Ruling: A writ of habeas corpus extends to all cases of illegal confinement or detention by which any person is deprived of his liberty, or by which the rightful custody of any person is withheld from the person entitled to it. As a general rule, the writ of habeas corpus will not issue where the person alleged to be restrained of his liberty is in the custody of an officer under a process issued by the court which has jurisdiction to do so. Its essential object and purpose is to inquire into all manner of involuntary restraint and to relieve a person from it if such restraint is illegal. In the case at bar, Major Aquino stands charged in court martial proceedings for alleged violations of Article 67 (Attempting to Begin or Create Mutiny) and Article 96 (Conduct Unbecoming an Officer and Gentleman) of the Articles of War. The legality of Major Aquino’s restraint having been settled, the privilege of the writ is unavailing. VILLAVICENCIO vs. LUKBAN

The Fraternal

TAU MU TAU MU

Issue: WON the denial of petitioner’s petition for writ of habeas corpus in the person of Major Aquino was valid.

KITY

TAU MU TAU MU TAU MU TAU MU TAU MU TAU MU

CONSTITUTIONAL LAW II Order of Saint Thomas More Atty. Philip John Pojas/Atty. Rovyne G. Jumao-as, RN University College of Law or whoever are acting in their place and stead, be directed to immediately produce the body of Major Aquino and explain forthwith why he should not be set at liberty without delay.

CASE: WRIT OF HABEAS DATA

Facts: The private respondents spouses Sanson filed a complaint for forcible entry and damages against the. The private respondents alleged in their complaint that: (1) they are the registered owners the disputed land; (2) they were the disputed land's prior possessors when the petitioners - armed with bolos and carrying suspected firearms and together with unidentified persons numbering 120 - entered the disputed land by force and intimidation, without the private respondents' permission and against the objections of the private respondents' security men, and built thereon a nipa and bamboo structure. The petitioners denied the material allegations of the complaint. They essentially claimed that: (1) they are the actual and prior possessors of the disputed land; (2) on the contrary, the private respondents are the intruders; and (3) the private respondents' certificate of title to the disputed property is spurious. They asked for the dismissal of the complaint and interposed a counterclaim for damages. Subsequently, private respondents filed a motion for demolition. Meanwhile, the petitioners opposed the motion for demolition. The respondent Judge nevertheless issued via a Special Order a writ of demolition after the Sherif's written notice to the petitioners to voluntarily demolish their house/s to allow the private respondents to efectively take actual possession of the land. When the sheriif used the notice to vacate and for demolition, petitioners filed a petition which contains and prays for three remedies, namely: a petition for certiorari; the issuance of a writ of habeas data under the Rule on the Writ of Habeas Data; and finally, the issuance of the writ of amparo under the Rule on the Writ of Amparo. The petitioners alleged in their motion that a petition for a WRIT OF HABEAS DATA is prayed for so that the PNP may release the report on the burning of the homes of the petitioners and the acts of violence employed against them by the private respondents, furnishing the Court and the petitioners with copy of the same; and that petitioners apply for a WRIT OF HABEAS DATA commanding the PNP to produce the police report pertaining to the burning of the houses of the petitioners in the land in dispute and likewise the investigation report if an investigation was conducted by the PNP." Issue: WON the issuance of the writ of habeas date is valid. Ruling: The SC ruled that the petitions for certiorari and issuance of a writ of habeas data fatally defective, both in substance and in form. The petition for the issuance of the writ of amparo, on the other hand, is fatally defective with respect to content and substance.

ACADCOM 2010; Contributors: Gene Geocaniga, Jarissa Guiani, Darlene Magabilen TAU MU Page 165 of 179

that it breaks the expectation of impunity in the commission of these ofenses; it is curative in that it facilitates the subsequent punishment of perpetrators as it will inevitably yield leads to subsequent investigation and action. In the long run, the goal of both the preventive and curative roles is to deter the further commission of extralegal killings and enforced disappearances.  It is similar to habeas corpus, only broader. This is practiced in Mexico. And petition can be filed for deprivation of any constitutional right.

Facts: Petitioners are settlers in a certain parcel of land situated in Pasig City. Their dwellings/houses have either been demolished as of the time of filing of the petition, or is about to be demolished pursuant to a court judgment. Hence, they filed a petition for writ of Amparo before the SC alleging that they were deprived of their liberty, freedom and/or rights to shelter enshrined and embodied in our Constitution, as the result of these nefarious activities of both the Private and Public Respondents.

TAU MU TAU MU TAU MU TAU MU TAU MU TAU MU TAU MU TAU MU

 The1987 Constitution does not explicitly provide for the writ of Amparo. But the second paragraph of Article VIII, Section 1 of the 1987 Constitution, the Grave Abuse Clause, provides for the judicial power "to determine whether or not there has been a grave abuse of discretion amounting to lack or excess of jurisdiction on the part of any branch or instrumentality of the Government." The Clause accords a similar general protection to human rights extended by the Amparo contra leyes, Amparo casacion, and Amparo administrativo. Amparo libertad is comparable to the remedy of habeas corpus found in several provisions of the 1987 Constitution.  Writ of Amparo ofers a better remedy to extralegal killings and enforced disappearances and threats thereof. The remedy provides rapid judicial relief as it partakes of a summary proceeding that requires only substantial evidence to make the appropriate reliefs available to the petitioner; it is not an action to determine criminal guilt requiring proof beyond reasonable doubt, or liability for damages requiring preponderance of evidence, or administrative responsibility requiring substantial evidence that will require full and exhaustive proceedings.  The writ of Amparo serves both preventive and curative roles in addressing the problem of extralegal killings and enforced disappearances. It is preventive in

Ateneo de Davao

TAU MU TAU MU

CHARACTERISTICS OF ENFORCED DISAPPEARANCES: 1. An arrest, detention or abduction of a person by a government official or organized groups or private individuals acting with the direct or indirect acquiescence of the government; 2. The refusal of the State to disclose the fate or whereabouts of the person concerned or a refusal to acknowledge the deprivation of liberty which places such persons outside the protection of law.

The Fraternal

TAU MU TAU MU

WRIT OF AMPARO  It was promulgated on October 24, 2007 "in light of the prevalence of extralegal killing and enforced disappearances." It was an exercise for the Court's expanded power to promulgate rules to protect the people's constitutional rights, which made its maiden appearance in the 1987 Constitution in response to the Filipino experience of the martial law regime.  The Amparo Rule was intended to address the intractable problem of "extralegal killings" and "enforced disappearances," its coverage, in its present form, is confined to these two instances or to threats thereof. "Extralegal killings" are "killings committed without due process of law, i.e., without legal safeguards or judicial proceedings."

KITY

TAU MU TAU MU TAU MU TAU MU TAU MU TAU MU

CONSTITUTIONAL LAW II Order of Saint Thomas More Atty. Philip John Pojas/Atty. Rovyne G. Jumao-as, RN University College of Law The petitioner’s allegations obviously lack what the Rule on Writ of Habeas Data requires as a minimum, thus rendering the petition fatally deficient. Specifically, there were no concrete allegations of unjustified or unlawful violation of the right to privacy related to the right to life, liberty or security. The petition likewise has not alleged, much less demonstrated, any need for information under the control of police authorities other than those it has already set forth as integral annexes. The necessity or justification for the issuance of the writ, based on the insufficiency of previous eforts made to secure information, has not also been shown. In sum, the prayer for the issuance of a writ of habeas data is nothing more than the "fishing expedition" that the SC - in the course of drafting the Rule on habeas data had in mind in defining what the purpose of a writ of habeas data is not. In these lights, the outright denial of the petition for the issuance of the writ of habeas data is fully in order.

CASES: WRIT OF AMPARO CANLAS vs. NAPICO HOMEOWNERS 554 SCRA 208

Issue: WON the issuance of a writ of Amparo is proper. Ruling: The Rule on the Writ of Amparo provides that, “The petition for a writ of amparo is a remedy available to any person whose right to life, liberty and security is violated or threatened with violation by an unlawful act or omission of a public official or employee, or of a private individual or entity. The writ shall cover extralegal killings and enforced disappearances or threats thereof. The threatened demolition of a dwelling by virtue of a final judgment of the court, which in this case was affirmed with finality by this Court is not included among the enumeration of rights as stated in the above-mentioned provision for which the remedy of a writ of amparo is made available. Their claim to their dwelling, assuming they still have any despite the final and executory judgment adverse to them, does not constitute right to life, liberty and security. There is, therefore, no legal basis for the issuance of the writ of amparo. Under Section 6 of the same rules, the court shall issue the writ upon the filing of the petition, only if on its face, the court ought to issue said writ. Thus, under said provision,. “Upon the filing of the petition, the court, justice or judge shall immediately order the issuance of the writ if on its face it ought to issue. The clerk of court shall issue the writ under the seal of the court; or in case of urgent necessity, the justice or the judge may issue the writ under his or her own hand, and may deputize any officer or person to serve it. The writ shall also set the date and time for summary hearing of the petition which shall not be later than seven (7) days from the date of its issuance. “ Considering that there is no legal basis for its issuance, as in this case, the writ will not be issued and the petition will be dismissed outright. This new remedy of writ of amparo is intended for the protection of the highest possible rights of any person, which is his or her right to life, liberty and security. SECRETARY OF NATIONAL DEFENSE vs. MANALO OCTOBER 7, 2008

TAU MU

Facts: The case at bar involves the rights to life, liberty and security in the first petition for a writ of Amparo filed before the SC. This case was originally a Petition for Prohibition, Injunction, and Temporary ACADCOM 2010; Contributors: Gene Geocaniga, Jarissa Guiani, Darlene Magabilen TAU MU Page 166 of 179

The Fraternal Ateneo de Davao

TAU MU TAU MU TAU MU TAU MU

from victims of extralegal killings and enforced disappearances. The writ of Amparo is a tool that gives voice to preys of silent guns and prisoners behind secret walls. Hence, the petition was dismissed.

Section 16. All persons shall have the right to a speedy disposition of their cases before all judicial, quasi-judicial, or administrative bodies.  This provision guarantees the right generally to a

speedy disposition of cases. It covers the periods before, during and after trial.  It gives a broader protection that Sec. 14 (2) --- right to speedy trial.  This provision applies to civil, criminal, and administrative cases. SECTION 14 vs. SECTION 16 While the rights of an accused only apply to the trial phase of criminal cases, the right to a speedy disposition of cases covers ALL phases of JUDICIAL, QUASI-JUDICIAL or ADMINISTRATIVE proceedings. In short, the right to a speedy trial granted in Sec. 14 is applicable only to criminal cases.

TAU MU

FACTORS TO BE CONSIDERED: 1. length of delay 2. reason for the delay 3. assertion of the right or failure to assert it 4. prejudice caused by the delay

TAU MU

 Factors to be considered are the same as the factors of speedy trial. As mentioned, the right of speedy disposition of cases is broader than the rights of the accused to speedy trial. If the delay will focus on the trial itself, then it falls under speedy trial.

TAU MU

Issue: WON respondents have a right to the privilege of the writ of Amparo.

KITY

TAU MU TAU MU TAU MU TAU MU TAU MU TAU MU

CONSTITUTIONAL LAW II Order of Saint Thomas More Atty. Philip John Pojas/Atty. Rovyne G. Jumao-as, RN University College of Law Restraining Order (TRO) filed before the SC respondents (therein petitioners) to stop herein petitioners (therein respondents) and/or their officers and agents from depriving them of their right to liberty and other basic rights. In SC’s resolution, it ordered (1) ordered the Secretary of the Department of National Defense and the Chief of Staf of the AFP, their agents, representatives, or persons acting in their stead, including but not limited to the Citizens Armed Forces Geographical Unit (CAFGU) to submit their Comment; and (2) enjoined them from causing the arrest of therein petitioners, or otherwise restricting, curtailing, abridging, or depriving them of their right to life, liberty, and other basic rights as guaranteed under Article III, Section 1 of the 1987 Constitution. While the Petition was pending, the Rule on the Writ of Amparo took efect. Petitioners filed a Manifestation and Omnibus Motion to Treat Existing Petition as Amparo Petition, to Admit Supporting Affidavits, and to Grant Interim and Final Amparo Reliefs. They prayed that: (1) the petition be considered a Petition for the Writ of Amparo under Sec. 26 of the Amparo Rule; (2) the Court issue the writ commanding therein respondents to make a verified return within the period provided by law and containing the specific matter required by law; (3) they be granted the interim reliefs allowed by the Amparo Rule and all other reliefs prayed for in the petition but not covered by the Amparo Rule; (4) the Court, after hearing, render judgment as required in Sec. 18 of the Amparo Rule; and (5) all other just and equitable reliefs. The Court resolved to treat the Petition as a petition under the Amparo Rule and accordingly, the privilege of writ of Amparo was granted.



TAU MU

Ruling: The production order under the Amparo Rule should not be confused with a search warrant for law enforcement under Article III, Section 2 of the 1987 Constitution. This Constitutional provision is a protection of the people from the unreasonable intrusion of the government, not a protection of the government from the demand of the people such as respondents. Instead, the Amparo production order may be likened to the production of documents or things under Section 1, Rule 27 of the Rules of Civil Procedure which provides in relevant part. Petitioners assert that the disclosure of the present places of assignment of M/Sgt. Hilario aka Rollie Castillo and Donald Caigas, as well as the submission of a list of medical personnel, is irrelevant, improper, immaterial, and unnecessary in the resolution of the petition for a writ of Amparo. They add that it will unnecessarily compromise and jeopardize the exercise of official functions and duties of military officers and even unwittingly and unnecessarily expose them to threat of personal injury or even death. On the contrary, the disclosure of the present places of assignment of M/Sgt. Hilario aka Rollie Castillo and Donald Caigas, whom respondents both directly implicated as perpetrators behind their abduction and detention, is relevant in ensuring the safety of respondents by avoiding their areas of territorial jurisdiction. Such disclosure would also help ensure that these military officers can be served with notices and court processes in relation to any investigation and action for violation of the respondents' rights. The list of medical personnel is also relevant in securing information to create the medical history of respondents and make appropriate medical interventions, when applicable and necessary. In blatant violation of our hard-won guarantees to life, liberty and security, these rights are snufed out ACADCOM 2010; Contributors: Gene Geocaniga, Jarissa TAU MU Page 167 of 179

The remedy for the violation of this right is MANDAMUS --- same remedy as to violation of the right to a speedy trial [ROQUE vs. OMBUDSMAN (5/12/1999)]. CASES: RIGHT VIOLATED CERVANTES vs. SANDIGANBAYAN 307 SCRA 149 (1999)

TAU MU TAU MU

Facts: On March 6, 1986, a complaint for falsification was filed against accused before the Tanodbayan. On Oct. 2, 1986, accused submitted his counter-affidavit. On May 18, 1992, or more than 6 years later, the Office of the Special Prosecutor filed an information before the Sandiganbayan against him for violation of R.A. No. 3019. Accused filed a motion to quash on the ground of violation of his right to speedy disposition of cases.

TAU MU TAU MU

Ruling: The information should be quashed. Accused was deprived of his right to speedy disposition of the case. It took 6 years from the filing of the initiatory complaint before the information for the ofense was filed in court. It is the duty of the Special Prosecutor to speedily resolve the complaint, as mandated by the Constitution, regardless of whether accused did not object to the delay or that the delay was with his acquiescence provided that it was not due to causes directly attributable to him. LOPEZ vs. OMBUDSMAN 364 SCRA 568 (2001)

TAU MU

Facts: On Dec. 20, 1993, members of the COA Audit Team submitted a Joint-Affidavit Complaint against respondent before the Office of the Ombudsman. On April 22, 1994, respondent submitted his counterGuiani, Darlene Magabilen

TAU MU TAU MU TAU MU TAU MU TAU MU

Facts: Licaros was Vice Chairman of Home Savings Bank who was charged as an accessory to the crime of robber before the Sandiganbayan in an information dated July 6, 1982. he asked for a separate trial and on August 26, 1986, completed the presentation of his evidence and submitted the case for decision. The Sandiganbayan deferred decision until such time that the other accused have completed presentation of their own evidence, which took place on June 20, 1990. On March 23, 2000, or after about 10 years, he filed a motion to dismiss the case on the ground that the Sandiganbayan failed to decide the case thus violating his right to speedy disposition of cases. Should the case be dismissed?

TAU MU

LICAROS vs. SANDIGANBAYAN 370 SCRA 394 (2001)

HOFER vs. HRET 428 SCRA 383

TAU MU

Ruling: Yes. From the time of the arraignment of accused in 1992, there was an unexplained interval of inactivity of close to 5 years in the Sandiganbayan. Granting that the delay was caused by the separate motions for reinvestigation filed by the other accused, there was no explanation why the reinvestigation was unduly stretched. Apparently, the Ombudsman did not complete the reinvestigation. In addition, it cannot be said that petitioner failed to assert his right because he wrote the Ombudsman about the prejudice caused him by the cases. For all these past 11 years, petitioner has remained under a cloud and has been deprived of his retirement after serving the government for over 40 years because of the inaction of the Ombudsman. The delay here measures up to unreasonableness which entitles him to a dismissal of the cases.

the time it is deemed submitted for decision. Even if we were to consider the period provided under Sec. 15 (1) VIII, of the Constitution, which is 12 months from submission of the case for decision, the Sandiganbayan still failed to perform its duty to decide within the period fixed by law. Indubitably, there has been a transgression of the right to a speedy disposition of his case due to inaction on the part of the Sandiganbayan. Considering the 10 year-delay in the resolution of the case; the fact that petitioner has sufered vexation and oppression due to the delay ; that he did not sleep on his rights but has in fact asserted it; that he did not contribute in any manner to the delay; that the Sandiganbayan did not give any valid reason to justify the delay; and that he is charged as a mere accessory, dismissal is justified.

TAU MU

Facts: On May 1991, accused was charged with 2 counts of falsification of a public document. Because of some pre-trial motions, accused was arraigned only on July 28, 1992. Since his co-accused sought a reinvestigation of the cases trial did not proceed. Accused retired in 1994 but failed to get his benefits due to the pendency of the criminal cases. On March 20, 1997, accused wrote to the Ombudsman a request for the payment of his retirement benefits. On October 12, 1998, accused filed a motion to dismiss invoking his right to speedy trial. Should the cases be dismissed?

Ateneo de Davao

TAU MU TAU MU

ABARDO vs. SANDIGANBAYAN 355 SCRA 64 (2001)

The Fraternal

TAU MU TAU MU

Ruling: Yes. The preliminary investigation was resolved close to 4 years after the last reply affidavit was submitted. It took the Deputy Ombudsman 8 months to approve the resolution of the investigator and close to another year for the Ombudsman to approve the recommendation. The issues are not sufficiently complex to justify the length of time for their resolution. There is no statement that voluminous documentary and testimonial evidence are involved. The failure of the Ombudsman to resolve the complaints for almost 4 years is violative of the right of respondent. In such event, he is entitled to the dismissal of the cases filed against him.

KITY

TAU MU TAU MU TAU MU TAU MU TAU MU TAU MU

CONSTITUTIONAL LAW II Order of Saint Thomas More Atty. Philip John Pojas/Atty. Rovyne G. Jumao-as, RN University College of Law affidavit. More than four years later, on July 17, 1988, the Office of the Ombudsman-Mindanao issued a resolution recommending that respondent be charged with 30 counts of violation of the Anti-Graft and Corrupt Practices Act. Was there an unjustifiable delay on the part of the Ombudsman in resolving the complaint against respondent which violated his right to a speedy disposition of cases?

Facts: Petitioner Hofer and respondent Cabilao, were congressional candidates in the lone congressional district of Zamboanga Sibugay during the May 2001 national and local elections. Respondent was proclaimed the duly elected congressional representative. Claiming that massive vote buying, tampering of election returns and other irregularities were committed, petitioner filed with the HRET an election protest. HRET dismissed petitioner’s protest and ruled that petitioner’s failure to take necessary steps to prosecute this case justify its dismissal. Issue: WON there was a violation of respondent Cabilao’s right to speedy disposition of case. Ruling: The election protest filed by petitioner is a serious charge which, if true, could unseat protestee as Representative of her district. Hence, the observance of the HRET Rules in conjunction with the Rules of Court, must be taken seriously. Section 59 of the 1998 HRET Rules is explicit. Unfortunately, petitioner did not comply with it. In fact, despite the lapse of six (6) months, she failed to present her evidence. Such inaction shows her utter lack of interest to prosecute her case. As held in one case, “by their very nature and given the public interest involved in the determination of the results of an election, the controversies arising from the canvass must be resolved speedily, otherwise the will of the electorate would be frustrated. And the delay brought about by the tactics resorted to by petitioner is precisely the very evil sought to be prevented by election statutes and controlling case law on the matter." TATAD vs. SANDIGANBAYAN

TAU MU

Facts: Initially, the complaints were filed in 1974 against Tatad while he was still connected with Malacañang. In 1979, he resigned so the complaints were filed with the Tanodbayan. In 1982, the submission of evidence was completed. In 1985, the Tanodbayan recommended the filing of charges against him. Tatad claimed that there was a violation of his right to speedy disposition. Subject matter was the time between 1982 to 1985 since it was this time when the evidence were presented and the Tanodbayan should have recommended the filing of the charges at that time. The argument of the Solicitor General is that: In past decisions, absence of preliminary investigation is not a fatal defect and the may be cured, thus, the more reason that delay should not also be a fatal defect (NOTE: In past cases, fiscal can just file charges without PI).

Ruling: Yes. Under Sec. 6 of PD No. 1606, the Sandiganbayan has only 90 days to decide a case from ACADCOM 2010; Contributors: Gene Geocaniga, Jarissa Guiani, Darlene Magabilen TAU MU Page 168 of 179

CASES: RIGHT NOT VIOLATED BINAY vs. SANDIGANBAYAN 316 SCRA 65 (1999)

TAU MU TAU MU TAU MU TAU MU

DIMAYACYAC vs. CA 430 SCRA 121 (2004)

TAU MU

Ruling: No. The concept of “speedy disposition of cases”, like “speedy trial” is flexible. It is consistent with reasonable delay. The preliminary investigation in the subject cases against accused took more than 1 year and 4 months to finish. But such a happenstance alone, or any like delay, for that matter, should not be cause for an unfettered abdication by the court of its duty to try cases. The delay adverted to in the cases under consideration does not measure up to unreasonableness. It cannot be said that petitioners found themselves in a situation oppressive to their rights simply by reason of the delay without more.

TAU MU

Facts: Accused were employed with the National Food Authority. On Dec. 1991, a complaint for falsification of public document was filed against him before the Ombudsman. On Jan. 15, 1993, the last rejoinder was filed and the complaint was submitted for resolution. On May 30, 1994, or after 1 year and 4 months, the office of the Ombudsman issued a resolution ordering that they be charged with various crimes. Was their right to speedy disposition of cases violated?

TAU MU

DANSAL vs. FERNANDEZ 327 SCRA 145 (2000)

TAU MU TAU MU

Ruling: No. The concept of speedy disposition is a relative term. Factors to determine whether the right has been denied include the length of the delay, the reasons for such delay, the assertion or failure to assert such right by the accused and the prejudice caused by the delay. A mere mathematical reckoning of the time involved, therefore, would not be sufficient. In this case, there was no undue delay. The cases involved complex issues and voluminous documents. The prosecution had to review the findings of the Commission on Audit to determine probable cause. Thus, the length of time consumed before the resolution was issued is justified.

The Fraternal Ateneo de Davao

TAU MU TAU MU

Facts: On Sept. 7, 1994, the Office of the Ombudsman filed 3 information against Mayor Binay for violations of Art. 220 of the Revised Penal Code and 2 for violations of Sec. 3 (e) of the Anti-Graft Law before the Sandiganbayan. Considering that the affidavitcomplaint was filed before the Office of the Tanodbayan on July 27, 1988 for preliminary investigation, was there a violation of the right of accused to speedy disposition of cases?

KITY

TAU MU TAU MU TAU MU TAU MU TAU MU TAU MU

CONSTITUTIONAL LAW II Order of Saint Thomas More Atty. Philip John Pojas/Atty. Rovyne G. Jumao-as, RN University College of Law Ruling: The court held that long delayed in the termination of the preliminary investigation is violative of the right to speedy trial. The court also held that the absence of PI can be corrected. But undue delay in the conduct of the preliminary investigation cannot be corrected because until now, man still has not invented a device to set back time.

dismissed for the inordinate delay in the conduct of preliminary investigation for the purpose of filing the proper information, which is a violation of the accused’s constitutional right to due process of law and to speedy disposition of cases. Issue: WON petitioner’s constitutional right to a speedy disposition of his case has been violated. Ruling: The SC ruled in the negative. The right to a speedy disposition of cases, like the right to a speedy trial, is deemed violated only when the proceedings is attended by vexatious, capricious, and oppressive delays; or when unjustified postponements of the trial are asked for and secured, or when without cause or unjustifiable motive, a long period of time is allowed to elapse without the party having his case tried. In the determination of whether or not that right has been violated, the factors that may be considered and balanced are: the length of the delay the reasons for such delay, the assertion or failure to assert such right by the accused, and the prejudice caused by the delay. A mere mathematical reckoning of the time involved, therefore, would not be sufficient. In the application of the constitutional guarantee of the right to speedy disposition of cases, particular regard must also be taken of the facts and circumstances peculiar to each case. The records clearly show is that petitioner never asserted his right to a speedy disposition of his case. The only ground he raised in assailing the subsequent filing of the two informations is that he will be subjected to double jeopardy. It was only the OSG that brought to light the issue on petitioner’s right to a speedy disposition of his case, and only when the case was brought to the appellate court on certiorari. Even in this petition before the SC, petitioner did not raise the issue of his right to a speedy disposition of his case. Again, it was only the OSG that presented such issue in the Brief for the State which was only then adopted by petitioner through a Manifestation. The SC is not convinced that the filing of the informations against petitioner after two years was an unreasonable delay. Petitioner himself did not really believe that there was any violation of his right to a speedy disposition of the case against him. As held in one case, petitioner’s failure to assert their right to a speedy disposition of their cases was deemed to have waived such. Hence, the SC dismissed/denied this case. BERNAT vs. SANDIGANBAYAN 428 SCRA 787 (2004) Facts: Special civil action in the SC. Certiorari. On August 14, 1991, petitioner along with several coaccused, were charged by the Sandiganbayan with violation of the Section 3(e) of RA 3019, otherwise known as the Anti-Graft and Corruption Practices Act. The case submitted for decision more than 8 years had lapsed. Issue: Whether there is a violation of petitioner’s constitutional right to a speedy disposition of his case. Ruling: The right is deemed violated only when the proceedings are attended by vexations, capricious and oppressive delays. It is not a mere mathematical reckoning of time. Factors that should be considered are (1) length of delay; (2) reasons for the delay: (3) assertion or failure to assert such right by the accused: and (4) prejudice caused by the delay. Petitioner failed reasonably to assert his constitutional right to speedy trial.

TAU MU

Facts: Information for falsification of public documents was filed against petitioner along with some others. Before his arraignment, petitioner moved to quash the information on two (2) grounds. First, that the officer who filed the information had no legal authority to do so, and second, that more than one ofense was charged in the information. Pending resolution of the motion to quash, petitioner was arraigned. Subsequently., the OSG advances the view that CASTILLO vs. SANDIGANBAYAN the criminal case against herein petitioner may be 328 SCRA 69 (2000) ACADCOM 2010; Contributors: Gene Geocaniga, Jarissa Guiani, Darlene Magabilen TAU MU Page 169 of 179

TAU MU

Facts: The office of the Tanodbayan recommended the filing of an Information against accused for violation of RA No. 3019 on Oct. 10, 1987. however, it was only on Nov. 5, 1990, or after 3 years when an Information was filed before the Sandiganbayan by the Ombudsman. Was the right of accused to speedy disposition of cases violate?

TAU MU TAU MU TAU MU TAU MU TAU MU

TY-DAZO vs. SANDIGANBAYAN 374 SCRA 200 (2001)

TAU MU

Facts: As a result of an audit by the Commission on Audit on the Davao del Norte School of Fisheries, petitioners were ordered to submit their counteraffidavits on August 14, 1992, which they did on Dec. 3, 1992. On October 10, 1994, the investigator came up with a resolution which was approved by the Deputy Ombudsman on March 13, 1997 and by the Ombudsman on April 28, 1998. Based on the resolution, an information for violation of Republic Act. No. 3019 was filed against petitioners before the Sandiganbayan. Was there a violation of their right to speedy disposition of cases? Ruling: No. The right to speedy disposition is deemed violated only when the proceedings is attended by vexations, capricious and oppressive delays. The concept is relative and flexible. In determining whether the right has been violated, the factors that may be considered and balanced are as follows: (1) the length of the delay; (2) the reasons for the delay; (3) the assertion or failure to assert such right by the accused; and (4) the prejudice caused by the delay. In this case, while it took the investigator some 4 years to prepare a resolution, he had to go over the lengthy COA report and counter-affidavits as well as numerous receipts and other evidence. Moreover, it was only after the case was set for arraignment when petitioners raised the issue of delay. Their silence may, therefore, be interpreted as a waiver of the right.

TAU MU TAU MU

DELA PENA vs. SANDIGANBAYAN 360 SCRA 470 (2001)

The Fraternal Ateneo de Davao

TAU MU TAU MU

Ruling: No. The right to speedy disposition of cases is deemed violated only when the proceeding is attended by vexatious, capricious and oppressive delays. It was not so in this case. In 1988, the Zaldivar vs. Gonzales case was decided where it was held that the incumbent Tanodbayan lost his right to conduct preliminary investigation so that resolutions remained pending awaiting the appointment of an Ombudsman. When one was appointed, it took some time before his Office could be fully operational. Finally, this Court has taken notice that the Sandiganbayan has a heavy case load. The structural reorganization of prosecutorial agencies, procedural changes brought about by frequent amendments, as well as the Sandiganbayan’s case load are valid reasons for the delay in disposition of petitioner’s case.

KITY

TAU MU TAU MU TAU MU TAU MU TAU MU TAU MU

CONSTITUTIONAL LAW II Order of Saint Thomas More Atty. Philip John Pojas/Atty. Rovyne G. Jumao-as, RN University College of Law

TAU MU TAU MU

Facts: On Feb. 19, 1993, the police caught a mini-truck owned by petitioner, then Mayor of Salcedo, Eastern Samar, loaded with illegally cut logs. A complaint for violation of the Anti-Graft and Corrupt Practices Act was files on Sept. 1, 1993 before the Ombudsman, but an information was filed with the Sandiganbayan only on Feb. 17, 1995. She moved for a reinvestigation on July 4, 1995, but the court denied it on March 5, 1999. Meanwhile the complain for violation of PD No. 705 was file with the Ombudsman on May 11, 1993; but the corresponding information was filed with the Sandiganbayan only on April 28, 1997. She filed a motion for reinvestigation which was resolved only on ACADCOM 2010; Contributors: Gene Geocaniga, Jarissa TAU MU Page 170 of 179

January 4, 1999. Was the right of accused to speedy disposition of cases violated? Ruling: No. The right to a speedy disposition of cases is deemed violated only when the proceedings is attended by vexations, capricious and oppressive delays. A mere mathematical reckoning of the time involved would not be sufficient. In this case, reliance by petitioner in Tatad is misplaced. There is no showing that the charge against her is politically motivated. In addition, unlike in Tatad, the established procedure for preliminary investigation was observed in this case. Finally, petitioner contributed to the delay by filing a motion for reinvestigation. The bare allegation that it took the Ombudsman more than 3 years to terminated the preliminary investigation is not enough. A mere mathematical reckoning of the time involved is insufficient. Section 17. No person shall be compelled to be a witness against himself. PRIVILEGE OF RIGHT AGAINST SELFINCRIMINATION The right against self-incrimination has its roots in the common law and is based on humanitarian and practical considerations. Humanitarian because it is intended to prevent the State, with all its coercive powers, from extracting from the suspect testimony that may convict him. Practical because a person subjected to such compulsion is likely to perjure himself for his own protection. This right is available not only criminal prosecutions but also in all other government proceedings, including civil actions and administrative or legislative investigations. It may be claimed not only by the person accused of an ofense but by any witness to whom an incriminating question is addressed. SCOPE OF RIGHT 1. Testimonial self-incrimination – when one is required to answer a question which will incriminate him. This involves the use of intelligence, or acts which are not merely mechanical. 2. Exempts the accused from producing documents or articles demanded of him which are incriminating. Exceptions: a. When the state has the right to inspect the same under police power; b. Corporate documents, even if the same would incriminate the corporate officers; c. Public documents which are matters of public concern (CF: right of people to information) 3. It does not extend to non-testimonial acts, physical acts like blood test to determine if the accused was infected with a sexual disease. EXCEPTION: a. If the test will subject the accused to unnecessary and existent humiliation b. If the accused is required to submit a specimen of his handwriting to be used as evidence against him to a charge of forgery 4. It also exempts the accused to do a reenactment of the crime he is being charged of.

TAU MU

NON-TESTIMONIAL PHYSICAL ACTS ARE ALLOWED AS EVIDENCE SO LONG AS: 1. They are purely mechanical and does not involve the use of intelligence or imagination Guiani, Darlene Magabilen

 At the heart of Section 17 is the right to remain

TAU MU TAU MU TAU MU TAU MU

NOTE: a. Beltran vs. Samson – The accused was asked to give a sample of his handwriting. This is not only a mechanical act because it still needs the use of intelligence; b. When the accused is asked to take the lie detector test even if the questions asked are unrelated to the crime – not merely a mechanical act; c. When the accused is asked to re-enact the crime, the right attaches as this is not a mechanical act; d. Cases where the right was not violated:

TAU MU

WHAT ARE NOT COVERED BY THIS RIGHT: Non-testimonial acts such as blood tests to determine if the accused has STD. In other words, acts which do not involve the use of intelligence or mechanical acts; unless the act is communicative in nature, or if the test will subject the accused to unnecessary and existent humiliation.

TAU MU

DISTINCTION BETWEEN AN ACCUSED AND AN ORDINARY WITNESS 1. An accused can refuse to take the witness stand by invoking the right against self-incrimination. 2. An ordinary witness cannot refuse to take the stand. He can only refuse to answer specific questions which would incriminate him in the commission of an ofense.

TAU MU

WHEN IS A QUESTION INCRIMINATING: A question tends to incriminate when the answer of the accused or the witness would establish a fact which would be a necessary link in a chain of evidence to prove the commission of a crime by the accused or the witness.

TAU MU

silent --- so that accused cannot be forced to testify --in accord with the rules of fair play.  There is no right against incrimination per se, only self-incrimination.  This right guarantees that you cannot be forced to present evidence against yourself.  It is important for a person to know when a question is incriminating and when it is not.  An incriminating question is a question that leads to an admission to the commission of a crime.

TAU MU TAU MU

BASES OF THIS RIGHT: 1. Principle of humanity, to prevent the extortion of confessions under duress or compulsion; 2. Public policy, to prevent the witnesses or the accused from committing perjury.

The Fraternal Ateneo de Davao

TAU MU TAU MU

 What is PROHIBITED is the use of physical or moral compulsion to extort communication from the witness or to otherwise elicit evidence which would not exist were it not for the actions compelled from the witness.  The right does NOT PROHIBIT the examination of the body of the accused or the use of findings with respect to his body as physical evidence. Hence, the fingerprinting of an accused would not violate the right against self-incrimination. However, obtaining a sample of the handwriting of the accused would violate this right if he is charged for falsification.  The accused cannot be compelled to produce a private document in his possession which might tend to incriminate him. However, a third person in custody of the document may be compelled to produce it.

KITY

TAU MU TAU MU TAU MU TAU MU TAU MU TAU MU

CONSTITUTIONAL LAW II Order of Saint Thomas More Atty. Philip John Pojas/Atty. Rovyne G. Jumao-as, RN University College of Law 2. And if it will not subject the accused to unnecessary and causes humiliation

1. US vs. Tan: where the accused was examined for gonorrhea which might have been transmitted to the victim. 2. When the accused was asked to strip so he can be examined for scratch marks. 3. When accused was asked to go through a pregnancy test. 4. When one is dusted with florescent powder 5. When placed in a police line-up WHEN THE RIGHT CAN BE IINVOKED: 1. In criminal cases 2. In administrative proceedings if the accused is liable to a penalty (Ex. Forfeiture of property) Note: This right can be waived, however, the SC has held that there is no implied waiver of this right. WHO CAN INVOKE THE RIGHT: Only natural persons. Judicial persons are subject to the visitorial powers of the state in order to determine compliance with the conditions of the charter granted to them. SCOPE OF THE RIGHT AGAINST SELFINCRIMINATION IN VARIOUS PROCEEDINGS: 1. In CRIMINAL CASES, this is a prohibition against inquiry. It means that the accused cannot be compelled to take the witness stand. But if you are a witness and not the accused, the right extends only to refusal to answer incriminating questions. The witness has no right to refuse to take the witness stand. o Editor’s Note: Under the rules on evidence, there can be waiver if the accused takes the witness stand. But he can refuse to answer questions which are not related to his direct testimony. MEANING:  SCOPE: absolute/total prohibition of inquiry (Chavez Case).  The accused cannot be forced to take the witness stand and testify.  Accused can refuse to answer questions at the outset.  Witnesses in a criminal case can invoke this right optionally only --- right is not absolute for them --otherwise there would be a violation of accused’s right to compulsory process.  If the accused chooses to testify, he is deemed to have waived the protection of this right.  Waiver is only to the extent of the matter he testified to --- problematic --- waiver as to matter x: accused should answer questions only as to that matter, if he is asked about other matters, he can invoke right. 2. In CIVIL CASES, the right is just an option to refuse since there is no accused in civil cases. Anybody can be called to the witness stand, the only thing they have is the option to refuse questions which would tend to incriminate them. MEANING:  The right is only an option to refuse to answer an incriminating question at the moment it is asked.  Defendant must take the witness stand and wait for the incriminating question to be asked before he can refuse to answer it otherwise he can be jailed for contempt --- thus the right is not absolute in civil cases.

TAU MU

3. In ADMINISTRATIVE PROCEEDINGS, same as rule in civil cases. In short, as a general rule, proceedings other than criminal, the right against self incrimination

ACADCOM 2010; Contributors: Gene Geocaniga, Jarissa Guiani, Darlene Magabilen TAU MU Page 171 of 179

MEANING:  The same principle for civil cases applies to administrative proceedings.  Right can be invoked only in cases when the proceeding is penal in nature.

KITY

TAU MU TAU MU

Q: WHEN IS A QUESTION CONSIDERED INCRIMINATING? A: When the question tends to expose the person to the accusation of a crime or tend to establish guilt against him.

TAU MU TAU MU

TESTIMONIAL/NON-TESTIMONIAL I. ACCUSED IS EXEMPT FROM TESTIFYING  This right covers only testimonial rights except: 1. acts which require the use of intelligence; example: writing your signature, copying certain words 2. acts that communicate in nature. II. THE ACCUSED IS EXEMPT FROM PRODUCING DOCUMENTS  EXCEPTIONS: 1. Corporation or their officers (PASECO Case); 2. Public officials (ALMONTE vs. VASQUEZ).  Corporations are only creature of the state, they cannot resist the mandate of the state if they should be asked to produce certain documents.  Corporation officers cannot invoke this right when asked to produce documents of the corporation which may incriminate them as well because they are only acting in behalf of the corporation --- it is not self-incrimination because it is the corporation that is incriminated.  Public officials cannot invoke section 17 because the public has the right to information.

TAU MU

Q: WHAT IF A LIE DETECTOR IS USED? A: No jurisprudence yet on this. Atty. DELA BANDA: it is safe to assume that it would violate section 17 because answering the questions would require the use of intelligence

TAU MU

Q: WHAT ABOUT DRUG-TESTING? A: It is non-testimonial because the technologist will only get your urine sample.

TAU MU

TAU MU TAU MU TAU MU TAU MU

DOJ: a. employees cannot be compelled to take it --because it would be a compulsion on their part; b. people applying for a job in the government can be required to be tested --- if they refuse, the choice is still theirs, there is no compulsion

TAU MU

4. In LEGISLATIVE PROCEEDINGS – one can refuse to answer incriminating questions.  The principles for civil and administrative proceedings apply.

medical

The Fraternal Ateneo de Davao

TAU MU TAU MU TAU MU TAU MU TAU MU TAU MU

CONSTITUTIONAL LAW II Order of Saint Thomas More Atty. Philip John Pojas/Atty. Rovyne G. Jumao-as, RN University College of Law is merely an option to refuse to answer incriminating questions. Exceptions in administrative proceedings: a. PASCUAL vs. BOARD – An administrative proceeding against a doctor for malpractice. The SC ruled that he cannot be forced to take the witness stand without his consent since the penalty is so severe; b. CABAL vs. CAPUNAN – when the penalty involves forfeiture of one’s property.

A: Atty. DELA BANDA: it might fall in the exception where section 17 applies because it requires the use of intelligence --- imagination!  GALMAN CASE  IMMUNITY IS THE ONLY WAY TO DEFEAT THE RIGHT GIVEN BY SECTION 17. 1. USE AND FRUIT IMMUNITY – Prohibits the use of the testimony and its fruits in any manner in connection with the criminal prosecution of the witness. It merely grants the witness immunity fro use of any statement given before the court, but not immunity from prosecution by reason of or on the basis thereof. Ex. A, B, C, D are the accused. D is the state witness. Now D's testimony cannot be used in any manner in connection with the criminal prosecution in that particular case. His testimony cannot be used to convict him in that particular case. But this does not mean that he cannot be convicted, his testimony cannot be used to as evidence against him in that particular case. 2. TRANSACTIONAL IMMUNITY – makes the witness immune from criminal prosecution for the ofense for which his compelled testimony relates. So this is immunity from prosecution. CASES: RIGHT AGAINST SELF-INCRIMINATION PEOPLE vs. GALLARDE 325 SCRA 835 (2001) Facts: Accused was a suspect in a rape with homicide case. After his arrest, he was asked by a Kagawad to pull down his shorts at police headquarters and he complied. Photographs of him were also taken after the incident and presented during trial. Are the pictures admissible in evidence? Ruling: Yes. The constitutional right of an accused against self-incrimination proscribes the use of physical or moral compulsion to extort communications and not the inclusion of his body in evidence when it may be material. Purely mechanical acts are not included in the prohibition as the accused does not thereby speak his guilt, hence the assistance and guiding hand of counsel is not required. The essence of the right against self-incrimination is testimonial compulsion, that is the giving of evidence against himself through a testimonial act. Hence, the taking of pictures of an accused even without the assistance of counsel, being a purely mechanical act, is not a violation of his constitutional right against self-incrimination. PEOPLE vs. RONDERO 320 SCRA 383 (1999) Facts: Accused was charged with rape with homicide. When the corpse of the 9-year old victim was found, tightly gripped in her right hand were hair strands. To enable the National Bureau of Investigation to conduct an examination on the hair strands, it sent a fax message to the police that hair strands be pulled, no cut, from the suspect, from the four regions of his head for comparison with the specimen. By then the accused was detained and he now claims that his hair strands were taken by the police without his consent. Was his right against self-incrimination violated?

TAU MU

Ruling: No. What is proscribed by the right against self-incrimination is the use of physical or moral compulsion to extort communication from the accused, ACADCOM 2010; Contributors: Gene Geocaniga, Jarissa Guiani, Darlene Magabilen TAU MU Page 172 of 179 Q: WHAT ABOUT SPERM TESTING?

PEOPLE vs. YATAR 428 SCRA 504 (2004)

TAU MU TAU MU TAU MU TAU MU

Facts: Senator Miriam Defensor Santiago introduced Senate Resolution 455, "directing an inquiry in aid of legislation on the anomalous losses incurred by the Philippines Overseas Telecommunications Corporation (POTC), Philippine Communications Satellite Corporation (PHILCOMSAT), and PHILCOMSAT Holdings Corporation (PHC) due to the alleged improprieties in their operations by their respective Board of Directors. Chief of Staf Inocencio, under the authority of Senator Gordon, wrote Chairman Sabio of the PCGG inviting him to be one of the resource persons in the public meeting jointly conducted by the Committee on Government Corporations and Public Enterprises and Committee on Public Services. The purpose of the public meeting was to deliberate on Senate Res. No. 455. Chairman Sabio declined the invitation because of prior commitment and at the same time, he invoked Section 4(b) of E.O. No. 1 which provides that “No member or staf of the Commission shall be required to testify or produce evidence in any judicial, legislative or administrative proceeding concerning matters within its official cognizance.” Several notices by virtue of a subpoena ad testificandum was issued to the petitioner-directors but still, they did not comply with the notice.

TAU MU

IN RE: SABIO 504 SCRA 704

TAU MU TAU MU

Ruling: The contention is untenable. The kernel of the right is not against all compulsion but against testimonial compulsion. The right against selfincrimination is simply against the legal process of extracting from the lips of the accused an admission of guilt. It does not apply where the evidence sought to be excluded is not an incrimination but as part of object evidence. A person may be compelled to submit to fingerprinting, photographing, paraffin, blood and DNA, as there is no testimonial compulsion is involved. It must also be noted that appellant in this case submitted himself for blood sampling which was conducted in open court on March 30, 2000, in the presence of counsel.

The Fraternal Ateneo de Davao

TAU MU TAU MU

Facts: Yatar was sentenced to death for special complex crime of rape with homicide. Subsequent testing showed that the DNA of the sperm specimen from the vagina of the victim was identical to the semen of the appellant’s gene type. Issue: Whether the blood sample taken from appellant as well as the DNA tests were conducted in violation of his right against self-incrimination.

KITY

TAU MU TAU MU TAU MU TAU MU TAU MU TAU MU

CONSTITUTIONAL LAW II Order of Saint Thomas More Atty. Philip John Pojas/Atty. Rovyne G. Jumao-as, RN University College of Law and not the inclusion of his body in evidence when it may be material. For instance, substance emitted from the body of the accused may be received as evidence in prosecution for rape, and morphine forced out of his mouth may also be used as evidence against him. Consequently, although accused insists that hair samples were forcibly taken from him and submitted to the NBI for forensic examination, the hair samples may be admitted in evidence against him for what is proscribed is the use of testimonial compulsion or any evidence communicative in nature acquired from the accused under duress.

officers of Philcomsat Holdings Corporation only when the incriminating question is being asked, since they have no way of knowing in advance the nature or efect of the questions to be asked of them. That this right may possibly be violated or abused is no ground for denying respondent Senate Committees their power of inquiry. The consolation is that when this power is abused, such issue may be presented before the courts. At this juncture, what is important is that respondent Senate Committees have sufficient Rules to guide them when the right against self-incrimination is invoked. Sec. 19 reads: Privilege Against SelfIncrimination - A witness can invoke his right against self-incrimination only when a question tends to elicit an answer that will incriminate him is propounded to him. However, he may ofer to answer any question in an executive session. No person can refuse to testify or be placed under oath or affirmation or answer questions before an incriminatory question is asked. His invocation of such right does not by itself excuse him from his duty to give testimony. In such a case, the Committee, by a majority vote of the members present there being a quorum, shall determine whether the right has been properly invoked. If the Committee decides otherwise, it shall resume its investigation and the question or questions previously refused to be answered shall be repeated to the witness. If the latter continues to refuse to answer the question, the Committee may punish him for contempt for contumacious conduct. The same directors and officers contend that the Senate is barred from inquiring into the same issues being litigated before the CA and the Sandiganbayan. Suffice it to state that the Senate Rules of Procedure Governing Inquiries in Aid of Legislation provide that the filing or pendency of any prosecution of criminal or administrative action should not stop or abate any inquiry to carry out a legislative purpose. So long as the constitutional rights of witnesses, like Chairman Sabio and his Commissioners, will be respected by respondent Senate Committees, it is their duty to cooperate with them in their eforts to obtain the facts needed for intelligent legislative action. The unremitting obligation of every citizen is to respond to subpoenae, to respect the dignity of the Congress and its Committees, and to testify fully with respect to matters within the realm of proper investigation. LASERNA vs. DANGEROUS DRUGS BOARD (G.R. No. 158633) SJS vs. DANGEROUS DRUGS BOARD (G.R. No. 157870) NOVEMBER 3, 2008

TAU MU

Facts: Section 36 of RA 9165 (Comprehensive Dangerous Drugs Act of 2002) provides that Authorized drug testing shall be done by any government forensic laboratories or by any of the drug testing laboratories accredited and monitored by the DOH to safeguard the quality of the test results. The drug testing shall employ, among others, two (2) testing methods, the screening test which will determine the positive result as well as the type of drug used and the confirmatory test which will confirm a positive screening test. The following shall be subjected to undergo drug testing: (c) Students of secondary and tertiary schools; (d) Officers and employees of public and private office; (f) All persons charged before the prosecutor's office with Issue: WON the subpoenae violated petitionera criminal ofense having an imposable penalty of directors right againt self-incrimination. imprisonment of not less than six years and one day shall undergo a mandatory drug test; and (g) All Ruling: The subpoenae did not violate their right candidates for public office whether appointed or against self-incrimination. It must be emphasized that elected both in the national or local government shall this right maybe invoked by the said directors and undergo a mandatory drug test. ACADCOM 2010; Contributors: Gene Geocaniga, Jarissa Guiani, Darlene Magabilen TAU MU Page 173 of 179

TAU MU

TAU MU

TAU MU

TAU MU TAU MU TAU MU TAU MU TAU MU TAU MU TAU MU TAU MU TAU MU

Facts: Paynor was charged with the crime of murder. On the strength of a positive identification of a witness, appellant was detained at the police station.

TAU MU TAU MU

Ruling: Unlike the situation covered by Sec. 36(c) and (d) of RA 9165, the Court finds no valid justification for mandatory drug testing for persons accused of crimes. In the case of students, the constitutional viability of the mandatory, random, and suspicionless drug testing for students emanates primarily from the waiver by the students of their right to privacy when they seek entry to the school, and from their voluntarily submitting their persons to the parental authority of school authorities. In the case of private and public employees, the constitutional soundness of the mandatory, random, and suspicionless drug testing proceeds from the reasonableness of the drug test policy and requirement. However, the SC find the situation entirely diferent in the case of persons charged before the public prosecutor's office with criminal ofenses punishable with six (6) years and one (1) day imprisonment. The operative concepts in the mandatory drug testing are "randomness" and "suspicionless." In the case of persons charged with a crime before the prosecutor's office, a mandatory drug testing can never be random or suspicionless. The ideas of randomness and being suspicionless are antithetical to their being made defendants in a criminal complaint. They are not randomly picked; neither are they beyond suspicion. When persons suspected of committing a crime are charged, they are singled out and are impleaded against their will. The persons thus charged, by the bare fact of being haled before the prosecutor's office and peaceably submitting themselves to drug testing, if that be the case, do not necessarily consent to the procedure, let alone waive their right to privacy. To impose mandatory drug testing on the accused is a blatant attempt to harness a medical test as a tool for criminal prosecution, contrary to the stated objectives of RA 9165. Drug testing in this case would violate a persons' right to privacy. Worse still, the accused persons are veritably forced to incriminate themselves. Thus, in this case, Section 36 (f) of RA 9165 was declared unconstitutional and Section © and (d) were declared constitutional.. PEOPLE vs. PAYNOR 261 SCRA 615 (1996)

The Fraternal Ateneo de Davao

TAU MU TAU MU

Issue: WON the aforesaid provision violated the constitutional right against self-incrimination.

KITY

TAU MU TAU MU TAU MU TAU MU TAU MU TAU MU

CONSTITUTIONAL LAW II Order of Saint Thomas More Atty. Philip John Pojas/Atty. Rovyne G. Jumao-as, RN University College of Law Petitioner Laserna, Jr., as citizen and taxpayer, seeks in his Petition for Certiorari and Prohibition that the aforementioned provision of RA9165 be struck down as unconstitutional for infringing on the constitutional right to privacy, the right against unreasonable search and seizure, and the right against self - incrimination, and for being contrary to the due process and equal protection guarantees. On the other hand, Social Justice Society, a registered political party, seeks to prohibit the Dangerous Drugs Board and the Philippine Drug Enforcement Agency from enforcing paragraphs (c), (d), (f), and (g) of Sec. 36 of RA 9165 on the ground that they are constitutionally infirm. For one, the provisions constitute undue delegation of legislative power when they give unbridled discretion to schools and employers to determine the manner of drug testing. For another, the provisions trench in the equal protection clause inasmuch as they can be used to harass a student or an employee deemed undesirable. And for a third, a person's constitutional right against unreasonable searches is also breached by said provisions.

Issue: Whether there was a violation of his constitutional right when the police stripped him of his clothing and personal items and the same were later introduced as evidence during trial. Ruling: The protection of the accused under custodial investigation, which is invoked by the appellant, refers to testimonial compulsion. Section 12, Article 3 of the Constitution provides that such accused shall have the right to be informed of his right to remain silent, the right to counsel and the right to waive the right to counsel in the presence of the counsel, and that any confession or admission obtained in violation of his rights shall be inadmissible in evidence against him. As held in People vs. Gamboa, this constitutional right applies only against testimonial compulsion and not when the body of the accused is proposed to be examined. In fact, an accused may validly be compelled to be photographed or measured, or his garments or shoes removed or replaced, or to move his body to enable the foregoing things to be done, without running afoul of the proscription against testimonial compulsion. CHAVEZ vs. CA Facts: The accused was charged with theft of a motor vehicle. During the trial the prosecutor called on the accused as a witness. The lawyer objected but was overruled. According to the judge, you only object when the statement is under scrutiny for trial. The accused was convicted and as stated in the decision, the accused was described as the star witness against himself. Ruling: The court held that in criminal cases the accused may refuse altogether to take the witness stand and answer all questions. The accused can invoke Sec. 17. Neither a witness can be compelled to take the witness stand. In contrary to the theory of the judge, accused need wait until the incriminating question is asked. He can refuse altogether to take the witness stand.  This is called a PROHIBITION OF INQUIRY wherein a witness can refuse altogether to take the witness stand or to answer all questions whether incriminating or not.  The court also stated that it should be only if the accused is allowed to take the stand since the purpose of the fiscal is mainly to incriminate him. This would be contrary to the rights of the accused. NOTE: The right can also be waived if the accused willingly takes the stand. VILLAFLOR vs. SUMMERS Facts: The accused was charged for adultery and upon motion of the prosecutor, the judge ordered her examined to find out if she is pregnant. She refused and was detained for contempt. She went to the SC. Ruling: The court ruled that the right extends to testimonial self incrimination only. Order of the Court was only for an ocular inspection of the body of the accused. So as not to embarrass the witness, the court ordered the physicians not to do any other means more than necessary. (NOTE: If this case happened now, the woman can also invoke the right to privacy and it will most probably be ordered in favor of the woman) BELTRAN vs. SAMSON Facts: Accused was charged with falsification and the fiscal ordered him to take a specimen of his writing and afterwards will compare his handwriting with the

ACADCOM 2010; Contributors: Gene Geocaniga, Jarissa Guiani, Darlene Magabilen TAU MU Page 174 of 179

Ruling: The court held this as a violation of the right to self-incrimination. Asking the accused to write is a positive estimated act. Writing is more than moving the hand, the body or the fingers. It is more than just a mechanical act because it requires the use of intelligence and attention. DISTINCTION BETWEEN VILLAFLOR CASES:

THE

BELTRAN

AND

TAU MU TAU MU TAU MU

SECTION 18 CANNOT BE INVOKED IN THE FOLLOWING: 1. In times of war, citizens can be forced to render military service or civil service as required by the Constitution [Art. II Sec. 4]; 2. In return-to-work orders;  If an employee participates in a strike, the employer can force him to resume work, otherwise he is deemed resigned.  This NOT compulsion because if this is not allowed the employer would be prejudiced because employees refuse to work for flimsy reasons. 3. If you are appointed as counsel de officio of the accused;

TAU MU

INVOLUNTARY SERVITUDE It is every condition of enforced or compulsory service of one to another no matter under what form such servitude may be disguised.

TAU MU

NOTE: There is no law authorizing as a penalty for a crime to be sufered to work. If in any case, the judge has to embody in the decision the order to work.

TAU MU

SECOND PARAGRAPH Freedom from Involuntary Servitude It is the liberty not to be compelled against your will to work whether paid or not.

TAU MU

Political Prisoner – a person detained solely for what he believes in (caveat: not a definite meaning). Under Amnesty International - the definition is broader such that they are those represented for acts done pursuant to his political beliefs.

TAU MU

FIRST PARAGRAPH The first Part speaks of the freedom of conscience or the freedom of thoughts. What is protected by this section are the political beliefs (the principles are similar to freedom of religion).

TAU MU

Section 18. (1) No person shall be detained solely by reason of his political beliefs and aspirations. (2) No involuntary servitude in any form shall exist except as a punishment for a crime whereof the party shall have been duly convicted.

TAU MU TAU MU

Evidence Villaflor was only asked to explain something already in existence, whereas in Beltran, accused was asked to produce evidence not yet in existence.

The Fraternal Ateneo de Davao

TAU MU TAU MU

Classify the act In the Villaflor case, the accused was not asked to perform a positive act. She was only asked to perform a negative act, or to present an examination of herself, whereas, in the Beltran case, the accused was asked to perform a positive act.

KITY

TAU MU TAU MU TAU MU TAU MU TAU MU TAU MU

CONSTITUTIONAL LAW II Order of Saint Thomas More Atty. Philip John Pojas/Atty. Rovyne G. Jumao-as, RN University College of Law writing of the falsified documents. The accused refused invoking the right against self-incrimination.

4. Punishment for a crime for which the party has been duly convicted 5. When he is enlisted in the services of a merchant vessel; (ROBERTSON vs. BALDWIN) 6. Posse Comitatus for the apprehension of criminals. The state may require some groups of people to assist in the pursuit of some criminals under the Police power 7. "Patria Potestas" unemancipated minors under parental power. (They must be minors under parental authority) Q: Can a person who ate in a restaurant and did not pay, be compelled to render service for the establishment? A: No. He can still invoke section 18 which prohibits involuntary servitude. CASES: INVOLUNTARY SERVITUDE ACLARACION vs. GATMAITAN Facts: Petitioner was a stenographer with the CFI. After working one year with the court, he transferred to another government entity, but he left some untranscribed notes of some cases on appeal. The court ordered him to finish the transcription, but he refused. He was reprimanded fro his refusal. (This case became moot and academic because there was a settlement subject to the condition that he will transcribe the notes) Ruling: The court held that the RTC stenographer can still be compelled to transcribe notes on cases on appeal without violating the right against involuntary servitude. This is part of the inherent power of the court necessary for the ordinance and efficient administration of justice. SARMIENTO vs. TUICO 162 SCRA 676 (1988) Facts: Petition for certiorari to review the order of the NLRC. Asian Transmission Corporation terminated the services of the petitioner for allegedly carrying deadly weapon in the premises. The striking workers were ordered to return to work immediately. Issue: Whether the return to work order results to involuntary servitude. Ruling: while one purpose of the return-to-work order is to protect the workers who might otherwise be locked out by the employer for threatening or waging the strike, the more important reason is to prevent impairment of the national interest in case the operations of the company are disrupted by a refusal of the strikers to return to work as directed. The return to work order imposes a duty. It is not considered violative of the right against involuntary servitude. Section 19. (1) Excessive fines shall not be imposed, nor cruel, degrading or inhuman punishment inflicted. Neither shall death penalty be imposed, unless, for compelling reasons involving heinous crimes, the Congress hereafter provides for it. Any death penalty already imposed shall be reduced to reclusion perpetua. (2) The employment of physical, psychological, or degrading punishment against any prisoner or detainee or the use of substandard or inadequate penal facilities under subhuman conditions shall be dealt with by law. Section 19 is not self-executing.

ACADCOM 2010; Contributors: Gene Geocaniga, Jarissa Guiani, Darlene Magabilen TAU MU Page 175 of 179

TAU MU TAU MU TAU MU TAU MU

INSTANCES WHEN PUNISHMENT MAY ALSO BE DEEMED CRUEL AND UNUSUAL: 1. If it inherently involves so much pain and sufering that civilized people cannot tolerate it. 2. If in the sense of being previously unknown it is imposed for a given ofense

TAU MU

 To be cruel and unusual or excessive punishment w/i the meaning of the constitution, the penalty must be flagrantly disproportionate to the ofense no matter under what circumstances the ofense may be committed.  If the punishment involves torture or lingering death.  Unconstitutional punishment implies something inhuman and barbarous, something more than the mere extinguishment of life.

TAU MU

CRUEL, DEGRADING and INHUMAN PUNISHMENT

TAU MU

2 CONCEPTS OF CRUEL AND UNUSUAL PUNISHMENT: 1. Those which public sentiment would regard as cruel and obsolete to law, refers to form, not on the severity or seriousness of the punishment. An example – crucifixion 2. Those which are disproportionate to the ofense as to shock the moral senses.

TAU MU

II. Cruel, degrading or inhuman punishment. So when is punishment considered cruel, inhuman or degrading? If it is flagrantly and plainly oppressive, wholly disproportionate to the ofense as to shock the moral sense of the community. When it involves torture or lingering death.

TAU MU

So this only applies to criminal cases because fines are imposed only in criminal cases. And it only applies to the fine imposed. If the court goes beyond the range provided for by law, then that is excessive fines. So what is the yardstick to measure that the fine imposed is not excessive. (what must the judge consider when imposing a fine) 1. The financial condition of the convict; 2. The amount fixed should be within the limit established by law; 3. The mitigating and aggravating circumstances attending the crime

TAU MU TAU MU

I. Excessive fines shall not be imposed. What is a fine? Fine is a penalty. So for example A was convicted of Arson with homicide. And the court awarded moral damages of 500 million, can A invoke his rights against excessive fines? No. Fines are provided by RPC, damages are not fines. Damages are civil liability.

The Fraternal Ateneo de Davao

TAU MU TAU MU

3. Death penalty unless for compelling reasons involving heinous crimes; 4. Right to be protected against physical, psychological or degrading punishment; 5. That any prisoner or detainee shall be entitled to adequated, standard penal facilities.

KITY

TAU MU TAU MU TAU MU TAU MU TAU MU TAU MU

CONSTITUTIONAL LAW II Order of Saint Thomas More Atty. Philip John Pojas/Atty. Rovyne G. Jumao-as, RN University College of Law 5 RIGHTS GUARANTEED BY SECTION 19: 1. Right to be protected against cruel and inhuman punishments; 2. Excessive fines shall not be imposed; Yardstick to measure that the fine imposed is not excessive. (What the judge must consider when imposing a fine) a. The financial condition of the convict; b. The amount fixed should be within the limit established by law; c. The mitigating and aggravating circumstances attending the crime

3. If it is excessive and serves a legislative purpose. 4. If popular sentiments abhors it even if not excessive or with a legislative purpose. DEATH PENALTY  Congress can re-impose death penalty (conditions):

a. b. c.  SC: If enough to

by enacting a law; for heinous crimes; there must be compelling reason. a crime is heinous, it is compelling reason impose the death penalty

EXCESSIVE FINE A fine is excessive, when under any circumstance, it is disproportionate to the ofense. Note: Fr. Bernas says that the accused cannot be convicted of the crime to which the punishment is attached if the court finds that the punishment is cruel, degrading or inhuman. Reason: Without a valid penalty, the law is not a penal law. STANDARDS USED 1. The punishment must not be so severe as to be degrading to the dignity of human beings. 2. It must not be applied arbitrarily. 3. It must not be unacceptable to contemporary society 4. It must not be excessive, i.e. it must serve a penal purpose more efectively than a less severe punishment would.  Penalty is cruel, degrading and inhuman if it is flagrantly and plainly oppressive, wholly disproportionate to the ofense as to shock the moral sense of the community. When it involves torture or lingering death. Q: What is the yardstick to measure if the penalty is justified? A: The penalty imposed should be proportional to the ofense committed. Take note that public officers may be meted out graver penalties because of public office is a public trust. LIM vs. CA: Estafa under PD 818 is punished as severely as reclusion perpetua. Q: Is the penalty disproportionate to the offense committed? A: No, because this kind of penalty is actually prescribed by law. It is actually provided for in PD 818 that for this kind of estafa, this is the penalty. Punishment authorized by a statute is not cruel, degrading or disproportionate to the nature of the ofense unless it is flagrantly and plainly oppressive. ZALDIVAR vs. SANDIGANBAYAN: indefinite suspension is not cruel, degrading or inhuman. III. Death Penalty. Under the Constitution, there is no death penalty. When the Constitution was ratified in 1987, it had no death penalty. But it had a provision that said that: Neither shall the death penalty be imposed, so that is the general rule, unless for compelling reasons involving heinous crimes, the Congress hereafter provides for it. So the Congress enacts a law providing for the death penalty. (RA 7659) The first person sentenced to die under RA 7659 was Echegaray. His lawyers questioned the death penalty claiming that this is cruel and inhuman punishment. The SC said that all punishments are cruel. What the Constitution means is something inhuman, and barbarous. Something more than just extinguishment

ACADCOM 2010; Contributors: Gene Geocaniga, Jarissa Guiani, Darlene Magabilen TAU MU Page 176 of 179

TAU MU TAU MU TAU MU TAU MU TAU MU TAU MU TAU MU TAU MU

Ruling: The evil of a crime may take various forms. There are crimes, like murder, rape, kidnapping, which are, by their nature despicable, either because life was callously taken or the victim is utterly dehumanized. There are crimes, like bribery, destructive arson, drug ofenses involving government officials, in which the abdomination lies in the significance and implications of the subject criminal acts in the scheme of the larger socio-political and economic context. There is no need for Congress to state the compelling reason for each and every heinous crime and statistical proof that such compelling reason actually exists. The elements of heinousness and compulsion are inseparable and are in fact interspersed with each other. The Court has no doubt that insofar as heinousness is concerned Congress has correctly identified the crimes warranting the death penalty. The claim of accused that the death penalty for the crime of rape is cruel and inhuman is based on a decision of the United States Supreme Court. In turn, said Court made a finding that (1) the public has manifested its rejection of the death penalty as a proper punishment for the crime of rape by the willful omission by state legislatures to include rape in their death penalty statutes, and (2) while the crime involves a dastardly contemptuous violation of a woman’s spiritual integrity and psychological balance, it does not involve the taking of life. The first ground, however has no bearing on Philippine experience and the context of our culture. As to the second, we disagree with the notion that whether or not the crime warrants the death penalty is the attendance of the circumstance of the death of the victim because it goes back to the idea of retributive justice, of “an eye for an eye.” The death penalty is imposed because the perpetrators have committed unforgivable acts causing irreparable and substantial injury to the victim and society so that they must be permanently prevented form repetition of their acts.

TAU MU

Facts: Accused was convicted of incestuous rape and sentenced to death under R.A. No. 7659 which reimposed the death penalty. He now argues that the death penalty law is unconstitutional for having been imposed in the absence of compelling reasons therefore and that the death penalty for rape is a cruel, excessive and inhuman punishment.

TAU MU TAU MU

PEOPLE vs. ECHEGARAY 267 SCRA 682 (1997)

The Fraternal Ateneo de Davao

TAU MU TAU MU

CASES: PROHIBITED PENALTIES

KITY

TAU MU TAU MU TAU MU TAU MU TAU MU TAU MU

CONSTITUTIONAL LAW II Order of Saint Thomas More Atty. Philip John Pojas/Atty. Rovyne G. Jumao-as, RN University College of Law of life. The death penalty is not cruel and inhuman because cruelty should be inherent in the method and not the punishment itself. The death penalty requires two reasons: so there must be a compelling reason and it only involves heinous crimes. So when is a crime considered heinous? They are crimes that by their nature are despicable. There are other crimes like bribery, destructive arson and drug ofenses, the abomination lies in the significance and the implication of the subject criminal act in the scheme of the larger socio-political and economic context. (What is the implication or the efect on society). What is this compelling reason? That the crime is heinous is reason compelling enough to bring back the death penalty. Heinous crime=compelling reason = death penalty justified. Kasi diba sabi ng SC it is not the penalty, it is the method. So balik na naman ang lawyers ni Echegaray saying that lethal injection is cruel and inhuman daw. The SC said that any infliction of pain in the lethal injection is incidental to the execution of the death penalty.

ECHEGARAY vs. SECRETARY 297 SCRA 754 (2002) Facts: In 1997, Congress passed R.A. No. 8177, designating death by lethal injection as the method of carrying out the death penalty. Pursuant to the law, the Secretary of Justice promulgated the implementing rules and directed the Director of the Bureau of Corrections to prepare the Lethal Injection Manual. Echegaray, a death convict, argues that death by the lethal injection is unconstitutional for being a cruel, degrading and inhuman punishment. Ruling: All punishments are cruel but of course the Constitution does not mean that crime, for this reason, is to go unpunished. It implies something inhuman and barbarous, something more that the mere extinguishments of life. The cruelty against which the Constitution protects a convicted man is cruelty inherent in the method of punishment, not the necessary sufering involved in any method employed to extinguish life humanely. Any infliction of pain in lethal injection is merely incidental in carrying out the execution of the death penalty. Courts in other countries have found that the few minutes of pain does not rise to a level of constitutional violation. Likewise, what is cruel and unusual must draw its meaning from the evolving standards of decency, and the widespread of lethal injection indicates that it comports with contemporary norms. LIM vs. PEOPLE 390 SCRA 194 (2002) Facts: The constitutionality of PD 818, a decree which amended Article 315 of the Revised Penal Code by increasing the penalties for estafa committed by means of bouncing checks, is being challenged in this petition for certiorari, for being violative of the due process clause, the right to bail and the provision against cruel, degrading or inhuman punishment enshrined under the Constitution. Petitioner spouses issued to private respondent two postdated checks. One check was dishonored upon presentment for having been drawn against insufficient funds. When petitioners reneged on their promise to cover the amount, PR filed a complaint for estafa against them. The trial court ruled against petitioner spouses. Issue: WON PD 818 was enacted in contravention of Section 19 of Article III of the Constitution Ruling: A punishment authorized by statute is not cruel, degrading or disproportionate to the nature of the ofense unless it is flagrantly and plainly oppressive and wholly disproportionate to the nature of the ofense as to shock the moral sense of the community. It takes more than merely being harsh, excessive, out of proportion or severe for a penalty to be obnoxious to the Constitution. Based on this principle, the Court has consistently overruled contentions of the defense that the penalty of fine or imprisonment authorized by the statute involved is cruel and degrading. In one case, the SC ruled that prohibition against cruel and unusual punishment is generally aimed at the form or character of the punishment rather than its severity in respect of its duration or amount, and applies to punishments which never existed in America or which public sentiment regards as cruel or obsolete. This refers, for instance, to those inflicted at the whipping post or in the pillory, to burning at the stake, breaking on the wheel, disemboweling and the like. The fact that the penalty is severe provides insufficient basis to declare a law unconstitutional and does not, by that circumstance alone, make it cruel and inhuman.

ACADCOM 2010; Contributors: Gene Geocaniga, Jarissa Guiani, Darlene Magabilen TAU MU Page 177 of 179

TAU MU TAU MU

2) Thus, if an accused fails to pay the fine imposed upon him, this may result in his subsidiary imprisonment because his liability is ex delicto and not ex contractu.

TAU MU

DEFINITION OF DEBT UNDER SECTION 20: 1) Debt refers to a CONTRACTUAL obligation, whether express or implied, resulting in any liability to pay money. Thus, all other types of obligations are not within the scope of this prohibition.

TAU MU

In non-payment of a debt, the debt assured in this section refers only to CONTRACTUAL DEBT (that which arises from contracts). Reason: this kind of contract is civil in nature.

TAU MU

NOTE: However, poll tax, as a condition precedent for the right to vote might be contrary to the Constitutional prohibition on literacy, property, or other substantive requirement as precedent to the right to vote.

TAU MU

According to Fr. Bernas, poll tax refers to residence certificate or cedula and the tax paid is a precedent to the exercise of the right of sufrage.

TAU MU

POLL TAX Tax paid as a condition precedent for the exercise of sufrage.

TAU MU

SEC. 20: No person shall be imprisoned for nonpayment of debt or poll tax (cedula)

TAU MU

Ruling: The court held that malversation is a serious ofense as it involves betrayal of public trust so the penalty cannot be considered unduly severe. At any rate, the remedy is to ask Congress to amend the law.

TAU MU TAU MU

Facts: Accused who was a municipal treasurer was accused of misappropriating the amount of P21,940.70. He was sentenced to 11 to 16 years of imprisonment. On appeal, he argues that the Revised Penal Code was enacted in 1952 and since then the peso has devaluated so that the penalty imposed on him has become obsolete, excessive and oppressive inflation. He pointed out that the penalty was severe as to shock the moral senses.

The Fraternal Ateneo de Davao

TAU MU TAU MU

AGBANLOG vs. PEOPLE

KITY

TAU MU TAU MU TAU MU TAU MU TAU MU TAU MU

CONSTITUTIONAL LAW II Order of Saint Thomas More Atty. Philip John Pojas/Atty. Rovyne G. Jumao-as, RN University College of Law In this case, the increase in the penalty, far from being cruel and degrading, was motivated by a laudable purpose, namely, to efectuate the repression of an evil that undermines the country’s commercial and economic growth, and to serve as a necessary precaution to deter people from issuing bouncing checks. The fact that PD 818 did not increase the amounts corresponding to the new penalties only proves that the amount is immaterial and inconsequential. What the law sought to avert was the proliferation of estafa cases committed by means of bouncing checks. Taking into account the salutary purpose for which said law was decreed, we conclude that PD 818 does not violate Section 19 of Article III of the Constitution. Moreover, when a law is questioned before the Court, the presumption is in favor of its constitutionality. To justify its nullification, there must be a clear and unmistakable breach of the Constitution, not a doubtful and argumentative one. T he burden of proving the invalidity of a law rests on those who challenge it. In this case, petitioners failed to present clear and convincing proof to defeat the presumption of constitutionality of PD 818.

3) A FRAUDULENT debt may result in the imprisonment of the debtor if: A. The fraudulent debt constitutes a crime such as estafa and B. The accused has been duly convicted. NON-PAYMENT OF POLL TAX  poll tax can be imposed but you cannot be imprisoned for not paying it  imposition of tax on right to vote --- BERNAS: you still cannot go to jail for not paying it because its imposition is prohibited by the Constitution  no jurisprudence yet NON-IMPRISONMENT FOR NON-PAYMENT OF DEBT  contractual debt only  you cannot be imprisoned for non-payment of contractual debt but you can be proceeded against for estafa  LABOR CODE: minimum wage --- if you pay your employee the amount less than that of the minimum wage, you cannot be imprisoned for it CASES: NON-PAYMENT OF DEBT VERGARA vs. GEDORIO 402 SCRA 520(2003) Facts: Petitioners are tenants of property and were arrested for indirect contempt for not complying with the probate courts orders directing them to pay monthly rentals. Some of the petitioners, together with the other tenants, informed the probate court that they are “freezing” their monthly rentals as they are in a quandary as to whom to pay the rentals. The probate court found them guilty and ordered them to pay a fine and undergo imprisonment until they comply with the order to pay rentals. Issue: Whether the order to undergo imprisonment violates the right to non-imprisonment of dent. Ruling: The Constitution provides that no person shall be imprisoned for debt. Debt refers to a civil debt or one not arising from a criminal ofense. It means that any liability to pay arising out of a contract, express or implied. The payment of rentals is covered by the Constitutional guarantee against imprisonment. PEOPLE vs. DELA CRUZ Facts: The accused sold meat for P0.30 and at that time, there was a price ceiling of P0.20 established by law. He was charged and convicted for overpricing. The sentence was 5 years imprisonment plus P5,000.00 fine. Also he cannot exchange in retail trade for 5 years (It was 195). The conviction was challenged as violative of his right against cruel and unusual punishment. Ruling: The court held the punishment as not shocking nor unusual. When seen with the policy of the government in protecting the public welfare, it is not disproportionate. Although the accused only earned P0.10, it is possible that he will earn a lot more if he will not be prosecuted. The damage to the government cannot be measured in terms of what he earned. This does not fall under the meaning of cruel and unusual act according to the 2 concepts. PEOPLE vs. DACUYCUY Facts: Violation of the Magna Carta for public school teachers provided for penalty of fine (P500-P1000) and

ACADCOM 2010; Contributors: Gene Geocaniga, Jarissa Guiani, Darlene Magabilen TAU MU Page 178 of 179

Ruling: the court ruled giving the 2 concepts for cruel and unusual. It is cruel an unusual when it is barbarous, obsolete or unknown to law, and when it is disproportionate as to shock the moral conscience. The law does not violate the right against cruel and unusual punishment based on the 2 concepts. If one of the 2 is violated, the law is unconstitutional. However, it is unconstitutional for being an undue delegation of legislative power. LOZANO vs. MARTINEZ

Trust Receipt Law Issue: Whether the trust receipt law violates the right on non-imprisonment for non-payment of debt.

TAU MU TAU MU

Ruling: The court held that the accused is wrong. Indemnity arises ex-delicto and not ex-contractu. The right only applies to debts arising from contracts. Also the judge was wrong because subsidiary imprisonment applies only to FINES and not to indemnity.

TAU MU

Facts: Accused was sentenced to imprisonment and ordered to pay an indemnity of P200.00 to the event that he is insolvent, he will serve subsidiary imprisonment. Accused challenged this for being violative of the right against imprisonment for nonpayment of debt.

TAU MU

AJENO vs. INSERTO

TAU MU

Ruling: The court rules No. The law does not seek to enforce payment of loan under the pain of imprisonment. The law punishes dishonesty and abuse of confidence in handling money or goods to the prejudice of others. What is protected is not the loan, which is a separate contract, but the security or property given.

TAU MU

PEOPLE vs. NITAFAN

TAU MU TAU MU

Ruling: The court held BP 22 does not permit the failure of the maker to pay his debt but only the making and issuance of a worthless check. The purpose of the law is not to force people to pay their debts under the pain of imprisonment. The purpose is to prohibit the issuance of worthless checks. This was likened to introducing garbage to the bloodstream of the economy.

The Fraternal Ateneo de Davao

TAU MU TAU MU

Bouncing Checks Law Issue: Whether or not BP 22 violates the right against non-imprisonment for non-payment of debt.

KITY

TAU MU TAU MU TAU MU TAU MU TAU MU TAU MU

CONSTITUTIONAL LAW II Order of Saint Thomas More Atty. Philip John Pojas/Atty. Rovyne G. Jumao-as, RN University College of Law imprisonment at the discretion of the Court. The accused challenged this as cruel and unusual.

TAU MU TAU MU TAU MU

ACADCOM 2010; Contributors: Gene Geocaniga, Jarissa Guiani, Darlene Magabilen TAU MU Page 179 of 179

View more...

Comments

Copyright ©2017 KUPDF Inc.
SUPPORT KUPDF